Clinical Integration Exam 8 Combined
Botulinum should be suspected in any infant with?
poor feeding and sucking, constipation, dilated pupils, weak cry, poor tone, and respiratory distress Sensory examination and mental status are normal.
What are the biases that twin studies are faced with?
1- Most serious: greater degree of environmental sharing among MZ twins 2- Somatic mutations - can affect one twin and not the other plus more
What are the mechanisms that are responsible for generating antibody diversity in somatic cells?
1- Multiple germline immunoglobulin genes 2- Somatic Recombination (VDJ recombination) 3- Junctional diversity 4- Somatic hypermutation 5- Multiple combinations of Heavy and Light chains
After ACh is released into the synaptic cleft what 2 receptors can it bind to?
1- Muscarinic Ach receptors- GPCR 2- Nicotinic Ach receptors - ligand gated ion channels these 2 channels share little structural similarity
What is Familial Combined Hyperlipoproteinemia? What factors can influence the phenotypic presentation?
Familial combined hyperlipoproteinemia (FCHL) is an autosomal dominant polygenic disorder that affects 1% to 2% of the population. Factors such as diet, glucose intolerance, and medications can influence the phenotypic presentation
What occurs as a result of Lysosomal acid lipase deficiency (LAL-D)?
Hepatic steatosis and dyslipidemia with elevated LDL-C, elevated triglycerides, and reduced HDL-C LAL-D is referred to as Wolman disease in infants and children or as cholesterol ester storage disease (CESD) in adults. Patients with LAL-D develop early atherosclerosis and progressive liver disease, with associated high rates of mortality at a young age
What does comparisons of correlations and concordance rates in MZ and DZ twins allow the estimation of?
Heritability a measure of the percentage of population variation in a disease that can be attributed to genes
What can be used to measure correlations and concordance rates in MZ and DZ twins?
Heritability of multifactorial traits heritability is the percentage of population variation in a trait that is due to genes (statistically, it is the proportion of the total variance of a trait that is caused by genes/ formula for estimating heritability (h) from twin correlations or concordance is as follows: h= Cmz- Cdz/ 1- Cdz cMZ is the concordance rate (or intraclass correlation) for MZ twins and cDZ is the concordance rate (or intraclass correlation) for DZ twins
What is the composition of most T cells (90%)?
Heterodimers- Alpha chain and Beta chain 10%- Heterodimers- alpha-beta or gamma-sigma receptors
What are Th2 involved in? What cytokines does it secrete?
Helps to fight multicellular parasites and are involved in allergic responses IL-4, IL-5, IL-6 and IL-13
What does IDL use for its transport into the liver?
Hepatic LDL receptors- apoprotein B-100 and E receptors
MOA and Use: methacholine
MOA: Cholinomimetic; ulate muscarinic receptor activity. USE: Inhalation, diagnosis of asthma
MOA and USE: lovastatin
MOA: Competitively inhibit HMG-CoA reductase (rate limiting step in cholesterol synthesis). USE: Treat high LDL, ASCVD, Diabetes. ** ↓↓↓ LDL, ↓TG, ↑HDL
MOA and USE: fluvastatin
MOA: Competitively inhibit HMG-CoA reductase (rate limiting step in cholesterol synthesis). USE: hypercholesterolemia, familial hypercholesterolemia, coronary atherosclerosis, prophylaxis for coronary atherosclerosis
T or F The combination of a statin with a bile acid sequestrant or cholesterol absorption inhibitor results in additive LDL decreases and is not associated with significant drug interactions
True
What are some adverse effects associated with Mipomersen use?
1- Injection site reactions 2- Flu-like symptoms 3- Increases in C-reactive protein 4- Increased transaminases
What 3 agents can be conisdered in the management of patients with refractory LDL elevations?
1- Neomycin 2- Lomitapide 3- Mipomersen
What 2 things does the Principal mechanism of T cell-mediated (CD8) killing of target involve?
1- Perforins 2- Granzymes are preformed mediators contained in the lysosome-like granules of CTLs (cytotoxic T cells)
What is the composition of each antibody?
2 Identical heavy chains (linked together by disulfide bonds) 2 Identical light chains
What is MHC restriction?
T cell receptors recognize peptides only in combination with MHC molecules on cell surfaces
Inhibitors of cholesterol absorption
Cholesterol absorption inhibitors reduce cholesterol absorption by the small intestine. Although this involves reduced absorption o dietary cholesterol, the more important e ect is reduced reabsorption o biliary cholesterol, which comprises the majority o intestinal cholesterol. Whereas statins and bile acid sequestrants reduce LDL cholesterol principally by increasing LDL clearance via the LDL receptor, inhibitors o cholesterol absorption also appear to reduce LDL cholesterol by inhibiting hepatic production o VLDL. The two available cholesterol absorption inhibitors are plant sterols and ezetimibe . Plant sterols and stanols are naturally present in vegetables and ruits, and they may be consumed in larger amounts rom nutritional supplements. Plant sterols and stanols are similar in molecular structure to cholesterol but are substantially more hydrophobic. As a result, plant sterols and stanols displace cholesterol rom micelles, increasing the excretion o cholesterol in the stool. The plant sterols and stanols are themselves poorly absorbed. Based on their mechanism o action, gram quantities o plant sterols and stanols are required to reduce plasma LDL-cholesterol concentrations by approximately 15%. Because an average diet contains 200-400 mg o plant sterols and stanols, these molecules must be highly enriched in dietary supplements (to approximately 2 grams) in order to be effective Ezetimibe decreases cholesterol transport rom micelles into enterocytes by selectively inhibiting cholesterol uptake through the brush border protein NPC1L1 At therapeutic concentrations, ezetimibe reduces intestinal cholesterol absorption by about 50%, without reducing the absorption o triglycerides or at-soluble vitamins The end result o reduced cholesterol absorption, achieved by either plant sterols and stanols or ezetimibe, is a decrease in LDL-cholesterol concentrations in the plasma. A reduction in cholesterol absorption presumably decreases the cholesterol content o chylomicrons and there ore decreases the transport o cholesterol rom the intestine to the liver. Within the liver, cholesterol derived rom chylomicron remnants contributes to the cholesterol that is packaged into VLDL particles. There ore, inhibiting cholesterol absorption can reduce cholesterol incorporation into VLDL and decrease LDL-cholesterol concentrations in the plasma. Importantly, reduced hepatic cholesterol content also leads to up-regulation o the LDL receptor, which contributes to the mechanism o LDL lowering by cholesterol absorption inhibitors. A single daily dose o ezetimibe lowers LDL-cholesterol concentrations by up to about 20%. Ezetimibe also lowers triglyceride concentrations by about 8% and elevates HDL cholesterol to a small extent (approximately 3%). Ezetimibe is particularly e ective in combination with a statin, or the ollowing reason. The reduction in hepatic cholesterol content due to inhibition o cholesterol absorption leads to a compensatory increase in hepatic cholesterol synthesis that partially o sets the benef ts o reducing absorption. By combining ezetimibe with a statin, the compensatory increase in hepatic cholesterol synthesis is prevented. This approach reduces LDL-cholesterol concentrations by an additional 15% compared with the e ect o the statin alone. The e ect is similar throughout the statin dose range. Unlike bile acid sequestrants (which are not absorbed), ezetimibe is rapidly absorbed by the enterocyte and extensively glucuronidated, so that systemic concentrations o both unmodif ed and glucuronidated orms can be measured. Ezetimibe undergoes enterohepatic circulation up to several times each day in conjunction with meals. Cholesterol absorption inhibitors have exhibited good sa ety prof les, with ew i any adverse e ects. Ezetimibe can increase plasma concentrations o cyclosporine, which should be monitored whenever these two drugs are co-administered
MOA and USE: colesevelam
"MOA: Inhibitor of bile acid absorption. bind to bile acids and preventing enterhepatic circulation. USE: hypercholesterolemia, diabetes mellitus type 2
What do Twin studies show about Schizophrenia?
1- A 47% concordance rate for MZ twins, compared with only 12% for DZ twins 2- The concordance rate for MZ twins reared apart, 46%, is about the same as the rate for MZ twins reared together. 3- The risk of developing the disease for offspring of a schizophrenic parent who are adopted by normal parents is about 10%, approximately the same as the risk when the offspring are raised by a schizophrenic biological parent.
What are the 2 most important systems that determine blood transfusion compatability?
1- ABO antigen system 2- Rh red cell antigen system red blood cell surface markers can cause immune reactions during blood transfusions
What are examples of omega-3 fatty acids?
1- Eicosapentaenoic acid (EPA) 2- Docosahexaenoic acid (DHA) also referred to as Fish oils
In individuals who are prone to allergies what does exposure to allergens result in the activation and production of what?
1- Activation of Th2 and Tfh cells 2- Production of IgE antibody Most individuals do not mount strong Th2 responses to environmental antigens. For unknown reasons, when some individuals encounter certain antigens, such as proteins in pollen, certain foods, insect venoms, or animal dander, or if they are treated with certain drugs such as penicillin, there is a strong Th2 response
How are X-linked dominant disorders transmitted? example of this inheritance pattern?
1- Affected heterozygous mother to half her sons and half her daughters 2- Affected male parent to all his daughters but none of his sons, if the female parent is unaffected eg: Vitamin D- resistant Rickets
What are the 2 key components of the adaptive immune response?
1- B lymphocytes (B cells) 2- T lymphocytes (T cells)
What are the 4 gene regions encode the heavy chains?
1- C- constant region 2- V- variable region 3- J - joining region 4- D- diversity region- located in bw the joining and variable regions
What are the 3 distinct segments that encode the light chain?
1- C: Constant region 2- V: Variable region 3- J: region that joins the constant region and variable regions
The antigens that form immune complexes may be either?
1- Exogenous eg: foreign particle that is injected or produced by an infectious microbe 2- Endogenous eg: if the individual produces antibody against self antigens (autoimmunity)
What are the 2 available fibrates in the US?
1- Gemfibrozil 2- Fenofibrate
What are the mechanisms by which spherical HDL stimulates cholesterol efflux?
1- Interaction of apoAI on HDL with SR-BI on the plasma membrane promotes cholesterol efflux 2- Macrophages express not only ABCA1 and SR-BI but also ABCG1, which also mediates cholesterol efflux to spherical HDL 3- Spherical HDL particles may promote cholesterol effux in the absence of binding to a specific cell surface protein
What 2 proteins enhance the capacity of HDL to remove cellular cholesterol ?
1- LCAT 2- PLTP they prevent the surface coat of the particle from becoming saturated with cholesterol.
What are characteristic clinical findings in Familial hypercholesterolemia?
1- Premature CHD (before 45 years in men and before 55 years in women) 2- Tendon xanthomas
What is the frequency of twin births?
1/100 births in populations of European ancestry They are slightly more common among African americans nad bit less common among Asians
What are Eosinophils activated by?
IL-5 which is produced by Th2, innate lymphoid cells and mast cells
What is the most effective treatment for improving muscle strength in Lambert Eaton MS? How much is prescribed and how often?
3,4- DAP works by blocking potassium (K+) channel efflux in nerve terminals so that action potential duration is increased. Calcium channels (Ca2+) can then be open for a longer time and allow greater acetylcholine release to stimulate muscle at the end plate 5 to 10 mg every 3 to 4 hours Pyridostigmine 60 mg every 4 hours is also used to improve symptoms.
What percent Serum antibody testing for AchR AB (binding antibody) positive in generalized MG patients vs ocular?
80% - generalized 50%- purely ocular Anti MuSK antibody is detected in a portion of seronegative patients, usually men
What is the principal transmitter at the Neuromuscular junction?
Acetylcholine (Ach)
What chromosome are several dozen of the polymorphisms associated with prostate cancer located on?
8q24 which contains polymorphisms associated with several other cancers as well (colon, pancreas, and esophagus) Although the 8q24 region contains no protein-coding genes, it contains enhancer elements that affect expression of the MYC oncogene, located about 250 kb from 8q24.
What is Wiskott-Aldrich syndrome? What is it caused by? treatment?
A X-linked recessive disorder that involves deficiencies of platelets and progressive T cell dysfunction Loss of function mutation in WAS gene, whose protein product is expressed on hematopoietic cells, where it relays signals from the cell surface to the cytoskeleton Gene therapy (like SCID)
What drugs are used in the treatment of Organophosphate poisoning?
A combination: 1- Antimuscarinic agent (e.g., atropine) 2- AChE reactivator, such as one of the pyridinium oximes (i.e., pralidoxime, trimedoxime, obidoxime, and HI6) 3-Diazepam are used for the treatment of OP poisoning in humans.
A patient you have been treating for severe allergies has been doing some research on why he gets such severe symptoms when he eats peanuts, but his brother doesn't. He tells you that it's due to the IgE antibodies which are attached to mast cells, and when they detect the antigen (peanuts in his case) the mast cells release cytokines causing him to have the severe symptoms. He also tells you that there is a thing called late-phase reaction, but doesn't know what leukocytes are involved in it. As his rheumatologist, what leukocyte is most likely responsible for the late-phase reaction? A.Neutrophils B. Basophils C. T Cells D. Plasma Cells E. Memory B Cells
A.Neutrophils
Lipid-poor pre-beta HDL particles, apoAI and apoAII can promote cholesterol efflux by interacting with what?
ABCA1
What are Neostigmine and Physostigmine hydrolyzed by?
ACHe
A small percentages of mutations are caused by what gene located on Chromosome 21?
APP gene encodes APP itself mutations in this gene lead to accumulation of the longer protein product It is interesting that this gene is present in three copies in persons with trisomy 21, where the extra gene copy leads to amyloid deposition and the frequent occurrence of earlyonset AD in Down syndrome patients
What are Neuromuscular junction (NMJ) diseases caused by?
Abnormal neuromuscular transmission of the action potential from the nerve terminal to the muscle
absorption of cholesterol and triglycerides
Absorption of cholesterol and triglycerides. Exogenous cholesterol and triglycerides are simultaneously absorbed rom the intestinal lumen by di erent mechanisms. Cholesterol is taken up rom micelles through a regulatory channel named NPC1L1. A raction o the cholesterol is pumped back into the lumen by ABCG5/G8, a heterodimeric ATP-dependent plasma membrane protein. The remainder o the cholesterol is converted to cholesteryl esters by ACAT. Triglycerides are taken up as atty acids and monoglycerides, which are re-esterif ed to triglycerides by DGAT. Ezetimibe inhibits cholesterol uptake through NPC1L1.
What do Acetylcholinesterase inhibitors bind to and inhibit? What does this result in?
Acetylcholinesterase (AChE) Increased concentration of endogenously released ACh in the synaptic cleft agents in this class are referred to as indirectly acting ACh receptor agonists bc they generally do not activate receptors directly
What can the muscular paralysis and autonomic blockade caused by nicotinic receptor antagonist (eg: Pancuronium) be reversed by?
Acetylcholinesterase inhibitors
What is the treatment of Alzheimer's disease?
Acetylcholinesterase inhibitors
Clinical applications
Acetylcholinesterase inhibitors have a number o clinical applications, including (1) increasing transmission at the neuromuscular junction, (2) increasing parasympathetic tone, and (3) increasing central cholinergic activity (e.g., to treat symptoms o f AD) Because o their ability to increase the activity o endogenous ACh, AChE inhibitors are especially use ul in diseases o the neuromuscular junction, where the primary de ect is an insu f cient quantity o either ACh or AChR. In myasthenia gravis, autoantibodies are generated against N M receptors. These antibodies both induce N M receptor internalization and block the ability o ACh to activate the receptors. As a result, patients with myasthenia gravis present with signif cant weakness (recall the description o Chie Opechancanough in the introductory case). Eaton-Lambert syndrome is also characterized by muscle weakness, but this disorder is caused by autoantibodies generated against Ca 2 channels; both presynaptic Ca 2 entry and the subsequent release o ACh in response to axon terminal depolarization are attenuated. Certain anticholinergic drugs, such as tubocurare, also cause weakness or paralysis by acting as competitive antagonists at the nAChR, preventing ACh rom binding to the receptor and causing nondepolarizing blockade o cholinergic transmission. Acetylcholinesterase inhibitors (such as the physostigmine used in the introductory case) improve all three o these conditions by increasing the concentration o endogenously released ACh at the neuromuscular junction and thereby increasing ACh signaling. Because ACh binding to N M receptors results in muscle cell depolarization, AChE inhibitors are ineffective at reversing the action of agents that cause paralysis by inducing sustained depolarization, such as succinylcholine (see below). In act, AChE inhibitors in su f ciently high doses can exacerbate existing weakness and paralysis caused by depolarizing blockade. Thus, it is o undamental importance that the cause o the muscle weakness should be determined be ore treatment is initiated. Short-acting AChE inhibitors such as edrophonium are ideal or such diagnostic purposes. Edrophonium mitigates weakness i the blockade is attributable to competitive AChR antagonists or to diseases such as myasthenia gravis or Eaton-Lambert syndrome. In contrast, i muscle strength decreases urther with edrophonium administration, then depolarizing blockade may be suspected. The short hal -li e o edrophonium ensures that exacerbation o the latter condition will last or a minimal amount o time. For chronic treatment o myasthenia gravis, longer acting AChE inhibitors such as pyridostigmine , neostigmine , and ambenonium are the pre erred agents AChE inhibitors exert other therapeutic e ects by potentiating parasympathetic actions in target tissues. Topical application o AChE inhibitors to the cornea o the eye decreases intraocular pressure by acilitating the out ow o aqueous humor. The main e ect o AChE inhibitors on the gastrointestinal system is an increase in smooth muscle motility because o enhancement o ganglionic transmission at Auerbach's plexus, although these agents also cause increased secretion o gastric acid and saliva. Neostigmine, the most popular drug or this application, is typically used or relie o abdominal distention. The use o anticholinesterases in reversing anticholinergic drug poisoning is also well established. The agent o choice or this indication is typically physostigmine ; its tertiary amine structure allows it ready access to the brain and spinal cord, where it can counteract the CNS e ects o anticholinergic toxicity. Acetylcholinesterase inhibitors are also used to treat the symptoms o AD dementia and other conditions causing dementia (e.g., Parkinson's disease with dementia, di use Lewy body dementia, vascular-ischemic dementia), brain injury (e.g., traumatic brain injury), and cognitive impairment (e.g., cognitive impairment associated with multiple sclerosis and schizophrenia). Donepezil and rivastigmine are secondgeneration AChE inhibitors indicated or the treatment o AD dementia in the mild, moderate, and severe stages; galantamine is a second-generation AChE inhibitor indicated or the treatment o AD dementia in the mild and moderate stages. Rivastigmine is also approved by the US Food and Drug Administration (FDA) or the treatment o Parkinson's disease with dementia. Tacrine is a f rst-generation AChE inhibitor that is no longer in clinical use—it had the disadvantage o our-times-daily dosing and it had the potential to cause hepatic toxicity In both short-term (24-52 weeks) e f cacy trials and long-term clinical e ectiveness studies, these AChE inhibitors have demonstrated benef cial e ects in producing improvement or stabilization o symptoms and in slowing the progression o cognitive, unctional, and behavioral decline in AD dementia. Although there are mechanistic and pharmacokinetic di erences among these drugs (Table 10-4), there are no signif cant di erences in their e f cacy in the treatment o AD. For example, rivastigmine is a "pseudoirreversible" cholinesterase inhibitor because it orms a labile carbamoylate complex with AChE (and BuChE), inactivating the enzyme until the covalent bond is broken. While rivastigmine is available as a twice-daily oral preparation, it is now used mostly as a once-daily transdermal patch ormulation. Galantamine is both a reversible AChE inhibitor and an allosteric (potentiating) nicotinic receptor ligand. All o these drugs exhibit linear pharmacokinetics, and their time to peak plasma concentration (T max ) values and elimination hal -lives are prolonged in elderly patients With appropriately slow titration, these medications are generally well tolerated and have a avorable adverse e ect prof le (with the exception o tacrine, which is no longer used clinically; see above). While these medications are relatively selective or AChE in the CNS, the most common adverse e ects—including nausea, vomiting, anorexia, atulence, loose stools, diarrhea, and abdominal cramping—are related to peripheral cholinomimetic effects on the GI tract. The rivastigmine transdermal patch can also cause skin irritation, redness, and rash at the site o application. The adverse e ects o AChE inhibitors may occur in 5-20% o patients, are usually mild and transient, and are related to the dose and rate o dose escalation. For the oral preparations, the adverse GI e ects o AChE inhibitors can be minimized by administering the drug a ter a meal or in combination with memantine, an NMDA channel blocker that is indicated or the treatment o moderate to severe AD. For transdermal rivastigmine, adverse e ects can be minimized by applying the patch to a di erent site each day. These medications may also increase the risk o syncope, particularly in susceptible individuals and with overdose. Use o these agents is contraindicated in patients with unstable or severe cardiac disease (particularly cardiac arrhythmias), uncontrolled epilepsy, unexplained or recurrent syncope, or active peptic ulcer disease.
What can be demonstrated in Lipoprotein lipase deficiency after refridgeration?
After refrigeration for 12 hours, a creamy top layer (increased chylomicrons) or turbid plasma infranatant (increased VLDL), or both, can be demonstrated
What does Hypertriglyceridemia most commonly develop with?
Age, weight gain, obesity and diabetes
Acetylcholinesterase inhibitors
Agents in this class bind to and inhibit AChE, thereby increasing the concentration o endogenously released ACh in the synaptic cle t. The accumulated ACh subsequently activates nearby cholinergic receptors. Agents in this class are also re erred to as indirectly acting ACh receptor agonists because they generally do not activate receptors directly. It is important to note that a ew AChE inhibitors have a direct action as well. For example, neostigmine , a quaternary carbamate, not only blocks AChE but also binds to and activates nAChRs at the neuromuscular junction
Muscarinic receptor agonists
Agents in this class are divided structurally into choline esters and alkaloids (Fig. 10-9). The choline esters are charged, highly hydrophilic molecules that are poorly absorbed by the oral route and ine f ciently distributed to the CNS. Choline esters include acetylcholine, methacholine, carbachol, and bethanechol (Table 10-5). Acetylcholine is not administered in clinical settings because o its broad actions and its extremely rapid hydrolysis by AChE and pseudocholinesterase. Methacholine is at least three times more resistant to hydrolysis by AChE than is ACh. This agent is relatively selective or cardiovascular muscarinic cholinergic receptors, and it has relatively little affinity or nicotinic cholinergic receptors. Although methacholine can stimulate receptors expressed on cardiovascular tissues, the magnitude o its response is unpredictable. This act has limited its use as a vasodilator or cardiac vagomimetic (i.e., a drug that mimics the cardiac response to vagus nerve [parasympathetic] stimulation, which typically includes bradycardia, decreased contractility, and compensatory sympathetic re exes). Currently, methacholine is used only in the diagnosis o asthma; in this application, the bronchial hyperreactivity that is characteristic o asthma causes an exaggerated bronchoconstriction response to parasympathomimetics Both carbachol and bethanechol are resistant to cholinesterases because, in these drugs, a carbamoyl group is substituted or the acetyl ester group o ACh (Fig. 10-9). This resistance to AChE extends their duration o action and allows time or distribution o the intact drug to areas o lower blood ow. Carbachol has enhanced nicotinic activity relative to other choline esters. This agent cannot be used systemically because its nicotinic action at autonomic ganglia leads to unpredictable responses. Instead, the agent is used principally as a topical miotic agent, typically in the treatment o glaucoma. Local application o the drug to the cornea o the eye results in both pupillary constriction (miosis) and decreased intraocular pressure. Bethanechol is almost completely selective or muscarinic receptors. It is an agent o choice or promoting GI and urinary tract motility, particularly or postoperative, postpartum, and drug-related urinary retention and or hypotonic neurogenic bladder. In contrast to the choline esters, the alkaloids vary greatly in structure. Some are amphipathic, while others are highly charged. Most o these agents are tertiary amines, although a ew are quaternary amines with protonated or permanently charged nitrogens substituting or the choline-centered N o ACh. The amphipathic nature o the tertiary amine alkaloids permits absorption through the GI mucosa and penetration into the CNS. Muscarine is an example o a quaternary amine alkaloid that has low bioavailability because o its permanently charged nature. Most alkaloids are primarily o value in pharmacologic research. The most clinically used alkaloid is pilocarpine , a miotic agent and a sialagogue (saliva-inducing agent) used to treat xerostomia (dryness o the mouth secondary to reduced salivary secretion). Cevimeline , an M 1 and M 3 agonist, is used to treat xerostomia in Sjögren's syndrome
What is the mode of Ehlers-Danlos syndromes?
All 3 Mendelian patterns - autosomal dominant, autosomal recessive and X-linked
What ages are affected and what is the prevelance in MG?
All ages affected but: Higher in women younger than 40 and men over 50 20/100,000
Receptor Agonists
All cholinergic receptor agonists bind to the ACh binding site o cholinergic receptors. Receptor agonists can be divided into muscarinic and nicotinic receptor-selective agents, although some cross-reactivity exists with virtually all o these agents. Muscarinic receptor agonists are used clinically in the diagnosis o asthma and as miotics (agents that cause pupil constriction). Nicotinic receptor agonists are used clinically or induction o muscle paralysis.
What does mast cell activation result from the binding of?
Allergen to 2 or more IgE antibodies on the cell
X-linked disorders
All sex-linked disorders are X-linked, and almost all are recessive. Several genes are located in the "male-specific region of Y"; all of these are related to spermatogenesis. Males with mutations affecting the Y-linked genes are usually infertile, and hence there is no Y-linked inheritance. As discussed later, a few additional genes with homologues on the X chromosome have been mapped to the Y chromosome, but only a few rare disorders resulting from mutations in such genes have been described. X-linked recessive inheritance accounts for a small number of well-defined clinical conditions. The Y chromosome, for the most part, is not homologous to the X, and so mutant genes on the X do not have corresponding alleles on the Y. Thus, the male is said to be hemizygous for X-linked mutant genes, so these disorders are expressed in the male. Other features that characterize these disorders are as follows: • An affected male does not transmit the disorder to his sons, but all daughters are carriers. Sons of heterozygous women have, of course, one chance in two of receiving the mutant gene. • The heterozygous female usually does not express the full phenotypic change because of the paired normal allele. Because of the random inactivation of one of the X chromosomes in the female, however, females have a variable proportion of cells in which the mutant X chromosome is active. Thus, it is remotely possible for the normal allele to be inactivated in most cells, permitting full expression of heterozygous X-linked conditions in the female. Much more commonly, the normal allele is inactivated in only some of the cells, and thus the heterozygous female expresses the disorder partially. An illustrative condition is glucose-6-phosphate dehydrogenase (G6PD) deficiency. Transmitted on the X chromosome, this enzyme deficiency, which predisposes to red cell hemolysis in patients receiving certain types of drugs (Chapter 14), is expressed principally in males. In the female, a proportion of the red cells may be derived from precursors with inactivation of the normal allele. Such red cells are at the same risk for undergoing hemolysis as are the red cells in the hemizygous male. Thus, the female is not only a carrier of this trait but also is susceptible to drug-induced hemolytic reactions. Because the proportion of defective red cells in heterozygous females depends on the random inactivation of one of the X chromosomes, however, the severity of the hemolytic reaction is almost always less in heterozygous women than in hemizygous men. Most of the X-linked conditions listed in Table 5-3 are covered elsewhere in the text. There are only a few X-linked dominant conditions. They are caused by dominant disease-associated alleles on the X chromosome. These disorders are transmitted by an affected heterozygous female to half her sons and half her daughters and by an affected male parent to all his daughters but none of his sons, if the female parent is unaffected. Vitamin D-resistant rickets is an example of this type of inheritance.
What is a positive Tensilon test?
An unequivocal improvement in strength in an affected muscle after 2-5 minutes from administration of 2 mg incremental diagnoses up to 10mg
What is the most severe form of Immediate hypersensitivtiy? What are the common producers that cause it?
Anaphylaxis a systemic reaction characterized by edema in many tissues, including the larynx, accompanied by a fall in blood pressure and bronchoconstriction bee stings, injected or ingested penicillin-family antibiotics, and ingested nuts or shellfish.
Treatment of immediate hypersensitivity reactions
Anaphylaxis Bronchial asthma Various allergic diseases Epinephrine Corticosteroids Leukotriene antagonists Phosphodiesterase inhibitors Desensitization (repeated administration of low doses of allergens) Causes vascular smooth muscle cell contraction, increases cardiac output (to counter shock), and inhibits bronchial smooth muscle cell contraction Reduce inflammation Relax bronchial smooth muscle and reduce inflammation Relax bronchial smooth muscles Unknown; may inhibit IgE production and increase production of other Ig isotypes; may induce T cell tolerance Anti-IgE antibody Neutralizes and eliminates IgE Antihistamines Block actions of histamine on vessels and smooth muscles Cromolyn Inhibits mast cell degranulation
ABO and Rh blood groups
Another component of the immune system involves red blood cell surface molecules that can cause an immune reaction during blood transfusions. The ABO and Rh red- cell antigen systems were discussed in Chapter 3 as early examples of polymorphic marker loci. They are also the most important systems that determine blood transfusion compatibility.
What causes the delay in delayed type hypersensitivtiy?
Because it takes several hours for circulating effector T lymphocytes to home to the site of antigen challenge, respond to the antigen at this site, and secrete cytokines that induce a detectable reaction.
What is the functions of Statin?
Competitively inhibit the activity of HMG-CoA reductase which is the rate limiting step in cholesterol synthesis inhibition of HMG-CoA results in modest decrease in cellular cholesterol synthesis
What do Nicotinic acetylcholine receptors (nAChRs) mediate cholinergic transmission via?
Direct ligand-gated conductance The binding of two ACh molecules to one nAChR elicits a conformational change in the receptor that creates a monovalent cation selective pore through the cell membrane.
What type of traits make up the largest category of Mendelian disorders? When do they occur?
Autosomal recessive traits when both alleles at a given gene locus are mutated
How does Somatic recombination (VDJ recombination) occur?
As immunoglobulin molecules are formed during B lymphocyte maturation, a specific combination of of single V and J segments is selected for the light chain and another V, D and J segments are selected for the heavy chain Because there are many possible combinations of single V, J, and D segments, somatic recombination can generate approximately 100,000 to 1,000,000 different types of antibody molecules
What do Cytokines produced by mast cells stimulate the recruitment of? What reaction do they cause?
Leukocytes Late phase reaction
What occurs in Junctional diversity?
As the V,D,J regions are assembled slight variations occur in the position in which they are joined, and small numbers of nucleotides may be deleted or inserted at the junctions joining the regions This creates even more variation in antibody amino acid specific
How are gain of function mutations almost always transmitted?
Autosomal dominant eg: Huntington disease
What is familial hypercholesterolemia?
Autosomal dominant disease involving defects in the LDL receptor Mutations in the gene encoding the LDL receptor result in one of four molecular defects: 1- Lack of receptor synthesis 2- Failure to reach the plasma membrane 3- Defective LDL binding 4-Failure to internalize bound LDL particle
What type of disorder is Familial hypertriglyceridemia? What is it characterized by?
Autosomal dominant disorder Overproduction of hepatic VLDL the exact defect or mutation is unknown
Receptor antagonists
Antagonists o AChRs act by binding directly to the agonist site and competitively blocking stimulation o the receptor by endogenous ACh or exogenously administered receptor agonists.
What do mature B lymphocytes secrete?
Antibodies combat infections AKA Humoral immune system
People with A,B or AB antigens on their erythrocyte surfaces possess what?
Antibodies against all other ABO antigens in their blood stream eg: if a type B person received type A or AB blood, his/her anti-A antibodies would produce a severe and possibly fatal reaction Type O persons since they lack A or B antigen have both anti-A and anti-B antibodies and so would react strongly to blood of the other 3 types (A,B and AB)
What occurs in a Type III hypersensitivity reaction?
Antibodies against soluble antigens may form complexes with the antigens, and the immune complexes may deposit in blood vessels in various tissues, causing inflammation and tissue injury
What are Apolipoproteins?
Are amphipathic proteins that intercalate into the surface coat of lipoproteins AKA apoproteins They may act as ligands or lipoprotein receptors or may activate enzymatic activities in the plasma.
What do Cholinergic neurons play an essential role in?
Arousal and attention levels of Ach throughout the brain increase during wakefulness and REM sleep and decrease during inattentive states and non-REM/slow wave sleep (SWS)
normal process of cholesterol metabolism and transport
Approximately 7% of the body's cholesterol circulates in the plasma, predominantly in the form of LDL. As might be expected, the amount of plasma cholesterol is influenced by its synthesis and catabolism, and the liver plays a crucial role in both these processes (Fig. 5-6). The first step in this complex sequence is the secretion of very-lowdensity lipoproteins (VLDLs) by the liver into the bloodstream. VLDL particles are rich in triglycerides, but they contain lesser amounts of cholesteryl esters. When a VLDL particle reaches the capillaries of adipose tissue or muscle, it is cleaved by lipoprotein lipase, a process that extracts most of the triglycerides. The resulting molecule, called intermediate-density lipoprotein (IDL), is reduced in triglyceride content and enriched in cholesteryl esters, but it retains two of the three apoproteins (B-100 and E) present in the parent VLDL particle (Fig. 5-6). After release from the capillary endothelium, the IDL particles have one of two fates. Approximately 50% of newly formed IDL is rapidly taken up by the liver by receptor-mediated transport. The receptor responsible for the binding of IDL to the liver cell membrane recognizes both apoprotein B-100 and apoprotein E. It is called the LDL receptor, however, because it is also involved in the hepatic clearance of LDL (described later). In the liver cells, IDL is recycled to generate VLDL. The IDL particles not taken up by the liver are subjected to further metabolic processing that removes most of the remaining triglycerides and apoprotein E, yielding cholesterol-rich LDL particles. IDL is the immediate and major source of plasma LDL. There seem to be two mechanisms for removal of LDL from plasma, one mediated by an LDL receptor and the other by a receptor for oxidized LDL (scavenger receptor), described later Although many cell types, including fibroblasts, lymphocytes, smooth muscle cells, hepatocytes, and adrenocortical cells, possess high-affinity LDL receptors, approximately 70% of the plasma LDL is cleared by the liver, using a quite sophisticated transport process (Fig. 5-7). The first step involves binding of LDL to cell surface receptors, which are clustered in specialized regions of the plasma membrane called coated pits (Chapter 1). After binding, the coated pits containing the receptor-bound LDL are internalized by invagination to form coated vesicles, after which they migrate within the cell to fuse with the lysosomes. Here the LDL dissociates from the receptor, which is recycled to the surface. In the lysosomes the LDL molecule is enzymatically degraded; the apoprotein part is hydrolyzed to amino acids, whereas the cholesteryl esters are broken down to free cholesterol. This free cholesterol, in turn, crosses the lysosomal membrane to enter the cytoplasm, where it is used for membrane synthesis and as a regulator of cholesterol homeostasis. The exit of cholesterol from the lysosomes requires the action of two proteins, called NPC1 and NPC2 (see "Niemann-Pick Disease Type C"). Three separate processes are affected by the released intracellular cholesterol, as follows: • Cholesterol suppresses cholesterol synthesis within the cell by inhibiting the activity of the enzyme 3-hydroxy3-methylglutaryl coenzyme A (HMG CoA) reductase, which is the rate-limiting enzyme in the synthetic pathway . • Cholesterol activates the enzyme acyl-coenzyme A: cholesterol acyltransferase, favoring esterification and storage of excess cholesterol . • Cholesterol suppresses the synthesis of LDL receptors, thus protecting the cells from excessive accumulation of cholesterol. As mentioned earlier, familial hypercholesterolemia results from mutations in the gene encoding the receptor for LDL. Heterozygotes with familial hypercholesterolemia possess only 50% of the normal number of highaffinity LDL receptors, because they have only one normal gene. As a result of this defect in transport, the catabolism of LDL by the receptor-dependent pathways is impaired, and the plasma level of LDL increases approximately twofold. Homozygotes have virtually no normal LDL receptors in their cells and have much higher levels of circulating LDL. In addition to defective LDL clearance, both the homozygotes and heterozygotes have increased synthesis of LDL. The mechanism of increased synthesis that contributes to hypercholesterolemia also results from a lack of LDL receptors (Fig. 5-6). IDL, the immediate precursor of plasma LDL, also uses hepatic LDL receptors (apoprotein B-100 and E receptors) for its transport into the liver. In familial hypercholesterolemia, impaired IDL transport into the liver secondarily diverts a greater proportion of plasma IDL into the precursor pool for plasma LDL The transport of LDL via the scavenger receptor seems to occur at least in part into the cells of the mononuclear phagocyte system. Monocytes and macrophages have receptors for chemically altered (e.g., acetylated or oxidized) LDL. Normally the amount of LDL transported along this scavenger receptor pathway is less than that mediated by the LDL receptor-dependent mechanisms. In the face of hypercholesterolemia, however, there is a marked increase in the scavenger receptor-mediated traffic of LDL cholesterol into the cells of the mononuclear phagocyte system and possibly the vascular walls (Chapter 11). This increase is responsible for the appearance of xanthomas and contributes to the pathogenesis of premature atherosclerosis. The molecular genetics of familial hypercholesterolemia is extremely complex. More than 900 mutations involving the LDL receptor gene, including insertions, deletions, and missense and nonsense mutations, have been identified. These can be classified into five groups (Fig. 5-8): Class I mutations are relatively uncommon and lead to a complete failure of synthesis of the receptor protein (null allele). Class II mutations are fairly common; they encode receptor proteins that accumulate in the endoplasmic reticulum because their folding defects make it impossible for them to be transported to the Golgi complex. Class III mutations affect the LDL-binding domain of the receptor; the encoded proteins reach the cell surface but fail to bind LDL or do so poorly. Class IV mutations encode proteins that are synthesized and transported to the cell surface efficiently They bind LDL normally, but they fail to localize in coated pits, and hence the bound LDL is not internalized. Class V mutations encode proteins that are expressed on the cell surface, can bind LDL, and can be internalized; however, the pH-dependent dissociation of the receptor and the bound LDL fails to occur. Such receptors are trapped in the endosome, where they are degraded, and hence they fail to recycle to the cell surface The discovery of the critical role of LDL receptors in cholesterol homeostasis has led to the rational design of drugs that lower plasma cholesterol by increasing the number of LDL receptors. One strategy is based on the ability of certain drugs (statins) to suppress intracellular cholesterol synthesis by inhibiting the enzyme HMG CoA reductase. This, in turn, allows greater synthesis of LDL receptors (Fig. 5-8). Statins have been widely and successfully used for secondary prevention of ischemic heart disease. They exemplify rational design of drugs based on an understanding of pathophysiology
What adverse effects can antimuscarinic agents such as Atropine and Scopalamine cause?
At low levels: Bradycardia and sedation at low to medium levels o muscarinic blockade, and tachycardia and At high levels: CNS hyperexcitation (with delirium, hallucinations, and seizures) at higher level Other adverse effects may include blurred vision (cycloplegia and mydriasis), dry mouth, ileus, urinary retention, flushing and fever, agitation, and tachycardia
What are high plasma levels of LDL major risk factors for the development of?
Atherosclerosis and subsequent cardiovascular disease
What drug is available during a Tensilon test due to the fact that bradycardia and hypotension are possible side effects?
Atropine
What was one of the first drugs used in the treatment of symptoms associated with Parkinson's syndrome?
Atropine still used at times to ameliorate tremor and rigidity in patients with PD
autism spectrum disorder
Autism spectrum disorder (ASD) is characterized by impairments in social interaction and communication, as well as restricted, repetitive, and stereotyped behaviors and activities. The definition of ASD has been broadened considerably in the past decade or so (e.g., it now includes Asperger syndrome), resulting in a much higher current prevalence estimate (approximately 1% worldwide). ASD is 3-4 times more common in males than in females and is usually diagnosed during the first three years of life. As predicted by the multifactorial threshold model, the empirical recurrence risk is substantially higher in the siblings of females with ASD. Genetic factors play a large role in the etiology of ASD, with twin studies suggesting a heritability of approximately 70%. Many large, well-controlled studies have investigated the relationship between ASD and vaccination (in particular for measles, mumps, and rubella), and no relationship has been found. Approximately 10% of ASD cases have a known mendelian condition or genetic syndrome, most of which have been discussed in previous chapters. These include Fragile X syndrome (1-2% of cases), Rett syndrome (0.5%), tuberous sclerosis (1%), and neurofibromatosis type 1 (<1%). The most frequent cytogenetic abnormality seen in persons with ASD (1-3%) is a maternally transmitted duplication of chromosome 15q11-q13, the region that is deleted in Prader-Willi and Angelman syndromes (see Chapter 5). A 600 kb duplication or deletion of 16p11.2 is seen in about 1% of ASD cases Numerous GWAS and sequencing studies have been carried out to identify additional loci associated with ASD, and more than 100 susceptibility loci (each with small effect) have been identified. An increased number of de novo mutations are seen in children with ASD, most of which are inherited paternally (these are detected by comparing the DNA sequences of affected cases with those of their unaffected parents). This may help to explain the increased risk of autism among the offspring of older fathers. In addition, an increase in the number of copy number variants (CNVs) is seen in persons with autism Psychiatric disorders, such as schizophrenia, bipolar disorder, and ASD, present a number of challenges for genetic analysis. These diseases are undoubtedly heterogeneous, reflecting the influence of numerous genetic and environmental factors (the same is true of other psychiatric diseases, such as attention hyperactivity deficit disorder, which are not discussed here). This genetic heterogeneity is to be expected, in part because at least one-third of human genes are expressed in the brain. Also, the definition of a psychiatric phenotype is not always straightforward, and it can change through time. Several measures are being taken to improve the likelihood of finding genes underlying these conditions. Phenotypes are being defined in standardized and rigorous fashion. Larger sample sizes of affected persons (often tens of thousands), with more rigorous phenotype definition, are being collected in efforts to increase the power to detect linkage and association. Heterogeneity can be decreased by studying clinically defined subtypes of these diseases and by carrying out studies in genetically homogeneous populations. Marked familial aggregation, as well as familial cooccurrence, has been observed for schizophrenia, bipolar disorder, and autism spectrum disorder. Casecontrol and family studies have revealed evidence that these conditions are caused by brainexpressed genes that encode neurotransmitters, receptors, ion channels, and neurotransmitterrelated enzymes.
Autonomic effects
Autonomic activity can be classif ed as either tonic activity, which accounts or end organ stimulation at rest, or phasic activity, which triggers an elevated response to changing conditions. Neurotransmission through autonomic ganglia is complicated because several distinct receptor types contribute to the complex changes observed in postganglionic neurons. The generalized postsynaptic response to presynaptic impulses can be separated into our distinct components (Fig. 10-7). The primary event in the postsynaptic ganglionic response is a rapid depolarization mediated by nicotinic ACh receptors in the cell membrane of postganglionic neurons . The mechanism is similar to that in the NMJ, in that an inward current elicits a near-immediate excitatory postsynaptic potential (EPSP) o 10-50 ms in duration. Typically, the amplitude o such an EPSP is only a ew millivolts, and many such events must sum or the postsynaptic cell membrane to reach the threshold or f ring an action potential (Fig. 10-7A). The three remaining events o ganglionic transmission modulate this primary signal and are known as the slow EPSP, the IPSP (inhibitory postsynaptic potential), and the late, slow EPSP. The slow EPSP occurs a ter a latency o 1 second and is mediated by M 1 muscarinic ACh receptors. The duration o this e ect is 10-30 seconds (Fig. 10-7C). The IPSP is largely a product o catecholamine (i.e., dopamine and norepinephrine) stimulation o dopaminergic and -adrenergic receptors (see Chapter 11, Adrenergic Pharmacology), although some IPSPs in a ew ganglia are mediated by M 2 muscarinic receptors. The latency and duration o the IPSPs generally vary between those o the ast and slow EPSPs. The late, slow EPSP is mediated by a decrease in potassium conductance induced by stimulation o receptors or peptide transmitters (i.e., angiotensin, substance P, and luteinizing hormone-releasing hormone). Lasting or several minutes, the late, slow EPSP is thought to have a role in the long-term regulation o postsynaptic neuron sensitivity to repetitive depolarization. One pharmacologic consequence o such a complex pattern o depolarization in autonomic ganglia is that drugs selective or the IPSP, slow EPSP, and late, slow EPSP are generally not capable o eliminating ganglionic transmission. Instead, such agents alter only the e f ciency o transmission. For example, methacholine , a muscarinic receptor agonist, has modulatory e ects on autonomic ganglia that resemble the stimulation o slow EPSPs (see below). Blockade o excitatory transmission through autonomic ganglia relies on inhibition o the nAChRs that mediate ast EPSPs. The overall effect of ganglionic blockade is complex and depends on the relative predominance of sympathetic and parasympathetic tone at the various end organs (Table 10-2). For example, the heart is in uenced at rest primarily by the parasympathetic system, whose tonic e ect is a slowing o the heart rate. Thus, blockade o autonomic ganglia that innervate the heart by moderate to high doses o the antimuscarinic agent atropine results in blockade o vagal slowing o the sinoatrial node and hence in relative tachycardia . (It should be noted that in low doses, the central parasympathetic stimulating e ects o atropine predominate, initially resulting in bradycardia prior to its peripheral vagolytic action.) Blood vessels, in contrast, are innervated only by the sympathetic system. Because the normal e ect o sympathetic stimulation is to cause vasoconstriction, ganglionic blockade results in vasodilation. It is important to realize, however, that the responses described above ignore the presence o muscarinic ACh receptors at many o the end organs. When stimulated directly by cholinergic agents, such receptors o ten mediate a response that overrides the response produced by ganglionic blockade. In general, the expected net cardiovascular e ects o muscarinic blockade produced by clinical doses o atropine in a healthy adult with a normal hemodynamic state are mild tachycardia, with or without ushing o the skin, and no pro ound e ect on blood pressure. The muscarinic receptor subtypes expressed in visceral smooth muscle, cardiac muscle, secretory glands, and endothelial cells mediate highly diverse responses to cholinergic stimulation. These e ects are detailed in Table 10-3. In general, these end-organ e ects tend to predominate over ganglionic in uences. That is, or systemically administered cholinergic agonists, the overall response is generally similar to that caused by direct stimulation o the postganglionic e ector sites and o ten di erent rom that caused by ganglionic stimulation alone
What type of disease is X-linked agammaglobulinemia (XLA)? What is it characterized by? Mutation in what gene? How are they treated?
B cell immunodeficiency most patients are male and lack B cells completely and have no IgA, IgE, IgM or IgD. They have some amount of IgG they get from their mother but this can run out in infancy as the get infections BTK gene- encodes B cell tyrosine kinase necessary for B cell maturation large amounts of Gamma-globulin
A 38-year-old female has brought her 9-month-old to her pediatrician's office, as she has noticed that she has been losing weight gradually over the past 6 months. She states that her diet has been normal, and even an excess of eating, that there has been gradual weight loss. She also has noticed very oily, foul smelling stools when changing her diaper. What could be the primary mechanism behind the child's weight loss? A) An upregulation of LPL within the muscle and adipose B) A decrease in the amount of apoCII found on the surface of the chylomicron C) A decrease in the amount of apo E presence on the chylomicron D) An upregulation of LDL receptors on the liver E) An increase in the amount of GPI linked protein
B) A decrease in the amount of apoCII found on the surface of the chylomicron
Understand
Because of their ability to increase activity of endogenous ACh, AChE inhibitors are especially useful in diseases of the NMJ where the primary defect is an insufficient quantity of either ACh or AChR eg: treats for eg: Myasthenia gravis, Lambert eaton Acetylcholinesterase inhibitors (such as the physostigmine used in the introductory case) improve all of these conditions by increasing the concentration of endogenously released ACh at the neuromuscular junction and thereby increasing ACh signaling.
How is plaque stability enhanced with Statin therapy?
Because the fibrous cap that overlies the lipid-rich plaque becomes thicker This effect may be attributable to decreased macrophage infiltration and inhibition of vascular smooth muscle proliferation
What is the main treatment used for Marfan Syndrome?
Beta blockers likely act by reducing heart rate and aortic wall stress.
What is the agent of choice for promoting GI and urinary tract motility, particularly for postoperative, postpartum and drug related urinary retention and for hypotonic neurologic bladder?
Bethanechol
What is cholesterol converted by the liver?
Bile acids
What is LDL- cholesterol used to synthesize in the liver?
Bile acids which are secreted into the intestinal lumen along with free cholesterol
the genetic basis of antibody diversity
Because the immune system cannot "know" in advance which types of microbes it will encounter, the system must contain a huge reservoir of structurally diverse immune cells so that at least a few cells can respond (i.e., bind) to any invading microbe. Indeed, the humoral immune system is capable of generating approximately 100 billion structurally distinct antibodies. At one time, it was thought that because each antibody has a unique amino acid sequence, each must be encoded by a different gene. However, this one gene-one antibody hypothesis could not possibly be correct, because the human genome has only 20,000 to 25,000 protein-coding genes. Further study has shown that several mechanisms are responsible for generating antibody diversity in somatic cells: 1. Multiple Germline Immunoglobulin Genes Molecular genetic studies (cloning and DNA sequencing) have shown that for each heavy and light chain, an individual has more than 80 different V segments located contiguously in his or her germline and six different J segments. There are at least 30 D segments in the heavy chain region. 2. Somatic Recombination (VDJ Recombination) As immunoglobulin molecules are formed during B lymphocyte maturation, a specific combination of single V and J segments is selected for the light chain, and another combination of V, D, and J segments is selected for the heavy chain. This is accomplished by deleting the DNA sequences separating the single V, J, and D segments before they are transcribed into mRNA (Fig. 9-6). The deletion process is carried out in part by recombinases (encoded by the RAG1 and RAG2 genes), which initiate double-strand DNA breaks at specific DNA sequences that flank the V and D gene segments. After the deletion of all but one V, D, and J segment, the nondeleted segments are joined by ligases. This cutting-and-pasting process is known as somatic recombination (in contrast to the germline recombination that takes place during meiosis). Somatic recombination produces a distinctive result: unlike most other cells of the body, whose DNA sequences are identical to one another, mature B lymphocytes vary in terms of their rearranged immunoglobulin DNA sequences. Because there are many possible combinations of single V, J, and D segments, somatic recombination can generate approximately 100,000 to 1,000,000 different types of antibody molecules 3. Junctional Diversity As the V, D, and J regions are assembled, slight variations occur in the position at which they are joined, and small numbers of nucleotides may be deleted or inserted at the junctions joining the regions. This creates even more variation in antibody amino acid sequence. 4. Somatic Hypermutation Typically, only a small subset of B cells has cell-surface receptors (immunoglobulins) that can bind to a specific foreign antigen, and their binding affinity is usually low. Once this subset of B cells is stimulated by a foreign antigen, they undergo an affinity maturation process characterized by somatic hypermutation of the V segments of immunoglobulin genes, as mentioned previously. An enzyme termed activation-induced deaminase causes cytosine bases to be replaced by uracil. Error-prone DNA polymerases are recruited, and DNA repair processes are modified so that mutations can persist in the DNA sequence. Consequently, the mutation rate of these gene segments is approximately 10−3 per base pair per generation (recall that the mutation rate in the human genome is normally only 10−8 per base pair per generation). This causes much additional variation in immunoglobulin-encoding DNA sequences and thus in the antigen-binding properties of the encoded immunoglobulins. Because mutation is a random process, most of the new receptors have poor binding affinity and are thus not selected. Eventually, however, somatic hypermutation produces a subset of immunoglobulins that have high-affinity binding to the foreign antigen, and the B cells that harbor these immunoglobulins are selected to proliferate extensively. The end result is a population of mature plasma cells that secrete antibodies that are highly specific to the invading pathogen. 5. Multiple Combinations of Heavy and Light Chains Further diversity is created by the random combination of different heavy and light chains in assembling the immunoglobulin molecule. Each of these mechanisms contributes to antibody diversity. Considering all of them together, it has been estimated that as many as 1011 distinct antibodies can potentially be produced. Mechanisms that produce antibody diversity include multiple germline immunoglobulin gene segments, somatic recombination of the immunoglobulin gene segments, junctional diversity, somatic hypermutation, and the potential for multiple combinations of heavy and light chains.
What does Hemicholinum-3 block and thus prevent?
Blocks High affinity transporter for Choline(Na+/choline cotransporter) Prevents uptake of choline required for Ach Synthesis
What does a stroke refer to? What can it result from>
Brain damage caused by a sudden and sustained loss of blood flow to the brain can result from arterial obstruction (ischemic stroke accounts for upto 80% of stroke cases) Is the 4th leading cause of mortality in the US- appx 140,000 deaths per year One's risk of having a stroke increases by two- to threefold if a parent has had a stroke.
What is the secondly most diagnosed cancer (after skin cancer) in women?
Breast cancer affects appx 12% of american women who live to age 85 Invasive breast cancer was diagnosed in approximately 230,000 American women in 2014
What is the most common form of respiratory allergy in which inhaled allergens stimulate bronchial mast cells to release mediators including leukotrienes?
Bronchial asthma
How do antibodies against tissue antigens and immune complexes cause inflammation? What pathway do the antibodies activate?
By attracting and activating leukocytes Classical complement pathway When leukocytes are activated at sites of antibody deposition, these cells release reactive oxygen species and lysosomal enzymes that damage the adjacent tissues
How do Antigen-antibody mediated complexes (Type III HS) cause tissue damage?
By eliciting inflammation at the sites of deposition
A 45-year-old female went to her primary care physician with complaints of progressing back, ankle, and finger joint pain for the past year. After a physical exam and receiving lab results, the doctor diagnosis her with Rheumatoid Arthritis; and placed her on methotrexate. What is the correct type of hypersensitivity reaction and its effector immune cell for Rheumatoid Arthritis? A - Type 1, IgE B - Type 2, IgM C - Type 4, T-Cell D - Type 1, Mast Cell E - Type 4, B-Cell
C - Type 4, T-Cell
A 26 year old female presents to her general practitioner with extreme fatigue. She has noticed muscle weakness in her arms and legs that seems to improve with rest. She has also noticed her eyelids starting to droop and has had difficulty swallowing when eating a meal. Lab evaluation confirm high levels of antibodies associated with Myasthenia Gravis. Which of the following is the target for these antibodies? A) TSH receptors B) Insulin receptors C) Acetylcholine receptors D) Neutrophil granule proteins E) Streptococcal cell wall antigen
C) Acetylcholine receptors
A 65-year-old female is in the ER in critical condition. She was in a serious accident and is currently undergoing an operation to receive a tissue transplant. Her husband is sitting in the waiting room waiting to hear the results. After the operation, a doctor came out and told the husband that unfortunately his wife rejected the transplant. Which of the following is a possible reason that the patient rejected the transplant? A. the recipient has type A blood and the donor had type O blood B. the class II alleles in the donor and recipient differed C. foreign MHC molecules on donor cells stimulated cytotoxic T cells to attack the cells D. the recipient's B cells identified the donor's foreign MHC molecules and attacked the cells E. the recipient's helper T cells did not secrete cytokines and were unable to stimulate B cells to cause antibody formation
C. foreign MHC molecules on donor cells stimulated cytotoxic T cells to attack the cells
Jennifer, a third-year internal medicine resident, is quizzing her sister, a first year medical student, on pharmacological treatments for dyslipidemia. Jennifer starts off by describing a drug that can lower elevated cholesterol levels by interfering with the transport of cholesterol at the intestinal brush border. Which dyslipidemia drug is she referring to? A.) Gemfibrozil B.) Nicotinic acid C.) Ezetimibe D.) Omega-3 fatty acids E.) Lovastatin
C.) Ezetimibe
What chemotactic agent directs the migration of polymorphonuclear leukocytes?
C5a anaphylatoxins (C3a and C5a)
CNS effects
CNS unctions o ACh include modulation o sleep, wake ulness, learning, and memory; suppression o pain at the spinal cord level; and essential roles in neural plasticity, early neural development, immunosuppression, and epilepsy. Both nicotinic and muscarinic receptors are expressed in central neurons. Nicotinic receptors are primarily involved as presynaptic heteroreceptors that modulate the release o other neurotransmitters, such as glutamate, whereas muscarinic presynaptic receptors are primarily autoreceptors that modulate the release o ACh. While the past two decades have improved understanding o the diversity o subunits and molecular properties o neuronal nicotinic receptors, important questions remain about the anatomical distributions and unctional roles o di erent neuronal receptor subtypes in the CNS and o their changes in disease states and during nicotine abuse (as occurs with smoking). As part o the ascending reticular activating system , cholinergic neurons play an important role in arousal and attention (see Fig. 9-8). Levels o ACh throughout the brain increase during wake ulness and REM sleep and decrease during inattentive states and non-REM/slow-wave sleep (SWS). During an awake or aroused state, cholinergic projections rom the pedunculopontine nucleus, the lateral tegmental nucleus, and the nucleus basalis o Meynert (NBM) are all active. Because the NBM projects di usely throughout the cortex and hippocampus (see Fig. 9-8), activation o the NBM causes a global increase in ACh levels. Acetylcholine markedly potentiates the excitatory e ects o other inputs to its cortical target cells without a ecting the baseline activity o these neurons, an e ect that likely derives rom its modulation o excitatory neurotransmitter release. This primed state is thought to improve the ability o such neurons to process incoming inputs. For the brain as a whole, the result is a heightened state o responsiveness. The cholinergic link to memory processes is supported by evidence rom diverse experimental models. Whereas elevated ACh levels during wake ulness appear to benef t memory encoding processes, consolidation o hippocampusmediated, episodic, explicit memories benef t rom SWS, when ACh levels are at their minimum. By artif cially keeping ACh levels elevated during SWS (e.g., by administration o an AChE inhibitor), consolidation o newly acquired explicit learning and episodic memories can be disrupted. Current understanding o the interplay among ACh, sleep, and memory is as ollows. During awake states, ACh prevents inter erence with initial learning in the hippocampus by suppressing retrieval o previously stored memories (to prevent them rom inter ering with new encoding), but release o this suppression is necessary to allow consolidation o new memories. During sleep (in particular, during SWS), lower ACh levels are required or proper consolidation o newly acquired memories because stronger excitatory eedback transmission is needed to reactivate memories or consolidation within neocortical brain areas. There ore, it may be use ul to remember to sleep, as sleep is needed to remember, or at least to remember better. The clinical importance o ACh or cognitive unction is illustrated by the pathophysiology and treatment o Alzheimer's disease (AD) and other neurodegenerative dementias, including di use Lewy body dementia (DLB) and Parkinson's disease with dementia (PDD). Neurodegenerative dementias and brain injury produce central cholinergic dys unction. Patients with these conditions mani est cognitive, unctional, and behavioral def cits that are at least partially related to cholinergic def cits and amenable to symptomatic treatment with procholinergic medications. An example is the symptomatic treatment o AD with acetylcholinesterase inhibitors. Acetylcholine also plays a role in pain modulation through inhibition o spinal nociceptive transmission. Cholinergic neurons located in the rostral ventromedial medulla extend processes to the superf cial lamina o the dorsal horn at all levels o the spinal cord, where secondary neurons in a erent sensory pathways are located. ACh released by the cholinergic neurons is believed to bind to muscarinic ACh receptors located on secondary sensory neurons specif c or pain transmission, resulting in suppression o action potential f ring in these cells and consequently in analgesia (see Chapter 18). Clinically, the analgesic properties o ACh can be demonstrated by injecting AChE inhibitors into the spinal fluid. Acetylcholine has CNS e ects unrelated to its role as a neurotransmitter. ACh has been observed to inhibit neurite growth. During the early phases o neural development, when such growth is essential, AChE levels are increased. The presence o ACh in chick limb buds and myotomes suggests other, morphogenetic roles or this compound. Lesioning o rat cholinergic neurons during development results in cortical abnormalities, including aberrant growth and positioning o pyramidal cell dendrites, altered cortical connectivity, and gross cognitive de ects. These abnormal f ndings are also observed in etal alcohol syndrome and Rett syndrome, both o which demonstrate dramatically reduced numbers o cholinergic neurons in the brain. There is also some evidence or an immunomodulatory role or ACh, as many cells o the immune system both release ACh and express ACh receptors. Finally, mutations have been identif ed in nicotinic ACh receptor genes that are responsible or autosomal dominant nocturnal rontal lobe epilepsy (ADNFLE); this milestone in epilepsy research is the f rst demonstration that alterations in a ligand-gated ion channel can cause epilepsy.
There are two mechanisms for removal of LDL from the plasma. The major pathway is removal via binding to LDL receptors. The minor pathway is through a receptor for oxidized LDL. Which cells are mainly involved in the removal of chemically altered LDL? A: Neutrophils and monocytes B: Macrophages and Neutrophils C: Monocytes and Macrophages D: Monocytes and Eosinophils E: CD8+ and Macrophages
C: Monocytes and Macrophages
What occurs in T cell (CD8) mediated reactions?
CD8 CTLs kill antigen expressing target cells
In virus infected cells what are viral peptides displayed by MHC I molecules recognized by?
CD8 T lymphocytes
CI and Side Effects: mivacurium
CI: Hypersensitivity to drug in neonates; SIDE EFFECTS: anaphylaxis, hypertension, respiratory failure, salivation, flushing
What is Ipratropium (synthetic quaternary ammonium compound) used in the treatment of?
COPD (Chronic obstructive pulmonary disease) more better at treating COPD due to the fact that the major reversible bronchoconstrictive component in COPD is mediated by cholinergic neural tone less effective in treating asthma as compared to Beta adrenergic agonists
Examples of Primary immunodeficiency diseases
CONDITION MODE OF INHERITANCE BRIEF DESCRIPTION X-linked agammaglobulinemia XR Absence of B cells leads to recurrent bacterial infections SCID (γ chain cytokine receptor defect or ADA deficiency) XR, AR T-cell deficiency leading also to impairment of humoral immune response; fatal unless treated by bone marrow transplantation or gene therapy SCID due to Jak3 deficiency AR Protein kinase deficiency leading to T-cell deficiency, NK cell deficiency, and impaired humoral immune response. SCID (also Omenn syndrome) due to RAG1 or RAG2 deficiency AR Lack of recombinase activity impairs VDJ recombination, which leads to B-cell and T-cell deficiency SCID due to interleukin-7 α chain deficiency AR T-cell deficiency leading to impaired B-cell response Zap70 kinase deficiency AR Lack of cytotoxic T cells; defective helper T cells; impaired antibody response Purine nucleoside phosphorylase deficiency AR Purine metabolism disorder leading to T-cell deficiency Bare lymphocyte syndrome (BLS) AR Deficient MHC class I expression (TAP2 mutation) leads to T-cell and B-cell deficiency in type 1 BLS; mutations in transcription factors for MHC class II genes lead to a relative lack of helper T cells in type 2 BLS Complement system defects Mostly AR Increased susceptibility to bacterial and other infections DiGeorge anomaly AD, sporadic Congenital malformations include abnormal facial features, congenital heart disease, and thymus abnormality leading to T-cell deficiency Ataxia telangiectasia AR DNA repair defect characterized by unsteady gait (ataxia), telangiectasia (dilated capillaries), and thymus abnormality producing T-cell deficiency Wiskott-Aldrich syndrome XR Abnormal, small platelets, eczema, and abnormal T cells causing susceptibility to opportunistic infections Chediak-Higashi syndrome AR Partial albinism, defective lysosomal assembly, giant cytoplasmic granules, abnormal natural killer cells, and neutrophils leading to recurrent bacterial infections Leukocyte adhesion deficiency AR Mutations in integrin receptor genes produce phagocytes that cannot recognize and ingest microorganisms, which results in severe bacterial infections Chronic granulomatous disease XR, AR Phagocytes ingest microbes but cannot kill them; leads to formation of granulomas and recurrent infections Hyper IgE syndrome AD, AR Recurrent staphylococcal infections, markedly elevated serum IgE levels, progressively coarse facial features IRAK-4 deficiency Toll-like receptor defect caused by deficiency of interleukin-1 receptor associated kinase-4 (IRAK-4), resulting in extracellular bacterial (especially Streptococcus pneumoniae) infections
Alleles of what gene is strongly associated with Type I diabetes?
CTLA4 (cyotoxic lymphocyte associated-4 gene) encodes an inhibitory T-cell receptor associated with other autoimmune diseases, such as rheumatoid arthritis and celiac disease. Another gene associated with type 1 diabetes susceptibility,
What are some of the causes of stroke?
Can be caused by more than a dozen single gene disorders: 1- Sickle cell disease 2- MELAS (mitochondrial myopathy, encephelopathy, lacitc acidosis, and stroke) 3- Cerebral autosomal dominant Arteriopathy with subcortical infarcts and leukoencephelopathy (CADASIL)- characterized by recurrent strokes and dementia caused by mutations in NOTCH3 gene
What is the second leading cause of death in the United States?
Cancer It is well established that many major types of cancer (e.g., breast, colon, prostate, ovarian) cluster strongly in families. This is due both to shared genes and shared environmental factors environmental factors such as Tobacco use accounts for 1/3 of all cancer - most known cause of cancer Diet (i.e., carcinogenic substances and the lack of "anticancer" components such as fiber, fruits, and vegetables) is another leading cause of cancer and may also account for as much as one third of cancer cases
What is LEMS associated with in upto 60% of cases?
Cancer, usually small cell lung carcinoma
How does ADCC cause destruction of cells?
Cells that are coated with IgG antibody are killed by a variety of effector cells, mainly NK cells and macrophages, which bind to the target by their receptors for the Fc fragment of IgG and cell lysis proceeds without phagocytosis
What should patients with LEMS be screened and monitored with especially if they are smokers and over the age of 50?
Chest CT
What test should be performed to rule out Thymoma in Myasthenia gravis?
Chest CT
What do micelles solubilize in bile for transport from the liver to the small intestine?
Cholesterol in this way, micelles serve as a functional counterpart to HDL particles in plasma
Genetically Determined Adverse Reactions to Drug
Certain genetically determined enzyme deficiencies are unmasked only after exposure of the affected individual to certain drugs. This special area of genetics, called pharmacogenetics, is of considerable clinical importance. The classic example of drug-induced injury in the genetically susceptible individual is associated with a deficiency of the enzyme G6PD. Under normal conditions glucose-6 phosphate-dehydrogenase (G6PD) deficiency does not result in disease, but on administration, for example, of the antimalarial drug primaquine, a severe hemolytic anemia results (Chapter 14). In recent years an increasing number of polymorphisms of genes encoding drug-metabolizing enzymes, transporters, and receptors have been identified. In some cases these genetic factors have major impact on drug sensitivity and adverse reactions. It is hoped that advances in pharmacogenetics will lead to patient-tailored therapy, an example of "personalized medicine." With this overview of the biochemical basis of singlegene disorders, we now consider selected examples grouped according to the underlying defect.
What is QT (LQT) syndrome characterized by? What is it indicative of?
Characteristically elongated QT interval in the electrocardiogram of affected individuals Delayed cardiac repolarization
What is Bethanechol resistant to? What does this resistance result in?
Cholinesterases bc the carbamoyl group is substituted for the acetyl ester group of Ach This resistance to AChE extends their duration of action and allows time or distribution of the intact drug to areas of lower blood flow
What are babies born with Trisomy 13 often suffer from?
Cleft lip/palate it is quite common for congential malformations to be associated with other disorders
What is the diagnosis of Familial hypercholesterolemia usually made on the basis of? What factors identifies patients at risk for familial hypercholesterolemia?
Clinical features: 1- Elevated total cholesterol - (>300 mg/dL) 2- Elevated total LDL cholesteorl (? 250 mg/dL) A person with an individual or premature Congestive heart disease and tendon xanthomas
What are the clinical manifestations of Apolipoprotein C-II deficiency?
Clinical manifestations are similar to those of LPL deficiency including: hypertriglyceridemia (>1000 mg/dL) and symptoms of pancreatitis, eruptive xanthomas, lipemia retinalis, and hepatosplenomegaly
What are Ehlers-Danlos syndromes?
Clinically and genetically heterogeneous group of disorders that result from some defect in the synthesis or structure of fibrillar collagen
What is Botulinum Toxin A produced by? What does it degrade and thus prevent?
Clostridium botulinum SNAP-25 Synaptic vesicle fusion with the axon terminal (presynaptic membrane)- thus prevents the release of Ach
What identifies disorder of VLDL metabolism and thus Familial hypertriglyceridemia ?
Cloudy infranatant after overnight refrigeration of plasma
A 45 year old woman presents with painful joints and rash that have both been increasing in severity over the past several months. Due to the pandemic, the physician's assessment is being done remotely. The physician suggests testing for lupus erythematosus. It is likely that this disease is caused by: A) A viral illness B) Acute autoimmune response C) Foodborne illness D) Chronic autoimmune response E) Malignancy
D) Chronic autoimmune response
Tissues rich in what are frequently involved in most variants of Ehlers danlos syndromes?
Collagen eg: skin, ligaments and joints Because the abnormal collagen fibers lack adequate tensile strength, skin is hyperextensible, and the joints are hypermobile
What is the Th1 (subset of helper t cell) primarily involved in? What cytokines does it secrete?
Combating intracellular pathogens IL-2, INF-gamma, and TNF- beta (TNF= tumor necrosis factor)
What is Familial hypertriglyceridemia ?
Common autosomal dominant disorder characterized by hypertriglyceridemia with normal LDL-cholesterol concentrations HDL cholesterol is often reduced Although the underlying de ect in this disorder is unknown, it is hypothesized to be a de ect in bile acid metabolism, leading to increased hepatic production o triglyceride-rich VLDL A strong family history of premature coronary heart disease is usually absent If that approach is unsuccessful at reducing triglyceride concentrations below 500 mg/dL, a fibrate should be considered Drug therapy should be initiated if triglycerides exceed 1,000 mg/dL.
What do adoption studies consist of?
Comparing disease rates among the adopted offspring of affected parents with the rates among adopted offspring of unaffected parents
What do patients with mixed hyperlipidemia exhibit ?
Complex lipid profiles that may consist: 1- Elevated total cholesterol 2- Elevated LDL cholesterol 3- Elevated triglyceride concentrations 4- HDL is often reduced
What type of condition is often seen with trisomy 13,18 and 21?
Congenital heart defects
What does Polygenic hypercholesterolemia lead to an increased propensity for? What treatment is recommended?
Congestive heart disease LDL lowering agents is recommended based on the risk factors (table 69.7- table in picture)
Low HDL concentrations can also be a risk for what?
Congestive heart disease (CHD) In the Framingham Heart Study, every decrease in HDL of 5 mg/dL increased the risk for myocardial infarction Both lifestyle modifications (e.g., diet low in saturated fat, exercise) and pharmacologic therapy (e.g., nicotinic acid, fibrate) can improve HDL levels
Mast cells are present in all ....... tissues especially under epithelia and they are usually adjacent to?
Connective tissues Blood vessels
What occurs in Dilated Cardiomyopathy? What is the end result?
Consists of increased size and impaired contraction of ventricles Impaired pumping of the heart occurs in 1/2500 people is familial in about 1/3 of affected persons, although autosomal dominant mutations are most common, mutations can also be X-linked or mitochondrial
What is an common example of tissue injury resulting from DTH reactions (type IV)? What may it be evoked by? What does it present as?
Contact dermatitis Contact with Urishiol, the antigenic component of Poison ivy or poison oak Vesicular Dermatitis
What is the fundamental defect in the 2 types of EDS- Arthrochalasia type and Dermatosparaxis type ?
Conversion of type I procollagen to collagen This step in collagen synthesis involves cleavage of noncollagen peptides at the N terminus and C terminus of the procollagen molecule. This is accomplished by N-terminal-specific and C-terminal-specific peptidases
What is the most common underlying cause of heart disease? What is it caused by?
Coronary artery disease (CAD) Atherosclerosis- narrowing of the coronary arteries resulting from the formation of lipid-laden lesions this narrowing impedes blood flow to the heart and can eventually result in a myocardial infarction (death of heart tissue caused by inadequate supply of oxygen)
A patient received his second round of vaccination for an unspecific infectious disease that contains antigen from the infectious microbe. After injection, the patient experiences painful, localized swelling at the injection site. Which of the following would most likely explain the patient's presentation after treatment? A. Type 1 hypersensitivity to the vaccine B. Autoimmune reaction to the vaccine C. Serum sickness D. Arthus reaction E. Systemic lupus erythematous
D. Arthus reaction
In disorders with inflammation such as asthma what is used to inhibit inflammation?
Corticosteroids
inhibitors of VLDL secretion
Currently approved inhibitors o VLDL secretion act by two di erent mechanisms (Fig. 20-3). Lomitapide is a small molecule that inhibits lipid trans er by binding to MTP, while mipomersen is a synthetic single-strand antisense oligonucleotide that binds to the apoB100 mRNA and thereby reduces apoB protein levels. The net e ect o each drug is to reduce VLDL secretion Lomitapide is approved or use in patients with homozygous amilial hypercholesterolemia (HoFH). At therapeutic doses, lomitapide reduces LDL-C by 30-50% in these patients. Adverse e ects include gastrointestinal distress due to at malabsorption, reductions in plasma vitamin E levels, and transaminase elevations that are correlated with increased hepatic at content. Transaminase elevations have not been associated with parallel increases in plasma bilirubin concentrations and generally normalize with continued lomitapide treatment. Mipomersen is indicated or the treatment o patients with homozygous FH who are already prescribed maximal medical therapy. Adverse e ects may include injection site reactions, u-like symptoms, increases in C-reactive protein, and increased transaminases. Similar to lomitapide, the liver test abnormalities likely correspond to an increase in liver at content. This increased at content appears to remain stable over 1 year of treatment and is reversible with cessation of therapy.
What are the major side effects associated with Niacin?
Cutaneous flushing and Pruritis (itching) The flushing is mediated by the G protein-coupled niacin receptor and involves the release of prostaglandins D2 and E2 within the skin, can be mitigated by the use of NSAIDs eg: Aspirin - important adverse effects of niacin include hyperuricemia, impaired insulin sensitivity, hepatotoxicity, and the potentiation of statin-induced myopathy. Hyperuricemia may precipitate gout. Impaired insulin sensitivity may precipitate diabetes in patients at risk, and niacin should be used with caution in diabetic patients. Rarely, niacin may cause myopathy.
What may CD4 T cells secrete after reacting against cell or tissue antigens? What do they induce?
Cytokines Local inflammation and activation of macrophages
Upon repeat exposure to an antigen what do Th1 cells secrete? What are they responsible for?
Cytokines - mainly IFN range responsible for many of the manifestations of delayed-type hypersensitivity
What is MHC I presentation essential for?
Cytotoxic T cell response (CD8)
A 25-year-old woman presents to a new primary care physician for a yearly physical examination. While taking the patient's family history, the physician notices that the patient's mother was diagnosed with breast cancer at age 40, and the patient's grandmother on her mother's side of the family died from breast cancer at age 55. Based on this history, the physician is concerned and recommends genetic testing. What is/are the most important gene(s) known to predispose women to developing hereditary breast cancer? A) MSH2 and MLH1 B) APC C) SMAD4 D) BRCA1 and BRCA2 E) STK11
D) BRCA1 and BRCA2
Recurrence risks for First, second and third degree relatives of probands
DISEASE PREVALENCE IN GENERAL POPULATION DEGREE OF RELATION FIRST DEGREE SECOND DEGREE THIRD DEGREE Cleft lip/palate 0.001 0.04 0.007 0.003 Club foot 0.001 0.025 0.005 0.002 Congenital hip dislocation 0.002 0.005 0.006 0.004
Drugs commonly used for the treatment of dyslipidemia
DRUG CLASS LDL (% CHANGE) HDL (% CHANGE) TRIGLYCERIDES (% CHANGE) SIDE EFFECTS HMG-CoA inhibitors ↓ 20-60 ↑ 5-10 ↓ 10-30 Liver toxicity, myositis, rhabdomyolysis; enhanced warfarin effect Cholesterol absorption inhibitors ↓ 17 No effect ↓ 7-8 Abnormal liver enzymes in combination with an HMG-CoA inhibitor, myalgia, hepatitis, rhabdomyolysis, pancreatitis, potential increase in cancer risk and cancer death Bile acid sequestrants ↓ 15-30 Slight increase No effect Nausea, bloating, cramping, abnormal liver function; interferes with absorption of other drugs such as warfarin and thyroxine Fibric acid ↓ 5-20 ↑ 5-20 ↓ 35-50 Nausea, cramping, myalgias, liver toxicity, enhanced warfarin effect Nicotinic acid ↓ 10-25 ↑ 15-35 ↓ 25-30 Hepatotoxicity, hyperuricemia, hyperglycemia, flushing, pruritus, nausea, vomiting, diarrhea Omega-3 fatty acids ↑ 4-49 ↑ 5-9 ↓ 23-45 Eructation, taste perversion, dyspepsia
What reactions are often used to determine if people have been previously exposed to and have responded to antigen?
Delayed type hypersensitivity (DTH) eg: a DTH reaction to a mycobacterial antigen, PPD (purified protein derivative), is an indicator of a T cell response to the mycobacteria. This is the basis for the PPD skin test, used to detect past or active mycobacterial infection
What is the typical reaction mediated by T cell cytokines?
Delayed-type hypersensitivity so called because it occurs 24 to 48 hours after an individual previously exposed to a protein antigen is challenged with the antigen (i.e., the reaction is delayed).
What is the primary function of apoB containing lipoproteins?
Deliver fatty acids in the form of triglycerides to muscle tissue for use in ATP biogenesis and to adipose tissue for storage
What type of conditions can Botulinum toxin be used to treat? Examples?
Diseases associated with increased muscle tone eg: Torticollis, Achalasia, Strabismus, Bleopharospasm and other focal dystonias Botulinum toxin is also approved or cosmetic treatment of facial lines or wrinkles and is used to treat various headache and pain syndromes (e.g., by intrathecal delivery into the spinal fluid
What confirms the diagnosis in Lipoprotein lipase deficiency?
Documentation of diminished LPL activity
What are fibrates the preferred therapy for patients with?
Dysbetalipoproteinemia Fibrates (e.g., fenofibrate) can be used together with statins in cases of combined hyperlipidemia or when HDL cholesterol is decreased
What do defects in the production or removal of lipoproteins result in?
Dyslipidemia both genetic and acquired conditions have been implicated in the pathogenesis of lipid disorders
What is Dyslipidemia defined by?
Dyslipidemia is defined by a total cholesterol, triglyceride, or LDL level greater than the 90th percentile or an HDL level lower than the 10th percentile for the general population
ehlers danlos syndromes
EDSs comprise a clinically and genetically heterogeneous group of disorders that result from some defect in the synthesis or structure of fibrillar collagen. Other disorders resulting from mutations affecting collagen synthesis include osteogenesis imperfecta (Chapter 26), Alport syndrome (Chapter 20), and epidermolysis bullosa (Chapter 25). Biosynthesis of collagen is a complex process (Chapter 1) that can be disturbed by genetic errors that may affect any one of the numerous structural collagen genes or enzymes necessary for posttranscriptional modifications of collagen. Hence, the mode of inheritance of EDS encompasses all three Mendelian patterns. On the basis of clinical and molecular characteristics, six variants of EDS have been recognized. These are listed in Table 5-5. It is beyond the scope of this book to discuss each variant individually; instead, the important clinical features common to most variants are summarized and clinical manifestations are correlated with the underlying molecular defects in collagen synthesis or structure. As might be expected, tissues rich in collagen, such as skin, ligaments, and joints, are frequently involved in most variants of EDS. Because the abnormal collagen fibers lack adequate tensile strength, skin is hyperextensible, and the joints are hypermobile. These features permit grotesque contortions, such as bending the thumb backward to touch the forearm and bending the knee forward to create almost a right angle. It is believed that most contortionists have one of the EDSs. A predisposition to joint dislocation, however, is one of the prices paid for this virtuosity. The skin is extraordinarily stretchable, extremely fragile, and vulnerable to trauma. Minor injuries produce gaping defects, and surgical repair or intervention is accomplished with great difficulty because of the lack of normal tensile strength. The basic defect in connective tissue may lead to serious internal complications. These include rupture of the colon and large arteries (vascular EDS), ocular fragility with rupture of cornea and retinal detachment (kyphoscoliosis EDS), and diaphragmatic hernia (classic EDS). The biochemical and molecular bases of these abnormalities are known in several forms of EDS. These are described briefly, because they offer some insights into the perplexing clinical heterogeneity of EDS. Perhaps the best characterized is the kyphoscoliosis type, the most common autosomal recessive form of EDS. It results from mutations in the gene encoding lysyl hydroxylase, an enzyme necessary for hydroxylation of lysine residues during collagen synthesis. Affected patients have markedly reduced levels of this enzyme. Because hydroxylysine is essential for the cross-linking of collagen fibers, a deficiency of lysyl hydroxylase results in the synthesis of collagen that lacks normal structural stability. The vascular type of EDS results from abnormalities of type III collagen. This form is genetically heterogeneous, because at least three distinct types of mutations affecting the COL3A1 gene encoding collagen type III can give rise to this variant. Some affect the rate of synthesis of pro-α1 (III) chains, others affect the secretion of type III procollagen, and still others lead to the synthesis of structurally abnormal type III collagen. Some mutant alleles behave as dominant negatives (see discussion under "Autosomal Dominant Disorders") and thus produce severe phenotypic effects. These molecular studies provide a rational basis for the pattern of transmission and clinical features that are characteristic of this variant. First, because vascular-type EDS results from mutations involving a structural protein (rather than an enzyme protein), an autosomal dominant pattern of inheritance would be expected. Second, because blood vessels and intestines are known to be rich in collagen type III, an abnormality of this collagen is consistent with severe structural defects (e.g., vulnerability to spontaneous rupture) in these organs. In two forms of EDS—arthrochalasia type and dermatosparaxis type—the fundamental defect is in the conversion of type I procollagen to collagen. This step in collagen synthesis involves cleavage of noncollagen peptides at the N terminus and C terminus of the procollagen molecule. This is accomplished by N-terminal-specific and C-terminal-specific peptidases. The defect in the conversion of procollagen to collagen in the arthrochalasia type has been traced to mutations that affect one of the two type I collagen genes, COL1A1 and COL1A2. As a result, structurally abnormal pro-α 1 (I) or pro-α2 (I) chains that resist cleavage of N-terminal peptides are formed. In patients with a single mutant allele, only 50% of the type I collagen chains are abnormal, but because these chains interfere with the formation of normal collagen helices, heterozygotes manifest the disease. In contrast, the related dermatosparaxis type is caused by mutations in the procollagen-N-peptidase genes, essential for the cleavage of collagens. Because in this case the disease is caused by an enzyme deficiency, it follows an autosomal recessive form of inheritance. Finally, in classic type of EDS, molecular analysis suggests that genes other than those that encode collagen may also be involved. In 30% to 50% of these cases, mutations in the genes for type V collagen (COL5A1 and COL5A2) have been detected. Surprisingly, in the remaining cases, no other collagen gene abnormalities have been found despite clinical features typical of EDS. It is suspected that in some cases genetic defects that affect the biosynthesis of other extracellular matrix molecules that influence collagen synthesis indirectly may be involved. One example is an EDS-like condition caused by mutation in tenascin-X, a large multimeric protein, that affects the synthesis and fibril formation of type VI and type I collagens To summarize, the common thread in EDS is some abnormality of collagen. These disorders, however, are extremely heterogeneous. At the molecular level, a variety of defects, varying from mutations involving structural genes for collagen to those involving enzymes that are responsible for posttranscriptional modifications of mRNA, have been detected. Such molecular heterogeneity results in the expression of EDS as a clinically variable disorder with several patterns of inheritance
When does Familial hypercholesterolemia present? What cholesterol levels do they present with?
Early in life Elevated total cholesterol levels (600- 1000 mg/dL) and LDL-cholesterol (550 to 950 mg/dL) Triglyceride and HDL-cholesterol levels are normal
What is the key regulatory event that differentiates Chylomicron metabolism from VLDL metabolism?
Editing of apoB mRNA Within enterocytes but not hepatocytes, a protein named apoB editing complex-1 (apobec-1) is expressed. This protein constitutes the catalytic subunit of the apoB editing complex, which deaminates a cytosine at position 6666 of the apoB mRNA molecule.
What is an example of a short acting AChE inhibitor that is ideal for diagnostic purposes?
Edrophonium
What does the magnitude of LDL-cholesterol lowering depends on what?
Efficacy and dose of the statin that is administered 1- Statins reduce LDL-cholesterol concentrations by up to about 60% 2- Statins increase HDL-cholesterol concentrations by an average of 10% 3- Reduce triglyceride concentrations by up to about 40%, depending on statin dose and degree of hypertriglyceridemia
What type of patients is Niacin indicated for?
Elevations in both triglycerides and cholesterol usually in combination with a Statin
For most multifactorial diseases what type of risks have been derived?
Empirical risks ie: risks based on direct observation of data since risk estimation for multifactorial is more complex bc the number of genes contributing to the disease is usually not known
What does the stimulus of eating a meal promote?
Emptying of the gallbladder bile into the small intestine where the micelles and vesicles solubilize the digested lipids
An enzyme defect can lead to a metabolic block and a decreased amount of what?
End product that may be necessary for normal function eg: a deficiency of melanin may result from lack of tyrosinase, which is necessary for the biosynthesis of melanin from its precursor, tyrosine, resulting in the clinical condition called albinism
What beneficial effects do HDL particles directly exert on vascular tissue?
Enhancement of antioxidant enzyme activities that inhibit oxidation of LDL
Since monozygotic twins are genetically identical any differences among them should be only due to?
Environmental effects
Which leukocyte is a prominent component of many allergic reactions? What do they cause in tissues?
Eosinophils Injury
key concepts- familial hypercholesterolemia
Familial Hypercholesterolemia ■ Familial hypercholesterolemia is an autosomal dominant disorder caused by mutations in the gene encoding the LDL receptor. ■ Patients develop hypercholesterolemia as a consequence of impaired transport of LDL into the cells.■ In heterozygotes, elevated serum cholesterol greatly increases the risk of atherosclerosis and resultant coronary artery disease; homozygotes have an even greater increase in serum cholesterol and a higher frequency of ischemic heart disease. Cholesterol also deposits along tendon sheaths to produce xanthomas.
fibrates
Fibrates bind to and activate peroxisome proli eratoractivated receptor (PPAR ), a nuclear receptor expressed in hepatocytes, skeletal muscle, macrophages, and the heart. Upon binding o f brate, PPAR heterodimerizes with the retinoid X receptor (RXR). This heterodimer binds to peroxisome proli erator response elements (PPREs) in the promoter regions o specif c genes, activating transcription o these genes and thereby increasing protein expression. Activation o PPAR by f brates results in numerous changes in lipid metabolism that act collectively to decrease plasma triglyceride levels and increase plasma HDL (Fig. 20-12). The decrease in plasma triglyceride levels is caused in part by increased muscle expression o lipoprotein lipase, decreased hepatic expression o apolipoprotein CIII, and increased hepatic oxidation o atty acids. The increased muscle expression o LPL results in increased uptake o triglyceride-rich lipoproteins, with a resultant decrease in plasma triglyceride levels. Because apoCIII normally unctions to inhibit interaction o triglyceride-rich lipoproteins with their receptors, the decrease in hepatic production o apoCIII may potentiate the increased LPL activity. The mechanisms by which f brate-mediated PPAR activation raises plasma HDL depend at least in part on increased hepatic production o apolipoprotein AI. This would be expected to contribute directly to increased plasma HDL. Upregulation o ABCA1 in macrophages presumably promotes cholesterol e ux rom these cells in vivo. Hepatocytes also increase expression o SR-B1 in response to PPAR activation, providing a pathway or increased reverse cholesterol transport, with subsequent cholesterol excretion into bile. Fibrates also lower LDL levels modestly. The lower LDL levels result rom a PPAR -induced shi t in hepatocyte metabolism toward atty acid oxidation. PPAR increases the expression o numerous enzymes involved in atty acid transport and oxidation, thereby increasing atty acid catabolism and decreasing triglyceride synthesis and VLDL production. PPAR activation also results in LDL particles o larger size, which appear to be taken up more e f ciently by LDL receptors. Many o the e ects o PPAR on lipid metabolism remain the subject o basic and clinical investigation, which may lead to the development o more selective PPAR agonists that are capable o targeting selective aspects o lipid metabolism. Finally, f brates have a benef cial anti-in ammatory e ect, decreasing the vulnerability o atherosclerotic plaques to rupture. Gemf brozil and enof brate are the available f brates in the United States. Two other f brates, bezaf brate and ciprof brate , are available in Europe. Fibrates are indicated or treatment o hypertriglyceridemia as well as hypertriglyceridemia with low HDL. They lower triglycerides by up to 50%, raise HDL by up to 20%, and lower LDL by up to 15%. In addition, f brates are the pre erred therapy or patients with dysbetalipoproteinemia. Because o their greater e f cacy, statins are pre erred over f brates or treatment o increased LDL levels. However, f brates (e.g., enof brate) can be used together with statins in cases o combined hyperlipidemia or when HDL cholesterol is decreased. Gastrointestinal discom ort is the most common adverse e ect o f brates. Rare adverse e ects include myopathy and arrhythmias. Increases in liver transaminases occur in about 5% o patients. Gastrointestinal disturbances and myopathy are less common with enof brate than with gemf brozil. Fibrates displace war arin rom albumin binding sites, resulting in increased ree war arin concentrations. There ore, the response to war arin should be monitored when a f brate is co-administered. Gemf brozil-associated gallstone ormation is presumably a consequence o f brate-induced increases in biliary cholesterol excretion. However, screening or gallstones is not recommended. The e ect o a co-administered statin on fibrate metabolism is described above
Multiple germline immunoglobin genes
For each heavy and light chain, an individual has more than 80 different V segments located contiguously in his or her germline and six different J segments. There are at least 30 D segments in the heavy chain region.
How is ACh synthesized?
From Choline and Acetyl CoA via the enzyme Choline acetyltransferase
What does Bare lymphocyte syndrome result in the severe reduction of?
Functional B cells and T cells
Variants in what genes are associated with addiction to alcohol?
GABA (gamma-aminobutyric acid) receptors alcohol is shown to increase GABA(inhibitory Neutotransmitter- thus brings calming effect) release and allelic variation in GABA receptor genes might modulate this affect
Chromosome location and Function of Major immune response genes
GENE SYSTEM CHROMOSOME LOCATION GENE PRODUCT FUNCTION Immunoglobulin heavy chain (C, V, D, and J genes) 14q32 Heavy chain, the first part of antibody molecule, which binds foreign antigens Immunoglobulin κ light chain (C, V, and J genes) 2p13 Light chain, the second part of antibody molecule Immunoglobulin λ light chain (C, V, and J genes) 22q11 Light chain, the second part of antibody molecule (either κ or λ may be used) T-cell receptor α 14q11 One chain of the α-β T-cell receptor, which recognizes antigen with MHC molecule T-cell receptor β 7q35 The second chain of the α-β T-cell receptor T-cell receptor γ 7p15 One chain of the γ-δ T-cell receptor T-cell receptor δ 14q11 The second chain of the γ-δ T-cell receptor MHC (classes I, II, and III); includes TAP1 and TAP2 6p21 Cell-surface molecules that present peptides to T-cell receptors. TAP1 and TAP2 are transporter molecules that process foreign peptides and carry them to the endoplasmic reticulum. β2-microglobulin 15q21-22 Forms second chain of the class I MHC molecule RAG1, RAG2 11p13 Recombinases that participate in VDJ somatic recombination
What are common side effects that limit the use of bile acid sequestrants?
GI Symptoms eg: Nausea, bloating, cramping They can also interfere with the absorption of other drugs such as warfarin and thyroxine
What is Gemfibrozil associated with the formation of? what does it form as a consequence of?
Gallstones Fibrate induced increases in Biliary cholesterol excretion
What is the most common adverse effect associated with Fibrates?
Gastrointestinal discomfort
What are examples of Fibric acid derivatives? What do they increase?
Gemfibrozil and Fenofibrate Free fatty acid (FFA) oxidation in muscle and liver The reduced lipogenesis in the liver decreases VLDL and subsequent LDL production.
What do most mutations lead to the reduced production of?
Gene products or give rise to a dysfunctional or inactive protein
What chromosome regions/genes are associated with bipolar disorder? What drugs are used to treat it?
Genes that encode voltage gated calcium channels Ion-channel modulating drugs are frequently used as mood stabilizers
What does definitive diagnosis of Familial dysbetalipoproteinemia require?
Genetic testing to identify apo E2 homozygosity or mutation.
Antimuscarinic medications are contraindicated in patients with what?
Glaucoma
What is the inflammatory reaction associated with Th17 trigger the release of?
Greater neutrophil component
HDL formation
HDL ormation occurs mainly in the liver, although a small percentage is contributed by the small intestine. The earliest events occur when lipid-poor apoAI is secreted by the liver or intestine or dissociates rom lipoprotein particles in the plasma. These amphipathic apoAI molecules interact with ABCA1 , which is localized in the sinusoidal membrane o the hepatocyte or the basolateral membrane o the enterocyte. ABCA1 incorporates a small amount o membrane phospholipid and unesterif ed cholesterol into the apoAI molecule. The resulting small, disk-shaped particle, which consists mainly o phospholipid and apolipoprotein AI, is re erred to as nascent or pre- - HDL , due to its characteristic migration on agarose gels
What is Apo A-I a surface lipoprotein on? What does it metabolize?
HDL particles mobilizes cholesterol from intracellular pools and accepts cholesterol released during lipolysis of triglyceride-rich lipoproteins
What loci accounts for about 40-50% of the genetic susceptibility to Type I diabetes?
HLA Loci Approximately 95% of whites with type 1 diabetes have the HLA DR3 and/or DR4 alleles, whereas only about 50% of the general white population has either of these alleles
Secondary causes of hyperlipidemia
HYPERTRIGLYCERIDEMIA HYPERCHOLESTEROLEMIA Diabetes mellitus Hypothyroidism Chronic renal failure Nephrotic syndrome Hypothyroidism Anorexia nervosa Glycogen storage disease Acute intermittent porphyria Stress Cholestasis Sepsis Obstructive liver disease Alcohol excess Corticosteroid treatment Lipodystrophy Protease inhibitor therapy Pregnancy Oral estrogen replacement therapy Antihypertensive drugs: betablockers, diuretics Glucocorticoid treatment Protease inhibitor therapy Acute hepatitis Systemic lupus erythematosus
the genetics of common diseases
Having discussed the principles of multifactorial inheritance, we turn next to a discussion of the common multifactorial disorders themselves. Some of these disorders, the congenital malformations, are by definition present at birth. Others, including heart disease, cancer, diabetes, and most psychiatric disorders, are seen primarily in adolescents and adults. Because of their complexity, unraveling the genetics of these disorders is a daunting task. Nonetheless, significant progress is now being made
What disease is the leading cause of death worldwide?
Heart disease it accounts for appx 25% of all deaths in the United States
What type of core do Lipoproteins have? what type of surface?
Hydrophobic core (cholesteryl esters and triglycerides) Ampiphilic surface monolayer (phospholipids, unesterified cholesterol and apolipoproteins) ultracentrigution separates lipoproteins into 5 classes based on their density
Polygenic hypercholesterolemia
Hypercholesterolemia in a population is mostly due to small influences of many different genes. The exact nature of these genetic defects is poorly defined, but apo E may play a role in the pathogenesis. Apo E4 on chylomicrons and VLDL remnants has a high affinity for the LDL receptor. Elevated binding of apo E4-containing lipoproteins to LDL receptors may downregulate LDL receptor synthesis and increase circulating LDL levels. Environmental factors such as diet can influence production of chylomicrons and VLDL, resulting in downregulation of the LDL receptor in conditions with high apo E4. This leads to an increased propensity for CHD, and treatment with LDLlowering agents is recommended based on risk factors (see Table 69-7).
What cytokine do CD8 T cells produce? what inflammatory reactions are they involved in?
INF-gamma inflammatory reactions involving DTH(delayed type hypersensitivity), especially following virus infections and exposure to some contact sensitizing agents
What test can be used to differentiate ptosis caused by Myasthenia gravis vs other forms of ptosis?
Ice pack test improvement of 2mm or more in ptosis supports MG
What is Variable expressivity?
If a trait is seen in all individuals carrying the mutant gene but is expressed differently among individuals eg: manifestations of neurofibromatosis type 1 range from brownish spots on the skin to multiple skin tumors and skeletal deformities
Pharmacology
If diet and exercise modifications do not sustain a normal lipid profile, then drug therapy is appropriate (see Table 69-7). Likely benefit needs to be balanced against potential adverse effects when determining drug therapy. Many patients require two or three agents to achieve adequate control. HMG-CoA reductase is the rate-limiting enzyme involved in cholesterol biosynthesis. Inhibition of this enzyme decreases intracellular cholesterol pools and subsequently increases uptake of LDL cholesterol from the circulation. HMG-CoA reductase inhibitors (e.g., lovastatin, pravastatin, simvastatin, fluvastatin, atorvastatin, and rosuvastatin) increase cholesterol utilization, decrease VLDL synthesis, and increase HDL synthesis. As a result, lower LDL and triglyceride levels and higher HDL levels are observed with treatment. Meta-analysis of primary and secondary CHD prevention trials found reductions in all-cause and cardiovascular mortality rates with statin therapy. These agents limit progression and may even cause regression of coronary atherosclerosis. Therefore, they represent first-line therapy in the management of abnormal LDL-cholesterol levels. Elevated liver enzymes and muscle toxicity are potential dose-related complications. Myositis can occur with statins alone, but the risk is higher when statins are used in combination with nicotinic acid or fibric acid derivatives. Some of these agents can also potentiate the effect of warfarin. Cholesterol absorption inhibitors (e.g., ezetimibe) function by interfering with the transport of cholesterol at the intestinal brush border. They increase cholesterol utilization and decrease LDL-cholesterol levels. Despite reductions in LDL-cholesterol, improvements in cardiovascular events and mortality have not been reported with treatment. Ezetimibe may be used as a single agent or in combination with an HMG-CoA reductase inhibitor to lower LDL-cholesterol levels. In combination, this agent can increase serum transaminase levels and potentially increase the risk of cancer and cancer death Drugs that interfere with the absorption of cholesterol from the intestinal lumen increase cholesterol utilization and decrease circulating levels of cholesterol. Bile acid sequestrants (e.g., cholestyramine, colestipol, and colesevelam) bind bile acids in the intestinal lumen and increase fecal excretion. Subsequently, more LDL-cholesterol is used by the liver to synthesis bile acids. The decrease in cellular cholesterol pools upregulates LDL receptors and decreases the amount of LDL-cholesterol in the circulation. Mild increases in HDL-cholesterol are also seen with this agent as a result of increased intestinal HDL formation. Treatment is associated with a reduction in the incidence of CHD. Bile acid sequestrants may be used alone for mild lipid dysfunction or in combination with another lipid-loweing agent such as an HMG-CoA reductase inhibitor or. Abnormal liver function and gastrointestinal symptoms (e.g., nausea, bloating, cramping) are common side effects that limit the use of bile acid sequestrants. They can also interfere with the absorption of other drugs such as warfarin and thyroxine. Fibric acid derivatives such as gemfibrozil and fenofibrate increase FFA oxidation in muscle and liver. The reduced lipogenesis in the liver decreases VLDL and subsequent LDL production. Fibric acid derivatives also enhance LPL activity and HDL synthesis. As a result, treatment is usually associated with not only lower triglyceride and LDL levels, but also higher HDL levels. Reduced cardiovascular events have been demonstrated in a subset of individuals with high triglyceride (>200 mg/dL) and low HDL (<40 mg/dL) levels, but improvements in cardiovascular or all-cause mortality otherwise have not been confirmed with these agents. Liver toxicity and myositis are potential side effects of fibric acid derivatives, and they also interfere with the metabolism of warfarin, leading to a need for its dose adjustment. Nicotinic acid has an antilipolytic effect and therefore decreases the influx of FFA to the liver. As a result, hepatic VLDL synthesis and LDL production are reduced. Nicotinic acid also decreases HDL catabolism. Lower triglyceride and LDL levels and higher HDL levels are observed with treatment. In addition, nicotinic acid stimulates tissue plasminogen activator and prevents thrombosis. It is the preferred agent for the reduction of lipoprotein(a) or Lp(a) (discussed later). The cardioprotective effect of nicotinic acid may be linked to its effect on Lp(a) and HDL. Side effects include hepatotoxicity, hyperuricemia, hyperglycemia, and flushing Omega-3 fatty acids reduce VLDL production and subsequently lower triglyceride levels (by 35%). They also modestly increase HDL (3%) and LDL (5%). The impact on lipids can occur over months to years and requires treatment doses as high as 3 to 4 g of fish oil per day. However, reductions in death due to sudden cardiac events and CHD are observed within weeks of treatment initiation. This benefit can be seen with lower treatment doses (fish oil <2 g/day) and is most likely related to the impact of omega-3 fatty acid on cardiac electrophysiology. Omega-3 fatty acids constitute 30% to 50% of fish oil supplements and 85% of prescribed pharmacologic preparations (i.e., Lovaza and Vascepa). In clinical trials, both Lovaza and Vascepa 4 g/day lowered triglyceride levels by 45%. Fish oil supplements seem to be a reasonable, cost-effective means to reduce triglyceride levels; side effects include eructation, taste perversion, and dyspepsia. Other agents to consider are neomycin, lomitapide, and mipomersen. These agents can be considered in the management of patients with refractory LDL elevations. Neomycin complexes with bile acid and lowers LDL levels. It also inhibits production of apolipoprotein(a) in the liver and lowers Lp(a). It is recommended as adjuvant therapy for patients with familial hypercholesterolemia and Lp(a) excess. Important side effects include nephrotoxicity and ototoxicity. Lomitapide inhibits microsomal triglyceride transfer protein in the liver and decreases apo B. Significant reductions in LDL (up to 50%) are seen with treatment. Liver toxicity is a serious adverse event associated with this agent. Mipomersen is another agent approved for use in homozygous familial hypercholesterolemia. It binds apo B messenger RNA and inhibits apo B production. Apo B is a structural component of VLDL, IDL, and LDL. Treatment reduces LDL by up to 50%. Side effects include flu-like symptoms, injection site reactions, elevations in liver enzymes, and liver toxicity. The side effect profile and expense associated with both lomitapide and mipomersen limit the use of these agents to individuals with homozygous familial hypercholesterolemia
What is evidence for reversal of endothelial dysfunction with statin therapy?
Improved vasodilatory response of endothelium to nitric oxide could help with ischemia
What can occur if Ezetimibe is used in combination with HMG-CoA reductase inhibitors?
In combination, this agent can increase serum transaminase levels and potentially increase the risk of cancer and cancer death.
What is Type IV Cell mediated Hypersensitivity caused by?
Inflammation resulting from cytokines produced by CD4 T cells and CD8 T cells T cell-mediated hypersensitivity induced by environmental and self antigens is the cause of many chronic inflammatory diseases, including autoimmune diseases
In CD4 T cell mediated hypersensitivity reactions what do cytokines produced by the T cells induce ?
Inflammation that may be chronic and destructive
What does HDL inhibit the expression of?
Inflammatory mediators e.g: Intercellular adhesion molecule [ICAM] and vascular cell adhesion molecule [VCAM]) by vascular cells
What does Marfan syndrome result from?
Inherited defect in an extracellular glycoprotein called Fibrillin-1
Antibodies specific for cell and tissue antigens may deposit in tissues and cause what?
Injury by local inflammation may induce phagocytosis, destruction of cells or they interfere with normal cellular functions
What is the body's first line of defense?
Innate immune system is early part of the immune response and is capable of responding very rapidly to pathogens
What facilitates the receptor-mediated endocytosis of LDL particles and subsequent vesicle fusion with lysosomes?
Interaction of apoB100 with the LDL receptor LDL receptor is recycled to the cell surace, while the cholesteryl esters and triglycerides within the LDL particle are hydrolyzed by lysosomal acid lipase (LAL) to release unesterified cholesterol and fatty acids
What is the intermediate and major source of plasma LDL?
Intermediate density lipoprotein (IDL)
Where are Chylomicrons formed and what do they transport?
Intestine Dietary triglyceride
LDL particles that are not taken up by the LDL receptor expressing tissues may migrate into where?
Intima of blood vessels and bind to proteoglycans There, they are subject to oxidation or nonenzymatic glycosylation Oxidation of LDL results in lipid peroxidation and may create reactive aldehyde intermediates that fragment apoB100 The modified LDL is internalized by scavenger receptors (e.g., SR-A), which are expressed predominantly by mononuclear phagocytic cells.
What is used to estimate quantitative traits like blood pressure or height?
Intraclass correlation coefficient This statistic varies bw -1.0 and +1.0 and measures the degree of homogeneity of a trait in a sample of individuals
Properties of lipoproteins- lipoprotein class, density, origin, apolipoproteins, lipid
LIPOPROTEIN CLASS DENSITY (g/mL) ORIGIN APOLIPOPROTEINS LIPID Chylomicrons <0.95 Intestine C-II, E TG (85%), cholesterol (10%) VLDL <1.006 Liver B100, C-II, E TG (55%), cholesterol (20%) IDL 1.006-1.019 VLDL catabolism B100, E TG (25%), cholesterol (35%) LDL 1.019-1.063 IDL catabolism B100 TG (5%), cholesterol (60%) HDL 1.063-1.25 Liver, intestine A-I, E TG (5%), cholesterol (20%)
colorectal cancer
It is estimated that 1 in 20 Americans will develop colorectal cancer, and roughly one third of those with this cancer will die from it. With approximately 135,000 new cases and 50,000 deaths in the United States each year, colorectal cancer is second only to lung cancer in the total number of annual cancer deaths. Like breast cancer, it clusters in families; familial clustering of this form of cancer was reported in the medical literature as early as 1881. The risk of colorectal cancer in people with one affected first-degree relative is two to three times higher than that of the general population As discussed in Chapter 11, familial colorectal cancer can be the result of mutations in the APC tumor suppressor gene or in one of several DNA mismatch-repair genes (HNPCC). Another, less common, inherited cause of colorectal cancer is the autosomal dominant Peutz-Jeghers syndrome. About half of Peutz-Jeghers cases are caused by mutations in the STK11 tumor suppressor gene, which encodes a protein kinase. Juvenile intestinal polyposis, an autosomal dominant disease defined by the presence of 10 or more polyps before adulthood, can be caused by mutations in SMAD4 (see Chapter 11), in BMPRA1 (a receptor serine-threonine kinase gene), or, in rare cases, in PTEN. As with breast cancer, most colorectal cancer cases (>90%) are not inherited as mendelian conditions and are likely to be caused by a complex interaction of inherited and somatic genetic alterations and environmental factors. The latter risk factors include a lack of physical activity and a high-fat, lowfiber diet.
What is the autosomal recessive form of LQT syndrome known as? What is it associated with?
Jervell -lange- Nielsen syndrome Longer QT interval, a higher incidence of sudden cardiac death and sensorineural defects
What is Jervell- Lange- Nielsen syndrome caused by mutations in?
KCNQI or KCNE1 genetic testing enables more accurate diagnosis
recurrence risks and transmission patterns part 2
It is sometimes difficult to differentiate polygenic or multifactorial diseases from single-gene diseases that have reduced penetrance or variable expression. Large data sets and good family history data are necessary to make the distinction. Several criteria are usually used to define multifactorial inheritance . • The recurrence risk is higher if more than one family member is affected. For example, the sibling recurrence risk for a ventricular septal defect (VSD, a type of congenital heart defect) is 3% if one sibling has had a VSD but increases to approximately 10% if two siblings have had VSDs. In contrast, the recurrence risk for single-gene diseases remains the same regardless of the number of affected siblings. This increase does not mean that the family's risk has actually changed. Rather, it means that we now have more information about the family's true risk: because they have had two affected children, they are probably located higher on the liability distribution than a family with only one affected child. In other words, they have more risk factors (genetic and/or environmental) and are more likely to produce an affected child. • If the expression of the disease in the proband is more severe, the recurrence risk is higher. This is again consistent with the liability model, because a more-severe expression indicates that the affected person is at the extreme tail of the liability distribution (see Fig. 12-2). His or her relatives are thus at a higher risk to inherit disease-causing genes. For example, the occurrence of a bilateral (both sides) cleft lip/palate confers a higher recurrence risk on family members than does the occurrence of a unilateral (one side) cleft . • The recurrence risk is higher if the proband is of the less commonly affected sex (see, for example, the previous discussion of pyloric stenosis). This is because an affected individual of the less susceptible sex is usually at a more extreme position on the liability distribution. • The recurrence risk for the disease usually decreases rapidly in more remotely related relatives (Table 12-2). Although the recurrence risk for single-gene diseases decreases by 50% with each degree of relationship (e.g., an autosomal dominant disease has a 50% recurrence risk for offspring of affected persons, 25% for nieces or nephews, 12.5% for first cousins, and so on), it decreases much more quickly for multifactorial diseases. This reflects the fact that many genetic and environmental factors must combine to produce a trait. All of the necessary risk factors are unlikely to be present in less closely related family members. • If the prevalence of the disease in a population is f (which varies between zero and one), the risk for offspring and siblings of probands is approximately f . This does not hold true for single-gene traits, because their recurrence risks are independent of population prevalence. It is not an absolute rule for multifactorial traits either, but many such diseases do tend to conform to this prediction. Examination of the risks given in Table 12-2 shows that these three diseases follow the prediction fairly well. Risks for multifactorial diseases usually increase if more family members are affected, if the disease has more severe expression, and if the affected proband is a member of the less commonly affected sex. Recurrence risks decrease rapidly with moreremote degrees of relationship. In general, the sibling recurrence risk is approximately equal to the square root of the prevalence of the disease in the population
What effect do CD8 T cells have for antigens on host cells?
Kill them T cell-mediated autoimmune diseases, both CD4+ T cells and CD8+ T cells specific for self antigens are present, and both contribute to tissue injury.
What is the most common autosomal dominant form of EDS? What does it result from?
Kyphoscoliosis type mutations in the gene encoding lysyl hydroxylase, an enzyme necessary for hydroxylation of lysine residues during collagen synthesis. Because hydroxylysine is essential for the cross-linking of collagen fibers, a deficiency of lysyl hydroxylase results in the synthesis of collagen that lacks normal structural stability.
How is LDL cholesterol estimated?
LDL = TC- HDL - TG/5
Lambert-Eaton myasthenic syndrome(LEMS)- Definition, epidemiology and pathology
LambertEaston myasthenic syndrome (LEMS) is an acquired, presynaptic neuromuscular transmission disorder caused by antibodies against the P/Q type voltagegated calcium channel (VGCC). P/Q VGCC antibodies cause reduced Ca+ influx into the presynaptic nerve terminal resulting in decreased acetylcholine release and neuromuscular transmission failure. LEMS is associated with cancer, usually small cell lung carcinoma, in 60% of cases. LEMS may predate tumor detection by up to 3 years. LEMS is very rare and more common in men (3 : 1).
What is diabetes the leading cause of?
Leading cause of adult blindness, kidney failure, and lower-limb amputation and a major cause of heart disease and stroke
Which Stain is the least potent? Which are the most potent?
Least potent: Fluvastatin Most potent: Atorvastatin and Rosuvastatin
What does Apo A-I activate? What it its function?
Lecithin-cholesterol acyltransferase (LCAT) enzyme that esterifies cholesterol
What is the function of LCAT?
Lecithincholesterol acyltransferase (LCAT): binds preferentially to disk-shaped HDL and converts cholesterol molecules within the particle to cholesteryl esters The reaction also creates a lysophosphatidylcholine molecule, which dissociates rom the particle and binds to serum albumin. Because they are highly insoluble, cholesteryl esters migrate into the core o the HDL particle. The development o a hydrophobic core converts the pre- beta-HDL to a spherical alpha -HDL particle.
Genes that encode what play a significant role in human obesity?
Leptin and its receptor Leptin hormone is secreted by adipocytes(fat storage cells) and binds to receptors in the hypothalamus, the site of the body's appetite control center
Is the amount of LDL transported along the scavenger receptor pathway less than or more than that mediated by the LDL receptor dependent mechanisms?
Less
What is Peutz-Jeghers syndrome? Mutation in what causes it?
Less common inherited cause of colorectal cancer Is an autosomal dominant mutation STK11 tumor suppressor gene - encodes a protein kinase
What is Lipoprotein lipase (LPL)?
Lipolytic enzyme expressed on the endothelial surface of capillaries in muscle and at tissue LPL is a glycoprotein that is synthesized by myocytes and adipocytes and transported to the endothelial cell sur ace by a specific glycosylphosphatidylinositol (GPI)-linked protein, GPIHBP1 On the endothelial cell membrane, GPIHBP1 also serves to anchor LPL in place
What are VLDL particles metabolized by? What do they become?
Lipoprotein Lipase (LPL) Incrementally smaller, relatively more deficient in triglycerides, and relatively enriched in cholesteryl esters.
What are esterified complex lipids transported in?
Lipoprotein particles
What is Lp(a)? What occurs when Lp(a) is present with elevated levels?
Lp(a) is a specialized form of LDL that is assembled extracellularly from apolipoprotein(a) and LDL. interferes with fibrinolysis by competing with plasminogen. This leads to decreased thrombolysis and increased clot formation Lp(a) also binds macrophages, promoting foam cell formation and atherosclerotic plaques
What 3 muscarinic receptors are coupled to Gq (phospholipase C)?
M1 M3 M5 Gq--> odd numbers facilitate excitation of the cell
Where are M1, M2, & M3 receptors located?
M1- Cortical neurons and autonomic ganglia (Brain) M2- Cardiac muscle (Heart) M3- Smooth muscle and glandular tissue (Everything else)
What is a key member of the cellular immune response? Where is it found?
MHC I found on the surface of nearly all of the body's cells useful to fight against microorganisms eg: viruses which are very adept at quickly inserting themselves into the body's cells, hence they are inaccessible to antibodies, which are water-soluble proteins that cannot pass through the cell's lipid membrane
MOA and Use: donepezil
MOA: AchE inhibitor USE: Alzheimer's
MOA and Use: physostigmine
MOA: AchE inhibitor USE: Glaucoma, Atropine overdose
Mutations in what genes can lead to Hereditary nonpolyposis colorectal cancer (HNPCC)? What does this disease also confer?
MSH2 and MLH1 Increased risk of breast cancer Despite the significance of these genes, it should be emphasized that more than 90% of breast cancer cases are not inherited as mendelian disease
What is the Concordance rates for stroke death in MZ and DZ twins respectively?
MZ- 10% DZ- 5%
What are Lipoproteins?
Macromolecular aggreagates that transport triglycerides and cholesterol in the blood Circulating lipoproteins can be differentiated on the basis of density, size, and protein content As a general rule, larger, less dense lipoproteins have a higher percentage composition of lipids; Chylomicrons are the largest and least dense lipoprotein subclass HDLs are the smallest lipoproteins, containing the lowest lipid content and the highest proportion of protein
What do the 5 types of heavy chains determine?
Major class or isotype to which the immunoglobin or Ig belongs to: γ, µ, α, δ, and ε which correspond to immunoglobin isotypes IgG, IgM, IgA, IgD and IgE respectively
What do ingested allergens trigger in food allergies? What does the released Histamine cause?
Mast cell degranulation Increased Peristalsis - resulting in diarrhea and vomiting
Common autosomal recessive disorders
Metabolic Cystic fibrosis Phenylketonuria Galactosemia Homocystinuria Lysosomal storage diseases* α1-Antitrypsin deficiency Wilson disease Hemochromatosis Glycogen storage diseases* Hematopoietic Sickle cell anemia Thalassemias Endocrine Congenital adrenal hyperplasia Skeletal Ehlers-Danlos syndrome (some variants)* Alkaptonuria* Nervous Neurogenic muscular atrophies Friedreich ataxia Spinal muscular atrophy
What does the composition of apolipoprotein determine?
Metabolic fate of the lipoprotein For example, each LDL particle contains one apolipoprotein B (apoB) 100 molecule, which is a ligand or the low-density lipoprotein receptor; in turn, binding of LDL to the LDL receptor promotes cholesterol uptake into cells.
The addition of methyl group to Acetylcholine forms what drug? What is it more resistant to?
Methacholine more resistant to degradation by AChE and hence pocesses a longer duration of action
What types of drugs are Methecholine and Bethanechol?
Muscarinic receptor agonists Cholinomimetics
What is lipid absorption into the enterocytes of the duodenum and jejunum facilitated by?
Micelles Long-chain fatty acids and monoglycerides are taken up separately into the enterocyte by carrier-mediated transport and then re-esterified to form triglycerides by the enzyme diacylglycerol acyltransferase (DGAT) medium-chain fatty acids are absorbed directly into the portal blood and metabolized by the liver
What drug is indicated for the treatment of Homozygous Familial Hypercholesterolemia who are already prescribed maximal medical therapy?
Mipomersen
What is another agent approved in the use of homozygous familial hypercholesterolemia?
Mipomersen
What is an example of a rapidly degraded nicotinic receptor antagonist? clinical use?
Mivacurium neuromuscular blockade during surgical procedures
What does each lipoprotein particle consist of?
Monolayer of polar, amphipathic lipids that surrounds a hydrophobic core structurally, lipoproteins are microscopic spherical particles ranging from 5 to 1,000nm in diameter Each lipoprotein particle also contains one or more types of apolipoprotein
Morphologically what is DTH characterized by the accumulation of? Where do they accumualte?
Mononuclear cells, mainly CD4 T cells and macrophages around venules producing perivascular cuffing In fully developed lesions, the venules show marked endothelial hypertrophy, reflecting cytokinemediated endothelial activation.
The transport of LDL via the scavenger receptor seems to occur at least in part into the cells of what system?
Mononuclear phagocyte system Monocytes and macrophages have receptors for chemically altered (e.g., acetylated or oxidized) LDL
Twin studies usually consist of comparisons between?
Monozygotic and Dizygotic twins if both members of a twin pair share a trait (eg: cleft lip) they are said to be concordant if they do not share the trait, they are discordant for traits determined completely by genes, MZ twins should always be concordant, and DZ twins should be concordant less often
MG- Diagnosis
Most patients with MG who are optimally treated experience improvement or remission of their symptoms. About 10% of patients with MG experience refractory symptoms despite optimal treatment. Mortality is currently less than 5%.
What is the prognosis of MG?
Most pt with MG who are optimally treated experience improvement or remission of their symptoms About 10% of patients with MG experience refractory symptoms despite optimal treatment, mortality is currently less than 5%
What are the most common side effects associated with use of AChE in the CNS ?
Mostly GI related: nausea, vomiting, anorexia, fatulence, loose stools, diarrhea, and abdominal cramping
What are Nicotinic receptor agonists used clinically in the induction of?
Muscle paralysis
What causes familial Colorectal cancer?
Mutations in APC tumor suppressor gene or Mutations in one of several DNA mismatch repair genes (HNPCC)
What is the major cause of MODY?
Mutations in gene that encodes Glucokinase - rate limiting enzyme in the conversion of glucose to glucose-6-phosphate in the pancreas Another 40% of MODY cases are caused by mutations in any of five genes that encode transcription factors involved in pancreatic development or insulin regulation: hepatocyte nuclear factor-1α (HNF1α), hepatocyte nuclear factor-1β (HNF1β), hepatocyte nuclear factor-4α (HNF4α), insulin promoter factor-1 (IPF1), and neurogenic differentiation 1 (NEUROD1)
What occurs in Lipoprotein lipase deficiency? When and how does it manifest?
Mutations in the LPL gene resulting in deficiency of LPL synthesis or function lead to increased circulating chylomicron and VLDL particles and severe hypertriglyceridemia It manifests in childhood with triglyceride levels higher than 1000 mg/dL Heterozygous LPL deficiency occurs in 2% to 4% of the population and usually requires a precipitating factor, such as uncontrolled diabetes or estrogen therapy, to manifest the phenotype.These individuals have moderate hypertriglyceridemia (250 to 750 mg/dL) that can increase to levels greater than 1000 mg/dL with secondary factors . These individuals have moderate hypertriglyceridemia (250 to 750 mg/dL) that can increase to levels greater than 1000 mg/dL with secondary factors. This can result in the chylomicronemia syndrome
The exit of cholesterol from the lysosomes requires the action of what two proteins?
NPC1 and NPC2
Recommendations for Nutritional intake
NUTRIENT RECOMMENDED INTAKE Total Fat 25-35% of total calories Saturated <7% Polyunsaturated <10% Monounsaturated <20% Carbohydrates 50-60% of total calories Protein 15% of total calories Cholesterol <200 mg/day Fiber 20-30 g/da
How do naive CD4 T cells get activated?
Naive CD4 T cells recognize peptides displayed by dendritic cells and secrete IL-2 which functions as a autocrine growth factor to stimulate proliferation of the antigen responsive T cells
MTP in hepatocytes lipidates apoB100 to form what?
Nascent VLDL particles Under the continued influence of MTP, the nascent VLDL particles coalesce with larger triglyceride droplets and are secreted directly into the circulation VLDL particles may also acquire apoE, apoCI, apoCII, and apoCIII within the hepatocyte prior to secretion However, these apolipoproteins may also be trans erred to VLDL rom HDL in the circulation
What is the internalization of LDL particles into cells is regulated by what?
Negative feedback A negative cholesterol balance increases the expression of LDL receptors and subsequent uptake of cholesterol from the circulation. A positive cell cholesterol balance suppresses LDL receptor expression and decreases uptake of LDL-cholesterol into cells.
What has ACh been observed to inhibit the growth of?
Neurite During the early phases of neural development, when such growth is essential, AChE levels are increased. The presence of
What do Mast cell derived tumor necrosis factor (TNF) and IL-4 promote?
Neutrophil and Eosinophil rich inflammation
clinical commentary - neural tube defects
Neural tube defects (NTDs) include anencephaly, spina bifida, and encephalocele, as well as several other less-common forms (Fig. 12-3). They are one of the most important classes of birth defects, with a newborn prevalence of approximately 1 per 1000. There is considerable variation in the prevalence of NTDs among populations, with an especially high rate among some northern Chinese populations (as high as 6 per 1000 births). The prevalence of NTDs has been decreasing in many parts of the United States and Europe during the past three decades Normally the neural tube closes at about the fourth week of gestation. A defect in closure or a subsequent reopening of the neural tube results in an NTD. Spina bifida is the most commonly observed NTD and consists of a protrusion of spinal tissue through the vertebral column (the tissue usually includes meninges, spinal cord, and nerve roots). About 75% of spina bifida patients have secondary hydrocephalus, which sometimes in turn produces intellectual disability. Paralysis or muscle weakness, lack of sphincter control, and club feet are often observed. A study conducted in British Columbia showed that survival rates for spina bifida patients have improved dramatically over the past several decades. Less than 30% of such children born between 1952 and 1969 survived to 10 years of age, but 65% of those born between 1970 and 1986 survived to this age. Anencephaly is characterized by partial or complete absence of the cranial vault and calvarium and partial or complete absence of the cerebral hemispheres. At least two thirds of anencephalics are stillborn; term deliveries do not survive more than a few hours or days. Encephalocele consists of a protrusion of the brain into an enclosed sac. It is seldom compatible with survival NTDs are thought to arise from a combination of genetic and environmental factors. In most populations surveyed thus far, empirical recurrence risks for siblings of affected individuals range from 2% to 5%. Consistent with a multifactorial model, the recurrence risk increases with additional affected siblings. A Hungarian study showed that the overall prevalence of NTDs in that country was 1/300 births and that the sibling recurrence risks were 3%, 12%, and 25% after one, two, and three affected offspring, respectively. Recurrence risks tend to be slightly lower in populations with lower NTD prevalence rates, as predicted by the multifactorial model. Recurrence risk data support the idea that the major forms of NTDs are caused by shared or similar factors. An anencephalic conception increases the recurrence risk for subsequent spina bifida conceptions, and vice versa. NTDs can usually be diagnosed prenatally, sometimes by ultrasound and usually by an elevation in α-fetoprotein (AFP) in the maternal serum or amniotic fluid (see Chapter 13). A spina bifida lesion can be either open or closed (i.e., covered with a layer of skin). Open spina bifida is more likely to be detected by an AFP assay A major epidemiological finding is that mothers who supplement their diet with folic acid at the time of conception are less likely to produce children with NTDs. This result has been replicated in several different populations and is thus well confirmed. It has been estimated that approximately 50% to 70% of NTDs can be avoided simply by dietary folic acid supplementation. (Traditional prenatal vitamin supplements would not have an effect because administration does not usually begin until well after the time that the neural tube closes.) Since mothers would be likely to ingest similar amounts of folic acid from one pregnancy to the next, folic acid deficiency could well account for at least part of the elevated sibling recurrence risk for NTDs. Dietary folic acid is an important example of a nongenetic factor that contributes to familial clustering of a disease However, it is likely that there is genetic variation in response to folic acid, which helps to explain why most mothers with folic acid deficiency do not bear children with NTDs and why some who ingest adequate amounts of folic acid nonetheless bear children with NTDs. To address this issue, researchers are testing for associations between NTDs and variants in several genes whose products (e.g., methylene tetrahydrofolate reductase) are involved in folic acid metabolism (see Clinical Commentary 15-6 in Chapter 15 for further information on dietary folic acid supplementation and NTD prevention).
What is the preferred agent for the reduction of Lipoprotein(a) or Lp(a)?
Nicotinic acid The cardioprotective effect of nicotinic acid may be linked to its effect on Lp(a) and HDL.
What channel does dietary and biliary cholesterol from micelles enter the enterocyte ?
Niemann-Pick C1- like protein (NPC1L1) Some of this cholesterol is immediately pumped back into the intestinal lumen by the ATP-dependent action of a heterodimeric protein, ABCG5/ABCG8 (ABCG5/G8). The fraction of cholesterol that remains is esterified to a long-chain fatty acid by acetyl-CoA:cholesterol acyltransferase (ACAT).
niacin
Niacin (nicotinic acid, vitamin B 3 ) is a water-soluble vitamin. At physiologic concentrations, it is a substrate in the synthesis o nicotinamide adenine dinucleotide (NAD) and nicotinamide adenine dinucleotide phosphate (NADP), which are important co actors in intermediary metabolism The pharmacologic use o niacin necessitates large doses (1,500-3,000 mg/day) and is independent o the conversion o nicotinic acid to NAD or NADP (Fig. 20-13). Niacin decreases plasma LDL-cholesterol and triglyceride concentrations and increases HDL cholesterol. Studies have identif ed a G protein-coupled receptor on adipocytes that appears to mediate the metabolic changes associated with niacin administration. Stimulation o this receptor by niacin decreases adipocyte hormone-sensitive lipase activity, leading to reduced peripheral tissue triglyceride catabolism and there ore decreased flux of free fatty acids to the liver. This decreases the rate o hepatic triglyceride synthesis and VLDL production, leading to decreases in triglycerides (by up to 45%) and LDL (by up to 20%). Niacin also increases the hal -li e o apoAI, the major apolipoprotein in HDL. The increase in plasma apoAI increases plasma HDL concentrations by up to 30% and presumably augments reverse cholesterol transport. Pharmacologic doses o niacin are available as oral agents or daily administration. The major adverse e ects o niacin are cutaneous ushing and pruritus (itching). The ushing is mediated by the G protein-coupled niacin receptor and involves the release o prostaglandins D 2 and E 2 within the skin. It can be mitigated by pretreatment with aspirin or another nonsteroidal anti-in ammatory drug (NSAID). These adverse e ects usually disappear a ter several weeks o niacin use. Timed-release ormulations o niacin are associated with less cutaneous flushing than the immediate-release dosage form. In addition to ushing and pruritus, important adverse e ects o niacin include hyperuricemia, impaired insulin sensitivity, hepatotoxicity, and the potentiation o statin-induced myopathy. Hyperuricemia may precipitate gout. Impaired insulin sensitivity may precipitate diabetes in patients at risk, and niacin should be used with caution in diabetic patients. Rarely, niacin may cause myopathy. Concurrent administration o niacin with a statin slightly increases the risk o myopathy Niacin is indicated or patients with elevations o both triglycerides and cholesterol, usually in combination with a statin. Because niacin is currently the most e ective agent available or raising HDL, it may also be the drug o choice or patients with modestly elevated LDL and decreased HDL. It is not clear whether both the LDL-lowering and HDL-raising e ects o niacin contribute to improved clinical outcomes
Nicotinic receptors
Nicotinic acetylcholine receptors (nAChRs) mediate nicotinic cholinergic transmission via a process known as direct ligand-gated conductance (Fig. 10-2). The binding o two ACh molecules to one nAChR elicits a con ormational change in the receptor that creates a monovalent cationselective pore through the cell membrane. Open channels o the activated nAChR are equally permeable to K and Na ions. (Since the resting membrane potential is close to the Nernst potential or K and ar below the Nernst potential or Na , the predominant ion passing through the open nACR is Na .) A relatively small permeability to Ca 2 ions also results in important elevations o intracellular [Ca 2 ]. There ore, when open, these channels produce a net inward current that depolarizes the postsynaptic cell. Stimulation o multiple nAChRs may depolarize the cell su f ciently to generate action potentials and to open voltage-dependent calcium channels. The latter action, and the direct entry o Ca 2 through the nAChR pore, can lead to activation o several intracellular signaling pathways. Because ACh dissociates rapidly rom active-state receptor molecules and acetylcholinesterase rapidly degrades ree (unbound) ACh in the synaptic cle t (see below), the depolarization mediated by nAChRs is brie ( 10 ms). Although the simultaneous binding o two ACh molecules is required or channel opening, it is not necessary or both molecules to dissociate rom the receptor in order or the channel to open again; the binding o a second ACh molecule to a receptor that still has one ACh bound may, once again, result in channel opening. The kinetics o nAChR binding and channel opening are detailed in Figure 10-3. Structurally, the nicotinic acetylcholine receptor comprises f ve subunits, each o which has a mass o approximately 40 kilodaltons (Fig. 10-2A). Several types o nAChR subunits have been identif ed; these are designated , , , , and . All o these subunits share 35-50% homology with one another. Each receptor at the NMJ is composed o two subunits, one and one subunit, and either one or one subunit. (The 2 orm dominates at the neuromuscular junction in mature skeletal muscle, while the 2 orm is expressed in embryonic muscle.) Agonist molecules bind at a hydrophobic pocket that is ormed between each subunit and the adjacent, complementary subunit—this is the structural basis or the binding o two ACh molecules to each receptor. The con ormational change in the subunits induced by the binding o ACh initiates the overall changes in the pore that permit ion ow through the receptor (i.e., that open the channel). Besides simply opening and closing in response to ACh binding, nicotinic receptors also modulate their responses to various concentration prof les o ACh. The receptors react di erently to discrete, brie pulses o ACh than to neurotransmitter that is present continuously. As noted above, under normal conditions, a closed, resting-state channel responds to dual ACh binding by opening transiently, and the low a f nity o the receptor or ACh causes rapid dissociation o ACh rom the receptor and return o the receptor to its resting con ormation. In contrast, continuous exposure o the receptor to ACh causes it to undergo a change to a "desensitized" con ormation in which the channel is locked closed. The desensitized state is also characterized by a greatly increased affinity of the receptor or ACh, so that ACh remains bound to the receptor or a relatively long period o time. This prolonged binding o ACh to the desensitized con ormation delays the conversion o the receptor to its resting state and hence prolongs the time during which the receptor is incapable o being activated by agonist. Nicotinic cholinergic receptors at autonomic ganglia and in the central nervous system (termed N 2 or N N ) are similar to receptors at the NMJ ( N 1 or N M ), with the exception that the subunits in N N receptors are composed solely o and subunits. To complicate matters, however, nine di erent subunit types ( 2 - 10 ) and three subunit types ( 2 - 4 ) have been detected in neuronal tissues. ( 1 and 1 re er to the distinct subunit types ound at the NMJ.) This diversity o and subunit combinations is responsible or the variable responses o CNS and autonomic nAChRs to pharmacologic agents. Presynaptic nAChRs in the CNS modulate the release both o ACh itsel and o other excitatory and inhibitory neurotransmitters. This e ect may involve prolonged elevations o [Ca 2 ] in the presynaptic nerve terminals, which lead to inactivation o neuronal calcium channels.
What effect do Selective nicotinic receptor antagonists produce? What does it lead to?
Nondepolarizing (competitive) neuromuscular blockade during surgical procedures Flaccid paralysis
What do non-genetic risk factors for prostate cancer include?
Nongenetic risk factors for prostate cancer may include a high-fat diet. Because prostate cancer usually progresses slowly and because it can be detected by digital examination and by the prostate-specific antigen (PSA) test, fatal metastasis can usually be prevented
What type of distribution is displayed by polygenic/multifactorial traits?
Normal or Bell shaped distribution
What type of symptoms do patients mostly present with first in MG?
Ocular symptoms blurred vision, double vision and droopy eyelids but about 15% of pt present with bulbar symptoms first- dysarthria(slurred speech), dysphagia, shortness of breath or limb weakness
What does activation of chylomicrons and VLDL require the addition of?
Optimal complement of apoCII molecules which occurs by aqueous transfer of apoCII from HDL particles
What does Cholesterol ester transfer protein (CETP) transfer core HDL cholesterol esters to?
Other lipoproteins such as VLDL These lipoproteins deliver cholesterol to peripheral sites for hormone and cell membrane synthesis
What gene associated T cell regulation is associated with Type I diabetes?
PTPN22 is also associated with other autoimmune disorders, including rheumatoid arthritis and systemic lupus erythematosus.
What effect do Anticholinergic compounds that act on muscarinic receptors produce at target organs?
Parasympathetic effect by blocking normal cholinergic tone, these compounds allow sympathetic responses to dominate
What do antibodies bind to after being released from plasma cells? What effects can they have?
Pathogen's surface antigens 1- Neutralize the pathogen directly 2- Tags the pathogen for destruction by other components like complement proteins and phagocytes
What is Type I hypersensitivity reaction (immediate hypersensitivity)? What does this reaction usually depend on the production of?
Pathologic reaction that is caused by the release of mediators from mast cells IgE
What is Th17 responsible for?
Recruitment of leukocytes including neutrophils
mixed hyperlipidemia
Patients with mixed hyperlipidemia exhibit complex lipid prof les that may consist o elevated total cholesterol, LDL cholesterol, and triglyceride concentrations. HDL cholesterol is o ten reduced. Etiologies o mixed hyperlipidemia include amilial combined hyperlipidemia (FCHL), dysbetalipoproteinemia , and lysosomal acid lipase def ciency (LAL-D FCHL is a common disease associated with moderately elevated concentrations o asting triglycerides and total cholesterol and reduced concentrations o HDL cholesterol. These patients o ten present with other eatures o the metabolic syndrome , including abdominal obesity, glucose intolerance, and hypertension. The molecular de ects are still under investigation. Current hypotheses ocus on insulin resistance, which leads to increased lipolysis in at tissue. Fatty acids liberated rom at tissue return to the liver, where they are reassembled into triglycerides. The increase in triglycerides increases the production o VLDL particles, which leads to an increase in apoB-containing lipoproteins in the plasma. In part because o the complex phenotypes o FCHL, the underlying genetic de ects have remained elusive. Faith ul adherence to dietary modif cation may be an e ective means o controlling FCHL. However, drug treatment is o ten required, and statins are commonly utilized. Combination therapy that includes addition o a f brate or niacin may be necessary to normalize triglyceride and LDL-cholesterol concentrations, as well as to increase HDL cholesterol Dysbetalipoproteinemia is a disorder characterized by increased cholesterol-rich chylomicrons and IDL-like particles. These f ndings are the result o accumulated chylomicron and VLDL remnants, leading to both hypertriglyceridemia and hypercholesterolemia. ApoE has three iso orms (E2, E3, and E4) in humans, and apoE2 has been implicated in the disease. Chylomicrons and VLDL particles in patients with the homozygous apoE2/apoE2 phenotype have reduced a f nity or their lipoprotein receptors, leading to accumulation o remnant particles in the plasma. Although the apoE iso orm is present at birth, symptoms generally present in adult males and in postmenopausal emales. The mechanism underlying this delay in expression o the phenotype is unknown, and additional metabolic actors (e.g., obesity, diabetes, or hypothyroidism) may be required to unmask the disorder. Dysbetalipoproteinemia can be managed by decreased intake o at and cholesterol, along with weight reduction and omission o alcohol intake. In addition, niacin and f brates are e ective pharmacologic therapies. Lysosomal acid lipase def ciency (LAL-D) is a rare lysosomal storage disorder caused by mutations in the LIPA gene, which encodes lysosomal acid lipase. This mutation leads to def ciency o the enzyme, with a corresponding reduction in the ability o LDL-C to be processed normally by hepatocytes. The result is hepatic steatosis and dyslipidemia, with elevated LDL-C, elevated triglycerides, and reduced HDL-C. Clinically, LAL-D is re erred to as Wolman disease in in ants and children or as cholesterol ester storage disease (CESD) in adults. Patients with LAL-D develop early atherosclerosis and progressive liver disease, with associated high rates o mortality at a young age.
What differentiates Type II diabetes from Type I?
Persons with type 2 diabetes usually have some degree of endogenous insulin production, at least in earlier stages of the disease, and they can sometimes be treated successfully with dietary modification, oral drugs, or both patients with type 2 diabetes have insulin resistance (i.e., their cells have difficulty using insulin) and are more likely to be obese No HLA associations nor autoantibodies are seen commonly in this form of diabetes
What are organophosphates used as and what do they cause?
Pesticides and developed for chemical warefare even a small exposure can lead to death caused by respiratory failure
What process is responsible for depletion of cells coated with antibodies?
Phagocytosis Cells opsonized by IgG antibodies are recognized by phagocyte Fc receptors, which are specific for the Fc portions of some IgG subclasses
What does the Non-neuronal Cholinergic system (NNCS) regulate?
Physiologic processes including: growth, adhesion, migration and differentiation
What is the drug of choice for treating the CNS effects of anticholinergic overdose?
Physostigmine
What is the drug of choice for reversing anticholinergic drug poisoning?
Physostigmine its tertiary amine structure allows it ready access to the brain and spinal cord, where it can counteract the CNS e ects o anticholinergic toxicity.
Where is Acetylcholinesterase (AChE) concentrated?
Postsynaptic membrane choline that is freed there is efficiently transported back into the presynaptic terminal Because o its central importance to cholinergic transmission, a class o drugs known as acetylcholinesterase inhibitors has been designed to target AChE.
What drug is effective in improving muscle weakness for a short period of time in MG?
Prednisone long term use is associated with side effects
What is Juvenile intestinal polyposis defined by? What is it caused by?
Presence of 10 or more polyps before adulthood Mutations in SMAD4, BMPRA1 ( a receptor serine-threonine kinase gene) or in rare cases- PTEN
What does Vesamicol prevent? How?
Prevents storage of Ach into vesicles by blocking the ACh-H+ antiporter
The recurrence risk is higher if the expression of the disease in the ..... is more severe?
Proband eg: the occurrence of a bilateral (both sides) cleft lip/palate confers a higher recurrence risk on family members than does the occurrence of a unilateral (one side) cleft
What do patients with LPL deficiency exhibit?
Profound hypertriglyceridemia which is characterized by elevated chylomicrons during infancy and impaired removal of VLDL later in life Infants or young adults may present with pancreatitis, eruptive xanthomas, hepatomegaly, and splenomegaly attributable to the accumulation of lipid-laden foam cells Treatment consists of a fat-free diet and avoidance of substances that increase VLDL production by the liver, such as alcohol and glucocorticoids.
What is Alzheimer's disease characterized by?
Progressive dementia and memory loss and by the formation of amyloid plaques and neurofibrillary tangles in the brain, particularly in the cerebral cortex and hippocampus The plaques and tangles lead to progressive neuronal loss, and death usually occurs within 7 to 10 years after the first appearance of symptoms. The risk of developing AD doubles in persons who have an affected first-degree relative
What do patients with LEMS have fluctuating weakness in? What is a common symptom?
Proximal limb and trunk muscles with the lower limbs/extremities more severely affected than the upper ones. Difficulty walking
Why are depolarizing blockades with Succinylcholine only used for small durations?
Prolonged depolarization can lead to life-threatening electrolyte imbalances (caused by prolonged Na+ influx and K+ efflux)
What is an example of a disease that corresponds to threshold model and presents shortly after birth? What is it caused by?
Pyloric stenosis Narrowing or obstruction of the pylorus, the area between the stomach and the intestine
What drug is used in the treatment of Mysthenia Gravis? How much and how often is it prescribed?
Pyridostigmine 30 to 60 mg every 4 hours improves most symptoms in patients with Mysthenia Gravis
SCID can also arise from mutations in what 2 genes involved in VDJ recombinatino?
RAG1 or RAG2
Recurrence Risks for Relatives of Schizophrenic Probands, Based on Multiple Studies of Western European Population
RELATIONSHIP TO PROBAND RECURRENCE RISK (%) Monozygotic twin 44.3 Dizygotic twin 12.1 Offspring 9.4 Sibling 7.3 Niece or nephew 2.7 Grandchild 2.8 First cousin 1.6 Spouse 1.0
Recurrence risks (%) for Pyloric stenosis, subdivided by gender of affected probands and relatives
RELATIVES MALE PROBANDS FEMALE PROBANDS LONDON BELFAST LONDON BELFAST Brothers 3.8 9.6 9.2 12.5 Sisters 2.7 3.0 3.8 3.8
Therapeutic approach to reduce levels of low density lipoprotein-cholesterol
RISK CATEGORY TREATMENT GOAL: LDL (mg/dL) LIFESTYLE CHANGES: LDL (mg/dL) DRUG THERAPY: LDL (mg/dL) ≤One risk factor <160 ≥160 ≥160-190 ≥Two risk factors <130 ≥130 ≥130-160 CHD or CHD risk equivalent <100 (optional <70) ≥100 ≥100-130
What is the primary event in the postsynaptic ganglionic response?
Rapid depolarization mediated by nicotinic ACh receptors in the cell membrane postganglionic neuorons
What is higher if more than one family member is affected?
Recurrence risk eg: the sibling recurrence risk for a ventricular septal defect (VSD, a type of congenital heart defect) is 3% if one sibling has had a VSD but increases to appx 10% if two siblings have had VSDs
What is lower if the proband is of the less commonly affected sex?
Recurrence risk This is because an affected individual of the less susceptible sex is usually at a more extreme position on the liability distribution.
What do B cell immunodeficiency diseases render patients especially susceptible to?
Recurrent bacterial infections eg: Strep pneumoniae
The recurrence risk for the disease usually decreases rapidly for more ........ related relatives?
Remotely
If total cholesterol value is greater than 200 mg/dL or the HDL value is less than 40 mg/dL then what is required?
Repeat fasting lipid panel If the total cholesterol value is less than 200 mg/dL and the HDL value is greater than 40 mg/dL, then retesting is recommended every 5 years
Methacholine is at least 3 times more ....... to ........ by AChE than is ACh?
Resistant Hydrolysis
People with DD or Dd genotypes have what on their erythrocytes and are thus?
Rh antigen Rh positive people with dd genotype are Rh negative and do not have Rh antigen
What is the Rh blood group loci D responsible for?
Rh maternal-fetal incompatibility and the resulting disease Hemolytic disease of the newborn (HDN)
Reverse cholesterol transport
Reverse cholesterol transport. A. The process o reverse cholesterol transport begins when apoAI is secreted rom the liver. ApoAI in plasma interacts with ATP binding cassette protein AI (ABCA1), which incorporates a small amount o phospholipid and unesterif ed cholesterol rom hepatocyte plasma membranes to orm a discoidal-shaped pre- -HDL particle. Due to the activity o lecithin:cholesterol acyltrans erase (LCAT) in plasma, pre- -HDL particles mature to orm spherical -HDL. Spherical -HDL particles unction to accept excess unesterif ed cholesterol rom the plasma membranes o cells in a wide variety o tissues. The unesterif ed cholesterol is trans erred rom the cell to nearby HDL particles by interactions with ABCA1, ABCG1, and SR-BI, as well as by aqueous di usion through the plasma. As explained in panel B, LCAT and phospholipid trans er protein (PLTP) increase the capacity o HDL to accept unesterif ed cholesterol molecules rom cells by allowing or expansion o the core and the sur ace coat o the particle. Cholesteryl ester trans er protein (CETP) removes cholesteryl ester molecules rom HDL and replaces them with triglycerides rom remnant particles. HDL particles interact with scavenger receptor, class B type I (SR-BI), which mediates selective hepatic uptake o cholesterol and cholesteryl esters, but not apoAI. This process is acilitated when hepatic lipase hydrolyzes triglycerides rom the core o the particle. The remaining apoAI molecules may begin the cycle o reverse cholesterol transport again. B. LCAT, PLTP, and CETP promote the removal o excess cholesterol rom the plasma membranes o cells. LCAT removes a atty acid rom a phosphatidylcholine molecule in the sur ace coat o - (or pre- -) HDL and esterif es an unesterif ed cholesterol molecule on the sur ace o the particle. The resulting lysophosphatidylcholine (lyso-PC) becomes bound to albumin in the plasma, whereas the cholesteryl ester migrates spontaneously into the core o the lipoprotein particle. The unesterif ed cholesterol molecules that are consumed by LCAT are replaced by unesterif ed cholesterol rom cells. HDL phospholipids that are consumed by LCAT action are replaced with excess phospholipids rom remnant particles by the activity o PLTP. As described in panel A, CETP increases the e f ciency o cholesterol transport to the liver by exchanging cholesteryl ester molecules in -HDL or triglycerides in VLDL remnants. Unlike phospholipids, triglycerides, and cholesteryl esters (which require transport proteins), unesterif ed cholesterol and lyso-PC can di use over short distances in the plasma.
What do hepatocytes increase expression of in response to PPAR alpha activation? What does that provide?
SR-B1 provides pathway for increased reverse cholesterol transport , with subsequent cholesterol excretion into the bile
When mature HDL particles circulate to the liver what do they interact with?
SR-BI is the principal HDL receptor
Who should be screened for elevated Plasma Lipoprotein (a)? How is the diagnosis made?
Screening should be considered in individuals who have a family or personal history of premature CHD without dyslipidemia and in those for whom cholesterol-lowering therapy has failed. documenting Lp(a) levels higher than 30 mg/dL in a patient with premature Congestive heart disease (CHD)
Where do B cells and T cells encounter disease causing microorganisms?
Secondary lymphoid tissues eg: lymph nodes, spleen and tonsils
What is Secondary prevention? What is primary prevention?
Secondary prevention: Statins have been shown in numerous clinical trials to reduce mortality significantly after a myocardial infarction Primary prevention: lowering of LDL with statins can decrease mortality even in the absence of overt cardiovascular disease
The use of 3,4 DAP for the treatment of LEMS is contraindicated in patients with?
Seizures
What are examples of T cell mediated hypersensitivity reactions against environmental antigens?
Sensitivity to chemicals eg: therapeutic drugs and substances found in plants such as poison ivy
What is the process of coating mast cells with IgE called?
Sensitization because coating with IgE specific for an antigen makes the mast cells sensitive to activation by subsequent encounter with that antigen
What is usually found in all cases of LEMS patients that helps with diagnosis?
Serum antibodies against P/Q VGCC (voltage gated calcium channels)
What can high potency statins cause increases in?
Serum transaminase levels ie: Alanine transaminase (ALT) and aspartate transaminase (AST) observed elevations in ALT and AST most likely reflect an adaptive response of the liver to changes in cholesterol homeostasis True hepatotoxicity is indicated by ALT and AST elevations that are accompanied by elevations in serum bilirubin concentrations
disorders associated with defects in structural proteins
Several diseases caused by mutations in genes that encode structural proteins are listed in Table 5-4. Many are discussed elsewhere in the text. Only Marfan syndrome and Ehlers-Danlos syndromes (EDSs) are discussed here, because they affect connective tissue and hence involve multiple organ systems
What is Edrophonium Chloride (Tensilon)? How is it administered? What does it demonstrate?
Short acting acetylcholinesterase inhibitor Intravenously (IV) Demonstrates symptoms improvement in patients with MG
What is required to cause a disease with locus heterogeneity? example?
Single mutation Osteogenesis imperfecta
Where do Cholesterol inhibitors reduce cholesterol absorption by?
Small intestine involves reduction of dietary cholesterol but the more important effect is reduced reabsorption of biliary cholesterol inhibitors of cholesterol absorption also appear to reduce LDL cholesterol by inhibiting hepatic production of VLDL.
What is the rate limiting compenent of acetylcholine synthesis? What is it a target of?
Sodium-dependent high affinity transport system (Na+ choline cotransporter) Anticholinergic drugs (eg: Hemicholinum)
What occurs in Somatic hypermutation?
Somatic hypermutation is a process that allows B cells to mutate the genes that they use to produce antibodies. This enables the B cells to produce antibodies that are better able to bind to bacteria, viruses and other infections. Eventually somatic hypermutation produces a subset of immunoglobulins that have high affinity binding to the foreign antigen and the B cells that harbor these immunoglobulins are selected to proliferate extensively The end result is a population of mature plasma cells that secrete antibodies that are highly specific to the invading pathogen
cellular immune response
Some microorganisms, such as viruses, are very adept at quickly inserting themselves into the body's cells. Here, they are inaccessible to antibodies, which are water-soluble proteins that cannot pass through the cell's lipid membrane. A second component of the adaptive immune system, the cellular immune system, has evolved to combat such infections. A key member of the cellular immune response is the class I MHC molecule, which is found on the surfaces of nearly all of the body's cells. In a normal cell, the class I MHC molecule binds with small peptides (8 to 10 amino acids long) derived from the interior of the cell. It migrates to the cell's surface, carrying the peptide with it and displaying it outside the cell. Because this is one of the body's own peptides, no immune response is elicited. In an infected cell, however, the class I MHC molecule can bind to small peptides that are derived from the infecting organism. Cell-surface presentation of foreign peptides by the class I MHC molecule alerts the immune system, T cells in particular. Recall that T lymphocytes learn to recognize and tolerate self peptides (in conjunction with MHC molecules) while developing in the thymus, but they are highly intolerant of foreign peptides. The MHCpeptide complex binds to receptors on the appropriate T cell's surface, which prompts the T cell to emit a chemical that destroys the infected cell (Fig. 9-4). Because of their ability to destroy cells in this way, these T lymphocytes are termed cytotoxic T lymphocytes or killer T lymphocytes.* Each cytotoxic T lymphocyte can destroy one infected cell every 1 to 5 minutes. T cells are often alerted to the presence of an infection when circulating dendritic cells present the foreign peptides on their cell surfaces and migrate to secondary lymphoid tissues, where most of the T cells reside. As with B lymphocytes, only a small fraction of the body's T cells has receptors with binding affinity for the infecting pathogens. The immune system is capable of destroying the body's cells once they are infected. Peptides from the pathogen are displayed on cell surfaces by class I MHC molecules. These are recognized by cytotoxic (killer) T lymphocytes, which destroy the infected cell. In contrast to cytotoxic T cells, which destroy infected cells directly, helper T (TH) cells respond to the presence of pathogens by secreting cytokines. These molecules in turn stimulate the development of other components of the immune system, such as B cells and cytotoxic T cells. TH cells are classified into subsets, depending on which cytokines they secrete. For example, TH1 cells, which are involved primarily in combatting intracellular pathogens, secrete interleukin-2 (IL-2), interferon-γ, and tumor necrosis factor β. TH2 cells, which secrete IL-4, IL-5, IL-6, and IL-13, help to fight multicellular parasites and are involved in allergic responses. Other helper T-cell subsets include TH17 cells, which secrete IL-17, and TH22 cells, which secrete IL-22. As in the B-cell component of the adaptive immune system, a subset of long-lived T cells is retained (memory T cells) to quickly respond to a foreign pathogen should it be encountered again in the future. Yet another type of T cell, the regulatory T cell, helps to regulate the immune system so that self peptides are not inadvertently attacked. Helper T cells stimulate development of other components of the immune system, such as cytotoxic T cells and B cells, by secreting cytokines. Various subsets of helper T cells have been described, depending on which cytokines they secrete. Memory T cells remain long after an infection and ensure rapid response to a subsequent infection by the same pathogen. Regulatory T cells help to prevent the immune system from attacking the body's own cells.
What is an example of a Choline ester that has a high affinity for Nicotinic receptors and is resistant to AChE? What is its clinical use?
Succinylcholine Induces paralysis during surgery by means of depolarizing blockade In order to produce depolarizing blockade, the agent must persist at the neuromuscular junction and activate the nicotinic receptor channels continuously. Note that this effect is unlike the depolarization pattern seen in the generation of a standard action potential or end-plate potential, in which ACh is present at the neuromuscular junction or only a brie period o time.
Nicotinic receptor agonists
Succinylcholine is a choline ester that has high a f nity or nicotinic receptors and is resistant to AChE. It is used to induce paralysis during surgery by means o depolarizing blockade . This e ect can be caused by any direct nAChR agonist because such drugs activate receptor-associated channels and produce depolarization o the cell membrane In order to produce depolarizing blockade, the agent must persist at the neuromuscular junction and activate the nicotinic receptor channels continuously. Note that this e ect is unlike the depolarization pattern seen in the generation o a standard action potential or end-plate potential, in which ACh is present at the neuromuscular junction or only a brie period o time. The overall pattern is a brie period o excitation, mani ested by widespread asciculations in muscle cells, ollowed by f accid paralysis. The paralysis occurs or two reasons. First, the open cholinergic channels maintain the cell membrane in a depolarized state, e ecting inactivation o voltagegated sodium channels so that they cannot open to support urther action potentials. Second, the agonist-bound nAChRs spontaneously desensitize, preventing their opening and response to any subsequently delivered, additional agonist. Because o this mechanism, any nAChR agonist, including ACh, is capable o producing depolarizing blockade at su f ciently high concentrations . Generally, depolarizing blockade with succinylcholine is used or only short durations because prolonged depolarization can lead to li e-threatening electrolyte imbalances (caused by prolonged Na inf ux and K e f ux). Table 10-6 compares the e ects o depolarizing and nondepolarizing NMJ-blocking agents The concept o depolarizing blockade pertains to all cholinergic receptors and is not strictly limited to the NMJ . For example, this mechanism accounts or the paradoxical suppression o parasympathomimetic activity at autonomic ganglia by high levels o agonists, such as nicotine, that are selective or nicotinic receptors. The potential or inducing depolarizing blockade is partially responsible or the unpredictable e ects o nAChR agonists. Although muscarinic receptor agonists can also cause depolarizing blockade at autonomic ganglia, this e ect is obscured by the overwhelmingly parasympathomimetic responses seen at other neuron effector sites.
Where are MHC II molecules only found?
Surfaces of the immune system's APC eg: Phagocytes and B lymphocytes MHC II are heterodimers consisting of alpha and beta chain on chromosome 6
What is the absence of Aspartic acid at position 57 of the DQ polypeptide strongly associated with?
Susceptibility to type 1 diabetes in fact, those who do not have this amino acid at position 57 (and instead are homozygous for a different amino acid) are 100 times more likely to develop type 1 diabetes
What are the symptoms/characteristics associated with Botulism?
Symmetric descending flaccid paralysis starting with: blurred or double vision, ptosis, dysphagia, dry mouth, dysarthria, and muscle weakness Symptoms usually start 18 to 36 hours after ingesting contaminated food
After synthesis where is Ach transported for storage? What provides energy for this process?
Synaptic Vesicles ATPase
What does the release of Ach depend on the fusion of? What is this process dependent on?
Synaptic vesicle with the plasma membrane Axon terminal depolarization and opening of voltage gated calcium channels
Defects in what results in dyslipidemia? Prevalence of dyslipidemia?
Synthesis and catabolism of lipoproteins Prevalence of dyslipidemia in the United States is approximately 20% and varies with the population studied. An estimated 70% of individuals with premature coronary heart disease (CHD) have dyslipidemia
Common X-linked recessive disorders
System Disease Musculoskeletal Duchenne muscular dystrophy Blood Hemophilia A and B Chronic granulomatous disease Glucose-6-phosphate dehydrogenase deficiency Immune Agammaglobulinemia Wiskott-Aldrich syndrome Metabolic Diabetes insipidus Lesch-Nyhan syndrome Nervous Fragile X syndrome*
Hypertension
Systemic hypertension is a key risk factor for heart disease, stroke, and kidney disease. It is estimated that hypertension is responsible for approximately half of all cardiovascular mortality. Studies of blood pressure correlations within families yield heritability estimates of approximately 30% to 50% for both systolic and diastolic blood pressure. Heritability estimates based on twin studies tend to be higher (about 60%) and may be inflated because of greater similarities in the environments of MZ compared with DZ twins. The fact that the heritability estimates are substantially less than 100% indicates that environmental factors must also be significant causes of blood pressure variation. The most important environmental risk factors for hypertension are increased sodium intake, decreased physical activity, psychosocial stress, and obesity (as discussed later, the latter factor is itself influenced both by genes and environment). Blood pressure regulation is a highly complex process that is influenced by many physiological systems, including various aspects of kidney function, cellular ion transport, vascular tone, and heart function. Because of this complexity, much research is now focused on specific components that might influence blood pressure variation, such as the renin- angiotensin system (Fig. 12-8) (involved in sodium reabsorption and vasoconstriction); vasodilators such as nitric oxide and the kallikrein-kinin system; and ion transport systems such as adducin and sodium-lithium countertransport. These individual factors are more likely to be under the control of smaller numbers of genes than is blood pressure itself, simplifying the task of identifying these genes and their role in regulating blood pressure. For example, linkage and association studies have implicated several genes involved in the renin-angiotensin system (e.g., the genes that encode angiotensinogen, angiotensin-converting enzyme type 1, and angiotensin II type 1 receptor) in causing hypertension A small percentage of hypertension cases are the result of rare single-gene disorders, such as Liddle syndrome (low plasma aldosterone and hypertension caused by mutations that alter the ENaC epithelial sodium channel) and Gordon syndrome (hypertension, high serum potassium level, and increased renal salt reabsorption caused by mutations in the WNK1 or WNK4 kinase genes); see Table 12-6 for additional examples. More than 20 genes have been identified that can lead to rare inherited forms of hypertension, and many of them affect the reabsorption of water and salt by the kidney, which in turn affects blood volume and blood pressure. It is hoped that isolation and study of these genes, and the pathways in which they act, will lead to the identification of genetic factors underlying essential hypertension.‡ Large-scale genome scans, undertaken in more than 100,000 humans and in experimental animals such as mice and rats, have sought to identify additional loci (see Box 12-1) that might underlie essential hypertension. These studies have identified dozens of genes that influence susceptibility to hypertension. Although the individual effects of each of these genes is small (as expected for a multifactorial trait), they collectively explain a significant proportion of the genetic risk for hypertension. Heritability estimates for systolic and diastolic blood pressure range from 30% to 50%. A number of genes responsible for rare hypertension syndromes have been identified, and genome scans have implicated regions that might contain genes that underlie susceptibility to essential hypertension. Other risk factors for hypertension include increased sodium intake, lack of exercise, psychosocial stress, and obesity.
What reflexes are hypoactive or absent and may increase following short exercise in Lambert eaton syndrome patients?
Tendon reflexes
Immediate hypersensitivity develop as a result of the activation of?
Th2 cells in response to protein antigens or chemicals that bind to proteins
What is an example of an environmental factor causing a congenital malformation?
Thalidomide When ingested during early pregnancy it can cause phocomelia (severely short limbs) in babies Maternal exposure to retinoic acid, which is used to treat acne, can cause congenital defects of the heart, ear, and central nervous system Maternal rubella infection can cause congenital heart defects.
What occurs following the binding of IgE on the mast cell?
The FcεRI molecules that are carrying the IgE are cross-linked, triggering biochemical signals from the signal-transducing chains of FcεRI
The Rh system
The Rh blood group is encoded by two tightly linked loci, one of which is labeled D. The other locus produces Rh antigens labeled C and E through alternative splicing of the messenger RNA. The D locus is of primary interest because it is responsible for Rh maternal-fetal incompatibility and the resulting disease, hemolytic disease of the newborn (HDN). Persons with the DD or Dd genotype have the Rh antigen on their erythrocytes and are Rh-positive. The recessive homozygotes, with genotype dd, are Rh-negative and do not have the Rh antigen. About 85% of persons of European ancestry are Rh-positive and about 15% are Rh-negative. Unlike the ABO system, in which antibodies normally are formed in response to antigens presented by other organisms, anti-Rh antibody production requires a stimulus by the human Rh antigen itself. An Rh-negative person does not begin to produce anti-Rh antibodies unless he or she is exposed to the Rh antigen, usually through a blood transfusion or during pregnancy. Maternal-fetal incompatibility results when an Rh-positive man and an Rh-negative woman produce children. If the man's genotype is DD, all of their offspring will be Rh-positive and will have Rh antigens on their erythrocytes. If the man is a heterozygote, with genotype Dd, half of their children will be Rh-positive, on average. There are usually no difficulties with the first Rh- incompatible child, because very few of the fetus's red blood cells cross the placental barrier during gestation. When the placenta detaches at birth, a large number of fetal red blood cells typically enter the mother's blood stream. These cells, carrying the Rh antigens, stimulate production of anti-Rh antibodies by the mother. These antibodies persist in the blood stream for a long time, and if the next offspring is again Rh-positive, the mother's anti-Rh antibodies enter the fetus's bloodstream and destroy its red blood cells. As this destruction proceeds, the fetus becomes anemic and begins to release many erythroblasts (immature nucleated red cells) into its blood stream. This phenomenon is responsible for the descriptive term erythroblastosis fetalis. The anemia can lead to a spontaneous abortion or stillbirth. Because the maternal antibodies remain in the newborn's circulatory system, destruction of red cells can continue in the neonate. This causes a buildup of bilirubin and a jaundiced appearance shortly after birth. Without replacement transfusions, in which the child receives Rh-negative red cells, the bilirubin is deposited in the brain, producing cerebral damage and usually death. Infants who do not die can develop intellectual disability, cerebral palsy, and/or high-frequency deafness. Among persons of European descent, approximately 13% of all matings are Rh-incompatible. Fortunately, a simple therapy now exists to avoid Rh sensitization of the mother. During and after pregnancy, an Rh-negative mother is given injections of Rh immune globulin, which consists of anti-Rh antibodies. These antibodies destroy the fetal erythrocytes in the mother's blood stream before they stimulate production of maternal anti-Rh antibodies. Because the injected antibodies do not remain in the mother's blood stream for long, they do not affect subsequent offspring. To avoid sensitization, these injections must be administered with each pregnancy. The Rh-negative mother must also be careful not to receive a transfusion containing Rh-positive blood, because this would also stimulate production of anti-Rh antibodies. Maternal-fetal Rh incompatibility (Rh-negative mother and Rh-positive fetus) can produce hemolytic disease of the newborn if the mother's Rh antibodies attack the fetus. Administration of Rh immune globulin to the mother prevents this reaction. A rarer form of maternal-fetal incompatibility can result when a mother with type O blood carries a fetus with type A or B blood. The hemolytic disease produced by this combination is usually so mild that it does not require treatment. Interestingly, if the mother is also Rh-negative and the child is Rh-positive, the ABO incompatibility protects against the more severe Rh incompatibility. This is because any fetal red blood cells entering the mother's circulatory system are quickly destroyed by her anti-A or anti-B antibodies before she can form anti-Rh antibodies.
assembly of apoB containing lipoproteins
The cellular mechanisms by which chylomicrons and VLDL are assembled are quite similar. Regulation o the assembly process depends on the availability o apoB and triglycerides, as well as the activity o microsomal triglyceride-transfer protein (MTP) The gene that encodes apoB is transcribed principally in the intestine and the liver. Apart rom this tissue-specif c expression, there is little transcriptional regulation o the apoB gene. In contrast, a key regulatory event that di erentiates chylomicron metabolism rom VLDL metabolism is the editing o apoB mRNA (Fig. 20-2). Within enterocytes but not hepatocytes, a protein named apoB editing complex-1 (apobec-1) is expressed. This protein constitutes the catalytic subunit o the apoB editing complex, which deaminates a cytosine at position 6666 o the apoB mRNA molecule. Deamination converts the cytosine to uridine. As a result, the codon containing this nucleotide is converted rom glutamine to a premature stop codon. When translated, the intestinal orm apoB48 is 48% as long as the ulllength protein that is expressed in the liver and re erred to as apoB100 . As a consequence, chylomicrons, the apoB-containing lipoprotein produced by the intestine, contain apoB48, whereas VLDL particles produced by the liver contain apoB100 Figure 20-3 illustrates the cellular mechanisms by which apoB-containing lipoproteins are assembled and secreted. As the apoB protein is synthesized by ribosomes, it crosses into the endoplasmic reticulum. Within the endoplasmic reticulum, triglyceride molecules are added co-translationally to the elongating apoB protein (i.e., apoB is lipidated) by the action o a co actor protein, MTP. Once apoB has been ully synthesized, the nascent lipoprotein is enlarged in the Golgi apparatus; during this process, MTP adds additional triglycerides to the core o the particle. By unclear mechanisms, cholesteryl esters are also added to the core. Each lipoprotein particle assembled by this process contains a single molecule o apoB Because the triglyceride component o chylomicrons originates primarily rom the diet (Fig. 20-4), the assembly, secretion, and metabolism o chylomicrons are collectively re erred to as the exogenous pathway o lipoprotein metabolism. During digestion, cholesteryl esters and triglycerides in ood are hydrolyzed to orm unesterif ed cholesterol, ree atty acids, and monoglycerides. Bile acids, phospholipids, and cholesterol are secreted by the liver into bile and stored in the gallbladder during asting as micelles and vesicles, which are macromolecular lipid aggregates that orm due to the detergent properties o bile acid molecules. The stimulus o eating a meal promotes emptying o gallbladder bile into the small intestine, where the micelles and vesicles solubilize the digested lipids Lipid absorption into enterocytes o the duodenum and jejunum is acilitated mainly by micelles. Long-chain atty acids and monoglycerides are taken up separately into the enterocyte by carrier-mediated transport and then re-esterif ed to orm triglycerides by the enzyme diacylglycerol acyltransferase (DGAT). By contrast, medium-chain atty acids are absorbed directly into the portal blood and metabolized by the liver. Dietary and biliary cholesterol rom micelles enter the enterocyte via a protein channel named Niemann-Pick C1-like 1 protein (NPC1L1) . Some o this cholesterol is immediately pumped back into the intestinal lumen by the ATP-dependent action o a heterodimeric protein, ABCG5/ABCG8 (ABCG5/G8). The raction o cholesterol that remains is esterif ed to a long-chain atty acid by acetyl-CoA:cholesterol acyltransferase (ACAT). Once triglycerides and cholesteryl esters are packaged together with apoB48, apoA1 is added as an additional structural apolipoprotein and the chylomicron particle is exocytosed into the lymphatics or transport to the circulation via the thoracic duct. The plasma concentration o triglyceride-rich chylomicrons varies in proportion to dietary at intake. Very-low-density lipoproteins (VLDL) contain triglycerides that are assembled by the liver using plasma atty acids derived rom adipose tissue or synthesized de novo. For this reason, the assembly, secretion, and metabolism o VLDL are o ten re erred to as the endogenous pathway o lipoprotein metabolism. Hepatocytes synthesize triglycerides in response to increased ree atty acid ux to the liver. This typically occurs in response to asting, thereby ensuring a continuous supply o atty acids or delivery to muscle in the absence o triglycerides rom the diet. Interestingly, dietary saturated ats as well as carbohydrates also stimulate the synthesis o triglycerides within the liver. By cellular mechanisms that are similar to those that produce chylomicrons (Fig. 20-3), MTP in hepatocytes lipidates apoB100 to orm nascent VLDL particles. Under the continued in uence o MTP, the nascent VLDL particles coalesce with larger triglyceride droplets and are secreted directly into the circulation. VLDL particles may also acquire apoE, apoCI, apoCII, and apoCIII within the hepatocyte prior to secretion. However, these apolipoproteins may also be trans erred to VLDL rom HDL in the circulation The synthesis o apoB48 in the intestine and apoB100 in the liver is constitutive. This permits the immediate production o chylomicrons and VLDL particles when triglyceride molecules are available. In the absence o triglycerides, such as in enterocytes during asting, apoB is degraded by a variety o cellular mechanisms. Recent studies have revealed a role or sortilin in the cellular tra f cking o VLDL particles. Sortilin is encoded by the Sort1 gene, and genomewide association studies (GWAS) have shown that Sort1 is associated with reduced levels o LDL cholesterol (LDL-C). Sortilin acilitates the post-translational degradation o apoB by a lysosome-dependent mechanism
What are the clinical presentations of Familial defective apolipoprotein B100?
The clinical presentation is similar to familial hypercholesterolemia, with elevated total cholesterol and elevated LDL-cholesterol levels associated with premature CHD and tendon xanthomas the homozygous and heterozygous clinical forms of familial defective apo B100 are milder than familial hypercholesterolemia, because apo E-mediated clearance of remnant particles is still functional Total cholesterol concentration ranges from 350 to 550 mg/dL in the homozygous and 200 to 350 mg/dL in the heterozygous disorder
How do Statins lower cholesterol synthesis?
The decrease in cholesterol concentration activates a cellular signaling cascade culminating in the activation of sterol regulatory element binding protein 2 (SREBP2), a transcription actor that up-regulates expression of the gene encoding the LDL receptor. Increased LDL receptor expression causes increased uptake of plasma LDL and consequently decreases plasma LDL-cholesterol concentration
pharmacological classes and agents
The decision to treat dyslipidemia is largely dependent on the calculated cardiovascular risk. A number o clinical algorithms exist or determining initiation o therapy. Goals or lipid lowering were established in the 2001 National Cholesterol Education Program Adult Treatment Panel III (ATP III) guidelines, which were updated in 2004 based on the results o several additional large, randomized clinical trials. These guidelines provide target LDL levels based on 10-year risk o death rom cardiovascular disease (Table 20-3) and have been generally adopted in clinical practice. In 2013, the American College o Cardiology and American Heart Association (ACC/AHA) published new guidelines on the treatment o cholesterol. These guidelines no longer utilize baseline LDL-C as an indication to initiate treatment, nor do they establish LDL-C goals o treatment. Instead, the new guidelines def ne our discrete "statin benef t groups" (Table 20-4). Both sets o guidelines emphasize the importance o therapeutic li estyle changes (TLCs), which include reduction o dietary saturated at and cholesterol intake, weight reduction, increased physical activity, avoidance o tobacco products, and, possibly, stress reduction Successful dietary therapy can reduce total cholesterol by up to about 25%, depending on adherence and the metabolic basis or elevated cholesterol concentrations. I this approach is unsuccess ul or insu f cient to normalize lipid levels, drug therapy is generally recommended. Five well-established classes o drugs are available or pharmacologic modification o lipid metabolism. Three o these classes (inhibitors) o cholesterol synthesis, bile acid sequestrants, and cholesterol absorption inhibitors) have relatively well-def ned e ects on lipid metabolism. While the overall e ects o the other two classes (f brates and niacin) are clear, their molecular mechanisms o actions are diverse and remain subjects o active investigation. The inhibitors o cholesterol synthesis (i.e., HMG-CoA reductase inhibitors, also known as statins ) are the most important class due to their welldemonstrated e f cacy in reducing cardiovascular morbidity and mortality. However, agents in each o the other classes act as important adjunctive therapies and may be the agents o choice or patients with certain specif c causes o dyslipidemia. The newest therapies or treating rare diseases and individuals with inadequate response to maximum medical management include VLDL secretion inhibitors and PCSK9 inhibitors.
What are adoption studies used to estimate?
The genetic contribution to a multifactorial trait provide a second means of estimating the influence of genes on multifactorial diseases
What occurs in a cross-reaction?
The immune system mistakes peptides from some of the body's normal cells for microbial peptides eg: infections of HLA-B27- positive persons with specific microbes eg: Klebsiella eg: Rheumatic fever- streptococcal infection initiates cross reactivity between streptococcus and cardiac myosin and thus leads to heart problems
Both MHC I and II are highly polymorphic, what does this increase the ability of?
The individuals and populations to respond to a wide variety of pathogens
psychiatric disorders
The major psychiatric diseases, including schizophrenia, bipolar disorder, and autism spectrum disorder, have been the subjects of numerous genetic studies. Twin, adoption, and family studies have shown that these disorders aggregate in families. Moreover, schizophrenia, biopolar disorder, and autism tend to occur together in the same families. In part, this reflects the fact that some of the same genes influence susceptibility to each of these conditions
What is the clinical importance of Ach for cognitive function illustrated by?
The pathophysiology and treatment of Alzheimer's disease (AD) and other neurodegenerative dementias, including diffuse Lewy body dementia (DLB) and Parkinson's disease with dementia (PDD) Neurodegenerative dementias and brain injury produce central cholinergic dysfunction. Patients with these conditions manifest cognitive, functional, and behavioral deficits that are at least partially related to cholinergic deficits and amenable to symptomatic treatment with procholinergic medications
Diseases caused by lymphocytes
The role of T lymphocytes in human immunologic diseases has been increasingly recognized with improved methods for identifying and isolating these cells from lesions and through animal models of human disease in which a pathogenic role of T cells is established by experiments. In fact, much of the recent interest in the pathogenesis and treatment of human autoimmune diseases has focused on disorders in which tissue injury is caused mainly by T lymphocytes.
What does the further reduction in size of LDL particle result in?
The transfer of apoE to HDL As a consequence, LDL is a distinct , cholesteryl ester-enriched lipoprotein with apoB100 as its only apolipoprotein
What are autoantibodies?
These are antibodies directed against "self" antigens.
What is the function of KIR (killer immunoglobulin like receptors)?
These receptors inhibit natural killer cells when they bind to MHC class 1 molecules on the surface of normal cells but activate them when MHC class 1 molecules are absent
What happens when Mast cells sensitized by IgE are exposed to the allergen?
They become activated and secrete their mediators
What happens once chylomicrons and VLDL once they acquire apoCII?
They bind to LPL which hydrolyzes triglycerides from the core of the lipoprotein
What are polygenic traits?
Traits in which variation is thought to be caused by the combined effects of multiple genes When environmental factors are also believed to cause variation in the trait, which is usually the case, the term multifactorial is used
How does Transient neonatal MG occur in appx 12% of newborns?
Transplacental passive transfer of antibodies from the mother to the fetus
T or F Organophosphates have long duration of AChE inhibition and that increases even more after it undergoes aging, thus making organophosphate inhibition essentially irreversible?
True
T or F Simultaneous binding of 2 ACh molecules are required for channel opening in Nicotinic receptors?
True
Twin studies
Twins occur with a frequency of about 1/100 births in populations of European ancestry. They are slightly more common among Africans and a bit less common among Asians. Monozygotic (MZ, or identical) twins originate when the developing embryo divides to form two separate but genetically identical embryos. Because they are genetically identical, MZ twins are an example of natural clones. Their physical appearances can be strikingly similar (Fig. 12-5). Dizygotic (DZ, or fraternal) twins are the result of a double ovulation followed by the fertilization of each egg by a different sperm.† Thus, DZ twins are genetically no more similar than siblings. Because two different sperm cells are required to fertilize the two eggs, it is possible for each DZ twin to have a different father. Because MZ twins are genetically identical, any differences between them should be due only to environmental effects. MZ twins should thus resemble each other very closely for traits that are strongly influenced by genes. DZ twins provide a convenient comparison: their environmental differences should be similar to those of MZ twins, but their genetic differences are as great as those between siblings. Twin studies thus usually consist of comparisons between MZ and DZ twins. If both members of a twin pair share a trait (e.g., cleft lip), they are said to be concordant. If they do not share the trait, they are discordant. For a trait determined completely by genes, MZ twins should always be concordant, and DZ twins should be concordant less often. Like siblings, DZ twins share only 50% of their DNA because each parent transmits half of his or her DNA to each offspring. Concordance rates can differ between opposite-sex DZ twin pairs and same-sex DZ pairs for some traits, such as those that have different frequencies in males and females. For such traits, only samesex DZ twin pairs should be used when comparing MZ and DZ concordance rates. A concordance estimate would not be appropriate for quantitative traits, such as blood pressure or height. Here the intraclass correlation coefficient is used. This statistic varies between −1.0 and +1.0 and measures the degree of homogeneity of a trait in a sample of individuals. For example, we may wish to assess the degree of similarity between twins for a trait such as height. The measurements are made in a collection of twins, and correlation coefficients are estimated separately for the MZ sample and the DZ sample. If a trait were determined entirely by genes, we would expect the correlation coefficient for MZ pairs to be 1.0 (i.e., each pair of twins would have exactly the same height). A correlation coefficient of 0.0 would mean that the similarity between MZ twins for the trait in question is no greater than chance. Because DZ twins share half of their DNA, we would expect a DZ correlation coefficient of 0.50 for a trait determined entirely by genes. Monozygotic (identical) twins are the result of an early cleavage of the embryo, whereas dizygotic (fraternal) twins are caused by the fertilization of two eggs by two sperm cells. Comparisons of concordance rates and correlations in MZ and DZ twins help to estimate the extent to which a trait is influenced by genes Concordance rates and correlation coefficients for a number of traits are given in Table 12-3. The concordance rates for contagious diseases like measles are quite similar in MZ and DZ twins. This is expected, because most contagious diseases are unlikely to be influenced markedly by genes. On the other hand, the concordance rates for schizophrenia are quite dissimilar between MZ and DZ twins, indicating a sizable genetic component for this disease. The MZ correlation for dermatoglyphics (fingerprints), a series of traits determined almost entirely by genes, is close to 1.0 Correlations and concordance rates in MZ and DZ twins can be used to measure the heritability of multifactorial traits. Essentially, heritability is the percentage of population variation in a trait that is due to genes (statistically, it is the proportion of the total variance of a trait that is caused by genes). A simple formula for estimating heritability (h) from twin correlations or concordance rates is as follows: where cMZ is the concordance rate (or intraclass correlation) for MZ twins and cDZ is the concordance rate (or intraclass correlation) for DZ twins. As this formula illustrates, traits that are largely determined by genes result in a heritability estimate that approaches 1.0 (i.e., cmz approaches 1.0, and cDZ approaches 0.5). As the difference between MZ and DZ concordance rates becomes smaller, heritability approaches zero. Correlations and concordance rates in other types of relatives (e.g., between parents and offspring) can also be used to measure heritability. Like recurrence risks, heritability values are specific for the population in which they are estimated. However, there is usually agreement from one population to another regarding the general range of heritability estimates of most traits (e.g., the heritability of height is almost always high, and the heritability of contagious diseases is almost always low). The same is true of empirical recurrence risks. Comparisons of correlations and concordance rates in MZ and DZ twins allow the estimation of heritability, a measure of the percentage of population variation in a disease that can be attributed to genes. At one time, twins were thought to provide a perfect "natural laboratory" in which to determine the relative influences of genetics and environment. But several difficulties arise. One of the most important is the assumption that the environments of MZ and DZ twins are equally similar. MZ twins are often treated more similarly than DZ twins. A greater similarity in environment can make MZ twins more concordant for a trait, inflating the apparent influence of genes. In addition, MZ twins may be more likely to seek the same type of environment, further reinforcing environmental similarity. On the other hand, it has been suggested that some MZ twins tend to develop personality differences in an attempt to assert their individuality. Another difficulty is that the uterine environments of different pairs of MZ twins can be more or less similar, depending on whether there are two amnions and two chorions, two amnions and one shared chorion, or one shared amnion and one shared chorion. In addition, somatic mutations can occur during mitotic divisions of the cells of MZ twin embryos after cleavage occurs. Thus, the MZ twins might not be quite "identical," especially if a mutation occurred early in the development of one of the twins. Finally, methylation patterns, which can influence the transcription of specific genes, become more dissimilar in MZ twin pairs as they age. This dissimilarity is greater when the twins adopt markedly different habits and lifestyles (e.g., when one twin smokes cigarettes and the other does not). Of the various problems with the twin method, the greater degree of environmental sharing among MZ twins is perhaps the most serious. One way to circumvent this problem, at least in part, is to study MZ twins who were raised in separate environments. Concordance among these twin pairs should be caused by genetic, rather than environmental, similarities. As one might expect, it is not easy to find such twin pairs. A major effort to do so has been undertaken by researchers at the University of Minnesota, whose studies have shown a remarkable congruence among MZ twins reared apart, even for many behavioral traits. However, these studies must be viewed with caution, because the sample sizes are relatively small and because many of the twin pairs had at least some contact with each other before they were studied. Although twin studies provide valuable information, they are also affected by certain biases. The most serious is greater environmental similarity between MZ twins than between DZ twins. Other biases include somatic mutations that might affect only one MZ twin and differences in the uterine environments of twins
type I diabetes
Type 1 diabetes, which is characterized by T-cell infiltration of the pancreas and destruction of the insulin-producing beta cells, usually (though not always) manifests before 40 years of age. Patients with type 1 diabetes must receive exogenous insulin to survive. In addition to T-cell infiltration of the pancreas, autoantibodies are formed against pancreatic cells, insulin, and enzymes such as glutamic acid decarboxylase; these autoantibodies can be observed long before clinical symptoms occur. These findings, along with a strong association between type 1 diabetes and the presence of several human leukocyte antigen (HLA) class II alleles, indicate that this is an autoimmune disease. Over the past few decades, the incidence of type 1 diabetes has increased substantially Siblings of persons with type 1 diabetes face a substantial elevation in risk: approximately 6%, as opposed to a risk of about 0.3% to 0.5% in the general population. The recurrence risk is also elevated when there is a diabetic parent, although this risk varies with the sex of the affected parent. The risk for offspring of diabetic mothers is only 1% to 3%, but it is 4% to 6% for the offspring of diabetic fathers. (Because type 1 diabetes affects males and females in roughly equal proportions in the general population, this risk difference is inconsistent with the sex-specific threshold model for multifactorial traits.) Twin studies show that the empirical risk for MZ twins of type 1 diabetes patients ranges from 30% to 50%. In contrast, the concordance rate for DZ twins is 5% to 10%. The fact that type 1 diabetes is not 100% concordant among identical twins indicates that genetic factors are not solely responsible for the disorder. There is evidence that specific viral infections contribute to the cause of type 1 diabetes in at least some persons, possibly by activating an autoimmune response. The association of specific HLA class II alleles and type 1 diabetes has been studied extensively, and it is estimated that the HLA loci account for about 40% to 50% of the genetic susceptibility to type 1 diabetes. Approximately 95% of whites with type 1 diabetes have the HLA DR3 and/or DR4 alleles, whereas only about 50% of the general white population has either of these alleles. If an affected proband and a sibling are both heterozygous for the DR3 and DR4 alleles, the sibling's risk of developing type 1 diabetes is nearly 20% (i.e., about 40 times higher than the risk in the general population). This association may in part reflect linkage disequilibrium between alleles of the DR locus and those of the HLA-DQ locus. The absence of aspartic acid at position 57 of the DQ polypeptide is strongly associated with susceptibility to type 1 diabetes; in fact, those who do not have this amino acid at position 57 (and instead are homozygous for a different amino acid) are 100 times more likely to develop type 1 diabetes. The aspartic acid substitution alters the shape of the HLA class II molecule and thus its ability to bind and present peptides to T cells (see Chapter 9). Altered T-cell recognition might help to protect persons with the aspartic acid substitution from an autoimmune episode. The insulin gene, which is located on the short arm of chromosome 11, is another logical candidate for type 1 diabetes susceptibility. Polymorphisms within and near this gene have been tested for association with type 1 diabetes. Intriguingly, a strong risk association is seen with allelic variation in a VNTR polymorphism (see Chapter 3) located just 5′ of the insulin gene. Differences in the number of VNTR repeat units might affect transcription of the insulin gene (possibly by altering chromatin structure), which would result in variation in susceptibility. It is estimated that inherited genetic variation in the insulin region accounts for approximately 10% of the familial clustering of type 1 diabetes. ve been used extensively to map additional genes that can cause type 1 diabetes. In addition, an animal model, the nonobese diabetic (NOD) mouse, has been used to identify diabetes susceptibility genes that could have similar roles in humans (see Box 12-1). These studies have identified dozens of additional genes associated with type 1 diabetes susceptibility. One of these is the CTLA4 (cytotoxic lymphocyte associated-4) gene, which encodes an inhibitory T-cell receptor. Several studies have demonstrated that alleles of CTLA4 are associated with an increased risk of type 1 diabetes. There is growing evidence that variation in CTLA4 is also associated with other autoimmune diseases, such as rheumatoid arthritis and celiac disease. Another gene associated with type 1 diabetes susceptibility, PTPN22, is involved in T-cell regulation and is also associated with other autoimmune disorders, including rheumatoid arthritis and systemic lupus erythematosus.
type II diabetes
Type 2 diabetes accounts for more than 90% of all diabetes cases, and its incidence is rising rapidly in populations with access to high-calorie diets. It currently affects approximately 10% to 20% of the adult populations of many developed countries. One study estimates that because of the rapid rate of increase of this disease, one third of Americans born in 2000 will eventually develop type 2 diabetes. A number of features distinguish type 2 diabetes from type 1 diabetes. Persons with type 2 diabetes usually have some degree of endogenous insulin production, at least in earlier stages of the disease, and they can sometimes be treated successfully with dietary modification, oral drugs, or both. In contrast to those with type 1 diabetes, patients with type 2 diabetes have insulin resistance (i.e., their cells have difficulty using insulin) and are more likely to be obese. This form of diabetes has traditionally been seen primarily in patients older than 40 years, but because of increasing obesity among adolescents and young adults, it is now increasing rapidly in this segment of the population. Neither HLA associations nor autoantibodies are seen commonly in this form of diabetes. MZ twin concordance rates are substantially higher than in type 1 diabetes, often exceeding 90% (because of age dependence, the concordance rate increases if older subjects are studied). The empirical recurrence risks for first-degree relatives of patients with type 2 diabetes are higher than those for type 1 patients, generally ranging from 15% to 40%. The differences between type 1 and type 2 diabetes are summarized in Table 12-7. The two most important risk factors for type 2 diabetes are a positive family history and obesity; the latter increases insulin resistance. The disease tends to rise in prevalence when populations adopt a diet and exercise pattern typical of United States and European populations. Increases have been seen, for example, among Japanese immigrants to the United States and among some native populations of the South Pacific, Australia, and the Americas. Several studies, conducted on both male and female subjects, have shown that regular exercise can substantially lower one's risk of developing type 2 diabetes, even among persons with a family history of the disease. This is partly because exercise reduces obesity. However, even in the absence of weight loss, exercise increases insulin sensitivity and improves glucose tolerance. Extensive linkage and genome-wide association analyses have identified more than 70 genes that contribute to type 2 diabetes susceptibility. The most significant gene identified thus far is TCF7L2, which encodes a transcription factor involved in secreting insulin. A variant of TCF7L2 is associated with a 50% increased risk of developing type 2 diabetes. A significant association has also been observed between type 2 diabetes and a common allele of the gene that encodes peroxisome proliferator-activated receptor-γ (PPAR-γ), a nuclear receptor that is involved in adipocyte differentiation and glucose metabolism. This receptor is the target of thiazolidinediones [TZDs], a class of drugs commonly used to increase insulin sensitivity in those with type 2 diabetes. Although the common PPARG allele confers only a 25% increase in the risk of developing type 2 diabetes, it is found in more than 75% of persons of European descent and thus helps to account for a significant fraction of type 2 diabetes cases. Variation in KCNJ11, which encodes a potassium channel necessary for glucose-stimulated insulin secretion, confers an additional 20% increase in type 2 diabetes susceptibility. The associations between diabetes susceptibility and each of these genes have been widely replicated in numerous populations. Many rare susceptibility alleles in each of these diabetes-associated genes are now being discovered through high-throughput DNA sequencing studies.
During digestion what are cholesteryl esters and triglycerides in food hydrolzyed to form?
Unesterified cholesterol, free fatty acids and monoglycerides Bile acids, phospholipids, and cholesterol are secreted by the liver into bile and stored in the gallbladder during fasting as micelles and vesicles, which are macromolecular lipid aggregates that form due to the detergent properties of bile acid molecules
What do Omega-3 fatty acids reduce production of and subsequently lower levels of? Increase levels of?
VLDL production Triglyceride levels (by 35%) Modestly increase HDL (3%) and LDL (5%) The impact on lipids can occur over months to years and requires treatment doses as high as 3 to 4 g of fish oil per day. However, reductions in death due to sudden cardiac events and CHD are observed within weeks of treatment initiation.
What explains the variable expression seen in Marfan syndrome?
Variable expression occurs due to the fact that many different mutations that affect the fibrillin locus, which number more than 600
What do mast cell mediators cause a rapid increaese in?
Vascular permeability and smooth muscle contraction
Effects of Acetylcholine on Muscarinic Receptors in Peripheral tissues
Vasculature (endothelial cells) Release o nitric oxide and vasodilation Eye iris (pupillae sphincter muscle) Contraction and miosis Ciliary muscle Contraction and accommodation o lens to near vision Salivary and lacrimal glands Thin and watery secretions Bronchi Constriction; ↑ secretions Heart Bradycardia, ↓ conduction velocity, AV block at high doses, slight ↓ in contractility Gastrointestinal tract ↑ Tone, secretions; relaxation o sphincters Urinary bladder Contraction o detrusor muscle; relaxation o sphincter Sweat glands Diaphoresis Reproductive tract, male Erection Uterus Variabl
What is it called if inflammatory lesions occur in blood vessels? renal glomeruli? joints?
Vasculitis Glomerulonephritis Arthritis
What do FIbrates displace from albumin binding sites? what does that result in?
Warfarin result in increased free Warfarin concentrations Therefore, the response to warfarin should be monitored when a fibrate is co-administered
What are the major contributors to liporprotein abnormalities ?
Western diets combined with sedentary lifestyles but a limited number of genetic causes of hyperlipidemia have also been identified.
delivery of HDL cholesterol to the liver
When mature HDL particles circulate to the liver, they interact with SR-BI, the principal HDL receptor (Fig. 20-9A). SR-BI is highly expressed on the sinusoidal plasma membranes o hepatocytes. In contrast to its action on most nonhepatic cells, where SR-BI mediates e f ux o excess cholesterol rom the membrane, SR-BI in the liver promotes selective uptake o lipids. In this process, the cholesterol and cholesteryl esters o HDL particles are taken up into the hepatocyte in the absence o uptake o apolipoproteins. During SR-BI-mediated selective lipid uptake, apoAI is liberated to participate in pre- -HDL ormation. The "li espan" o an HDL particle is 2-5 days, suggesting that each apoAI molecule can participate in many cycles o reverse cholesterol transport. Among the nonhepatic tissues that express high levels o SR-BI are the adrenal glands and gonads, presumably re ecting the requirement o these organs or cholesterol to support steroidogenesis. Delivery o cholesterol rom extrahepatic tissues to the liver is optimized by two additional proteins: cholesterol ester transfer protein (CETP) and hepatic lipase. CETP is a plasma protein that trans ers cholesteryl esters rom mature spherical HDL to the cores o remnant lipoproteins in exchange or a triglyceride molecule, which is inserted into the core o the HDL particle (Fig. 20-9B). This process allows the body to utilize remnant particles that have completed their unction o triglyceride transport or purposes o transporting cholesterol to the liver. Removal o cholesteryl ester molecules rom HDL appears to serve two unctions. First, it urther increases the capacity o HDL to take on additional cholesterol molecules rom cells. Second, it makes the process o selective uptake by SR-BI more e f cient. This is because hydrolysis o triglycerides by hepatic lipase on the hepatocyte sur ace acilitates the activity o SR-BI (Fig. 20-9A). As noted above, reverse cholesterol transport is the overall process by which HDL removes cholesterol rom macrophages and other extrahepatic tissues and returns it to the liver. The concept that increased plasma concentrations o HDL cholesterol may re ect increased rates o reverse cholesterol transport provides a possible explanation or the inverse relationship between plasma HDL levels and risk o cardiovascular disease. HDL particles also exert direct benef cial e ects on vascular tissue, including enhancement o antioxidant enzyme activities that inhibit oxidation o LDL. HDL also inhibits the expression o in ammatory mediators (e.g., intercellular adhesion molecule [ICAM] and vascular cell adhesion molecule [VCAM]) by vascular cells. Increased understanding o HDL metabolism may lead to the development o novel biochemical targets or increasing reverse cholesterol transport in order to slow or even reverse the progression o atherosclerosis
What is the most common cause of Chronic granulomatous disease? What can patients with CGD develop?
X linked mutation Pneumonia, lymph node infections, and abscesses in the skin, liver and other sites
All sex linked disorders are ...... and almost all are recessive?
X-linked Several genes are located in the "male-specific region of Y"; all of these are related to spermatogenesis. Males with mutations affecting the Y-linked genes are usually infertile, and hence there is no Y-linked inheritance
Are the rates of contagious diseases similar in MZ and DZ twins?
Yes because most contagious diseases are unlikely to be influenced markedly by genes on the other hand, the concordance rates for Schizophrenia are quite dissimilar between MZ and DZ twins , indicating a sizeable genetic component for this disease The MZ correlation for dermatoglyphics (fingerprints), a series of traits determined almost entirely by genes, is close to 1.0.
What does Electrodiagnostic testing with 3 Hz repetitive nerve stimulations (RNS) demonstrate in Myasthenia Gravis?
a compound muscle action potential (CMAP) decrement more than 10% in about 50% to 75% of patients with generalized MG but is abnormal in less than 50% of patients with purely ocular symptoms
In familial hypercholesterolemia waht does impaired IDL transport into the liver divert?
a greater proportion of plasma IDL into the precursor pool for plasma LDL
What occurs in DiGeorge sequence?
a lack of thymic development leads to T cell deficiency
What does treatment for Familial hypercholesterolemia require?
a low-fat (<20% of total calories), low cholesterol (<100 mg/day) diet in combination with drug therapy
A 16-year-old male presents to the clinic with a chief complaint of rhinitis, sneezing, difficulty breathing when he eats shrimp. He states he always ate shrimp in the past without any complications. Other than that, there are no issues. Based on his symptoms and history he was diagnosed with having allergies to shrimp. What immune mechanisms are responsible for his presentation? a) Th2 cells, IgE antibodies, mast cells, and eosinophils b) Th2 cells, IgG, and macrophages c) Complement proteins, IgG and mast cells d) CD4 T cells e) IgM or IgG immune complexes
a) Th2 cells, IgE antibodies, mast cells, and eosinophils
What is Ataxia-telangiectasia? What is caused by?
an autosomal recessive disorder that includes breast cancer in its presentation caused by defective DNA repair
What occurs in Graves disease?
antibodies against the receptor for thyroid stimulating hormone stimulate thyroid cells even in the absence of the hormone.
What does Mipomersen bind to ? what does it inhibit production of?
apo B messenger RNA inhibits apo B production Apo B is a structural component of VLDL, IDL, and LDL Treatment reduces LDL by up to 50%.
What is defective in Familial Defective Apolipoprotein B100? What does that result in?
apo B100 Impaired binding of LDL particles to the LDL receptor this disorder affects as many as 1 in 750 Caucasians with hypercholesterolemia
What are the 2 main apolipoproteins of HDL ?
apoAI and apoAII
A patient has recently been put on lovastatin to help manage their high LDL levels. The doctor states that it also has a secondary prevention that it helps with. What is the secondary prevention that the physician is mentioning? a) An overall reduction in Inflammation in the body. b) Reduce mortality after a myocardial infarction. c) A reversal of endothelial dysfunction d) A decrease in thrombosis. e) An improvement in the stability of atherosclerotic plaques.
b) Reduce mortality after a myocardial infarction.
Adam is a 20-year-old male who presents to his primary care physician with back pain that causes him to be in a hunched over position. He is referred to an orthopedic physician who diagnoses him with ankylosing spondylitis. Adam then meets with a geneticist to determine if he has a genetic predisposition to the condition. Which gene is Adam most likely to carry which causes his condition? a. HLA-DQB1 b. HLA-B27 c. HLA-A3 d. HLA-A e. HLA-DQA1
b. HLA-B27
A patient presents to his physician for an annual checkup and mentioned a fatty, yellowish swelling on the palmer crease of his hand. The patient admits to being a heavy alcohol drinker. Lab work came back and showed elevated cholesterol levels (350 mg/dL) and elevated triglyceride levels (360 mg/dL). The physician suspects familial dysbetalipoproteinemia, but to confirm his assumption, what mutation does he expect to see in the genetic testing? a) LDL receptor mutation b) Apo B100 defect c) Apo E2 homozygosity d) LPL mutation e) Apo C-II mutation
c) Apo E2 homozygosity
What is the MOA of Organophosphates?- what do they inhibit and cause accumulation of? What effects do they produce throughout the nervous system?
cause inhibition of acetylcholinesterase (AChE) accumulation of acetylcholine at the cholinergic receptor sites producing continuous stimulation of cholinergic fibers
What are mast cells coated with in atopic individuals?
coated with IgE antibody specific for the antigen(s) to which the individual is allergic
Which of the following is NOT true regarding multifactorial inheritance? a) The recurrence risk is higher if the proband is of the less commonly affected sex b) The recurrence risk is higher if more than one family member is affected c) The recurrent risk for the disease usually decreases rapidly in more remotely related relatives d) If the expression of the disease in the proband is less severe, the recurrent risk is higher e) All of the above are true
d) If the expression of the disease in the proband is less severe, the recurrent risk is higher
What is Dysbetaliproteinemia? What is it caused by? What isoform is implicated?
disorder characterized by increased cholesterol-rich chylomicrons and increased IDL-like particles are the result of accumulated chylomicron and VLDL remnants, leading to both hypertriglyceridemia and hypercholesterolemia apoE2 isoform has been implicated in this disease - symptoms generally present in adult males and in postmenopausal females Dysbetalipoproteinemia can be managed by decreased intake of fat and cholesterol, along with weight reduction and omission of alcohol intake In addition, niacin and fibrates are effective pharmacologic therapies.
Which of the following muscarinic receptor antagonist drugs would be administered to prevent sea sickness? a) Tiotropium b) Atropine c) Methscopolamine d) Ipratropium e) Scopolamine
e) Scopolamine
What do etiologies of Mixed hyperlipidemia include?
mixed hyperlipidemia include: 1-Familial combined hyperlipidemia (FCHL) 2- Dysbetalipoproteinemia 3-Lysosomal acid lipase deficiency (LAL-D).
What occurs as a result of gain of function mutation of PCSK9? loss of function mutations?
gain of function: marked elevations in LDL-C loss of function: reduction in LDL-C
What are hypersensitivity reactions?
immune responses that are exaggerated or inappropriate against an antigen or allergen are injurious or pathologic in nature
What occurs in Bare lymphocyte syndrome?
immune system defects that results in lymphocytes that lack MHC molecules on their surfaces one form of these are caused by mutations in TAP2 genes
What conditions are associated with about 10% of Autism patients?
include Fragile X syndrome (1-2% of cases), Rett syndrome (0.5%), tuberous sclerosis (1%), and neurofibromatosis type 1 (<1%).
What does Lomitapide inhibit? What does it inhibit? What is a side effect?
inhibits microsomal triglyceride transfer protein in the liver decreases apo B. Significant reductions in LDL (up to 50%) are seen with treatment. Liver toxicity is a serious adverse event associated with this agent.
What are lipids?
insoluble or sparingly soluble molecules that are essential or membrane biogenesis and maintenance of membrane integrity They also serve as energy sources, hormone precursors, and signaling molecules
immune response proteins- genetic basis of structure and diversity immunoglobulin molecules and genes
lin) molecule is composed of four chains: an identical pair of longer heavy chains and an identical pair of shorter light chains, which are linked together by disulfide bonds. There are five different types of heavy chains (termed γ, µ, α, δ, and ε) and two types of light chains (κ and λ). The five types of heavy chains determine the major class (or isotype) to which an immunoglobulin (Ig) molecule belongs: γ, µ, α, δ, and ε correspond to the immunoglobulin isotypes IgG, IgM, IgA, IgD, and IgE, respectively. Immature B lymphocytes produce only IgM, but as they mature, a rearrangement of heavy chain genes called class switching occurs. This produces the other four major classes of immunoglobulins, each of which differs in amino acid composition, charge, size, and carbohydrate content. Each class tends to be localized in certain parts of the body, and each tends to respond to a different type of infection. The two types of light chains can be found in association with any of the five types of heavy chains. The heavy and light chains both contain a constant and a variable region, which are located at the carboxyl (C)-terminal and amino (N)-terminal ends of the chains, respectively. The arrangement of genes encoding the constant region determines the major class of the Ig molecule (e.g., IgA, IgE). The variable region is responsible for antigen recognition and binding and thus varies within immunoglobulin classes. Three distinct gene segments encode the light chains: C for the constant region, V for the variable region, and J for the region that joins the constant and variable regions. Four gene segments encode the heavy chains, with C, V, and J coding again for the constant, variable, and joining regions, respectively and a diversity (D) region located between the joining and variable regions Immunoglobulin molecules consist of two identical heavy chains and two identical light chains. The heavy chain constant region determines the major class to which an immunoglobulin belongs. The variable regions of the light and heavy chains recognize and bind antigens
What occurs as a consequence of receptor abnormality in familial hypercholesterolemia?
loss of feedback control and elevated levels of cholesterol that induce premature atherosclerosis, leading to a greatly increased risk of myocardial infarction.
What are the various CNS functions of ACh?
modulation of sleep, wakefulness, learning, and memory; suppression of pain at the spinal cord level; and essential roles in neural plasticity, early neural development, immunosuppression, and epilepsy.
Where is LDL receptor expressed?
on the surface of hepatocytes, macrophages, lymphocytes, adrenocortical cells, gonadal cells, and smooth muscle cells
What is the function of CETP?
plasma protein that transfers cholesteryl esters from mature spherical HDL to the cores of remnant lipoproteins in exchange for a triglyceride molecule, which is inserted into the core of the HDL particle This process allows the body to utilize remnant particles that have completed their unction of triglyceride transport or purposes of transporting cholesterol to the liver.
During SR-BI mediated selective lipid uptake, apoAI is liberated to participate in what?
pre-beta HDL formation
What is the mechanism of triglyceride lowering in Omega-3 fatty acids?
regulation of nuclear transcription factors, including SREBP-1c and PPAR alpha to cause reduced triglyceride biosynthesis and increased fatty acid oxidation in the liver.
If the prevalence of a disease in a population if f (which varies between zero and one), appx what is the risk for offspring and siblings of probands?
sq root of f This does not hold true for single-gene traits, because their recurrence risks are independent of population prevalence. It is not an absolute rule for multifactorial traits either, but many such diseases do tend to conform to this prediction. Examination of the risks given in Table 12-2 shows that these three diseases follow the prediction fairly well.
What is Serum sickness induced by? What does it lead to the formation of?
systemic administration of a protein antigen, which elicits an antibody response leads to the formation of circulating immune complexes
Dysfunction of the NNCS may contribute to the pathogenesis of disease of several organ systems including:
the skin (e.g., atopic dermatitis, pemphigus, psoriasis, vitiligo) the urinary (e.g., overactive bladder syndrome), gastrointestinal (e.g., gastroesophageal reflux disease, peptic ulcer disease, pancreatitis), immune (e.g., Sjogren's syndrome, rheumatoid arthritis, sepsis), pulmonary (e.g., asthma, chronic obstructive pulmonary disease, cystic fibrosis), musculoskeletal (e.g., osteoporosis, tendinosis), reproductive (e.g., sperm dysmotility), cardiovascular systems (e.g., atherosclerosis).
What are leukocytes activated by the engagement of? What does it result in?
their C3b and Fc receptors This results in the production of other substances that damage tissues, such as lysosomal enzymes, including proteases capable of digesting basement membrane, collagen, elastin, and cartilage, and reactive oxygen species.
What is the function of PLTP?
transfers phospholipids from the surface coat of apoB-containing remnant particles to the surface coat of HDL. By transferring phospholipids to the surface of HDL, PLTP also replaces the molecules that are consumed by the LCAT reaction. This allows the core of HDL to continue to enlarge
What is the LDL receptor involved in?
transport and metabolism of cholesterol
What occurs in Huntington's disease?
trinucleotide-repeat mutation affecting the Huntington gene gives rise to an abnormal protein, called huntingtin Huntingtin is toxic to neurons, and hence even heterozygotes develop a neurologic deficit
What happens Familial hypercholesterolemia remains untreated in homozygous individuals? Heterozygotes?
typically die of myocardial infarction before 20 years of age. The heterozygous form of familial hypercholesterolemia affects 1 in every 500 individuals, with the partial receptor defect resulting in cells that display half the normal number of fully functional LDL receptors Heterozygotes have less strongly elevated concentrations of total cholesterol (>300 to 600 mg/dL) and LDL-cholesterol (250 to 500 mg/dL) than do those with the homozygous form
What occurs in MODY (maturity onset diabetes of the young)?
typically occurs before 25 years of age and follows an autosomal dominant mode of inheritance. In contrast to type 2 diabetes, it is not associated with obesity accounts for 1% to 5% of all diabetes cases
What occurs in Reverse cholesterol transport?
uptake of cholesterol from peripheral cells to HDL and from HDL to liver for bile acid/sat syn and excretion in feces via bile to maintain chol homeostasis HDL plays a key role in cholesterol homeostasis by removing excess cholesterol from cells and transporting it in plasma to the liver.
What are the major causes of T cell mediated hypersensitivity reactions?
1- Autoimmunity 2- Exaggerated or persistent responses to environmental antigens
What 3 areas does Muscarinic cholinergic transmission occur mainly?
1- Autonomic ganglia 2- End organs innervated by the parasympathetic division of the ANS 3- CNS
What are the 3 chemical classes of AChE inhibitors?
(1) Simple alcohols with a quaternary ammonium group (Edrophonium) (2) Carbamic acid esters of alcohols bearing either quaternary or tertiary ammonium groups (neostagmine, physostigmine) (3) Organic derivatives of phosphoric acid (eg: Isoflurophate)
Under what conditions does Antibody-mediated cell destruction and phagocytosis occur?
(1) Transfusion reactions, in which cells from an incompatible donor react with and are opsonized by preformed antibody in the host (Chapter 14); (2) Hemolytic disease of the newborn (erythroblastosis fetalis), in which there is an antigenic difference between the mother and the fetus, and IgG anti erythrocyte antibodies from the mother cross the placenta and cause destruction of fetal red cells (Chapter 10) (3) Autoimmune hemolytic anemia, agranulocytosis, and thrombocytopenia, in which individuals produce antibodies to their own blood cells, which are then destroyed (Chapter 14); and (4) certain drug reactions, in which a drug acts as a "hapten" by attaching to plasma membrane proteins of red cells and antibodies are produced against the drug-protein complex.
What 4 categories can the mechanisms involved in single-gene disorders be classified into?
(1) enzyme defects and their consequences (2) defects in membrane receptors and transport systems (3) alterations in the structure, function, or quantity of nonenzyme proteins (4) mutations resulting in unusual reactions to drugs.
What are Cholinergic and anticholinergic drugs primarily used for?
(1) modulation of gastrointestinal motility, (2) xerostomia (dry mouth), (3) glaucoma, (4) motion sickness and antiemesis, (5) neuromuscular diseases such as myasthenia gravis and Eaton-Lambert syndrome, (6) acute neuromuscular blockade and reversal during surgery, (7) ganglionic blockade during aortic dissection, (8) dystonias (e.g., torticollis), headache, and pain syndromes, (9) reversal of vagal-mediated bradycardia, (10) mydriasis, (11) bronchodilation in chronic obstructive pulmonary disease, (12) bladder spasms and urinary incontinence, (13) cosmetic effects on skin lines and wrinkles, and (14) treatment o Alzheimer's disease and other neurodegenerative dementias.
cardiovascular diseases - Heart disease
. Heart disease is the leading cause of death worldwide, and it accounts for approximately 25% of all deaths in the United States. The most common underlying cause of heart disease is coronary artery disease (CAD), which is caused by atherosclerosis (a narrowing of the coronary arteries resulting from the formation of lipid-laden lesions). This narrowing impedes blood flow to the heart and can eventually result in a myocardial infarction (death of heart tissue caused by an inadequate supply of oxygen). When atherosclerosis occurs in arteries that supply blood to the brain, a stroke can result. A number of risk factors for CAD have been identified, including obesity, cigarette smoking, hypertension, elevated cholesterol level, and positive family history (usually defined as having one or more affected firstdegree relatives). Many studies have examined the role of family history in CAD, and they show that a person with a positive family history is at least twice as likely to suffer from CAD than is a person with no family history. Generally, these studies also show that the risk is higher if there are more affected relatives, if the affected relative is female (the less commonly affected sex) rather than male, and if the age of onset in the affected relative is early (before 55 years of age). For example, one study showed that men between the ages of 20 and 39 years had a threefold increase in CAD risk if they had one affected first-degree relative. This risk increased to 13-fold if there were two first-degree relatives affected with CAD before 55 years of age. What part do genes play in the familial clustering of CAD? Because of the key role of lipids in atherosclerosis, many studies have focused on the genetic determination of variation in circulating lipoprotein levels. An important advance was the identification of the gene that encodes the low-density lipoprotein (LDL) receptor. Heterozygosity for a mutation in this gene roughly doubles LDL cholesterol levels and is seen in approximately 1 in 500 persons. (This condition, known as familial hypercholesterolemia, is described further in Clinical Commentary 12-2.) Mutations in the gene encoding apolipoprotein B, which are seen in about 1 in 1000 persons, are another genetic cause of elevated LDL cholesterol. These mutations occur in the portion of the gene that is responsible for binding of apolipoprotein B to the LDL receptor, and they increase circulating LDL cholesterol levels by 50% to 100%. Dozens of other genes involved in lipid metabolism and transport have been identified, including those genes that encode various apolipoproteins (these are the protein components of lipoproteins) (Table 12-5). In addition, several genes whose protein products contribute to inflammation have been associated with CAD, reflecting the critical role of inflammation in generating atherosclerotic plaques. Functional analysis of these genes is leading to increased understanding and more effective treatment of CAD. Environmental factors, many of which are easily modified, are also important causes of CAD. There is abundant epidemiological evidence that cigarette smoking and obesity increase the risk of CAD, whereas exercise and a diet low in saturated fats decrease the risk. Indeed, the approximate 60% reduction in age-adjusted mortality due to CAD and stroke in the United States since 1950 is usually attributed to a decrease in the percentage of adults who smoke cigarettes, decreased consumption of saturated fats, improved medical care, and increased emphasis on healthy lifestyle factors such as exercise. Another form of heart disease is cardiomyopathy, an abnormality of the heart muscle that leads to inadequate cardiac function. Cardiomyopathy is a common cause of heart failure, resulting in approximately 10,000 deaths annually in the United States. Nearly 100 different genes have been linked to cardiomyopathy. Hypertrophic cardiomyopathy, one major form of the disease, is characterized by thickening (hypertrophy) of portions of the left ventricle and is seen in as many as 1 in 500 adults. About half of hypertrophic cardiomyopathy cases are familial and are caused by autosomal dominant mutations in any of the multiple genes that encode various components of the cardiac sarcomere. The most commonly mutated genes are those that encode the β-myosin heavy chain (35% of familial cases), myosin-binding protein C (20% of cases), and troponin T (15% of cases). In contrast to the hypertrophic form of cardiomyopathy, dilated cardiomyopathy, which is seen in about 1 in 2500 persons, consists of increased size and impaired contraction of the ventricles. The end result is impaired pumping of the heart. This disease is familial in about one third of affected persons; although autosomal dominant mutations are most common, mutations can also be X-linked or mitochondrial. The most commonly mutated gene encodes titin, a cytoskeletal protein. Other genes that can cause dilated cardiomyopathy encode other cytoskeletal proteins, including actin, cardiac troponin T, desmin, and components of the dystroglycan-sarcoglycan complex. (Recall from Chapter 5 that abnormalities of the latter proteins can also cause muscular dystrophies.) Mutations have also been identified in more than a dozen genes that cause the long QT (LQT) syndrome. LQT, which is seen in approximately 1 in 2000 persons, describes the characteristically elongated QT interval in the electrocardiogram of affected individuals, indicative of delayed cardiac repolarization. This disorder, which can be caused either by inherited mutations or by exposure to drugs that block potassium channels, predisposes affected persons to potentially fatal cardiac arrhythmia. An autosomal dominant form, known as Romano-Ward syndrome, can be caused by lossof-function mutations in genes that encode potassium channels (such as KCNQ1, KCNH2, KCNE1, KCNE2, or KCNJ2). These mutations delay cardiac repolarization. Gain-of-function mutations in several of these same genes have been shown to produce a shortened QT interval, as might be expected. Romano-Ward syndrome can also be caused by gain-of-function mutations in sodium or calcium channel genes (SCN5A and CACNA1C, respectively), which result in a prolonged depolarizing current. (Other examples of mutations that can cause LQT are given in Table 12-6.) An autosomal recessive form of LQT syndrome, known as Jervell-Lange-Nielsen syndrome, is less common than the Romano-Ward syndrome but is associated with a longer QT interval, a higher incidence of sudden cardiac death, and sensorineural deafness. This syndrome is caused by mutations in either KCNQ1 or KCNE1. Because LQT syndrome can be difficult to diagnose clinically, genetic testing enables more accurate diagnosis of affected persons. In addition, the identification of disease-causing genes and their protein products is now guiding the development of drug therapy to activate the encoded ion channels. Because cardiac arrhythmias account for most of the 300,000 sudden cardiac deaths that occur annually in the United States, a better understanding of the genetic defects underlying arrhythmia is of considerable public health significance. Heart disease aggregates in families. This aggregation is especially strong if there is early age of onset and if there are several affected relatives. Specific genes have been identified for some subsets of families with heart disease, and lifestyle changes (exercise, diet, avoidance of tobacco) can modify heart disease risks appreciably.
Stroke
. Stroke, which refers to brain damage caused by a sudden and sustained loss of blood flow to the brain, can result from arterial obstruction (ischemic stroke, which accounts for 80% of stroke cases) or breakage (hemorrhagic stroke). This disease is the fourth leading cause of mortality in the United States, accounting for approximately 140,000 deaths per year. As with heart disease, strokes cluster in families: One's risk of having a stroke increases by two- to threefold if a parent has had a stroke. The largest twin study conducted to date showed that concordance rates for stroke death in MZ and DZ twins were 10% and 5%, respectively. These figures imply that genes might influence one's susceptibility to this disease. Stroke can be caused by any of more than a dozen singlegene disorders, including sickle cell disease (see Chapter 3), MELAS (mitochondrial myopathy, encephalopathy, lactic acidosis, and stroke, a mitochondrial disorder discussed in Chapter 5), and cerebral autosomal dominant arteriopathy with subcortical infarcts and leukoencephalopathy (CADASIL, a condition characterized by recurrent strokes and dementia and caused by mutations in the NOTCH3 gene). Because blood clots are a common cause of stroke, it is expected that mutations in genes that encode coagulation factors might affect stroke susceptibility. For example, inherited deficiencies of protein C and protein S, both of which are coagulation inhibitors, are associated with an increased risk of stroke, especially in children. A specific mutation in clotting factor V, the factor V Leiden allele, causes resistance to activated protein C and thus produces an increased susceptibility to clotting. Heterozygosity for this allele, which is seen in approximately 5% of whites, produces a sevenfold increase in the risk of venous thrombosis (clots). In homozygotes, the risk increases to 100-fold. However, the evidence for an association between the factor V Leiden allele and stroke is inconsistent. In addition to family history and specific genes, several factors are known to increase the risk of stroke. These include hypertension, obesity, atherosclerosis, diabetes, and smoking. Stroke, which clusters in families, is associated with several singlegene disorders and with some inherited coagulation disorders
What are the 3 patterns of inheritance that mutations involving single genes typically follow?
1- Autosomal dominant 2- Autosomal recessive 3- X-linked
What are the reasons that depolarization mediated by nicotinic acetylcholine receptors (nAChRs) are brief?
1- ACh degrades rapidly from active-state receptor molecules 2-Acetylcholinesterase - both rapidly degrade free unbound Ach in the synaptic cleft
What do Cholinergic synaptic vesicles contain besides Ach that act as counter ions for Ach?
1- ATP 2- Heparin Sulfate proteoglycans By neutralizing the positive charge of ACh, these molecules disperse electrostatic forces that would otherwise prevent dense packing of ACh within the vesicle. (Released ATP also acts as a neurotransmitter, through purinergic receptors, to inhibit the release of ACh and norepinephrine from autonomic nerve endings.)
What are the 3 sources of Choline for ACh synthesis? Source for acetyl coA?
1- Acetylcholinesterase (35-50%) 2- Phosphatidylcholine- Plasma based stores of Choline 3- Phosphorylcholine- choline stored as phospholipids mainly derived from Glycolysis, produced via the enzyme pyruvate dehydrogenase
What is the sequence of events in the development of immediate hypersensitivity reaction?
1- Activation of Th2 and IL-4- secreting follicular helper T (Tfh) cells 2- They stimulate the production of IgE antibodies in response to an antigen, binding of the IgE to specific Fc receptors of mast cells 3- IgE then bind to antigens, crosslinking the bound IgE by the antigen and then release mast cell mediators
What does the therapy for FCH involve?
1- Affected individuals require a low-fat, low-cholesterol diet plus multiple lipid-lowering drugs to achieve target goals. 2- Fibric acid derivatives, which hydrolyze the triglyceride core of VLDL particles and increase LDL production, are recommended for treatment of the hypertriglyceridemia. 3- Patients with FCHL often additionally require a statin or niacin to lower their LDLcholesterol level.
What are the risk factors for Coronary heart disease (CHD)?
1- Age - men > 55 years and women > 45 years 2- Family history of premature CHD (affected male first degree relatives <55 years or female first degree-relative <65 years of age) 3- Smoking 4- Hypertension 5- Low HDL (<40 mg/dL) HDL concentrations higher than 60 mg/dL are cardioprotective. Overall, the level of evidence to support screening is fair (level B) but increases with age, male gender, and CHD (level A).
What are characteristics of X-linked disorders?
1- An affected male does not transmit the disorder to his sons, but all daughters are carriers 2- Sons of heterozygous women have one chance in two of receiving the mutant gene 3- The heterozygous female usually does not express the full phenotypic change because of the paired normal allele
What are the common drugs that treat immediate type hypersensitivity?- for Hay fever? Asthma?Anaphylaxis?
1- Anti-histamines for Hay fever 2- Agents that relax bronchial smooth muscle in asthma 3- Epinephrine in anaphylaxis
What are the recommended treatment options for Apolipoprotein C-II deficiency?
1- Appropriate management of secondary factors such as diabetes and hypothyroidism, 2- dietary fat restriction (<10% of calories) 3- Drug therapy (e.g., fibric acid derivatives). For severe hypertriglyceridemia, plasma transfusion (with apo C-II) can be considered.
What are the steps involved in the assembly and secretion of apoB containing lipoproteins?
1- As the apoB protein is synthesized by ribosomes, it crosses into the endoplasmic reticulum 2- Within the endoplasmic reticulum, triglyceride molecules are added co-translationally to the elongating apoB protein (i.e., apoB is lipidated) by the action of a cofactor protein, MTP 3- Once apoB has been fully synthesized, the nascent lipoprotein is enlarged in the Golgi apparatus; during this process, MTP adds additional triglycerides to the core of the particle 4- By unclear mechanisms, cholesteryl esters are also added to the core. 5- Each lipoprotein particle assembled by this process contains a single molecule of apoB
What 3 phases can the metabolic lifespan of apoB-containing lipoproteins be divided into?
1- Assembly 2- Intravascular metabolism 3- Receptor-mediated clearance
What are Muscarinic receptor agonists used clinically in the diagnosis of? What are they divided structurally into?
1- Asthma 2- Mitotics (agents that cause pupillary constriction) 1- Choline esters - inefficently distributed to CNS, poorly absorbed, eg: Acetylcholine, Methacholine, Bethanechol 2- Alkaloids
What are the different types of NMJ diseases?examples of each?
1- Autoimmune eg: Myasthenia gravis, Lambert Eaton syndrome 2- Hereditary eg: Congenital myasthenic syndromes 3- Toxic eg: Botulism, Organophosphate poisoning
What does the assembly of ApoB-containing lipoproteins depend on?
1- Availability of apoB and triglycerides 2- Activity of microsomal triglyceride-transfer protein (MTP) gene that encodes apoB is transcribed principally in the intestine and the liver.
What 2 drugs are used for long term treatment of MG?
1- Azathioprine 2- Myocophenolate also used as steroid-sparing agents
What are the ocular changes that occur in Marfan syndrome?
1- Bilateral subluxation or dislocation (usually outward and upward) of the lens, referred to as ectopia lentis.- most characteristic change
What are the steps involved in the formation of bile?
1- Bile acids are pumped into bile by the action of a canalicular membrane transport pump known as ABCB11 2- These bile acids stimulate the biliary secretion of phospholipids and cholesterol - Phospholipid and cholesterol secretion are mediated by two additional transporters: ABCB4 for phospholipids and a heterodimer of ABCG5 and ABCG8 for cholesterol 3- Large amounts of bile acids, phospholipids, and cholesterol are secreted into bile at approximate rates of 24, 11, and 1.2 grams each day, respectively. These molecules comprise the biliary lipids, which are stored in the gallbladder during fasting.
What are the steps involved in the clearance of LDL from the liver?
1- Binding of LDL to cell surface receptors, which are clustered in specialized regions of the plasma membrane called coated pits 2- After binding, the coated pits containing the receptor-bound LDL are internalized by invagination to form coated vesicles, after which they migrate within the cell to fuse with the lysosomes 3- There the LDL dissociates from the receptor, which is recycled to the surface 4- In the lysosomes the LDL molecule is enzymatically degraded; the apoprotein part is hydrolyzed to amino acids, whereas the cholesteryl esters are broken down to free cholesterol 5- This free cholesterol, in turn, crosses the lysosomal membrane to enter the cytoplasm, where it is used for membrane synthesis and as a regulator of cholesterol homeostasis.
In T cell mediated disease tissue injury is caused by inflammation induced by cytokines that are produced by mainly?
1- CD4 T cells 2- CD8 T cells
What 2 proteins is delivery of cholesterol from extrahepatic tisuses to the liver optimized by?
1- CETP (cholesterol ester transfer protein) 2- Hepatic lipase
What are the steps involved in the CTL mediated destruction of targets by Perforin and Granzyme?
1- CTLs (cyotoxic T cells-CD8) that recognize the target cells secrete a complex consisting of perforin, granzymes, and other proteins which enters target cells by endocytosis 2- In the target cell cytoplasm, perforin facilitates the release of the granzymes from the complex 3- Granzymes are proteases that cleave and activate caspases, which induce apoptosis of the target cells Activated CTLs also express Fas ligand, a molecule with homology to TNF, which can bind to Fas expressed on target cells and trigger apoptosis.
What are the characteristics of Type I Alcoholics?
1- Characterized by later onset (after 25 years of age) 2- Occurrence in both males and females, and greater psychological dependency on alcohol 3- Are more likely to be introverted, solitary drinkers 4- Type I is less likely to cluster in families
What does the hydrophobic core of a lipoprotein contain?
1- Cholesterol esters (cholesterol molecules linked by an ester bound to a fatty acid) 2- Triglycerides (three fatty acids esterified to a glycerol molecule)
How does Cholesterol exhibit negative feedback?
1- Cholesterol suppresses cholesterol synthesis within the cell by inhibiting the activity of the enzyme 3-hydroxy3-methylglutaryl coenzyme A (HMG CoA) reductase, which is the rate limiting step in the synthetic pathway 2- Cholesterol activates the enzyme acyl-coenzyme A: cholesterol acyltransferase, favoring esterification and storage of excess cholesterol 3- Cholesterol suppresses the synthesis of LDL receptors, thus protecting the cells from excessive accumulation of cholesterol.
What are 3 bile acid sequestrant drugs?
1- Cholestyramine 2- Colesevelam 3- Colestipol these drugs possess similar efficacy, causing up to 28% reductions in LDL levels at therapeutic concentrations drugs possess similar efficacy, causing up to 28% reductions in LDL levels at therapeutic concentrations.
What are the 5 classes of mutations involving LDL receptors?
1- Class I mutations are relatively uncommon and lead to a complete failure of synthesis of the receptor protein (null allele) 2- Class II mutations are fairly common; they encode receptor proteins that accumulate in the endoplasmic reticulum because their folding defects make it impossible for them to be transported to the Golgi complex 3- Class III mutations affect the LDL-binding domain of the receptor; the encoded proteins reach the cell surface but fail to bind LDL or do so poorly 4- Class IV mutations encode proteins that are synthesized and transported to the cell surface efficiently They bind LDL normally, but they fail to localize in coated pits, and hence the bound LDL is not internalized 5- Class V mutations encode proteins that are expressed on the cell surface, can bind LDL, and can be internalized however, the pH-dependent dissociation of the receptor and the bound LDL fails to occur. Such receptors are trapped in the endosome, where they are degraded, and hence they fail to recycle to the cell surface.
What is the diagnosis of MG based on the combination of?
1- Clinical history 2- Physical examination 3- Confirmatory tests
What 2 regions do both heavy and light chains contain?
1- Constant region 2- Variable region located at the Carboxyl and amino terminal respectively
What is evidence for decreased thrombosis with statin therapy?
1- Decrease in prothrombin activation 2-Decrease in tissue factor production
What does activation of Fibrates by PPAR alpha result in?
1- Decrease plasma triglyceride levels 2- Increase plasma HDL - The decrease in plasma triglyceride levels is caused in part by increased muscle expression of lipoprotein lipase, decreased hepatic expression of apolipoprotein CIII, and increased hepatic oxidation of fatty acids the decrease in hepatic production of apoCIII may potentiate the increased LPL activity.
What is the effect of Statins on triglyceride levels mediated by?
1- Decreased VLDL production 2- Increased clearance of remnant lipoproteins by the liver
What are some of the effects of Statins besides lowering cholesterol concentration (pleiotropic effects)?
1- Decreased inflammation 2- Reversal of endothelial dysfunction 3- Decreased thrombosis 4- Improved stability of atherosclerotic plaques
What 3 types of responses occur following the binding of IgE to mast cells and crosslinking of IgE receptors?
1- Degranulation - rapid release of granule contents 2- Synthesis and secretion of lipid mediators 3- Synthesis and secretion of cytokines
What are the 2 stores of Ach and roles do they play in Ach release?
1- Depot pool - consists of vesicles positioned near the plasma membrane of the axon terminal and axonal depolarization causes these vesicles to release Ach rapidly 2- Reserve pool - serves to refill the depot pool as it is being used , is required to sustain ACh release for a longer period of time
What factors exacerbate Familial hypertriglyceridemia by increasing VLDL? What is low HDL associated with in familial hypertriglyceridemia?
1- Diabetes 2- Alcohol 3- Estrogen therapy Increased catabolism
What 2 tests help differentiate between Myasthenia gravis, Lambert Eaton myasthenic syndrome (LEMS) and Guillan-Barre syndrome (GBS)?
1- Electrodiagnostic test 2- Serum anti-body tests
What is isolated Hypercholesterolemia characterized by?
1- Elevated levels of total plasma cholesterol and LDL cholesterol, 2-Normal concentrations of triglycerides
What are some side effects associated with Statin use?
1- Elevated liver enzymes and muscle toxicity are potential dose-related complications. 2- Myositis can occur with statins alone, but the risk is higher when statins are used in combination with nicotinic acid or fibric acid derivatives some of these agents can also potentiate the effect of warfarin
What are the principal leukocytes involved in late phase reactions?
1- Eosinophils 2- Neutrophils 3- Th2 cells
What are the 2 process responsible for Choline transport?
1- Facilitated diffusion (low affinity, Km=10-100) 2- Sodium (Na) dependent high affinity transport system (Km= 1-5 micrometers)
What are the causes of primary hypercholesterolemia?
1- Familial hypercholesterolemia, 2- Familial defective apoB100 3- Gain-of -function mutations in PCSK9 4- Familial combined hyperlipidemia (FCHL) 5-Polygenic hypercholesterolemia (most common)
What are the 3 major etiologies of hypertriglyceridemia?
1- Familial hypertriglyceridemia 2- Familial lipoprotein lipase (LPL) deficiency 3- apoCII deficiency
What 3 things are required for Dyslipidemia assesment?
1- Fasting total cholesterol 2- Fasting triglyceride 3- Lipoprotein assessments It is advisable to confirm dyslipidemia with two separate determinations.
What agents should be considered if the triglyceride level is greater than 500 mg/dL?
1- Fibrates 2- Fish oil 3- Nicotinic acid for levels lower than 500 mg/dL, statins are first-line therapy (level B).
What are the side effects associated with Mipomersen use?
1- Flu like symptoms 2- Injection site reactions 3- Elevations in liver enzymes 4- Liver toxicity The side effect profile and expense associated with both lomitapide and mipomersen limit the use of these agents to individuals with homozygous familial hypercholesterolemia.
What are the 2 ways in which the APCs alert the adaptive immune system to the presence of pathogens?
1- Foreign peptide is transported to the surface of the APC by MCH II - peptide + MHC II is recognized by T lymphocytes which have receptors that recognize MHC-peptide complex 2- APC display costimulatory molecules on their cell surfaces as a signal that foreign pathogens have been encountered - binding of MHC-peptide stimulates helper T cells to secrete Cytokines which are signaling proteins that mediate communication bw 2 cells
What are the 3 phases of the Pathogenesis of systemic immune complex disease?
1- Formation of immune complexes 2- Deposition of immune complexes 3- Inflammation of tissue injury
What is obesity an important risk factor for?
1- Heart disease 2- Stroke 3- Type 2 diabetes 4- Cancers of prostate, breast and colon
What are the side effects associated with use of Nicotinic acid?
1- Hepatotoxicity 2- Hyperuricemia 3- Hyperglycemia 4- Flushing
What are examples of cancers caused primarily by infectious agents?
1- Human papilloma virus for cervical cancer 2- Hepatits B and C for liver cancer appx 15% of worldwide cancer cases are caused primarily by infectious agents
What can dyslipidemias be divided into?
1- Hypercholesterolemia 2- Hypertriglyceridemia 3- Mixed hyperlipidemia 4- Disorders of HDL metabolism
What are bile acid sequestrants(Cholestyramine,Colesevelam) mainly used in the treatment of?
1- Hypercholesterolemia in young (less than 25 years old) 2- Patients for whom statins do not provide sufficient plasma LDL reductions
What are fibrates indicated for the treatment of?
1- Hypertriglyceridemia 2- Hypertriglyceridemia with low HDL They lower triglycerides by up to 50%, raise HDL by up to 20%, and lower LDL by up to 15%.
What do individuals with Familial hypertriglyceridemia have? diagnosis?
1- Hypertriglyceridemia (200 to 500 mg/dL) 2- Low HDL-cholesterol (<35 mg/dL) This diagnosis is considered in individuals who have a family and personal history of hypertriglyceridemia, CHD, and normal LDL levels
What are the 2 situations in which hypersensitivity reactions can occur?
1- In response to foreign antigens (microbes etc), may cause injury 2- Immune responses directed against self antigens as a result of the failure of self tolerance- responses against self are termed autoimmunity
What effects do HMG-CoA reductase inhibitors have?What are examples of HMG-CoA inhibitor drugs?
1- Increase cholesterol utilization 2- Decrease VLDL synthesis 3- Increase HDL synthesis As a result, lower LDL and triglyceride levels and higher HDL levels are observed with treatment. lovastatin, pravastatin, simvastatin, fluvastatin, atorvastatin, and rosuvastati
What does hypertriglyceridemia from type 2 diabetes results from?
1- Increased VLDL synthesis and secretion 2- Reduced chylomicron and VLDL catabolism by LPL apoCIII levels are increased in association with insulin resistance, and this reduces the catabolism of chylomicrons and VLDL particles Hypothyroidism is an important and common cause of secondary hyperlipidemia. Any patient with a lipid disorder should be screened or hypothyroidism
What is increased risk of cardiovascular mortality most closely linked to?
1- Increased levels of LDL cholesterol 2- Decreased levels of HDL cholesterol hypertriglyceridemia represents an independent risk factor.
What are the 3 clinical applications of Acetylcholinesterase inhibitors?
1- Increasing transmission at the neuromuscular junction 2- Increasing parasympathetic tone 3- Increasing central cholinergic activity (eg: to treat symptoms of Alzheimers disease)
What are Delayed type hypersensitivity reactions manifested by?
1- Infiltrates of T cells and blood monocytes in tissues 2- Edema and fibrin deposition caused by increased vascular permeability in response to cytokines produced by CD4 T cells 3- Tissue damage induced by leukocyte products, mainly from macrophages that are activated by the T cells
What is the therapy for immediate hypersensitivity aimed at? Common drugs?
1- Inhibiting mast cell degranulation 2- Antagonizing the effects of mast cell mediators 3- Reducing inflammation
What effects can occur as a result of G proteins by agonists binding to Muscarinic receptors?
1- Inhibition of Adenylyl cyclase (via Gi) 2- Stimulation of Phospholipase C (via Gq)
How does LDL hydrolysis via LPL affect homeostatic pathways?
1- Intracellular cholesterol inhibits HMG-CoA reductase, the enzyme that catalyzes the rate-limiting step in de novo cholesterol synthesis 2- Cholesterol activates ACAT to increase esterification and storage of cholesterol in the cell. 3- LDL receptor expression is down-regulated, reducing further uptake of cholesterol into the cells
What are the 2 functions of removal of cholesteryl ester molecules from HDL?
1- It further increases the capacity of HDL to take on additional cholesterol molecules from cells 2- It makes the process of selective uptake by SR-BI more efficient -This is because hydrolysis of triglycerides by hepatic lipase on the hepatocyte surface facilitates the activity o SR-BI
What does pre-beta HDL maturation to HDL occur as a result of what 2 circulating proteins?
1- LCAT (Lecithin cholesterol acetyltransferase) 2- PLTP (phospholipid transfer protein)
What do Low-fat diets that limit saturated fat content promote?
1- LDL receptor expression 2- Uptake of LDL-cholesterol from circulation
What are the steps involved in the formation of HDL?
1- Lipid-poor apoAI is secreted by the liver or intestine or dissociates from lipoprotein particles in the plasma. 2-These amphipathic apoAI molecules interact with ABCA1 , which is localized in the sinusoidal membrane of the hepatocyte or the basolateral membrane o the enterocyte. 3- ABCA1 incorporates a small amount o membrane phospholipid and unesterif ed cholesterol into the apoAI molecule. 4-The resulting small, disk-shaped particle, which consists mainly o phospholipid and apolipoprotein AI, is re erred to as nascent or pre-beta- HDL , due to its characteristic migration on agarose gels
What do Fibric acid derivates enhance the activity of?
1- Lipoprotein lipase (LPL) 2- HDL synthesis As a result, treatment is usually associated with not only lower triglyceride and LDL levels, but also higher HDL levels.
What are the 2 sites where HDL is synthesized?
1- Liver 2- Intestine Excess phospholipids, cholesterol, and apolipoproteins on remnant chylomicrons, VLDL, IDL, and LDL, are transferred to HDL particles and thus increase HDL mass
What are potential side effects of fibric acid derivatives?
1- Liver toxicity 2- Myositis they also interfere with the metabolism of warfarin, leading to a need for its dose adjustment.
What are the treatment options for Familial hypercholesterolemia in patients who do not tolerate the medications or who have limited receptor functions?
1- Liver transplantation provides functional receptors 2-Ileal bypass surgery- decreases gastrointestinal absorption of bile acids 3- LDL apheresis - removes excess LDL lomitapide and mipomersen may also be considered as adjuvant therapy.
What 2 tightly linked loci is the Rh blood group encoded by?
1- Locus D 2- Locus C and E- produces Rh antigens
What 2 drugs are inhibitors of VLDL secretion?
1- Lomitapide - inhibits lipid transfer by binding to MTP 2- Mipomersen -synthetic single-strand antisense oligonucleotide that binds to the apoB100 mRNA and thereby reduces apoB protein levels
What are the 2 mechanisms by which loss of fibrillin leads to the clinical manifestations of Marfan syndrome?
1- Loss of structural support in microfibril rich connective tissue 2- Excessive activation of TGF-β signaling
What are the 7 statin drugs approved fro use in hypercholesterolemia and mixed hyperlipidemia?
1- Lovastatin 2- Pravastatin 3- Simvastatin 4-Fuvastatin 5- Atorvastatin 6- Rosuvastatin 7- Pitavastatin considered first-line therapy for increased LDL levels their use is supported by numerous trials showing that statins decrease both cardiovascular related and total mortality, Stroke is also reduced they all act by the same mechanism and the main differences are attributable to potency and pharmacokinetic parameters
What are the parts of the treatment for Familial defective apolipoprotein B100?
1- Low-cholesterol, low-fat diet 2- Statin, bile acid resin and/or niacin this combination is recommended to lower cholesterol levels to target ranges
What 3 groups is the MHC (major histocompatability complex) classified into?
1- MHC I 2- MHC II 3- MHC III
What is Chlyomicronemia syndrome characterized by?
1- Marked hypertriglyceridemia (>1000 to 2000 mg/dL) 2- Pancreatitis 3- Eruptive xanthomas 4-Lipemia retinalis 5- Hepatosplenomegaly Visual inspection demonstrates lipemic plasma has lipoprotein lipase deficiency
MOA and Use: ipratropium
MOA: Block muscarinic recptors -> bronchodilation. USE: COPD and asthma, nasal discharge
What are the 2 broad classes of Cholinergic receptors?
1- Muscarinic acetylcholine receptors (mAChR) - are G protein-coupled receptors that are expressed at the terminal synapses of all parasympathetic postganglionic fibers and a few sympathetic postganglionic fibers in the CNS 2- Nicotinic acetylcholine receptors (nAChR) - ligand gated ion channels that are concentrated postsynpatically at many excitatory autonomic synapses and presynaptically in the CNS
LEMS should be suspected when the triad of what symptoms occur?
1- Muscle weakness 2- Dry mouth 3- Decreased or absent reflexes
What are the 2 forms that that gain of function mutations take on?
1- Mutations that result in an increase in a protein's enzyme activity 2- Mutations impart a wholly new activity completely unrelated to the affected protein's normal function
What do the biochemical mechanisms of autosomal dominant disorders depend on?
1- Nature of the mutation 2- Type of protein affected
What are the uses of Neomycin? It is recommended as an adjuvant therapy for patients with what? side effects?
1- Neomycin complexes with bile acid and lowers LDL levels. 2- inhibits production of apolipoprotein(a) in the liver and lowers Lp(a) Familial hypercholesterolemia and Lp(a) excess Nephortoxicity and Ototoxicity
What is included in the innate immune system?
1- Phagocytes (engulf and destroys microorganism) 2- Complement system 3- Natural killer (NK) cells
What 2 treatment techniques are used for MG patients with severe bulbar or generalized weakness, respiratory crisis and in refractory patients who do not respond to oral immunomodulating medications?
1- Plasmapheresis 2- IVIG (intravenous immunoglobulin)
What are the 2 most important risk factors for Type II diabetes?
1- Positive family history 2- Obesity (increases insulin resistance)
What are problems associated with Ehlers danlos?
1- Predisposition to joint dislocation 2- The skin is extraordinarily stretchable, extremely fragile, and vulnerable to trauma 3-Basic defect in connective tissue may lead to serious internal complications. These include rupture of the colon and large arteries (vascular EDS), ocular fragility with rupture of cornea and retinal detachment (kyphoscoliosis EDS), and diaphragmatic hernia (classic EDS).
Describe the characteristics of Type II alcoholics
1- Predominantly seen in males, typically occurs before 25 years of age (early onset) 2- Tend to be extroverted and thrill seeking 3- Type II is more difficult to treat successfully and tends to cluster more strongly in families
What are the biases in adoption studies?
1- Prenatal environmental influences could have long-lasting effects on an adopted child 2- Children are sometimes adopted after they are several years old, ensuring that some nongenetic influences have been imparted by the natural parents 3- Adoption agencies sometimes try to match the adoptive parents with the natural parents in terms of attributes such as socioeconomic status all of these factors could exaggerate the apparent influence of biological inheritance
Why is degradation of Ach essential?
1- Prevent unwanted activation of neighboring neurons or muscle cells 2- Ensure proper timing of signaling at the postsynpatic cells
What are the clinical uses of Scopolamine (tertiary amine)? How is it administered?
1- Prevention and treatment of motion sickness 2- Anti-emetic 3-Hospice setting, as an adjunct to end of life comfort care medications to effect mild sedation and management of oral secretions Transdermal patch
How do antimuscarinics help in the treatment of urinary incontinence and overactive bladder syndrome?
1- Promote detrusor muscle relaxation 2- Tightening of bladder syndrome these are the opposite of the effects produced by muscarinic stimulation Antimuscarinics currently approved or the treatment o overactive bladder include oxybutynin , propantheline , terodiline , tolterodine , fesoterodine , trospium , darifenacin , and solifenacin
What are examples of Arachidonic acid metabolites? What are their effects?
1- Prostaglandins - Vascular dilation 2- Leukotrienes- stimulate prolonged smooth muscle contraction
Mutations in the synthesis of an enzyme may result in? What is the consequence in either case?
1- Reduced activity 2- Reduced amount of a normal enzyme Metabolic block
What are the steps involved in the Process of Cholesterol metabolism and transport (normal)?
1- Secretion of very-lowdensity lipoproteins (VLDLs) by the liver into the bloodstream. 2- VLDL particle then reaches the capillaries of adipose tissue or muscle, it is cleaved by lipoprotein lipase, a process that extracts most of the triglycerides 3- The resulting molecule, called intermediate-density lipoprotein (IDL), is reduced in triglyceride content and enriched in cholesteryl esters but it retains two of the three apoproteins (B-100 and E) present in the parent VLDL particle 4- After release from the capillary endothelium, the IDL particles have one of two fates: a- Approximately 50% of newly formed IDL is rapidly taken up by the liver by receptor-mediated transport. -The receptor responsible for the binding of IDL to the liver cell membrane recognizes both apoprotein B-100 and apoprotein E. It is called the LDL receptor, however, because it is also involved in the hepatic clearance of LDL. In the liver cells, IDL is recycled to generate VLDL b- IDL particles not taken up by the liver are subjected to further metabolic processing that removes most of the remaining triglycerides and apoprotein E, yielding cholesterol-rich LDL particles
What are the steps involved in the uptake of the remnants of Chylomicrons and VLDL?
1- Sequestration of the particles within the space of disse between the fenestrated endothelium of the liver sinusoids and the sinusoidal (basolateral) plasma membrane of the hepatocytes 2- Once in the space o Disse, remnants are bound and sequestered by large heparan sulfate proteoglycans 3- The next step is particle remodeling within the space o Disse by the action o hepatic lipase , a lipolytic enzyme that is similar to LPL but is expressed by hepatocytes 4- Hepatic lipase appears to optimize the triglyceride content o remnant particles so that they can be cleared e f ciently by receptor-mediated mechanisms 5- The final phase of remnant clearance is receptor-mediated particle uptake (4 diff pathways for uptake)
What chances of developing Type I diabetes in families?
1- Siblings of persons with type 1 diabetes face a substantial elevation in risk: approximately 6%, as opposed to a risk of about 0.3% to 0.5% in the general population 2- The risk for offspring of diabetic mothers is only 1% to 3%, but it is 4% to 6% for the offspring of diabetic fathers. 3- Twin studies show that the empirical risk for MZ twins of type 1 diabetes patients ranges from 30% to 50%. In contrast, the concordance rate for DZ twins is 5% to 10% 4- If an affected proband and a sibling are both heterozygous for the DR3 and DR4 alleles, the sibling's risk of developing type 1 diabetes is nearly 20%
What are the functions of internalized LDL-cholesterol?
1- Synthesize hormones 2- Produce cell membranes 3- Store energy
What are clinical features of Familial hypercholesterolemia?
1- Tendon xanthomas (caused by intracellular and extracellular accumulation of cholesterol) 2- Arcus corneae (deposition of cholesterol in the cornea)
What features generally apply to most autosomal recessive disorders that distinguish them from autosomal dominant disorders?
1- The expression of the defect tends to be more uniform than in autosomal dominant disorders. 2- Complete penetrance is common 3- Onset is frequently early in life 4- Although new mutations associated with recessive disorders do occur, they are rarely detected clinically. Since the individual with a new mutation is an asymptomatic heterozygote, several generations may pass before the descendants of such a person mate with other heterozygotes and produce affected offspring 4- Many of the mutated genes encode enzymes. In heterozygotes, equal amounts of normal and defective enzyme are synthesized. Usually the natural "margin of safety" ensures that cells with half the usual complement of the enzyme function normally
Why is Ezetimibe particularly effective when in combination with Statin?
1- The reduction in hepatic cholesterol content due to inhibition of cholesterol absorption leads to a compensatory increase in hepatic cholesterol synthesis that partially offsets the benefits of reducing absorption 2- By combining ezetimibe with a statin, the compensatory increase in hepatic cholesterol synthesis is prevented. This approach reduces LDL-cholesterol concentrations by an additional 15% compared with the effect of the statin alone Ezetimibe undergoes enterohepatic circulation up to several times each day in conjunction with meals. Ezetimibe can increase plasma concentrations of cyclosporine, which should be monitored whenever these two drugs are co-administered
How do Bile acid resins lead to reductions in LDL concentrations?
1- The resin-bile acid complex cannot be reabsorbed in the distal ileum and is excreted in the stool. 2- Decreased bile acid reabsorption by the ileum partially interrupts enterohepatic bile acid circulation, causing hepatocytes to upregulate 7-alpha- hydroxylase, the rate-limiting enzyme in bile acid synthesis 3- The increase in bile acid synthesis decreases hepatocyte cholesterol concentration, leading to increased expression of the LDL receptor and enhanced LDL clearance from the circulation
What are Autosomal recessive disorders/traits characterized by?
1- The trait does not usually affect the parents of the affected individual, but siblings may show the disease 2- Siblings have one chance in four of having the trait (i.e., the recurrence risk is 25% for each birth) 3- If the mutant gene occurs with a low frequency in the population, there is a strong likelihood that the affected individual (proband) is the product of a consanguineous marriage.
What are the ways in which IFN-gamma(classically activated) macrophages are altered?
1- Their ability to phagocytose and kill microorganisms is markedly augmented; they express more class II MHC molecules on the surface, thus facilitating further antigen presentation 2- They secrete TNF, IL-1, and chemokines, which promote inflammation 3- They produce more IL-12, thereby amplifying the TH1 response.
What two categories do Autosomal dominant mutations arising from deleterious mutations arise from?
1- Those involved in regulation of complex metabolic pathways that are subject to feedback inhibition eg: Membrane receptors such as the low-density lipoprotein (LDL) receptor provide one such example; in familial hypercholesterolemia, 50% loss of LDL receptors results in a secondary elevation of cholesterol that, in turn, predisposes to atherosclerosis in affected heterozygotes. 2- Key structural proteins, such as collagen and cytoskeletal elements of the red cell membrane eg: spectrin
Autonomic activity of Ach can be classified as either?
1- Tonic activity - accounts for end organ stimulation at rest 2- Phasic activity - triggers an elevated response to changing conditions
What do individuals with familial dysbetalipoproteinemia have elevated levels of?
1- Total cholesterol (300 to 400 mg/dL) 2- Triglycerides (300-400 mg/dL)
Familial combined Hyperlipoproteinemia needs to be considered in all patients with what factors?
1- Total cholesterol level is greater than 250 mg/dL 2- Triglycerides greater than 175 mg/dL 3- HDL-cholesterol less than 35 mg/dL.
What is the treatment of Familial Hypertriglyceridemia?
1- Treatment starts with management of secondary factors that may exacerbate the condition. 2- Dietary fat restriction (<10% of calories) and drug therapy with fish oil, niacin, and fibric acid derivates should be initiated if target goals are not achieved.
What 2 classes of lipids play a significant (yet modifiable) role in pathogenesis of atherosclerosis?
1- Triglyceride 2- Cholesterol
What 3 constituents does the liver assemble to form VLDL?
1- Triglycerides (55% of VLDL mass) 2- Cholesterol (20%) 3- Surface apolipoproteins
What are Chylomicrons formed from?
1- Triglycerides (85% of chylomicron mass) 2- Cholesterol esters assembled with surface lipoproteins Chylomicrons enter into the circulation and acquire more surface apolipoproteins such as apo C-II and apo E from high-density lipoprotein (HDL) particles
What are the most important mediators produced by mast cells ? Where are they stored and released from?
1- Vasoactive amines 2- Proteases 3- Secreted products of Arachidonic acid metabolism and cytokines Granules proteases may cause damage to local dissues
What is low HDL associated with?
1- Visceral obesity 2- Insulin resistance
What are the 3 basic rules that autosomal dominant conditions are characterized by?
1- With every autosomal dominant disorder, some proportion of the patients do not have affected parents 2- Clinical features can be modified by variations in penetrance and expressivity 3- In many conditions the age at onset is delayed; symptoms and signs may not appear until adulthood (eg: Huntingtons disease)
How are Emperical risks calculated?
1- a large series of families is examined in which one child (the proband) has developed the disease 2- The relatives of each proband are surveyed in order to calculate the percentage who have also developed the disease. 3- For example in North America neural tube defects are seen in about 2% to 3% of the siblings of probands with this condition Thus, the recurrence risk for parents who have had one child with a neural tube defect is 2% to 3% In contrast to most single-gene diseases, recurrence risks for multifactorial diseases can change substantially from one population to another
What can lipoprotein particles be divided into (from a metabolic perspective)?
1- lipoproteins that participate in the delivery of triglyceride molecules to muscle and at tissue 2- Lipoproteins that are involved primarily in cholesterol transport ( HDL and the remnants o apoB-containing lipoproteins). (the apoB-containing lipoproteins, chylomicrons, and VLDL )
Elevated concentrations of HDL occur in what conditions? What is a common genetic cause for reduction in HDL levels?
1-Aerobic activity 2-Alcohol consumption 3-Estrogen use 4-Corticosteroid therapy reductions in CETP activity have been characterized as a relatively common genetic cause of increased HDL levels.
How do immune complexes get formed?
1-Protein antigen triggers an immune response that results in the formation of antibodies 2- Typically about a week after the injection of the protein, these antibodies are secreted into the blood, where they react with the antigen still present in the circulation and form antigen antibody complexes
Recommendations for screening for dyslipidemia
1. A fasting lipid profile is recommended at age 20 yr 2. Rescreen every 5 yr if • LDL <160 mg/dL in patients with 0-1 risk factor • LDL <130 mg/dL in patients with ≥2 risk factors 3. Rescreen every year if • LDL 130-159 mg/dL in patients with ≥2 risk factors • LDL <100 mg/dL in patients with CHD or CHD risk equivalent
What are the 4 pathways that allow for receptor-mediated VLDL/chylomicron uptake by the liver?
1. LDL receptor 2. LDL receptor-related protein (LRP) 3. heparan sulfate proteoglycans 4. combined activity of LRP and the heparan sulfate proteoglycans
What are the chances of a child developing disease when one of the parents is affected with an autosomal dominant condition and the other is unaffected?
1/2 chance 50%
Thymoma is found in what percent of patients with MG? What percent Thymaic hyperplasia?
10% of patients 65%
What is the increased risk of developing CAD if one has 2 first-degree relatives affected with CAD before the age of 55?
13 fold
Treatment of Dyslipidemia is initiated after what?
2 abnormal lipid findings Treatment of elevated total cholesterol and LDL-cholesterol can slow the development and progression of CHD Meta-analysis of primary and secondary prevention trials indicates that CHD mortality decreases by approximately 15% for every 10% reduction in serum cholesterol.
What is the lifespan of a HDL particle?
2-5 days suggesting that each apoAI molecule can participate in many cycles of reverse cholesterol transport. Among the nonhepatic tissues that express high levels o SR-BI are the adrenal glands and gonads, presumably re ecting the requirement o these organs or cholesterol to support steroidogenesis
What is the end result of reduced cholesterol absorption by Ezetimibe?
A decrease in LDL-cholesterol concentrations in the plasma inhibiting cholesterol absorption can reduce cholesterol incorporation into VLDL and decrease LDL-cholesterol concentrations in the plasma A single daily dose of ezetimibe lowers LDL-cholesterol concentrations by up to about 20% Ezetimibe also lowers triglyceride concentrations by about 8% and elevates HDL cholesterol to a small extent (approximately 3%
What is the primary treatment for Lipoprotein Lipase (LPL) deficiency?
A diet low in fat (<10% of total calories or 20 to 25 g/day) Secondary factors such as uncontrolled diabetes and alcohol use should be addressed, and VLDL-lowering agents (e.g., fibric acid derivatives, niacin) may be needed to prevent severe hypertriglyceridemia.
B cell response- humoral immune system
A major element of the adaptive immune response begins when specialized types of phagocytes, which are part of the innate immune system, engulf invading microbes and then present peptides derived from these microbes on their cell surfaces. These cells, which include macrophages and dendritic cells, are termed antigen-presenting cells (APCs). B cells are also capable of engulfing microbes and presenting foreign peptides on their cell surfaces. The APCs alert the adaptive immune system to the presence of pathogens in two ways. First, the foreign peptide is transported to the surface of the APC by a class II major histocompatibility complex (MHC) molecule, which carries the foreign peptide in a specialized groove (Fig. 9-2). This complex, which projects into the extracellular environment, is recognized by T lymphocytes, which have receptors on their surfaces that are capable of binding to the MHC-peptide complex. In addition, the APCs, upon encountering a pathogen, display costimulatory molecules on their cell surfaces as a signal that foreign pathogens have been encountered (Fig. 9-3). Binding to the MHC-peptide complex stimulates the helper T lymphocyte to secrete cytokines, which are signaling proteins that mediate communication between cells. In particular, these cytokines help to stimulate the subset of B lymphocytes whose cell surface receptors, termed immunoglobulins, can bind to the invading microorganism's peptides (see Fig. 9-3). The immunoglobulin's capacity to bind a specific foreign peptide (i.e., its affinity for the peptide) is determined by its shape and other characteristics. In the humoral immune response, foreign particles are displayed in conjunction with class II MHC molecules by antigen-presenting cells. These displayed molecules are recognized by helper T cells, which then stimulate proliferation of B cells whose receptors (immunoglobulins) can bind to the foreign pathogen It is estimated that upon initial exposure to a foreign microbe, as few as 1 in every 1 million B lymphocytes happens to produce cell-surface receptors capable of binding to the microbe. This number is too small to fight an infection effectively. Furthermore, the receptor's binding affinity is likely to be relatively poor. However, once this relatively small population of B lymphocytes is stimulated, they begin an adaptive process in which additional DNA sequence variation is generated via the process of somatic hypermutation (see later discussion). These DNA mutations, which are confined to the genes that encode the cell-surface receptors, in turn produce alterations in the receptors' binding characteristics (e.g., the shape of the protein). Some of these variant receptors possess a higher level of binding affinity for the microorganism. The B cells that produce these receptors are favorably selected because they bind the pathogen for a longer period of time. They thus proliferate rapidly. These B cells then become plasma cells, which secrete their cell-surface receptors, or immunoglobulins, into the blood stream. The secreted molecules, which are structurally identical to the receptors on the B cell's surface, are antibodies. It can now be seen how the adaptive immune system gets its name: it involves the initial selection of B cells and T cells whose receptors can bind with the pathogen, followed by subsequent fine-tuning (adaptation) of these cells to achieve higher binding affinity. During the B-cell response to a foreign peptide, the binding affinity of immunoglobulins for the invading pathogen increases. When mature, the B cell becomes an antibody-secreting plasma cell. After initial stimulation by the disease pathogen, the process of B-cell differentiation and maturation into antibody-producing plasma cells requires about 5 to 7 days for completion. Each plasma cell is capable of secreting approximately 10 million antibody molecules per hour. Antibodies bind to the pathogen's surface antigens (a term derived from "antibody generating") and may neutralize the microorganism directly. More often, the antibody tags the pathogen for destruction by other components of the immune system, such as complement proteins and phagocytes. Another important activity of the humoral immune response is the creation of memory B cells, a subset of highaffinity-binding B cells that persist in the body after the infection has subsided. These cells, which have already been highly selected for response to the pathogen, provide a more rapid response should the pathogen be encountered again later in the individual's life. Vaccinations are effective because they induce the formation of memory cells that can respond to a specific pathogen.
What is a Granuloma?
A microscopic aggregation of epithelioid cells, usually surrounded by a collar of lymphocytes
What do Memory B cells provide?
A more rapid response should the pathogen be encountered again later in the individuals life Vaccinations are effective because they induce the formation of memory cells that can respond to a specific pathogen.
Threshold model
A number of diseases do not follow the bell-shaped distribution. Instead, they appear to be either present or absent in individuals. Yet they do not follow the patterns expected of single-gene diseases. A commonly used explanation is that there is an underlying liability distribution for these diseases in a population (Fig. 12-2). Persons who are on the low end of the distribution have little chance of developing the disease in question (i.e., they have few of the alleles or environmental factors that would cause the disease). Those who are closer to the high end of the distribution have more of the diseasecausing alleles and environmental factors and are more likely to develop the disease. For multifactorial diseases that are either present or absent, a threshold of liability must be crossed before the disease is expressed. Below the threshold, the person appears unaffected; above it, he or she is affected by the disease A disease that is thought to correspond to this threshold model is pyloric stenosis, a disorder that manifests shortly after birth and is caused by a narrowing or obstruction of the pylorus, the area between the stomach and intestine. Chronic vomiting, constipation, weight loss, and electrolyte imbalance result from the condition, which can be corrected by surgery or sometimes resolves spontaneously. The prevalence of pyloric stenosis among whites is about 3/1000 live births. It is much more common in males than in females, affecting 1/200 males and 1/1000 females. It is thought that this difference in prevalence reflects two thresholds in the liability distribution; a lower one in males and a higher one in females (see Fig. 12-2). A lower male threshold implies that fewer disease-causing factors are required to generate the disorder in males. The liability threshold concept can explain the pattern of sibling recurrence risks for pyloric stenosis, shown in Table 12-1. Notice that males, having a lower threshold, always have a higher risk than females. However, the recurrence risk also depends on the sex of the proband. It is higher when the proband is female than when the proband is male. This reflects the concept that females, having a higher liability threshold, must be exposed to more disease-causing factors than males in order to develop the disease. Thus, a family with an affected female must have more genetic and environmental risk factors, producing a higher recurrence risk for pyloric stenosis in future offspring. In such a situation, we would expect that the highest risk category would be male relatives of female probands; Table 12-1 shows that this is indeed the case. A number of other congenital malformations are thought to correspond to this model. They include isolated* cleft lip and/or cleft palate, neural tube defects (anencephaly and spina bifida), club foot (talipes), and some forms of congenital heart disease The threshold model applies to many multifactorial diseases. It assumes that there is an underlying liability distribution in a population and that a threshold on this distribution must be passed before a disease is expressed
MHC and disease association
A number of diseases show significant associations with specific MHC alleles: persons who have the allele are much more likely to develop the disease than are those who lack it. Some examples, mentioned in earlier chapters, include the association of HLA-B27 (i.e., allele 27 of the HLA-B locus) with ankylosing spondylitis and of HLA-DQB1 with type 1 diabetes. An especially strong association is seen between several HLA-DR and -DQ alleles and narcolepsy, a disorder characterized by sudden and uncontrollable episodes of sleep. As Table 9-2 shows, most of the HLA-disease associations involve the class II MHC genes. In some cases the association between MHC alleles and a disease is caused by linkage disequilibrium. For example, the hemochromatosis locus is linked to HLA-A, and significant associations occur between HLA-A3 and the most common hemochromatosis-causing allele (see Chapter 8). There is no known causal link between HLA-A3 and this disorder, however. More likely, the association represents a past event in which the primary hemochromatosis mutation arose on a copy of chromosome 6 that had the HLA-A3 allele. Similarly, the association between HLA-DQB1 and HLA-DQA1 and narcolepsy is due to linkage disequilibrium between the HLADQ region and a nearby locus that causes narcolepsy (the hypocretin type 2 receptor gene). In other cases a causal association may exist. Some MHC- disease associations involve autoimmunity, in which the body's immune system attacks its own normal cells. For example, type 1 diabetes is characterized by T-cell infiltration of the pancreas and subsequent T-cell destruction of the insulin-producing beta cells. In some cases, autoimmunity involves "molecular mimicry." Here, a peptide that stimulates an immune response is so similar to the body's own peptides that the immune system begins to attack the body's own cells. This phenomenon helps to explain the onset of ankylosing spondylitis, another autoimmune disease. Infections of HLAB27-positive persons with specific microbes, such as Klebsiella, can lead to a cross-reaction in which the immune system mistakes peptides from some of the body's normal cells for microbial peptides. Another such example is given by rheumatic fever, in which a streptococcal infection initiates crossreactivity between streptococcus and cardiac myosin. In each of these scenarios, the body already has a small population of self-reactive T cells, but they remain inactive and quiteharmless until they are stimulated to proliferate by a foreign peptide that closely resembles a self peptide. Autoimmunity can also be caused by specific defects in the regulation of immune system components. For example, regulatory T cells help to prevent the formation of selfreactive immune cells and require a transcription factor, encoded by FOXP3, for their normal development. Mutations in FOXP3 result in a deficiency of regulatory T cells and an autoimmune disease called IPEX (immunodysregulation, polyendocrinopathy, enteropathy, X linked). Other common diseases that involve autoimmunity include rheumatoid arthritis, systemic lupus erythematosus, psoriasis, and multiple sclerosis. It is estimated that approximately 5% of the population suffers from some type of autoimmune disease. A significant number of diseases are associated with specific MHC alleles. Some of these associations are the result of linkage disequilibrium, but most are likely to result from causal associations involving autoimmunity
lipid disorders
A number of specific disorders of overproduction or impaired removal of lipoproteins result in dyslipidemia (see Tables 69-2 and 69-3). These disorders are often familial, but secondary causes also need to be considered. Comorbid conditions (diabetes, hypothyroidism), medications (estrogen, glucocorticoids, β-blockers), and lifestyle factors (diet, alcohol) can increase the production and clearance of lipoproteins. Addressing these factors can often normalize lipid levels. If abnormalities persist, evaluation of genetic factors and treatment with pharmacologic therapy may need to be considered.
What genes affect Schizophrenia? What do major drugs used to treat schizophrenia block?
A number of the genes in these regions encode components of the glutamatergic and dopaminergic neuronal signaling pathways Dopamine receptors
What is Molecular mimicry?
A peptide stimulates an immune response so similar to the body's own peptides that the immune system begins to attack the body's own cells eg: ankylosing spondylitis,
What is Chronic Granulomatous disease? What does it result in the formation of?
A primary immunodeficiency disorder in which phagocytes can ingest bacteria and fungi but then are unable to kill them this brings about a persistent cellular immune response to the ingested microbes, resulting in the formation of the granulomas
What is Familial hypercholesterolemia?
A receptor disease that is the consequence of a mutation in the gene encoding the receptor for LDL
What is Schizophrenia?
A severe emotional disorder characterized by delusions, hallucinations, retreat from reality and bizarre, withdrawn or inappropriate behavior The lifetime recurrence risk for schizophrenia among the offspring of one affected parent is approximately 8% to 10%, which is about 10 times higher than the risk in the general population. a person with an affected sibling and an affected parent has a risk of about 15% to 20%, and a person with two affected parents has a risk of 40% to 50%.
Cholinergic receptors
A ter ACh has been released into the synaptic cle t, it binds to one o two classes o receptors, usually on the membrane sur ace o the postsynaptic cell. Muscarinic acetylcholine receptors (mAChR) are seven-transmembrane-helix G proteincoupled receptors (GPCRs), and nicotinic acetylcholine receptors (nAChR) are ligand-gated ion channels. Although muscarinic receptors and nicotinic receptors are sensitive to the same neurotransmitter, these two classes of cholinergic receptors share little structural similarity .
What is Niacin?
A water soluble vitamin AKA Nicotinic acid, vitamin B3 At physiologic concentrations, it is a substrate in the synthesis of nicotinamide adenine dinucleotide (NAD) and nicotinamide adenine dinucleotide phosphate (NADP), which are important co factors in intermediary metabolism pharmacologic use of niacin necessitates large doses (1,500-3,000 mg/day) and is independent o the conversion of nicotinic acid to NAD or NADP
A 52-year-old male presents to the emergency department with sudden onset chest pain. The patient has a tall and slender build, a depressed sternum, and long tapering fingers. An Echocardiogram was ordered by the examining physician which revealed mitral valve prolapse and aortic valve regurgitation. What mutation is responsible for the disorder that is most likely associated with these findings? A) A mutation in Fibrillin-1 gene B) A mutation in LDLR gene C) A mutation in BTK gene D) A mutation in the PLOD1 gene E) A mutation in COL1A1 and COL1A2
A) A mutation in Fibrillin-1 gene
An 18-year-old caucasian male with a family history of CHD visits his Family physician and the clinical presentation shows elevated total cholesterol and LDL cholesterol levels as well as tendon xanthomas. Thus, the doctor diagnoses the patient with hypercholesterolemia. The doctor has to run further tests to rule out a lipid disorder in patients who have hypercholesterolemia as the homozygous form of this disorder can cause death due to Myocardial infarction. Which Receptor must be functional in order to rule out this disorder? A) Apo E B) apo B C) apo C-II D) apo100
A) Apo E
Marfan Syndrome is a connective tissue disorder that results in changes of the skeletal, eyes, and cardiovascular system. What is Marfan Syndrome's inheritance pattern and what gene does it affect? A) Autosomal Dominant, FBN1 B) Autosomal Recessive, COL1A1 C) X linked Dominant, COL1A1D D) X linked recessive, FBN1 E) Mitochondrial, FBN1
A) Autosomal Dominant, FBN1
A deficiency in CYP7A1 would cause an accumulation of cholesterol and other substances which would damage certain organs. Symptoms could include gallstones, and heart and peripheral vascular disease. This deficiency affects which of the following processes in lipid metabolism? A) Bile Acid Synthesis B) Delivery of Cholesterol to the Liver C) Cholesterol Synthesis D) Pumping of bile acids into bile E) Biliary Secretion of Phospholipids and cholesterol
A) Bile Acid Synthesis
A researcher is working in a lab studying the effects of various drugs that are used in the treatment of hyperlipidemia. The researcher wants to administer different drugs to mice to see how they alter lipid levels. In this experiment, the researcher used a drug that binds bile acids in the intestinal lumen and increases their fecal excretion. The drug is a success as it decreased LDL-cholesterol. Which of the following drugs did the technician use? A) Cholestyramine B) Ezetimibe C) Simvastatin D) Fish Oil E) Gemfibrozil
A) Cholestyramine
A 49-year-old, overweight male presents to his primary care physician for his first physical exam. Patient has a history of Type II Diabetes Mellitus and states his weight has been steadily increasing. Laboratory diagnostics indicate hypercholesterolemia. The physician prescribes a medication that prevents bile acid reabsorption within the intestines. Which of the following is a potentially adverse effect of the prescribed medication? A) Decreased ability to coagulate blood. B) Myoglobin releases muscle into the bloodstream. C) Acute renal failure D) Inflammation of the liver E) Increased serum creatinine
A) Decreased ability to coagulate blood.
Loretta, a 51-year-old female visits her gynecologist because she wants to start estrogen therapy. Menopause has been particularly difficult for her and her friend suggested that hormone therapy might help. Loretta also has a family history of Familial Hypertriglyceridemia. After talking it over with her, her doctor said it would not be advisable to begin estrogen therapy. What is the likely reason why Loretta cannot take it? A) Estrogen therapy can exacerbate Familial Hypertriglyceridemia B) Estrogen therapy would decrease VLDL levels C) Estrogen therapy can exacerbate menopause symptoms considering her age D) Estrogen therapy would increase HDL levels E) Estrogen therapy drastically decrease TG levels
A) Estrogen therapy can exacerbate Familial Hypertriglyceridemia
A laboratory technician working for a neurology research company is running an experiment involving the inhibition of cholinergic nerve transmission in hopes of finding a new potential therapy for neurodegenerative disorders. In this experiment, the technician used a drug that inhibits the synthesis of acetylcholine. Which of the following drugs did the technician use? A) Hemicholinium-3 B) Pyridostigmine C) Succinylcholine D) Scopolamine E) Pancuronium
A) Hemicholinium-3
A new diagnosis of hypercholesterolemia was made by a primary care physician after a patient's fasting lipid panel showed elevated triglycerides and LDL-cholesterol in the blood. In addition to a diet and exercise program, the patient is prescribed Lovastatin- the "first-line" drug of choice for patients with high LDL-cholesterol. Which of the following is the mechanism of action of this drug that results in the reduction of circulating LDL-cholesterol? A) Inhibition of HMG-CoA Reductase B) Binding to bile acids in the intestinal lumen C) Increasing FFA oxidation in muscle and liver D) Reducing VLDL production E) Decreasing FFA influx into liver
A) Inhibition of HMG-CoA Reductase
A 23-year-old male has been diagnosed with an immune-complex disorder responsible for his nephritis, skin lesions and arthritis. Which antigen is responsible for the disease causing these manifestations? A) Nuclear antigens B) Hepatitis B virus antigens C) Foreign proteins D) Streptococcal cell wall antigens
A) Nuclear antigens
What are the 4 major ABO blood types?
A, B, AB and O A,B and AB represent persons who carry the A,B or A and B antigens on their erythrocyte surfaces, people with type O have neither A or B antigens
A 24-year-old male presents to the clinic. He appears to be unusually tall with exceptionally long extremities and long thin fingers. He states that his back has been causing him pain for the last few months, but in the past couple weeks, the pain has become much more severe. Upon physical examination it is discovered that he has scoliosis as well as pectus excavatum. The attending physician suspects a disorder associated with defects in structural proteins. To confirm his diagnosis, he orders genetic testing. What type of inheritance pattern does this disorder typically present with and what would be the gene(s) that most likely carry the mutation? A. AD, FBN1 B. AR, FBN1 C. AD, COL3A1 D. AR, COL1A1, COL1A2 E. AD, COL5A1, COL5A2
A. AD, FBN1
Bernie presents to a local clinic for a purified protein derivative (PPD) skin test to screen for Tuberculosis. About 8 to 12 hours after having the test performed, reddening and an induration were observed at the site of injection. After hitting a peak at about 24 hours these features slowly subsided. Which features best characterize this type of hypersensitivity reaction? A. Accumulation of mononuclear cells that mainly consist of CD4+ T cells and macrophages. B. Antigen-Antibody complex deposition inducing complement activation. C. Accumulation of antibodies directed against cell surface antigens. D. Cross linkages of IgE bound to mast cells and basophils leading to the release of vasoactive mediators. E. Elevated IgE production and stimulation of strong TH2 responses
A. Accumulation of mononuclear cells that mainly consist of CD4+ T cells and macrophages.
A 32 year old female patient is diagnosed with a disorder that causes demyelination of the central nervous system. Which of the following explains how this disease leads to inflammation? A. Activated TH1 cells secrete IFN-gamma, which activates macrophages, increasing their phagocytic capabilities. B. Immune complexes and complement are deposited along blood vessels in the central nervous system, leading to necrosis and inflammation. C. Activated TH1 cells secrete IL-17 and IL-22, which recruit neutrophils to the reaction. D. Antibodies against myelin are deposited into the tissues, which generates chemotactic agents, recruiting macrophages and neutrophils to the site. E. Activated TH17 cells secrete IFN-gamma, which activates macrophages, increasing their phagocytic capabilities
A. Activated TH1 cells secrete IFN-gamma, which activates macrophages, increasing their phagocytic capabilities.
Ray is an 85-year-old male who has been showing signs of progressive cognitive impairment for the past few months. His family is concerned as Ray has been exhibiting memory loss whereas in the past, he was known for having a great memory. After a thorough examination, Ray's physician diagnosed him with late-onset Alzheimer disease. What risk factor is associated with the late-onset form of this disease? A. Allelic variation in the apolipoprotein E (APOE) locus B. Elevated leptin C. Allelic variation in the CTLA4 gene D. Mutations in the gene that encodes glucokinase E. Mutations in the APC tumor suppressor gene
A. Allelic variation in the apolipoprotein E (APOE) locus
A 55-year-old female presents to the clinic with complaints of muscle weakness that gets worse with continued exertion. She states that this started happening a few years ago, but has gotten worse and worse until she is unable to perform simple household tasks. The physician suspects Myasthenia Gravis. To confirm his suspicion, he administers Edrophonium. Within a minute, her symptoms improve, but shortly thereafter they return. She is then diagnosed with Myasthenia Gravis. What is the mechanism of this disease? A. Antibody inhibits acetylcholine binding, down-modulates receptors B. Antibody-mediated activation of proteases, disruption of intercellular adhesions C. Inflammation and macrophage activation D. Antibody-mediated stimulation of TSH receptors E. Opsonization and phagocytosis of platelets
A. Antibody inhibits acetylcholine binding, down-modulates receptors
Susie has not been feeling well for a couple of days. Susie has a fever and a cough. Her physician believes she is suffering from a viral infection. During an initial viral infection, which of the following adaptive immune response will occur? A. Antigen presenting cells present foreign peptide on Class 1 MHC molecules to CD8+ T cells B. Antigen presenting cells present foreign peptide on Class 1 MHC molecules to CD4+ T cells C. Cytotoxic T cells undergo somatic hypermutation upon encountering antigen. D. Antigen presenting cells are converted to memory cells that can respond to a specific pathogen E. Memory B cells undergo somatic hypermutation upon encountering the pathogen.
A. Antigen presenting cells present foreign peptide on Class 1 MHC molecules to CD8+ T cells
Walter is a 75 year old male that walks into your clinic. The physician did a medical history check and noticed that this man has been diagnosed with Alzheimer dementia. Before going any further with the checkup, the physician should know that Walter should avoid which drug at all cost? A. Atropine B. Methacholine C. Edrophonium D. Cevimeline E. Donepezil
A. Atropine
A 65 year old male presents to the clinic for a routine postop visit after his cholecystectomy procedure. Upon physical examination, it is observed that his abdomen is distended and he complains of urinary retention. What is the drug of choice for the treatment of urinary retention and corresponding class of drug? A. Bethanechol; muscarinic agonist B. Neostigmine; muscarinic antagonist C. Carbachol; muscarinic agonist D. Methacholine; nicotinic antagonist E. Bethanechol; muscarinic antagonist
A. Bethanechol; muscarinic agonist
John Cena recently started medical school and is currently learning about familial hypercholesterolemia in his genetics class. During class he started to have flashbacks about his best friend who he knew had familial hypercholesterolemia. His friend once mentioned to John that for some reason, his body only produces half of the normal number of LDL receptors. What gene mutation does John's friend have? A. Class I LDLR mutation B. Class II LDLR mutation C. Class III LDLR mutation D. Class IV KCNQ1 mutation E. Class V LDLR mutation
A. Class I LDLR mutation
1. Dr. Green is obtaining the medical history of her new patient who has a history of familial hypercholesterolemia and wants to be prescribed a new medication. The patient states that she has been taking a bile acid resin that has been giving her side effects. She explains that she likes to take all of her medications first thing in the morning so that she does not forget to take them later. The patient states that she takes the bile acid resin right after she takes digoxin. She explains that her cousin also has familial hypercholesterolemia and takes digoxin but does not experience side effects like she does even though they are both taking bile acid resins. Which of the following drugs is the patient's cousin most likely taking? A. Colesevelam B. Lomitapide C. Ezetimibe D. Cholestyramine E. Colestipol
A. Colesevelam
A 22-year-old female presents to her primary care physician with chief complaints of a fever, sore throat, pain with swallowing, and "tiny red spots" at the back of her throat. During the history and physical, she informs her doctor that this is the third time in the past 9 months that she is experiencing these particular symptoms. The symptoms lasted for 8 days the first time and 5 days the second time. Her doctor explained to her that she is having recurrent infections of Streptococcus pneumonia leading to the sickness more commonly known as Strep Throat and prescribed her with Penicillin. What is the basis of the process that allowed for a quicker recovery during her second infection? A. DNA sequence variations for B cell receptors were generated resulting in B cells with higher affinity B. Immune system cells with Killer Cell Immunoglobulin-like Receptors on its surface attacks the infectious cell C. There were V, D, and J segment recombination of T cell receptors D. Her body's immune system mistook her normal cells for peptides of Streptococcus pneumoniae E. None of the above
A. DNA sequence variations for B cell receptors were generated resulting in B cells with higher affinity
"A student conducting research on Acetylcholine is working with three different drugs. However, there was a mix up and he does not know which drug is which. To identify the drugs, he injects a different drug into three different mice. In the mouse injected with Drug 1, Acetylcholine is synthesized but is distributed throughout the cytoplasm of presynaptic neurons. Drug 2 causes Acetylcholine to accumulate in vesicles and not be released into the synaptic left, while Drug 3 causes a severe decrease in the synthesis of Acetylcholine. What are Drugs 1, 2, 3 respectively, along with their main target." A. Drug 1: Vesamicol and ACh-H+ antiporter Drug 2: Botulinum toxin and SNARE proteins Drug 3: Hemicholinium-3 and high affinity choline transporter B. Drug 1: Vesamicol and SNARE proteins Drug 2: Botulinum toxin and high affinity choline transporter Drug 3: Hemicholinium-3 and ACh-H+ antiporter C. Drug 1: Botulinum toxin and ACh-H+ antiporter Drug 2: Vesamicol and high affinity choline transporter Drug 3: Hemicholinium-3 and SNARE proteins D. Drug 1: Vesamicol and ACh-H+ antiporter Drug 2: Botulinum toxin and SNARE proteins Drug 3: Hexamethonium and high affinity choline transporter E. Drug 1: Hemicholinium-3 and high affinity choline transporter Drug 2: Vesamicol and ACh-H+ antiporter Drug 3: Botulinum toxin and SNARE proteins
A. Drug 1: Vesamicol and ACh-H+ antiporter Drug 2: Botulinum toxin and SNARE proteins Drug 3: Hemicholinium-3 and high affinity choline transporter
After moving to a new city, a 55-year-old male makes an appointment to become established with a new primary care physician. He states that both of his parents died of cardiovascular complications early in life despite never smoking and maintaining healthy diets. Although he has adopted the same lifestyle as his parents and exercises regularly, he has prominent xanthelasma and documented hyperlipidemia. His current therapy regimen includes 40 mg/day of Rosuvastatin and 10 mg/day of Ezetimibe. The primary care physician decides to prescribe a drug that helps prevent degradation of the LDL receptor. Which one of the following drugs exerts its effects through this specific mechanism of action? A. Evolocumab B. Niacin C. Cholestyramine D. Gemfibrozil E. EPA (Eicosapentaenoic Acid)
A. Evolocumab
A 30-year-old male presents for a regular checkup. The man is overweight and reports a high fat diet and a familial history of premature coronary heart disease. There are no present xanthomas or xanthelasmas. Further testing reveals high LDL-cholesterol, high VLDL-triglyceride, high levels of apoB, and a low ratio of LDL-cholesterol to apolipoprotein B100. What is the mechanism of this autosomal dominant polygenic disorder? A. Excess synthesis of VLDL and decreased VLDL catabolism B. Abnormal expression of ApoE2 C. Gene mutation of apo B100 D. Decreased thrombolysis and increased clot formation. E. Impairment of LDL receptor synthesis.
A. Excess synthesis of VLDL and decreased VLDL catabolism
A 16 year old female presents to the clinic with her mother, who is concerned about the patient's extreme mood swings. The mother explains that her daughter seems irritable, easily distracted, more talkative than usual, and engaging in more risky behaviors. What genes are likely involved in the development of this patient's condition? A. Genes that encode voltage gated calcium channels B. Genes that encode glutamatergic and dopaminergic neuronal signaling pathways C. No genes have been implicated in this condition D. Genes that encode components of GABA receptors E. Genes that encode for presenilin
A. Genes that encode voltage gated calcium channels
After getting the results from Sabrina's lipid panel she asks her doctor about HDL. Which of the following is FALSE about HDL? A. HDL is responsible for transporting apoB 100 to VLDL molecules as it leaves the liver. B. HDL synthesis starts out in the liver as apoA1. C. Nascent HDL are made into HDL by LCAT and PLTP. D. HDL helps bring cholesterol from Macrophages using apo1A which binds to SR-BI E. HDL deposits cholesterol into hepatocytes using apo1A which binds to SR-BI
A. HDL is responsible for transporting apoB 100 to VLDL molecules as it leaves the liver.
A 32-year-old woman presents to the doctor's office having been recently diagnosed with Type I diabetes. She believes she was predisposed to this condition due to a strong family history of Type I diabetes on her maternal side. Which of the following genes would be most associated with her condition? A. HLA B. TP53 C. APC D. MSH2 & MLH1 E. STK11
A. HLA
A 65 year old female was recently diagnosed with hyperlipidemia after her blood results came back with elevated levels of cholesterol and triglycerides. Her primary care physician informs her that her condition predisposes her to an increased risk of heart attack and stroke. After changing her diet to healthier foods and exercising regularly for a few months, she gets retested, hoping her lipid levels have gone down. The results show they are still elevated. She is prescribed a drug to help control her hyperlipidemia. A few weeks later, she makes an appointment with her PCP because she is experiencing muscle weakness, swelling, and pain (myositis). She does not believe it is just lactic acid build up from exercising. Her PCP determines it is a side effect of the drug she is taking. She is instructed to discontinue the drug and is prescribed a new one. What class of drug was the patient initially taking for her hyperlipidemia? A. HMG-CoA reductase inhibitor B. Omega-3 fatty acid C. Bile acid sequestrant D. Cholesterol absorption inhibitor E. Nicotinic acid
A. HMG-CoA reductase inhibitor
Which of the following is NOT true regarding autosomal recessive disorders? A. Incomplete penetrance B. Onset is frequently early in life C. New mutations do occur but are rarely detected clinically D. Expression of the defect tends to be uniform E. Many of the mutated genes encode enzymes
A. Incomplete penetrance
A 40-yr. old female goes to her doctor for an annual checkup, during this visit her doctor informs her that BMI indicates she is in the obese range. The doctor wants to explain to her about a hormone associated with obesity. The doctor is explaining to her about how high levels of this hormone promotes satiety and loss of appetite, what is the mechanism through which this hormone works? A. Leptin is secreted by adipose cells and binds to receptors in the hypothalamus, the body's appetite control center. B. Estrogen is secreted by adipose cells and binds to receptors in the hypothalamus, the body's appetite control center. C. Leptin is secreted by adipose cells and binds to receptors in the cerebellum, the body's appetite control center. D. Estrogen is secreted by adipose cells and binds to receptors in the cerebellum, the body's appetite control center. E. FTO is secreted by adipose cells and binds to receptors in the hypothalamus, the body's appetite control center.
A. Leptin is secreted by adipose cells and binds to receptors in the hypothalamus, the body's appetite control center.
A 45 years old women present to the clinic with a chief complaint of chest pain. During the physical examination physician notice corneal arcus and xanthelasma. Her Triglyceride and HDL cholesterol level was normal, however total cholesterol (800mg/dL) and LDL cholesterol (900mg/dL) was elevated. Name two the adjuvant therapy that should be considered for the treatment. A. Lomitapide and Mipomersen B. Omega-3 fatty acids and Niacin C. Lomitapide and Niacin. D. Mipomersen and Niacin E. Omega-3fatty acid and lomitapide
A. Lomitapide and Mipomersen
Tyrell presents with tremors while doing precise movements in hands and limbs, difficulty walking, blurred vision, and reduced sensation of touch. It is determined that his presentation is the result of pathogenic T cells causing demyelination in his central nervous system. What is the disease? Classify it in terms of hypersensitivity type. A. Multiple sclerosis; type IV hypersensitivity B. Rheumatoid arthritis; type IV hypersensitivity C. Myasthenia gravis; Type II hypersensitivity D. Graves' disease; type II hypersensitivity E. Multiple sclerosis; type III: hypersensitivity
A. Multiple sclerosis; type IV hypersensitivity
A 60-year-old man presented to his primary care physician after a recent surgical procedure with complaints of some urinary problems, specifically, difficulty urinating. His physician diagnosed him with postoperative urinary retention and prescribed him a type of choline ester drug known as Bethanecol for treatment of his symptoms. What is the main mechanism of action of this drug? A. Muscarinic receptor agonist B. Muscarinic receptor antagonist C. Nicotinic receptor agonist D. Nicotinic receptor antagonist E. Acetylcholinesterase inhibitor
A. Muscarinic receptor agonist
A 23-year-old male is diagnosed with familial hypercholesterolemia, which of the following is he most likely at an increased risk for? A. Myocardial infarction B. Stenosis C. Bradycardia D. Urinary retention E. Hyperthermia
A. Myocardial infarction
Joe a 78 year old male recently presented to his local clinic with symptoms of an overactive bladder including frequent urination and repeated episodes of nocturnal enuresis. After meeting with his doctor, Joe was prescribed oxybutynin, an antimuscarinic agent. Approximately 1 week later Joe was rushed to the ER after suffering a seizure. Joe's wife said that for the past week Joe had been experiencing blurred vision, dry mouth and was more irritable than normal. The night before, she said Joe seemed to be hallucinating. It was determined that Joe may have overdosed on his prescribed oxybutynin. Which drug should be administered to counteract Joe's symptoms? A. Neostigmine B. Atropine C. Physostigmine D. Pilocarpine E. Edrophonium
A. Neostigmine
Amanda just got home from the physician's office after being diagnosed with Familial Hypertriglyceridemia, an autosomal dominant disorder characterized by the over production of hepatic VLDL. Her doctor prescribed her 3 things and she was curious as to how they all worked. She googled the first drug and found that it worked by decreasing adipocyte hormone sensitive lipase activity, leading to reduced peripheral tissue triglyceride catabolism and a decreased flux of free fatty acids to the liver. She also found on another site that the drug increases the half-life of a major apolipoprotein in HDL, apoAI. What is the drug that she googled? A. Niacin B. Fenofibrate C. Ezetimibe D. Pitavastatin E. Scopolamine
A. Niacin
EMTs responded to a local elder care facility when a call came in regarding an unresponsive female at the facility. Patricia, 84, had been admitted to the care center about 5 years ago due to her worsening dementia. A well-documented history of aggressive and combative episodes resulted in Patricia being given intermittent Olanzapine to mitigate these outbursts. Medical history from the nurse was significant in that Patricia had felt very nauseous that morning, so she was given Scopolamine to ease the symptoms. She currently appears delirious and is unable to answer questions. Her face is warm to the touch and the nurse reports that although she has been very thirsty today, she has not been to the bathroom at all. What would be the best medication to reverse Patricia's current state? A. Physostigmine B. Mivacurium C. Quaternary antimuscarinics D. Edrophonium E. Pilocarpine
A. Physostigmine
A 26-year-old male sets up an appointment with his general physician due to his finding out of past familial cardiovascular history. During the visit, his doctor notes his BP is 132/82 mm Hg with a BMI of 40 lbs./sq. inch. The physician orders two fasting lipid panels to be completed one week apart over the next two weeks. The (averaged) results of the two tests are shown below. Given these results, which of the following pharmacological therapies would be most suited for this patient? Low Density Lipoprotein (mg/dL) 165 Triglycerides (mg/dL) 188 High Density Lipoprotein (mg/dL) 34 A. Pravastatin B. Colesevelam C. Ezetimibe D. Niacin E. Vascepa
A. Pravastatin
Holden was an unfortunate victim of a car accident involving an explosion. He sustained serious burns to his lower extremities before being removed by the jaws of life. At the ER, the physician considers skin grafting as an option for Holden's burn injuries. However, the physician is wary of potential complications in skin graft rejection. Which of the following could be the mechanism that is responsible for rejection of the skin graft? A. Release of perforins and granzymes from lysosome-like granules of CD8-T cells B. Chronic cytokine tissue damage from CD4+ T cell mediated inflammation C. Delayed type hypersensitivity D. Formation and deposition of antigen-antibody immune complexes in various tissues E. Anaphylaxis due to Type I hypersensitivity
A. Release of perforins and granzymes from lysosome-like granules of CD8-T cells
NTDs are one of the most important classes of birth defects, with a newborn prevalence of approximately 1 per 1000. A defect in closure or a subsequent reopening of the neural tube results in an NTD. A 5 y/o patient displays a protrusion of spinal tissue through the vertebral column intellectual disability, paralysis or muscle weakness, lack of sphincter control, and club feet. This most likely describes which of the following diseases? A. Spina Bifida with Secondary Hydrocephalus B. Meningomyelocele C. Anencephaly D. Closed Spina Bifida E. Encephalocele
A. Spina Bifida with Secondary Hydrocephalus
A 33-year old patient comes to the clinic to get his yearly physical. The doctor notices that the patient has been taking exogenous insulin to treat his autoimmune disease. What is the characteristic of this disease? A. T cell - infiltration of the pancreas and destruction of insulin-producing beta cells. B. Insulin resistance. C. Mutation in the gene that encodes glucokinase. D. Mutation in the gene that encodes alcohol dehydrogenase. E. Allelic variation in the apolipoprotein E.
A. T cell - infiltration of the pancreas and destruction of insulin-producing beta cells.
A 32 year old man came into the ER after being stung by a bee. He was noted to have an airway obstruction due to laryngeal edema. What is the mechanism of action of the drug the ER physician most likely administered? A. Vascular smooth muscle cell contraction, increases cardiac output, and inhibits bronchial smooth muscle cell contraction B. Neutralizes and eliminates IgE C. Blocks actions of histamine on vessels and smooth muscles D. Inhibits mast cell degranulation Blocks actions of cytokines
A. Vascular smooth muscle cell contraction, increases cardiac output, and inhibits bronchial smooth muscle cell contraction
A mother brings her 1-year old baby boy into the clinic because he has had a high fever and has been very fussy lately. She states 2 months ago the baby boy had a streptococcus pneumoniae infection. The clinic runs the test and finds the baby has another streptococcus pneumoniae infection. The repeated streptococcus pneumoniae infections raise suspicion in the doctor, and he tests for a primary immunodeficiency disease. The tests show he is correct. What is the most likely mode of inheritance for this immunodeficiency disease? A. X-linked recessive B. Autosomal recessive C. Autosomal dominant D. Mitochondrial inheritance E. Sporadic mutation
A. X-linked recessive
A 24-year-old male presents to the clinic with multiple plaques formed on his knuckles of both hands. The patient tells the physician that he has several family members who have elevated cholesterol levels. The doctor orders a lipid panel and confirms the patient has elevated LDL and total cholesterol levels. The physician diagnoses the patient with hypercholesterolemia and prescribes statin therapy. Statin therapy proves to be insufficient in lowering the patient's LDL levels so the physician supplements statin with Colesevelam. What is a potential concern/risk with taking Colesevelam? A. potential risk of bleeding gums B. Potential drug-drug interactions and severe toxicity C. Increased risk of gout D. Increased risk of developing rhabdomyolysis E. Decreased HDL levels
A. potential risk of bleeding gums
Immune complex injury called acute vasculitis is associated with necrosis of vessel wall as well as intense neutrophilic infiltration. The tissue contains deposits of immune complexes, complement, and plasma protein. What is the term to describe the appearance of the tissue destruction? A.) Fibrinoid Necrosis B.) Gangrene C.) Glomerular Deposits D.) Vascultitis E.) Erythematosus
A.) Fibrinoid Necrosis
A patient arrives in the clinic and is diagnosed with pyloric stenosis. While educating the patient regarding his or her condition, the physician also explains the concepts of threshold of liability and recurrent risk. In the case of pyloric stenosis, males have a lower threshold of liability than females. In the following scenarios, who would have the greatest risk of acquiring pyloric stenosis? A: The brother of an affected female B: The brother of an affected male C: The sister of an affected female D: The sister of an affected male E: The daughter of an affected female
A: The brother of an affected female
People with the presence of what allele are much less likely to become alcoholics?
ALDH2*2 allele of the ALDH2 gene (ALDH2*2) results in excessive accumulation of acetaldehyde and thus in facial flushing, nausea, palpitations, and lightheadedness.
What are Primary immunodeficiency diseases caused by?
Abnormalities in cells of the immune system and are usually produced by genetic alterations
What is cardiomyopathy? What is it a common cause of?
Abnormality of the heart muscle that leads to inadequate cardiac muscle Heart failure - results in appx 10,000 deaths annually in the US nearly 100 different genes have been linked to cardiomyopathy
What is hypertrophic cardiomyopathy caused by?
About half of hypertrophic cardiomyopathy cases are familial and are caused by autosomal dominant mutations in any of the multiple genes that encode various components of the cardiac sarcomere
Synthesis of Ach
Acetylcholine is synthesized in a single step rom choline and acetyl coenzyme A (acetyl CoA) by the enzyme choline acetyltransferase (ChAT) In the CNS, choline used or the synthesis o acetylcholine arises rom one o three sources. Approximately 35% to 50% o the choline generated by acetylcholinesterase in the synaptic cle t (see Fig. 10-1 and below) is transported back into the axon terminal, where it comprises about hal o the choline used in ACh synthesis. Plasma-based stores o choline may also be transported to the brain as part o phosphatidylcholine (a phospholipid), which is then metabolized to ree choline. (The incorporation o choline into phosphatidylcholine is essential, because choline itsel cannot cross the blood-brain barrier.) Choline is also stored in phospholipids as phosphorylcholine, where it can be used when needed. Acetyl CoA or the synthesis reaction is derived mainly rom glycolysis and is ultimately produced by the enzyme pyruvate dehydrogenase. Although the synthesis o acetyl CoA occurs at the inner membrane o mitochondria, choline acetyltrans erase is located in the cytoplasm. It is hypothesized that citrate serves as the carrier or acetyl CoA rom the mitochondrion to the cytoplasm, where the citrate is reed by citrate lyase. The choline acetyltransferase reaction is not the ratelimiting step in ACh synthesis. Rather, ACh synthesis is limited by the availability of the choline substrate, which depends on uptake of choline into the neuron . Two processes are responsible or choline transport. The f rst is low-a f nity (K m 10-100 M) acilitated di usion. This transport system is not saturable and is ound in cells that synthesize choline-containing phospholipids, such as the corneal epithelium. Far more important is a sodium-dependent, higha f nity transport system (K m 1-5 M) ound specif cally in cholinergic nerve terminals. Because the high-a f nity transporter is saturated at concentrations o choline 10 M, it sets an upper limit on the supply o choline or ACh synthesis. As the rate-limiting component in ACh synthesis, this transporter is a target or several anticholinergic drugs (e.g., hemicholinium-3 , see Fig. 10-1).
Physiologic effects of Cholinergic transmission- Neuromuscular junction
Acetylcholine is the principal neurotransmitter at the neuromuscular junction (Fig. 10-4). ACh is released by motor neurons and it binds to nicotinic receptors in the muscle cell membrane, resulting in motor end-plate depolarization. The extent o depolarization depends on the quantity o ACh released into the synaptic cle t. Release o ACh is quantal in nature; that is, ACh is released in discrete quantities by the presynaptic motor neuron. Each quantum o ACh corresponds to the contents o a single synaptic vesicle and elicits a small depolarization in the motor end-plate termed a miniature end-plate potential (MEPP) . Under resting conditions, sporadic MEPPs are detected at the motor end-plate, corresponding to a low baseline level o unstimulated ACh release that arises rom spontaneous synaptic vesicle usion with the motor axon's presynaptic membrane. In contrast, the arrival o an action potential at the motor axon terminal causes many more vesicles (up to thousands) to use with the neuronal membrane and release their ACh. At the motor endplate, the result is a relatively large depolarization termed the end-plate potential (EPP) (Fig. 10-5). The magnitude o an EPP is more than su f cient to trigger an action potential that propagates rom the end-plate throughout the muscle f ber and, hence, produces a single contraction or "twitch." Acetylcholine not only triggers muscle contraction as its primary effect at the NMJ, but also modulates its own action at this site . Presynaptic cholinergic receptors, located on the axon terminal o the motor neuron, respond to ACh binding by facilitating the mobilization o synaptic vesicles rom the reserve pool to the depot pool. This positive eedback loop, in which the release o ACh stimulates additional ACh release, is necessary to ensure su f cient ACh release when the nerve is stimulated with high requency ( 100 Hz). Despite this mechanism, the ACh output per nerve impulse wanes rapidly during prolonged high- requency stimulation. Fortunately, because an excess o ACh is released and an excess o ACh receptors is present, there is a large sa ety margin. Only when 50% or more o the postsynaptic receptors are desensitized is a decline in muscle tension observed during tetanic stimulation (a phenomenon known as tetanic fade ). Importantly, selective blockade o the modulatory presynaptic cholinergic receptors by antagonists such as hexamethonium prevents acilitation and causes rapid tetanic ade to occur under otherwise normal conditions (Fig. 10-6).
Biochem and Phys of cholinergic neurotransmission
Acetylcholine synthesis, storage, and release ollow a similar set o steps in all cholinergic neurons. The specif c e ects o ACh at a particular cholinergic synapse are largely determined by the ACh receptor type at that synapse. Cholinergic receptors are divided into two broad classes. Muscarinic acetylcholine receptors (mAChR) are G protein-coupled receptors that are expressed at the terminal synapses o all parasympathetic postganglionic f bers and a ew sympathetic postganglionic f bers, at autonomic ganglia, and in the CNS. Nicotinic acetylcholine receptors (nAChR) are ligand-gated ion channels that are concentrated postsynaptically at many excitatory autonomic synapses and presynaptically in the CNS. Acetylcholinesterase (AChE), the enzyme responsible or acetylcholine degradation, is also an important pharmacologic target. In this section, the biochemistry o each o these pharmacologic targets is described and the physiologic e ects o acetylcholine at the neuromuscular junction, in the autonomic nervous system, in the CNS, and in the nonneuronal cholinergic system are discussed. Cho
What enzyme is responsible for Acetylcholine degradation?
Acetylcholinesterase (AChE)
What is the transport of protons out of the vesicle (down the H+ gradient) coupled to uptake? via what channel?
Ach into the vesicle ACh-H+ antiport channel
Inhibition of what channel results in deficient ACh storage into vesicles and subsequent release? eg of a drug that causes this?
Ach- H+ antiport channel Vesamicol
What is Lambert-Easton myasthenic syndrome(LEMS)? What is it caused by?
Acquired presynaptic neuromuscular transmission disorder caused by antibodies against the P/Q type voltage-gated Calcium channels is very rare and is more common in men (3:1)
What do deposited antibodies activate and generate?
Activate complement and generate by-products including chemotactic agents (mainly C5a)
What do proteins on the surface of lipoproteins (ie apolipoproteins) activate? What do they then do?
Activate enzymes and Receptors Guide lipid metabolism
Within the circulation, chylomicrons and VLDL particles must be ......... in order to target triglyceride delivery to muscle and adipose tissue?
Activated
What is the inflammatory reaction associated with Th1 cells dominated by?
Activated Macrophages
What is the principal morphologic manifestation of immune complex injury?
Acute Vasculitis associated with necrosis of the vessel wall and intense neutrophilic infiltration The necrotic tissue and deposits of immune complexes, complement, and plasma protein appear as a smudgy eosinophilic area of tissue destruction, an appearance termed fibrinoid necrosis When deposited in the kidney, the complexes can be seen on immunofluorescence microscopy as granular lumpy deposits of immunoglobulin and complement and on electron microscopy as electron-dense deposits along the glomerular basement membrane
What disease is associated with HLA-B27? with HLA-DBQI? HLA-DR
Ankylosing spondylitis Type I diabetes Narcolepsy
What is the prototype systemic immune complex disease? What does it occur as a result of?
Acute serum sickness Administration of large amounts of foreign serum but in modern times the disease is infrequent, and usually seen in individuals who receive antibodies from other individuals or species
Systemic immune complex disease
Acute serum sickness is the prototype of a systemic immune complex disease; it was once a frequent sequela to the administration of large amounts of foreign serum (e.g., serum from immunized horses used for protection against diphtheria). In modern times the disease is infrequent, and usually seen in individuals who receive antibodies from other individuals or species. Nevertheless, it is an informative model that has taught us a great deal about systemic immune complex disorders.
Since the innate immune system typically helps deal with in an infection in its early stages but is unable to help overcome the infection, what happens next?
Adaptive immune system turns on is capable of adapting to features of the invading microorganism in order to mount a more specific and effective immune response only found in vertebraes
What are about 15% of SCID cases caused by?
Adenosine deaminase (ADA) deficiency, an autosomal recessive disorder of purine metabolism that results in a build up metabolites that are toxic to B and T cells can be treated with bone marrow transplant
Where are excess fatty acids stored?
Adipose tissue or are utilized in heptic lipoprotein synthesis The triglyceride-poor chylomicron remnant is then cleared from the circulation by hepatic LDL receptors. These receptors are activated by apo E, which is located on the surface of chylomicrons
What symptoms occur during the inflammation and tissue injury phase of immune complex disease?
After appx. 10 days after administration clinical features such as fever, urticaria, joint pains (arthralgias), lymph node enlargement, and proteinuria appear.
Storage and release of Ach
After its synthesis in the cytoplasm, ACh is transported into synaptic vesicles or storage. An ATPase that pumps protons into the vesicle provides the energy necessary or this process. Transport o protons out o the vesicle (i.e., down the H concentration gradient) is coupled to uptake o ACh into the vesicle (i.e., against the ACh concentration gradient) via an ACh-H antiport channel. This antiporter is a target or some anticholinergic drugs, such as vesamicol , and its inhibition results in a def cit o ACh storage and subsequent release (Fig. 10-1). Cholinergic synaptic vesicles contain not only ACh but also ATP and heparan sulfate proteoglycans, both of which serve as counter-ions for ACh. By neutralizing the positive charge of ACh, these molecules disperse electrostatic forces that would otherwise prevent dense packing of ACh within the vesicle. (Released ATP also acts as a neurotransmitter, through purinergic receptors, to inhibit the release of ACh and norepinephrine from autonomic nerve endings.) Release of ACh into the synaptic cleft occurs via fusion of the synaptic vesicle with the plasma membrane. This process depends on axon terminal depolarization and the opening of voltage-dependent calcium channels. The increase in intracellular Ca 2 facilitates the binding of synaptotagmin to the SNARE-complex proteins, which together mediate vesicle- membrane attachment and fusion. The result is that the contents of the vesicle are released as discrete "quanta" into the synaptic cleft. Two stores of ACh have distinct roles during the process of Ach release. One store, known as the depot pool, consists o vesicles positioned near the plasma membrane o the axon terminal. Axonal depolarization causes these vesicles to release ACh rapidly. The reserve pool serves to ref ll the depot pool as it is being used. An adequate rate o reserve pool mobilization is required to sustain ACh release or an extended period o time. O the two stores, the depot pool is replenished f rst by vesicles loaded with newly synthesized ACh; this process displaces some o the older depot pool vesicles into the reserve pool
What are some examples of secondary factors that can lead to hyperlipidemia?
Alcohol intake increases the synthesis of fatty acids which are then esterifed to glycerol to form triglycerides Therefore, excess alcohol consumption can result in increased VLDL production
Structural classes
All indirectly acting cholinergic agonists inter ere with the unction o AChE by binding to the active site o the enzyme. There are three chemical classes o such agents, including (1) simple alcohols with a quaternary ammonium group, (2) carbamic acid esters o alcohols bearing either quaternary or tertiary ammonium groups, and (3) organic derivatives o phosphoric acid (Fig. 10-8). The most important unctional di erence among these classes is pharmacokinetic Edrophonium is a simple alcohol that inhibits AChE by reversibly associating with the active site o the enzyme. Because o the noncovalent nature o the interaction between the alcohol and AChE, the enzyme-inhibitor complex lasts or only 2-10 minutes, resulting in a relatively rapid but completely reversible block The carbamic acid esters neostigmine and physostigmine are hydrolyzed by AChE, so a labile covalent bond is ormed between the drug and the enzyme. However, the rate at which this reaction occurs is many orders of magnitude lower than for ACh . The resulting enzyme-inhibitor complex has a hal li e o approximately 15-30 minutes, corresponding to an e ective inhibition lasting 3-8 hours. Organophosphates such as diisopropyl fluorophosphate have a molecular structure that resembles the transition state ormed in carboxyl ester hydrolysis. These compounds are hydrolyzed by AChE, but the resulting phosphorylated enzyme complex is extremely stable and dissociates with a hal -li e o hundreds o hours. Furthermore, the enzyme- organophosphate complex is subject to a process known as aging , in which oxygen-phosphorus bonds within the inhibitor are broken spontaneously in avor o stronger bonds between the enzyme and the inhibitor. Once aging occurs, the duration o AChE inhibition is increased even urther. Thus, organophosphate inhibition is essentially irreversible and the body must synthesize new AChE molecules to restore AChE activity. However, i strong nucleophiles (such as pralidoxime ) are administered be ore aging has occurred, it is possible to displace the organophosphate rom the inhibited AChE and recover enzymatic function.
Diagnosis and Differential diagnosis of Botulism
All suspected cases of botulism need to be reported to public health authorities immediately. Local health department and CDC laboratories can confirm the diagnosis by detecting the toxin in serum, stool, or gastric or wound aspirate specimens Electrodiagnostic testing can also confirm the diagnosis by demonstrating persistent posttetanic facilitation of CMAP of at least 20%, a decremental response greater than 10% with slow RNS, and increased jitter and blocking on SFEMG. Electrodiagnostic tests are also helpful in differentiating botulism from GuillainBarré syndrome and myasthenia gravis.
Botulism - diagnosis and differential diagnosis
All suspected cases of botulism need to be reported to public health authorities immediately. Local health department and CDC laboratories can confirm the diagnosis by detecting the toxin in serum, stool, or gastric or wound aspirate specimens. Electrodiagnostic testing can also confirm the diagnosis by demonstrating persistent posttetanic facilitation of CMAP of at least 20%, a decremental response greater than 10% with slow RNS, and increased jitter and blocking on SFEMG. Electrodiagnostic tests are also helpful in differentiating botulism from GuillainBarré syndrome and myasthenia gravis.
Clinical manifestations of immediate hypersensitivity reactions
Allergic rhinitis, sinusitis (hay fever) Food allergies Bronchial asthma Anaphylaxis (may be caused by drugs, bee sting, food) Increased mucus secretion; inflammation of upper airways, sinuses Increased peristalsis due to contraction of intestinal muscles Airway obstruction caused by bronchial smooth muscle hyperactivity; inflammation and tissue injury caused by late-phase reaction Fall in blood pressure (shock) caused by vascular dilation; airway obstruction due to bronchoconstriction and laryngeal edema Chemokines produced by mast cells and by epithelial cells in the tissues also contribute to leukocyte recruitment. Eosinophils and neutrophils liberate proteases, which cause tissue damage, and Th2 cells may exacerbate the reaction by producing more cytokines
What are the most frequent disorders of the immune system?
Allergies
Disorders in which IgE mediated hypersensitivity is prominent are also called?
Allergy or Atopy common types of allergy include hay fever, food allergies, bronchial asthma and anaphylaxis
What is ACh released by? What does it bind to and result in?
Alpha motor neurons Nicotinic receptors in the muscle cell membrane Motor end-plate depolarization
What do Mendelian disorders result from?
Alterations involving single genes the genetic defect may lead to the formation of an abnormal protein or a reduction in the output of the gene product
Clinical features
Although mitral valve lesions are more frequent, they are clinically less important than aortic lesions. Loss of connective tissue support in the mitral valve leaflets makes them soft and billowy, creating a so-called floppy valve (Chapter 12). Valvular lesions, along with lengthening of the chordae tendineae, frequently give rise to mitral regurgitation. Similar changes may affect the tricuspid and, rarely, the aortic valves. Echocardiography greatly enhances the ability to detect the cardiovascular abnormalities and is therefore extremely valuable in the diagnosis of Marfan syndrome. The great majority of deaths are caused by rupture of aortic dissections, followed in importance by cardiac failure. While the lesions just described typify Marfan syndrome, it must be emphasized that there is great variation in the clinical expression of this genetic disorder. Patients with prominent eye or cardiovascular changes may have few skeletal abnormalities, whereas others with striking changes in body habitus have no eye changes. Although variability in clinical expression may be seen within a family, interfamilial variability is much more common and extensive. Because of such variations, the clinical diagnosis of Marfan syndrome is currently based on the so called "revised Ghent criteria." These take into account family history, cardinal clinical signs in the absence of family history, and presence or absence of fibrillin mutation. In general, major involvement of two of the four organ systems (skeletal, cardiovascular, ocular, and skin) and minor involvement of another organ is required for diagnosis The variable expression of the Marfan defect is best explained on the basis of of the many different mutations that affect the fibrillin locus, which number more than 600. This genetic heterogeneity also poses formidable challenges in the diagnosis of Marfan syndrome. The evolving high throughput sequencing technologies discussed later in this chapter may overcome this problem in the future The mainstay of the medical treatment is administration of β blockers which likely act by reducing heart rate and aortic wall stress. In animal models inhibition of TGF-β action by use of specific antibodies has been found useful. Since lifelong use of such antibodies in humans is not feasible, other strategies to block TGF-β signaling are being tested. Blockade of angiotensin type 2 receptors accomplishes this effect in humans and several preliminary studies are very promising.
The innate, humoral, cellular immune systems- a comparison
Although the innate and adaptive immune systems are described separately, a great deal of interaction takes place between them, and the two systems fulfill complementary functions. The innate system, because it recognizes general features of pathogens, can react very quickly to foreign elements. While doing so, it signals the adaptive immune system to initiate a fine-tuned response to the pathogen. Without this signal, the adaptive immune system is incapable of responding to an infection. After several days during which the adaptive system "learns" the characteristics of the pathogen, it can launch a massive, specialized response. Through the creation of memory B and T cells, the adaptive immune system allows the organism to respond quickly and effectively to a pathogen should it be encountered again. No such memory cells exist for the innate immune system. The humoral immune system is specialized to combat extracellular infections, such as circulating bacteria and viruses. The cellular immune system combats intracellular infections, such as parasites and viruses within cells. However, this division of labor is not strict, and there is again a great deal of interaction between the humoral and cellular components of the immune system
adaptive immune system
Although the innate immune system typically helps to hold an infection in check in its early phases, it is sometimes incapable of overcoming the infection. This becomes the task of a more specialized component of the immune response, the adaptive immune system. As its name suggests, this part of the immune system is capable of changing, or adapting to features of the invading microorganism in order to mount a more specific and more effective immune response. The adaptive immune system is a more recent evolutionary development than the innate immune system and is found only in vertebrates. Key components of the adaptive immune response (Fig. 9-1) include T lymphocytes (or T cells) and B lymphocytes (or B cells). These cells develop in the body's primary lymphoid organs (bone marrow for B cells and the thymus for T cells). In the thymus, developing T cells are exposed to a wide variety of the body's peptides. Those that can recognize and tolerate the body's own peptides are selected, and those that would attack the body's peptides are eliminated. The B and T cells progress to secondary lymphoid tissues, such as the lymph nodes, spleen, and tonsils, where they encounter disease-causing microorganisms. Mature B lymphocytes secrete circulating antibodies, which combat infections. The B lymphocyte component of the immune system is sometimes called the humoral immune system because it produces antibodies that circulate in the blood stream. Helper T lymphocytes stimulate B lymphocytes and other types of T lymphocytes to respond to infections more effectively, and cytotoxic T lymphocytes can directly kill infected cells. Because of this direct interaction with infected cells, the T-cell component of the immune system is sometimes called the cellular immune system. It is estimated that the body contains several trillion B and T cells. B lymphocytes are a component of the adaptive immune system; they produce circulating antibodies in response to infection. T lymphocytes, another component of the adaptive immune system, interact directly with infected cells to kill these cells, and they aid in the B-cell response.
Alzheimer's Disease
Alzheimer disease (AD), which is responsible for 60% to 70% of cases of progressive cognitive impairment among the elderly, affects approximately 10% of the population older than 65 years and 40% of the population older than 85 years. Because of the aging of the population, the number of Americans with AD, which is currently about 5 million, continues to increase. Alzheimer disease is characterized by progressive dementia and memory loss and by the formation of amyloid plaques and neurofibrillary tangles in the brain, particularly in the cerebral cortex and hippocampus. The plaques and tangles lead to progressive neuronal loss, and death usually occurs within 7 to 10 years after the first appearance of symptoms. n affected first-degree relative. Although most cases do not appear to be caused by single genes, approximately 10% follow an autosomal dominant mode of transmission. About 3% to 5% of AD cases occur before age 65 years and are considered early onset; these are much more likely to be inherited in autosomal dominant fashion Alzheimer disease is a genetically heterogeneous disorder. Approximately half of early-onset cases can be attributed to mutations in any of three genes, all of which affect amyloid-β deposition. Two of the genes, presenilin 1 (PS1) and presenilin 2 (PS2), are very similar to each other, and their protein products are involved in cleavage of the amyloid-β precursor protein (APP) by γ-secretase (posttranslational modification; see Chapter 2). Gain-of-function mutations in PS1 or PS2 affect the cleavage of APP such that amyloid-producing forms of it accumulate excessively and are deposited in the brain (Fig. 12-9). This is thought to be a primary cause of AD. Mutations in PS1 typically result in especially early onset of AD, with the first occurrence of symptoms in the fifth decade of life A small percentage of cases of early-onset AD are caused by mutations of the gene (APP) that encodes APP itself, which is located on chromosome 21. These mutations disrupt normal secretase cleavage sites in APP (see Fig. 12-9), again leading to the accumulation of the longer protein product. It is interesting that this gene is present in three copies in persons with trisomy 21, where the extra gene copy leads to amyloid deposition and the frequent occurrence of earlyonset AD in Down syndrome patients (see Chapter 6). Highthroughput DNA sequencing studies have revealed an allele in APP that is protective against Alzheimer disease and may help to prevent cognitive decline An important risk factor for the more common late-onset form of AD is allelic variation in the apolipoprotein E (APOE) locus, which has three major alleles: ε2, ε3, and ε4. Studies conducted in diverse populations have shown that persons who have one copy of the ε4 allele are 2 to 5 times more likely to develop AD, and those with two copies of this allele are 5 to 10 times more likely to develop AD. The risk varies somewhat by population, with higher ε4-associated risks in whites and Japanese and relatively lower risks in Latin Americans and African Americans. Despite the strong association between ε4 and AD, approximately half of persons who develop late-onset AD do not have a copy of the ε4 allele, and many who are homozygous for ε4 remain free of AD even at advanced age. The apolipoprotein E protein product is not involved in cleavage of APP but instead appears to be associated with clearance of amyloid from the brain. Genome-wide association studies, based on microarrays and high-throughput sequencing, indicate that there are many additional genes associated with late-onset AD. Diseaseassociated variants that are relatively common in populations tend to confer only a modest increase in risk, as is the case with most multifactorial conditions. Less common diseaseassociated variants, revealed by high-throughput sequencing, confer a higher relative risk of AD but are generally rare in populations. AD has several features that have made it somewhat refractory to genetic analysis. Its genetic heterogeneity has already been described. In addition, because a definitive diagnosis can be obtained only by a brain autopsy, it is often difficult to diagnose AD in living family members (although clinical features and brain imaging studies can provide strong evidence that a person is affected with AD). Finally, because onset of the disease can occur very late in life, persons carrying an AD-predisposing mutation could die from another cause before developing the disease. They would then be misidentified as noncarriers. These types of difficulties arise not only with AD but with many other common adult diseases as well. Despite these obstacles, multiple AD-associated genes have now been identified, leading to a better understanding of the disorder and to the possibility of more effective AD treatment. Approximately 10% of AD cases are caused by autosomal dominant genes. Earlyonset cases cluster more strongly in families and are more likely to follow an autosomal dominant inheritance pattern. This disease is genetically heterogeneous, and many AD susceptibility genes have been identified. Three genes (encoding presenilin 1, presenilin 2, and amyloidβ precursor protein) cause earlyonset AD and affect the cleavage and processing of the amyloid precursor protein. The most significant gene associated with lateage onset AD encodes the apolipoprotein E protein
What disease is responsible for 60-70% of cases of progressive cognitive impairment among the eldery?
Alzheimer's disease affects appx 10% of the population older than 65 years and 40% of the population older than 85 years old
What occurs after immune complexes are deposited in tissues?
An Acute inflammatory reaction
What are Nonesterified Free fatty acids (FFA) travel as? complexed to?
Anions Albumin
What is the treatment for Maternal fetal Rh incompatability?
Anti-Rh antibody injections these antibodies destroy the fetal erythrocytes in the mother's blood stream before they stimulate production of maternal anti-Rh antibodies
What occurs in opsonization?
Antibodies bind to cells (eg: erythrocytes and plateletes), which then become opsonized and may be ingested and destroyed by host phagocytes
diseases caused by antibodies and antigen antibody complexes
Antibodies other than IgE may cause disease by binding to their target antigens in cells and tissues or by forming immune complexes that deposit in blood vessels (Fig. 11-7). Antibody-mediated hypersensitivity reactions have long been recognized as the basis of many chronic immunologic diseases in humans. Antibodies against cells or extracellular matrix components may deposit in any tissue that expresses the relevant target antigen. Diseases caused by such antibodies usually are specific for a particular tissue. Immune complexes often deposit in blood vessels, including vessels through which plasma is filtered at high pressure (e.g., in renal glomeruli and joint synovium). Therefore, immune complex diseases tend to be systemic and often manifest as widespread vasculitis, arthritis, and nephritis.
What occurs in a Type II hypersensitivity reaction?
Antibodies other than IgE( usually IgG or IgM) are directed against cell or tissue antigens can damage these cells or tissues or impair their functions
What causes muscle weakness in Myasthenia gravis?
Antibodies reactive with acetylcholine receptors in the motor end plates of skeletal muscles block neuromuscular transmission and therefore cause muscle weakness.
mechanisms of tissue injury and disease
Antibodies specific for cell and tissue antigens may deposit in tissues and cause injury by inducing local inflammation, they may induce phagocytosis and destruction of cells, or they interfere with normal cellular functions (Fig. 11-8) • Inflammation. Antibodies against tissue antigens and immune complexes deposited in vessels induce inflammation by attracting and activating leukocytes. IgG antibodies of the IgG1 and IgG3 subclasses bind to neutrophil and macrophage Fc receptors and activate these leukocytes, resulting in inflammation (see Chapter 8). The same antibodies, as well as IgM, activate the complement system by the classical pathway, resulting in the production of complement byproducts that recruit leukocytes and induce inflammation. When leukocytes are activated at sites of antibody deposition, these cells release reactive oxygen species and lysosomal enzymes that damage the adjacent tissues. • Opsonization and phagocytosis . If antibodies bind to cells, such as erythrocytes and platelets, the cells are opsonized and may be ingested and destroyed by host phagocytes. • Abnormal cellular responses. Some antibodies may cause disease without directly inducing tissue injury. For example, antibodies against hormone receptors may inhibit receptor function; in some cases of myasthenia gravis, antibodies against the acetylcholine receptor inhibit neuromuscular transmission, causing paralysis. Other antibodies may directly activate receptors, mimicking their physiologic ligands. In a form of hyperthyroidism called Graves disease, antibodies against the receptor for thyroidstimulating hormone stimulate thyroid cells even in the absence of the hormone.
Antibody mediated Type II hypersensitivity
Antibodies that react with antigens present on cell surfaces or in the extracellular matrix cause disease by destroying these cells, triggering inflammation, or interfering with normal functions. The antibodies may be specific for normal cell or tissue antigens (autoantibodies) or for exogenous antigens, such as chemical or microbial proteins, that bind to a cell surface or tissue matrix. The antibody-dependent mechanisms that cause tissue injury and disease are illustrated in Figure 6-16 and described next. These reactions are the cause of several important diseases (Table 6-3).
What does Antibody-dependent cellular cyototoxicity(ADCC) cause?
Antibody-mediated destruction of cells
What is the mechanism responsible for tissue injury in some forms of Glomerulonephritis, vascular rejection in graft organs?
Antibody-mediated inflammation
What is Graves disease? What occurs in this disorder?
Antibody-mediated stimulation of cell function antibodies against the thyroid-stimulating hormone receptor on thyroid epithelial cells stimulate the cells, resulting in hyperthyroidism
Muscarinic receptor antagonists
Anticholinergic compounds that act on muscarinic receptors are used to produce a parasympatholytic e ect in target organs. By blocking normal cholinergic tone, these compounds allow sympathetic responses to predominate (Table 10-2). The most commonly used anticholinergics are either naturally occurring alkaloids or synthetic quaternary ammonium compounds. The alkaloids are relatively selective or antagonist activity at muscarinic receptors, whereas the synthetic compounds also demonstrate substantial antagonism at nicotinic receptors. The prototypical muscarinic receptor antagonist is atropine , a naturally occurring alkaloid ound in the plant Atropa belladonna , or deadly nightshade. Belladonna derived its name rom Italian or "beauti ul woman"—during the Renaissance, women in Italy ingested or applied to their eyes extracts and juices o berries rom the plant to cause dilation o the pupils, which was considered a mark o beauty. Atropine is used clinically to induce mydriasis (pupil dilation) or ophthalmologic examinations, to reverse symptomatic sinus bradycardia, to inhibit excessive salivation and mucus secretion during surgery, to prevent vagal ref exes induced by surgical trauma o visceral organs, and to counteract the e ects o muscarine poisoning rom certain mushrooms (see Box 10-1). Because o its marginal activity at nicotinic receptors, extremely high doses o atropine are required or any e ects to be seen at the NMJ. Similarly, because nicotinic receptors are primarily responsible or excitatory transmission at autonomic ganglia, atropine produces only partial block at these sites and only at relatively high doses. Scopolamine (hyoscine hydrobromide) , a tertiary amine, di ers rom atropine by virtue o its substantial CNS e ects. Scopolamine is requently used or the prevention and treatment o motion sickness, as an antiemetic, and, in the hospice setting, as an adjunct to end-o -li e com ort care medications to e ect mild sedation and management o oral secretions. A transdermal patch system has been developed to e ect slow absorption and long duration o the anti-motion sickness e ect while avoiding a rapid rise in plasma levels and adverse CNS e ects (e.g., anterograde disruption o novel learning and memory encoding, inattention, and slowing o psychomotor speed). Scopolamine can also be used to ameliorate nausea, particularly that associated with chemotherapy, and can be administered intravenously during procedures in which minimizing oral secretions is desirable. Methscopolamine and glycopyrrolate are quaternary amine antimuscarinics with low CNS penetration that are used or their peripheral e ects to decrease oral secretions, decrease GI spasms, and, in the case o glycopyrrolate, prevent bradycardia during surgical procedures. Both drugs have delayed but measurable CNS and cognitive anticholinergic e ects. Pirenzepine , which is selective or M 1 and M 4 receptors, was a potential alternative to H 2 receptor antagonists in the treatment o peptic ulcer disease, but its use has been supplanted by the advent o the proton pump inhibitors Ipratropium , a synthetic quaternary ammonium compound, is more e ective than -adrenergic agonists in the treatment o chronic obstructive pulmonary disease (COPD) but less e ective in treating asthma. Tiotropium has similar, and possibly superior, e f cacy to ipratropium as a bronchodilator in the treatment o COPD. The superior e f cacy o ipratropium and tiotropium in COPD is likely due to the act that the major reversible bronchoconstrictive component in COPD is mediated by cholinergic neural tone (see Chapter 48). In poorly controlled asthma, the addition o tiotropium to inhaled glucocorticoids and long-acting betaagonists may signif cantly increase the time to f rst severe exacerbation and provide modest sustained bronchodilation Some antimuscarinic drugs are used in the treatment o urinary incontinence and overactive bladder syndrome. Muscarinic stimulation promotes voiding by causing (1) detrusor muscle contraction and (2) bladder trigone and sphincter muscle relaxation. Antimuscarinics produce the opposite e ects by promoting detrusor relaxation and tightening the bladder sphincter. Antimuscarinics currently approved or the treatment o overactive bladder include oxybutynin , propantheline , terodiline , tolterodine , fesoterodine , trospium , darifenacin , and solifenacin . Among these agents, oxybutynin, propantheline, tolterodine, esoterodine, and trospium are nonspecif c muscarinic receptor antagonists, whereas dari enacin and soli enacin are selective M 3 receptor antagonists. These agents appear to have similar clinical efficacy. Clinical trials suggest that tolterodine may cause less dry mouth than oxybutynin and that the newer M 3 -selective agents dari enacin and soli enacin may cause less dry mouth and constipation than the nonselective agents. Atropine, rom belladonna extract, was one o the drugs f rst used to treat symptoms o Parkinson's disease (PD). Antimuscarinics are still used at times to ameliorate tremor and rigidity in patients with PD. These medications include amantadine , biperiden , benztropine , procyclidine , and trihexyphenidyl . Although antimuscarinics may be help ul in the treatment o PD-related tremor and rigidity, the use of antimuscarinics in elderly and cognitively susceptible patients should be avoided because o the high risk o potential adverse e ects (see Box 10-2). Benztropine and trihexyphenidyl are commonly used to treat extrapyramidal symptoms, dystonias, and akathisia associated with neuroleptics; these adverse e ects are thought to be due to an imbalance between dopaminergic and cholinergic pathways secondary to excessive neuroleptic-induced dopamine antagonism. Trihexyphenidyl is also used to treat neuroleptic-induced hypersalivation syndrome. Antimuscarinic toxicity causes substantial morbidity and unctional impairment in the geriatric population (see Box 10-2). Depending on the dose, antimuscarinic agents such as atropine and scopolamine may cause bradycardia and sedation at low to medium levels o muscarinic blockade, and tachycardia and CNS hyperexcitation (with delirium, hallucinations, and seizures) at higher levels. Other adverse e ects may include blurred vision (cycloplegia and mydriasis), dry mouth, ileus, urinary retention, f ushing and ever, agitation, and tachycardia. Antimuscarinic medications are contraindicated in patients with glaucoma. Patients with angle-closure glaucoma, which may be precipitated in individuals with shallow anterior chambers, are especially at risk. Antimuscarinics should also be used with caution in patients with prostatic hypertrophy and in patients with dementia or cognitive impairment. Antimuscarinic toxicity is considered dangerous in in ants and children, who are exquisitely sensitive to the hyperthermic adverse e ects caused by an overdose. Symptomatic treatment may include controlled cooling and antiepileptic drugs, but slow administration o low doses o intravenous physostigmine may also be required. High doses o quaternary antimuscarinics and shortacting ganglionic blockers (such as trimethaphan) can cause parasympathetic ganglionic toxicity, mani ested as autonomic blockade and severe orthostatic hypotension. The antimuscarinic e ects may be treated with neostigmine, and the hypotension may require treatment with sympathomimetics such as phenylephrine.
What occurs in Type III HS?
Antigen combines with antibody in the circulation, creating immune complexes that typically deposit in vessel walls.
What are the specialized macrophages which are part of the innate immune system, whose function is to engulf invading microbes and present peptides derived from these microbes on their cell surfaces?
Antigen presenting cells (APC): 1- Macrophages 2- Dendritic cells B cells are also capable of engulfing microbes and presenting foreign peptides on their cell surfaces
What is the variable region of the antibody responsible for?
Antigen recognition and binding
Immune complex mediated type III hypersensitivity
Antigen-antibody complexes produce tissue damage mainly by eliciting inflammation at the sites of deposition. The pathologic reaction is usually initiated when antigen combines with antibody in the circulation, creating immune complexes that typically deposit in vessel walls. Less frequently, the complexes may be formed at sites where antigen has been "planted" previously (called in situ immune complexes). The antigens that form immune complexes may be exogenous, such as a foreign protein that is injected or produced by an infectious microbe, or endogenous, if the individual produces antibody against self antigens (autoimmunity). Examples of immune complex disorders and the antigens involved are listed in Table 6-4. Immune complex-mediated diseases tend be systemic, but often preferentially involve the kidney (glomerulonephritis), joints (arthritis), and small blood vessels (vasculitis), all of which are common sites of immune complex deposition
What effect does Nicotinic acid have? what does that result in a decrease of? What occurs as a result?
Antilipolytic effect decreases influx of Free fatty acids (FFA) to the liver Hepatic VLDL synthesis and LDL production are reduced Nicotinic acid also decreases HDL catabolism Lower triglyceride and LDL levels and higher HDL levels are observed with treatment
How does Poststreptococcal glomerulonephritis occur?
Antistreptococcal antibodies deposit in the kidney glomeruli leading to an inflammatory process can lead to renal failure, hematuria, proteinuria
Apolipoprotein C-II Deficiency
Apo C-II is an activating cofactor for LPL. Deficiency of apo C-II is a rare autosomal recessive disorder that leads to increased chylomicrons and VLDL particles in the circulation, resulting in severe hypertriglyceridemia. Clinical manifestations are similar to those of LPL deficiency, including hypertriglyceridemia (>1000 mg/dL) and symptoms of pancreatitis, eruptive xanthomas, lipemia retinalis, and hepatosplenomegaly. Treatment recommendations include appropriate management of secondary factors such as diabetes and hypothyroidism, dietary fat restriction (<10% of calories), and drug therapy (e.g., fibric acid derivatives). For severe hypertriglyceridemia, plasma transfusion (with apo C-II) can be considered
What occurs in Familial Dysbetaliproteinemia?
Apo E on the surface of lipoprotein particles binds LDL receptors and facilitates clearance of remnant particles from the circulation. In individuals who are homozygous for apo E2, LPL hydrolyzes the triglyceride core and the resulting cholesterol-rich chylomicrons. VLDL and IDL remnant particles accumulate in the circulation Expression of this phenotype usually requires a precipitating condition that increases lipoprotein production (e.g., diabetes, alcohol consumption) or decreases clearance (e.g., hypothyroidism
Familial dysbetalipoproteinemia
Apo E on the surface of lipoprotein particles binds LDL receptors and facilitates clearance of remnant particles from the circulation. The apo E2 allele has a lower affinity for LDL receptors than apo E3 or apo E4. In individuals who are homozygous for apo E2, LPL hydrolyzes the triglyceride core and the resulting cholesterol-rich chylomicrons. VLDL and IDL remnant particles accumulate in the circulation. Expression of this phenotype usually requires a precipitating condition that increases lipoprotein production (e.g., diabetes, alcohol consumption) or decreases clearance (e.g., hypothyroidism). In addition to the more common autosomal recessive mutation of apo E described earlier, several apo E mutations have been described that result in an autosomal dominant phenotype manifesting in childhood. Premature CHD, peripheral vascular disease, and xanthomas involving the palmer crease are characteristic clinical features. Individuals with familial dysbetalipoproteinemia have elevated levels of total cholesterol (300 to 400 mg/dL) and triglycerides (300 to 400 mg/dL). Definitive diagnosis requires genetic testing to identify apo E2 homozygosity or mutation. Treatment of coexisting conditions such as diabetes and hypothyroidism can normalize lipid levels in apo E2 homozygotes. If target levels are not achieved, dietary therapy and lipid-lowering drugs such as fibric acid derivatives and HMG-CoA reductase inhibitors should also be considered.
What is the main structural determinant of HDL and what does it participate in the formation of?
ApoAI Formation of the particle and its its interaction with its receptor, scavenger receptor class B type I (SR-BI) The function of apoAII is not well understood—it appears to play a role in maintaining the structural integrity of HDL.
formation and clearance of LDL particles
ApoB48-containing chylomicron remnants are completely cleared rom the plasma. By contrast, the presence o apoB100 alters the metabolism o VLDL remnants so that only approximately 50% are cleared by the pathways or remnant particles. The di erence is mani ested during the metabolism o the remnant particles by LPL. VLDL remnants are avidly metabolized by LPL, becoming an increment smaller, relatively more def cient in triglycerides, and relatively enriched in cholesteryl esters. When converted to remnants ollowing exchange o apolipoproteins with HDL, these more dense particles are called intermediate-density lipoproteins (IDL). Because IDL contain apoE, a raction o these particles (approximately 50%) may be cleared by the liver via remnant receptor pathways (Fig. 20-6). However, the remainder are converted to LDL by hepatic lipase, which urther hydrolyzes triglycerides in the core o IDL. The urther reduction in size o the particle results in the trans er o apoE to HDL. As a consequence, LDL is a distinct , cholesteryl ester-enriched lipoprotein with apoB100 as its only apolipoprotein (Fig. 20-7A). The LDL receptor is the only receptor capable o clearing signif cant amounts o LDL rom the plasma. The LDL receptor is expressed on the sur ace o hepatocytes, macrophages, lymphocytes, adrenocortical cells, gonadal cells, and smooth muscle cells. Due to the lack o apoE, LDL particles are relatively weak ligands or the LDL receptor. As a result, the hal -li e o LDL in the circulation is markedly prolonged (2-4 days). This explains why LDL cholesterol accounts or approximately 65-75% o total plasma cholesterol. The uptake o LDL-C by the LDL receptor also appears to be promoted by sortilin (Fig. 20-7B). Interaction o apoB100 with the LDL receptor acilitates receptor-mediated endocytosis o LDL particles and subsequent vesicle usion with lysosomes (Fig. 20-7B). The LDL receptor is recycled to the cell sur ace, while the cholesteryl esters and triglycerides within the LDL particle are hydrolyzed by lysosomal acid lipase (LAL) to release unesterif ed cholesterol and atty acids. These hydrolysis products a ect three major homeostatic pathways. First, intracellular cholesterol inhibits HMG-CoA reductase, the enzyme that catalyzes the rate-limiting step in de novo cholesterol synthesis. Second, cholesterol activates ACAT to increase esterif cation and storage o cholesterol in the cell. Third, LDL receptor expression is down-regulated, reducing urther uptake o cholesterol into the cells. The majority o LDL receptors (70%) are expressed on the sur ace o hepatocytes. As a result, the liver is primarily responsible or the removal o LDL particles rom the circulation. Proprotein convertase subtilisin-like kexin type 9 (PCSK9) is a plasma protein that regulates LDL receptor activity. PCSK9 is synthesized as a 72-kDa proPCSK9 proprotein that is autocatalytically cleaved in the endoplasmic reticulum to orm the mature protein. It then enters the secretory pathway, and with the assistance o sortilin in the trans Golgi network, it is secreted into the plasma. PCSK9 then binds to the epidermal growth actor-like repeat A (EGFA) moti o the LDL receptor. This complex is targeted to lysosomes or degradation (Fig. 20-7B). Gain-o - unction mutations in PCSK9 result in marked elevations in LDL-C, whereas loss-o - unction mutations reduce LDL-C LDL particles that are not taken up by LDL receptorexpressing tissues may migrate into the intima o blood vessels and bind to proteoglycans (Fig. 20-8). There, they are subject to oxidation or nonenzymatic glycosylation. Oxidation o LDL results in lipid peroxidation and may create reactive aldehyde intermediates that ragment apoB100. The modif ed LDL is internalized by scavenger receptors (e.g., SR-A), which are expressed predominantly by mononuclear phagocytic cells. Unlike the LDL receptor, scavenger receptors are not down-regulated when the phagocytic cells begin to accumulate cholesterol. As a result, the continued accumulation o oxidized LDL in macrophages can lead to foam cell ormation (cholesterol-rich macrophages). These oam cells may undergo apoptotic or necrotic death, releasing ree radicals and proteolytic enzymes. Oxidized LDL also causes up-regulation o cytokine production, impairs endothelial unction, and increases expression o endothelial adhesion molecules. All o these e ects increase the local in ammatory response and promote atherosclerosis. Foam cells are a major constituent o atherosclerotic lesions, and excessive oam cell death can destabilize atherosclerotic plaques. This is attributable in part to the liberation o matrix metalloproteinases. Because plaque rupture is the main cause o acute ischemic cardiovascular events, particularly heart attacks and strokes, high plasma levels of LDL are a major risk factor for the development of atherosclerosis and subsequent cardiovascular disease . This is why Jake's doctor became concerned when he discovered that Jake had very high plasma concentrations o LDL.
The mechanisms by which fibrate-mediated PPAR alpha activation raises plasma HDL depend at least in part on increased hepatic production of what?
Apolipoprotein A1 This would be expected to contribute directly to increased plasma HDL Upregulation of ABCA1 in macrophages presumably promotes cholesterol efflux from these cells in vivo
What does limiting intake of saturated and trans-unsaturated fatty acid require?
Appropriate calories substitutions Increasing carbohydrate content to achieve this goal can increase the hepatic synthesis of triglyceride. Dietary substitution with soluble fibers (e.g., oat bran) has been recommended, because these fibers have a limited effect on triglyceride levels They also bind bile acids in the gut and thereby decrease cholesterol levels Other polyunsaturated fats, such as omega-3 fatty acids, are cardioprotective. They are abundant in fatty fish, flaxseed oil, canola oil, and nuts. They reduce VLDL production, inhibit platelet aggregation, and decrease CHD. Even two servings per week of fatty fish such as salmon can be beneficial.
What type of disease is Myasthenia gravis? What is it caused by?
Autoimmune disease Antibodies against the postsynaptic acetylcholine receptors (AChR Ab) in the neuromuscular junction
A 35-year-old female presents to the Neurology clinic with complaints of blurred vision, muscle weakness and paralysis. Symptoms have been ongoing for 4-5 months. After a number of tests have been performed, the physician diagnoses her with Myasthenia Gravis. What type of hypersensitivity is this? A) Immediate Type I Hypersensitivity B) Antibody-Mediated Type II Hypersensitivity C) Immune Complex Mediated Type III Hypersensitivity D) CD4+ T Cell-Mediated Type IV Hypersensitivity E) CD8+ T Cell Mediated Type IV Hypersensitivity
B) Antibody-Mediated Type II Hypersensitivity
congenital malformations
Approximately 2% of newborns present with a congenital malformation (i.e., one that is present at birth); most of these conditions are considered to be multifactorial in etiology. Some of the more common congenital malformations are listed in Table 12-4. In general, sibling recurrence risks for most of these disorders range from 1% to 5%. Some congenital malformations, such as cleft lip/palate and pyloric stenosis, are relatively easy to repair and thus do not cause lasting problems. Others, such as neural tube defects, usually have more severe consequences. Although some cases of congenital malformations can occur in the absence of any other problems, it is quite common for them to be associated with other disorders. For example, cleft lip/ palate is often seen in babies with trisomy 13, and congenital heart defects are seen in many syndromes, including trisomy of chromosomes 13, 18, and 21. Considerable progress is now being made in isolating single genes that can cause congenital malformations. Many of these, including the HOX, PAX, and TBX families of genes, were discussed in Chapter 10. Another example is the RET proto-oncogene, which is responsible for some cases of Hirschsprung disease. However, the causes of many cases of this disorder remain undiscovered. Indeed, most of the genetic factors that contribute to important congenital malformations (e.g., neural tube defects, common congenital heart defects, cleft lip/palate) are as yet unidentified. Environmental factors have also been shown to cause some congenital malformations. An example is thalidomide, a sedative used during pregnancy in the early 1960s (and recently reintroduced for the treatment of dermatological conditions such as leprosy). When ingested during early pregnancy, this drug often can cause phocomelia (severely shortened limbs) in babies. Maternal exposure to retinoic acid, which is used to treat acne, can cause congenital defects of the heart, ear, and central nervous system. Maternal rubella infection can cause congenital heart defects. Other environmental factors that can cause congenital malformations are discussed in Chapter 15. Congenital malformations are seen in roughly 1 of every 50 live births. Most of them are considered to be multifactorial disorders. Specific genes and environmental causes have been detected for some congenital malformations, but the causes of most congenital malformations remain largely unknown.
What occurs in individuals that are Homozygotes for familial hypercholesterolemia?
Are much more severely affected and may have five fold to six-fold elevations in plasma cholesterol levels. Skin xanthomas and coronary, cerebral, and peripheral vascular atherosclerosis may develop in these individuals at an early age.
Effects of Autonomic Ganglionic blockade on Tissues
Arterioles Sympathetic (adrenergic) Vasodilation; ↑ peripheral blood f ow; hypotension Veins Sympathetic (adrenergic) Vasodilation; pooling o blood; ↓ venous return; ↓ cardiac output Heart Parasympathetic (cholinergic) Tachycardia Iris Parasympathetic (cholinergic) Mydriasis (pupil dilation) Ciliary muscle Parasympathetic (cholinergic) Cycloplegia ( ocused to ar vision) Gastrointestinal tract Parasympathetic (cholinergic) ↓ Tone and motility; constipation; ↓ secretions Urinary bladder Parasympathetic (cholinergic) Urinary retention Salivary glands Parasympathetic (cholinergic) Xerostomia (dry mouth) Sweat glands Sympathetic (cholinergic) Anhidrosis (absence o sweating
What is an example of a Local immune complex disease ?
Arthus reaction
receptor mediated clearance of apoB containing lipoproteins
As LPL continues to hydrolyze triglycerides rom chylomicrons and VLDL, the particles become progressively depleted o triglycerides and relatively enriched in cholesterol. Once approximately 50% o the triglycerides have been removed, the particles lose their a f nity or LPL and dissociate rom the enzyme. The exchangeable apolipoproteins apoAI and apoCII (as well as apoCI and apoCIII) are then trans erred to HDL in exchange or apoE (Fig. 20-6A), which serves as a high-a f nity ligand or receptor-mediated clearance o the particles. Upon acquiring apoE, the particles are termed chylomicron or VLDL remnants Remnants o chylomicrons and VLDL are taken up by the liver in a three-step process (Fig. 20-6B). The f rst step is the sequesteration of the particles within the space of Dis the fenestrated endothelium o the liver sinusoids and the sinusoidal (basolateral) plasma membrane o the hepatocytes. Sequestration requires that the remnant particles become small enough during lipolysis to f t between the endothelial cells. Once in the space o Disse, remnants are bound and sequestered by large heparan sulfate proteoglycans. The next step is particle remodeling within the space o Disse by the action o hepatic lipase , a lipolytic enzyme that is similar to LPL but is expressed by hepatocytes. Hepatic lipase appears to optimize the triglyceride content o remnant particles so that they can be cleared e f ciently by receptor-mediated mechanisms. The f nal phase o remnant clearance is receptor-mediated particle uptake. This is accomplished by one o our pathways. At the sinusoidal hepatocyte plasma membrane, remnant particles may be bound and taken up by the LDL receptor , the LDL receptor-related protein (LRP), or heparan sul ate proteoglycans. A ourth pathway is mediated by the combined activities o LRP and heparan sul ate proteoglycans. These redundant mechanisms allow or e f cient particle clearance, so that the hal -li e o remnants in the plasma is approximately 30 minutes
Human antibody-mediated diseases (Type II hypersensitivity)
Autoimmune hemolytic anemia Pemphigus vulgaris Goodpasture syndrome Rheumatic fever Myasthenia gravis Graves disease (hyperthyroidism) Pernicious anemia Bleeding Hemolysis, anemia Skin vesicles (bullae) Nephritis, lung hemorrhage Myocarditis, arthritis Muscle weakness, paralysis Hyperthyroidism Abnormal erythropoiesis, anemia Platelet membrane proteins (gpIIb/IIIa integrin) Erythrocyte membrane proteins (Rh blood group antigens, I antigen) Proteins in intercellular junctions of epidermal cells (desmoglein) Noncollagenous protein in basement membranes of kidney glomeruli and lung alveoli Streptococcal cell wall antigen; antibody crossreacts with myocardial antigen Acetylcholine receptor Thyroid stimulating hormone (TSH) receptor Intrinsic factor of gastric parietal cells Opsonization and phagocytosis of platelets Opsonization and phagocytosis of erythrocytes Antibody-mediated activation of proteases, disruption of intercellular adhesions Complement and Fc receptor-mediated inflammation Inflammation, macrophage activation Antibody inhibits acetycholine binding, down-modulates receptors Antibody-mediated stimulation of TSH receptors Neutralization of intrinsic factor, decreased absorption of vitamin B12
defects in receptors and transport systems
As we discussed in chapter 1, biologically active substances have to be actively transported across the cell membrane. In some cases transport is achieved by receptor-mediated endocytosis. A genetic defect in a receptor-mediated transport system is exemplified by familial hypercholesterolemia, in which reduced synthesis or function of LDL receptors leads to defective transport of LDL into the cells and secondarily to excessive cholesterol synthesis by complex intermediary mechanisms. In cystic fibrosis the transport system for chloride ions in exocrine glands, sweat ducts, lungs, and pancreas is defective. By mechanisms not fully understood, impaired chloride transport leads to serious injury to the lungs and pancreas (Chapter 10).
assembly and secretion of apolipoproteinE containing lipoproteins
Assembly and secretion of apolipoprotein B-containing lipoproteins. Chylomicrons and VLDL particles are assembled and secreted by similar mechanisms in the enterocyte and hepatocyte, respectively. The apoB mRNA (i.e., apoB48 or apoB100 mRNA) is translated by ribosomes to yield a protein that enters the lumen o the endoplasmic reticulum. I triglycerides are available, the apoB protein is lipidated by the action o microsomal triglyceride-trans er protein (MTP) in two distinct steps, accumulating triglyceride as well as cholesteryl ester molecules. The resulting chylomicron or VLDL particle is secreted by exocytosis into the lymphatics by enterocytes or into the plasma by hepatocytes. In the absence o triglycerides, the apoB protein is degraded (not shown). Sortilin (not shown) regulates the intracellular tra f cking o VLDL particles and can direct apoB to lysosomal-dependent degradation, thereby decreasing its secretion. Mipomersen inhibits translation o apoB by binding to apoB mRNA. Lomitapide inhibits lipidation o apoB by binding to MTP.
Epidemiology of alcoholism
At some point in their lives, alcoholism (alcohol dependence) is diagnosed in approximately 10% of men and 3% to 5% of women in the United States. More than 100 studies have shown that this disease clusters in families: the risk of developing alcoholism among persons with one affected parent is three to five times higher than for those with unaffected parents. Most twin studies have yielded concordance rates for DZ twins of less than 30% and for MZ twins in excess of 60%, with an estimated heritability of approximately 50%. Adoption studies have shown that the offspring of an alcoholic parent, even when raised by nonalcoholic parents, have a fourfold increased risk of developing the disorder.
alcoholism
At some point in their lives, alcoholism (alcohol dependence) is diagnosed in approximately 10% of men and 3% to 5% of women in the United States. More than 100 studies have shown that this disease clusters in families: the risk of developing alcoholism among persons with one affected parent is three to five times higher than for those with unaffected parents. Most twin studies have yielded concordance rates for DZ twins of less than 30% and for MZ twins in excess of 60%, with an estimated heritability of approximately 50%. Adoption studies have shown that the offspring of an alcoholic parent, even when raised by nonalcoholic parents, have a fourfold increased risk of developing the disorder. To control for possible prenatal effects from an alcoholic mother, some studies have included the offspring of alcoholic fathers only. The results have remained the same. These data argue that there may be genes that predispose some people to alcoholism. Alcohol dependence can be divided into different subtypes, and some researchers distinguish two major types of alcoholism. Type I is characterized by a later age of onset (after 25 years of age), occurrence in both males and females, and greater psychological dependency on alcohol. Type I alcoholics are more likely to be introverted, solitary drinkers. This form of alcoholism is less likely to cluster in families (one study yielded a heritability estimate of 0.21), has a lesssevere course, and is more easily treated. Type II alcoholism is seen predominantly in males, typically occurs before 25 years of age, and tends to involve persons who are extroverted and thrill-seeking. This form is more difficult to treat successfully and tends to cluster more strongly in families, with heritability estimates ranging from 0.55 to more than 0.80. It has long been known that an individual's physiological response to alcohol can be influenced by variation in the key enzymes responsible for alcohol metabolism: alcohol dehydrogenase (ADH), which converts ethanol to acetaldehyde, and aldehyde dehydrogenase (ALDH), which converts acetaldehyde to acetate. In particular, an allele of the ALDH2 gene (ALDH2*2) results in excessive accumulation of acetaldehyde and thus in facial flushing, nausea, palpitations, and lightheadedness. Because of these unpleasant effects, persons who have the ALDH2*2 allele are much less likely to become alcoholics. This protective allele is common in some East Asian populations, with a frequency of approximately 40%, but is rare in most other populations. A number of genome scans have been undertaken in large cohorts of alcoholics and controls. One of the most consistent findings is that variants in genes that encode components of gamma-aminobutyric acid (GABA) receptors are associated with addiction to alcohol. This finding is biologically plausible, because the GABA neurotransmitter system inhibits excitatory signals in neurons, exerting a calming effect. Alcohol has been shown to increase GABA release, and allelic variation in GABA receptor genes might modulate this effect. It should be underscored that we refer to genes that might increase one's susceptibility to alcoholism. This is obviously a disease that requires an environmental component, regardless of one's genetic constitution. Twin and adoption studies show that alcoholism clusters quite strongly in families, reflecting a genetic contribution to this disease. Familial clustering is particularly strong for type II alcoholism (earlyonset form primarily affecting males).
What are examples of diseases where immunodeficiency is seen that involve DNA instability?
Ataxia telangiectasia Bloom syndrome Fanconi anemia
What are increased concentrations of certain lipoproteins in the circulation strongly associated with?
Atherosclerosis
What does dyslipidemia play a significant role in the development of?
Atherosclerosis Increased incidence of CHD with high LDL- and low HDL-cholesterol is well documented
Decreased HDL cholesterol is an independent risk factor for the development of what? What are some rare genetic defects in HDL metabolism ?
Atherosclerosis and cardiovascular disease apoAI, ABCA1, and LCAT - each of these defects are the results of decreased HDL
Fibrates have a beneficial anti-inflammatory effect decreases the vulnerability of what?
Atherosclerotic plaques to rupture
What is the prototypical muscarinic receptor agonist and what are its clinical uses?
Atropine 1- Induce mydriasis (pupil dilation) for ophthalmologic eye exam 2- Reverse symptomatic bradycardia 3- Inhibit excessive salivation and mucus secretion during surgery 4- Prevent Vagal reflexes induced by surgical trauma of visceral organs 5- Counteract the effects of muscarine poisoning from certain mushrooms 6- Ameliorate nausea, particularly that associated with chemotherapy marginal activity at nicotinic receptors, extremely high doses o atropine are required or any e ects to be seen at the NMJ. Similarly, because nicotinic receptors are primarily responsible or excitatory transmission at autonomic ganglia, atropine produces only partial block at these sites and only at relatively high doses
What occurs in Lambert-Eaton syndrome?
Autoantibodies against Ca2+ channels also characterized by muscle weakness
The antibodies that cause Antibody mediated diseases are most often what?
Autoantibodies against self antigens are less specific for foreign (eg: microbial) antigens
What occurs in Myasthenia gravis?
Autoantibodies are generated against Nicotinic Ach receptors (Nm) receptors at the NMJ These antibodies both induce Nm receptor internalization and block the ability of ACh to activate the receptors As a result, patients with myasthenia gravis present with significant weakness
Non-neuronal Ach act in an ..... and ..... fashion on nicotonic and muscarinic receptors?
Autocrine Paracrine
Examples of Antibody-mediated diseases (Type II Hypersensitivity)
Autoimmune hemolytic anemia Red cell membrane proteins (Rh blood group antigens, I antigen) Opsonization and phagocytosis of red cells Hemolysis, anemia Autoimmune thrombocytopenic purpura Platelet membrane proteins (Gpllb : Illa integrin) Opsonization and phagocytosis of platelets Bleeding Pemphigus vulgaris Proteins in intercellular junctions of epidermal cells (epidermal cadherin) Antibody-mediated activation of proteases, disruption of intercellular adhesions Skin vesicles (bullae) Vasculitis caused by ANCA Neutrophil granule proteins, presumably released from activated neutrophils Neutrophil degranulation and inflammation Vasculitis Goodpasture syndrome Noncollagenous protein in basement membranes of kidney glomeruli and lung alveoli Complement- and Fc receptor-mediated inflammation Nephritis, lung hemorrhage Acute rheumatic fever Streptococcal cell wall antigen; antibody cross-reacts with myocardial antigen Inflammation, macrophage activation Myocarditis, arthritis Myasthenia gravis Acetylcholine receptor Antibody inhibits acetylcholine binding, down-modulates receptors Muscle weakness, paralysis Graves disease (hyperthyroidism) TSH receptor Antibody-mediated stimulation of TSH receptors Hyperthyroidism Insulin-resistant diabetes Insulin receptor Antibody inhibits binding of insulin Hyperglycemia, ketoacidosis Pernicious anemia Intrinsic factor of gastric parietal cells Neutralization of intrinsic factor, decreased absorption of vitamin B12 Abnormal erythropoiesis, anemia
What symptoms occur in 75% of patients with LEMS?
Autonomic manifestations: dry mouth, impotence, decreased sweating, orthostatic hypotension, and slow pupillary reflexes
What type of inheritance pattern is seen in Vascular EDS?
Autosomal dominant because vascular-type EDS results from mutations involving a structural protein (rather than an enzyme protein) because blood vessels and intestines are known to be rich in collagen type III, an abnormality of this collagen is consistent with severe structural defects (e.g., vulnerability to spontaneous rupture) in these organs.
What is Familial defective apoB100?
Autosomal dominant disorder in which mutations in the apoB100 protein lead to decreased affinity of the LDL particle or LDL receptors - Gain-of -function mutations in PCSK9 have been identified in families with clinical features similar to FH - this disorder reflects increased PCSK9 function and decreased LDL receptor expression on cell surfaces
autosomal dominant disorderss
Autosomal dominant disorders are manifested in the heterozygous state, so at least one parent of an index case is usually affected; both males and females are affected, and both can transmit the condition. When an affected person marries an unaffected one, every child has one chance in two of having the disease. In addition to these basic rules, autosomal dominant conditions are characterized by the following: • With every autosomal dominant disorder, some proportion of patients do not have affected parents. Such patients owe their disorder to new mutations involv ing either the egg or the sperm from which they were derived. Their siblings are neither affected nor at increased risk for disease development. The proportion of patients who develop the disease as a result of a new mutation is related to the effect of the disease on reproductive capability. If a disease markedly reduces reproductive fitness, most cases would be expected to result from new mutations. Many new mutations seem to occur in germ cells of relatively older fathers. • Clinical features can be modified by variations in penetrance and expressivity. Some individuals inherit the mutant gene but are phenotypically normal. This is referred to as incomplete penetrance. Penetrance is expressed in mathematical terms. Thus, 50% penetrance indicates that 50% of those who carry the gene express the trait. In contrast to penetrance, if a trait is seen in all individuals carrying the mutant gene but is expressed differently among individuals, the phenomenon is called variable expressivity. For example, manifestations of neurofibromatosis type 1 range from brownish spots on the skin to multiple skin tumors and skeletal deformities. The mechanisms underlying incomplete penetrance and variable expressivity are not fully understood, but they most likely result from effects of other genes or environmental factors that modify the phenotypic expression of the mutant allele. For example, the phenotype of a patient with sickle cell anemia (resulting from mutation at the β-globin locus) is influenced by the genotype at the α-globin locus, because the latter influences the total amount of hemoglobin made (Chapter 14). The influence of environmental factors is exemplified by individuals heterozygous for familial hypercholesterolemia. The expression of the disease in the form of atherosclerosis is conditioned by the dietary intake of lipids. • In many conditions the age at onset is delayed; symptoms and signs may not appear until adulthood (as in Huntington disease). The biochemical mechanisms of autosomal dominant disorders depend upon the nature of the mutation and the type of protein affected. Most mutations lead to the reduced production of a gene product or give rise to a dysfunctional or inactive protein. Whether such a mutation gives rise to dominant or recessive disease depends on whether the remaining copy of the gene is capable of compensating for the loss. Thus, understanding the reasons why particular loss-of-function mutations give rise to dominant vs. recessive disease patterns requires an understanding of the biology. Many autosomal dominant diseases arising from deleterious mutations fall into one of a few familiar patterns: 1. Those involved in regulation of complex metabolic pathways that are subject to feedback inhibition. Membrane receptors such as the low-density lipoprotein (LDL) receptor provide one such example; in familial hypercholesterolemia, discussed later, a 50% loss of LDL receptors results in a secondary elevation of cholesterol that, in turn, predisposes to atherosclerosis in affected heterozygotes. 2. Key structural proteins, such as collagen and cytoskeletal elements of the red cell membrane (e.g., spectrin). The biochemical mechanisms by which a 50% reduction in the amounts of such proteins results in an abnormal phenotype are not fully understood. In some cases, especially when the gene encodes one subunit of a multimeric protein, the product of the mutant allele can interfere with the assembly of a functionally normal multimer. For example, the collagen molecule is a trimer in which the three collagen chains are arranged in a helical configuration. Each of the three collagen chains in the helix must be normal for the assembly and stability of the collagen molecule. Even with a single mutant collagen chain, normal collagen trimers cannot be formed, and hence there is a marked deficiency of collagen. In this instance the mutant allele is called dominant negative, because it impairs the function of a normal allele. This effect is illustrated by some forms of osteogenesis imperfecta, characterized by marked deficiency of collagen and severe skeletal abnormalities (Chapter 26). Less common than loss-of-function mutations are gainof-function mutations, which can take two forms. Some mutations result in an increase in a protein's normal function, for example, excessive enzymatic activity. In other cases, mutations impart a wholly new activity completely unrelated to the affected protein's normal function. The transmission of disorders produced by gain-of-function mutations is almost always autosomal dominant, as illustrated by Huntington disease (Chapter 28). In this disease the trinucleotide-repeat mutation affecting the Huntington gene (see later) gives rise to an abnormal protein, called huntingtin, that is toxic to neurons, and hence even heterozygotes develop a neurologic deficit Table 5-1 lists common autosomal dominant disorders. Many are discussed more logically in other chapters. A few conditions not considered elsewhere are discussed later in this chapter to illustrate important principles.
Clinical commentary- familial hypercholesterolemia
Autosomal dominant familial hypercholesterolemia (FH) is an important cause of heart disease, accounting for approximately 5% of myocardial infarctions (MIs) in persons younger than 60 years. FH is one of the most common autosomal dominant disorders: in most populations surveyed to date, about 1 in 500 persons is a heterozygote. Plasma cholesterol levels are approximately twice as high as normal (i.e., about 300-400 mg/ dL), resulting in substantially accelerated atherosclerosis and the occurrence of distinctive cholesterol deposits in skin and tendons, called xanthomas (Fig. 12-6). Data compiled from multiple studies showed that approximately 75% of men with FH developed coronary artery disease, and 50% had a fatal MI, by age 60 years. The corresponding percentages for women were lower (45% and 15%, respectively), because women generally develop heart disease at a later age than men Consistent with Hardy-Weinberg predictions (see Chapter 4), about 1/1,000,000 births is homozygous for the FH gene. Homozygotes are severely affected, with cholesterol levels ranging from 600 to 1200 mg/dL. Most homozygotes experience MIs before 20 years of age, and an MI at 18 months of age has been reported. Without treatment, most FH homozygotes die before the age of 30 years All cells require cholesterol as a component of their plasma membrane. They can either synthesize their own cholesterol, or preferentially they obtain it from the extracellular environment, where it is carried primarily by low-density lipoprotein (LDL). In a process known as endocytosis, LDL-bound cholesterol is taken into the cell via LDL receptors on the cell's surface. FH is caused by a reduction in the number of functional LDL receptors on cell surfaces. Cellular cholesterol uptake is reduced, and circulating cholesterol levels increase Much of what we know about endocytosis has been learned through the study of LDL receptors. The process of endocytosis and the processing of LDL in the cell is described in detail in Figure 12-7. These processes result in a fine-tuned regulation of cholesterol levels within cells, and they influence the level of circulating cholesterol as well. The identification of the LDL receptor gene (LDLR) in 1984 was a critical step in understanding exactly how LDL receptor defects cause FH. More than a thousand different mutations, two thirds of which are missense and nonsense substitutions, have been identified. Most of the remaining mutations are insertions and deletions, many of which arise from unequal crossovers (see Chapters 5 and 6) that occur between Alu repeat sequences (see Chapter 2) scattered throughout the gene. The LDLR mutations can be grouped into five broad classes, according to their effects on the activity of the receptor: • Class I mutations in LDLR result in no detectable protein product. Thus, heterozygotes would produce only half the normal number of LDL receptors. • Class II mutations result in production of the LDL receptor, but it is altered to the extent that it cannot leave the endoplasmic reticulum. It is eventually degraded . • Class III mutations produce an LDL receptor that is capable of migrating to the cell surface but is incapable of normal binding to LDL . • Class IV mutations, which are comparatively rare, produce receptors that are normal except that they do not migrate specifically to coated pits and thus cannot carry LDL into the cell. • Class V mutations produce an LDL receptor that cannot disassociate from the LDL particle after entry into the cell. The receptor cannot return to the cell surface and is degraded Each class of mutations reduces the number of effective LDL receptors, resulting in decreased LDL uptake and hence elevated levels of circulating cholesterol. The number of effective receptors is reduced by about half in FH heterozygotes, and homozygotes have virtually no functional LDL receptors. Understanding the defects that lead to FH has helped in the development of effective therapies for the disorder. Dietary reduction of cholesterol (primarily through the reduced intake of saturated fats) has only modest effects on cholesterol levels in FH heterozygotes. Because cholesterol is reabsorbed into the gut and then recycled through the liver (where most cholesterol synthesis takes place), serum cholesterol levels can be reduced by the administration of bile-acid absorbing resins, such as cholestyramine. The absorbed cholesterol is excreted. It is interesting that reduced recirculation from the gut causes the liver cells to form additional LDL receptors, lowering circulating cholesterol levels. However, the decrease in intracellular cholesterol also stimulates cholesterol synthesis by liver cells, so the overall reduction in plasma LDL is only about 15% to 20%. This treatment is much more effective when combined with one of the statin drugs (e.g., lovastatin, pravastatin), which reduce cholesterol synthesis by inhibiting 3-hydroxy-3methylglutaryl coenzyme A (HMG-CoA) reductase. Decreased synthesis leads to further production of LDL receptors. When these therapies are used in combination, serum cholesterol levels in FH heterozygotes can often be reduced to approximately normal levels. The picture is less encouraging for FH homozygotes. The therapies mentioned can enhance cholesterol elimination and reduce its synthesis, but they are largely ineffective in homozygotes because these persons have few or no LDL receptors. Liver transplants, which provide hepatocytes that have normal LDL receptors, have been successful in some cases, but this option is often limited by a lack of donors. Plasma exchange, carried out every 1 to 2 weeks, in combination with drug therapy, can reduce cholesterol levels by about 50%. However, this therapy is difficult to continue for long periods. Somatic cell gene therapy, in which hepatocytes carrying normal LDL receptor genes are introduced into the portal circulation, is now being tested (see Chapter 13). It might eventually prove to be an effective treatment for FH homozygotes. As discussed in the main text, FH can also be caused by inherited mutations in the gene that encodes apolipoprotein B. In addition, a small number of FH cases are caused by mutations in the gene that encodes PCSK9 (proprotein convertase subtilisin/kexin type 9), an enzyme that plays a key role in degrading LDL receptors. Gain-of-function mutations in the PCSK9 gene reduce the number of LDL receptors, causing FH. Loss-of-function mutations in this gene can increase the number of LDL receptors, resulting in exceptionally low circulating LDL levels. These findings have led to the development of drugs that inhibit PCSK9 activity, thus lowering LDL cholesterol levels. These drugs, which are in late-stage clinical trials, can reduce LDL cholesterol levels by approximately 50% in the general population of persons with hypercholesterolemia and produce significant effects even in those who are using statin drugs. The FH story illustrates how medical genetics research has made important contributions to both the understanding of basic cell biology and advancements in clinical therapy. The process of receptor-mediated endocytosis, elucidated largely by research on the LDL receptor defects, is of fundamental significance for cellular processes throughout the body. Equally, this research, by clarifying how cholesterol synthesis and uptake can be modified, has led to significant improvements in therapy for this important cause of heart disease. The discovery of rare mutations in PCSK9 has led to PCSK9 inhibitor drugs that may benefit millions of persons with high cholesterol levels.
Inborn errors of metabolism are almost always?
Autosomal recessive
What occurs in Familial lipoprotein lipase deficiency?
Autosomal recessive disorder caused by the absence of active LPL - This condition may be diagnosed by testing the plasma or lipase activity following an in fusion of heparin, which competes for binding sites on endothelial cells and dislodges LPL molecules into the plasma
autosomal recessive disorders
Autosomal recessive traits make up the largest category of Mendelian disorders. They occur when both alleles at a given gene locus are mutated. These disorders are characterized by the following features: (1) The trait does not usually affect the parents of the affected individual, but siblings may show the disease; (2) siblings have one chance in four of having the trait (i.e., the recurrence risk is 25% for each birth); and (3) if the mutant gene occurs with a low frequency in the population, there is a strong likelihood that the affected individual (proband) is the product of a consanguineous marriage. The following features generally apply to most autosomal recessive disorders and distinguish them from autosomal dominant diseases: • The expression of the defect tends to be more uniform than in autosomal dominant disorders. • Complete penetrance is common. • Onset is frequently early in life. • Although new mutations associated with recessive disorders do occur, they are rarely detected clinically. Since the individual with a new mutation is an asymptomatic heterozygote, several generations may pass before the descendants of such a person mate with other heterozygotes and produce affected offspring. • Many of the mutated genes encode enzymes. In heterozygotes, equal amounts of normal and defective enzyme are synthesized. Usually the natural "margin of safety" ensures that cells with half the usual complement of the enzyme function normally. Autosomal recessive disorders include almost all inborn errors of metabolism. The various consequences of enzyme deficiencies are discussed later. The more common of these conditions are listed in Table 5-2. Most are presented elsewhere; a few prototypes are discussed later in this chapter
A boy scout troop has just arrived at the emergency department. They had to cut their annual campout short because several of the members of the troop now appear to be ill. When finding food for lunch, the boys were excited because they found some delicious looking mushrooms in the woods to make a stew for their group! The boys are now presenting to the ED with diarrhea and abdominal cramps. Upon exam they are sweating, wheezing, and have miosis. Which of the following drugs is most likely given to the boy scouts to resolve their symptoms? A) Pralidoxime B) Atropine C) Bethanechol D) Methacholine E) Vesamicol
B) Atropine
Mark just underwent a hernia repair surgery and had difficulty urinating afterwards. The doctor prescribed a medication to help relieve his symptoms. Following administration, Mark begins experiencing bronchospasms and is unable to breathe. The doctor checks his chart and sees he is an asthma patient. What medication did the doctor prescribe? A) Methacholine B) Bethanechol C) Atropine D) Pancuronium E) Pilocarpine
B) Bethanechol
Douglas, a 38 -year-old construction worker, is seeking medical consultation regarding his chief complaint of shortness of breath and heart palpitations. His vitals upon entry are normal. He explains that the shortness of breath becomes much worse when he is under a heavy workload at the jobsite and that his symptoms as a whole have become more noticeable over time, precipitating his visit to the clinic. After running the appropriate tests, Douglas's physician diagnoses him with a familial, autosomal dominant disorder that has resulted in the thickening of portions of his left ventricle. The mutated gene causing the disorder most likely encodes for which of the following? A) Titin B) Cardiac β-myosin heavy chain C) Apolipoprotein B D) Cardiac potassium channel α subunit E) WNK 1 kinase
B) Cardiac β-myosin heavy chain
Acetylcholine that is released into the neuromuscular junction primarily triggers muscle contraction, however it also can have a secondary use. Which most likely is the effect of that secondary function? A) Decrease binding at pre-synaptic receptors B) Ensure sufficient acetylcholine release C) Decrease binding at post-synaptic receptors D) Reduce additional acetylcholine release E) Muscle spasm
B) Ensure sufficient acetylcholine release
A 25-year-old female schedules an appointment with her primary care physician for her annual physical. Her 31-year-old sister was recently diagnosed with peripheral artery disease and she is worried that she may be at risk of developing the disease as well. Her physician orders genetic testing and confirms she has a mutation in a gene that leads to decreased affinity of apo E2 for the LDL receptor. Which of the following disorders is she most at risk of developing? A) Lipoprotein lipase deficiency B) Familial dysbetalipoproteinemia C) Polygenic hypercholesterolemia D) Familial hypercholesterolemia E) Familial hypertriglyceridemia
B) Familial dysbetalipoproteinemia
Mark, a 49-year-old has been making visits to his primary care provider (PCP) for the past two years since he was diagnosed with Familial Hypercholesterolemia (FH). His PCP recently had him undergo a genetic test to determine what gene mutation is the cause of his disease. When he receives his results, he learns that he has an uncommon mutation causing his FH, the PCSK9 gene. What effect on the PCSK9 gene causes FH? A) Point mutation leading to increased numbers of LDL-receptors B) Gain of function mutation leading to decreased numbers of LDL-receptors C) No LDL receptors are produced D) Loss of function leading to increased numbers of LDL-receptors E) Insertion mutation leading to increased numbers of LDL-receptors
B) Gain of function mutation leading to decreased numbers of LDL-receptors
Two siblings, Arthur (M) and Jeanne (F) in their late 20s were experiencing fever, dark colored urine, fatigue, and pale skin after they both took a high dosage of aspirin. Arthur complains to the physician that his symptoms are significantly more severe than Jeanne's. Blood test results for the two return and revealed red cell hemolysis and low levels of glucose-6-phosphate dehydrogenase, with Arthur having a much higher proportion of defective red blood cells. They were then both diagnosed with glucose-6-phosphate dehydrogenase deficiency. Which of the following reasons might explain why Arthur's presentation of the disease is more severe than Jeanne's? A) Glucose-6-phosphate dehydrogenase deficiency is an autosomal dominant disorder and tends to show incomplete penetrance. B) Glucose-6-phosphate dehydrogenase deficiency is an X-linked recessive disorder. Arthur, being a male, is hemizygous for the disease, and experiences the full expression of the disease. Jeanne, being female, most likely does not experience the full expression of the disease due to random inactivation of the X-chromosomes. C) Glucose-6-phosphate dehydrogenase deficiency is an autosomal recessive disorder that displays variable expressivity. The difference in severity between Arthur and Jeanne's disorder is due to slightly different phenotypes being expressed, even though their genotypes are the same. D) Glucose-6-phosphate dehydrogenase deficiency is an X-linked dominant disorder. Arthur, being a male, is hemizygous for the disease, and experiences the full expression of the disease. Jeanne, being female, most likely does not experience the full expression of the disease due to random inactivation of the X-chromosomes. E) Glucose-6-phosphate dehydrogenase deficiency is a pleiotropic disease that can be caused by multiple different mutated genes. Arthur just had bad luck that he had mutations in genes that caused a more severe presentation of the disease than Jeanne.
B) Glucose-6-phosphate dehydrogenase deficiency is an X-linked recessive disorder. Arthur, being a male, is hemizygous for the disease, and experiences the full expression of the disease. Jeanne, being female, most likely does not experience the full expression of the disease due to random inactivation of the X-chromosomes.
A 47 year old woman visits her local clinic with the chief complaint of skin rash and painful joints. The physician notices a butterfly-like rash on her face. After finishing his examination, he diagnoses her with systemic lupus erythematosus (SLE). What type of hypersensitivity is that and what is the underlying mechanism? A) T cell mediated (type IV) hypersensitivity; cell killing by CD8+ t cells B) Immune complex mediated (type III) hypersensitivity; circulating antibody binds free antigen and cause inflammation at the site of deposition C) Immune complex mediated (type III) hypersensitivity; antibody binds to surface antigen and triggering inflammation D) Antibody mediated (type II) hypersensitivity; antibody binds to surface antigen and triggering inflammation E) Antibody mediated (type II) hypersensitivity; circulating antibody binds free antigen and cause inflammation at the site of deposition
B) Immune complex mediated (type III) hypersensitivity; circulating antibody binds free antigen and cause inflammation at the site of deposition
A 29-year-old male presents to an ophthalmologist saying that he was at work and noticed the appearance of a "black curtain" moving across his visual field and the sudden appearance of many floaters in his vision. The patient was diagnosed with a retinal detachment and was taken into surgery to repair the detachment. The patient also reported a history of hypotonia, joint laxity and congenital scoliosis, telling the physician that he had been diagnosed with the most common type of autosomal recessive Ehlers-Danlos Syndrome. Which of the following classifications of Ehlers-Danlos Syndrome does the patient most likely have? A) Classic B) Kyphoscoliosis C) Vascular D) Hypermobility E) Dermatosparaxis
B) Kyphoscoliosis
Bob is a 74 year old male that presents to his primary care physician with his wife, who states that she has noticed changes in Bob's cognitive function and behavior within the past three months. She explains that he forgets her name quite often and has frequent mood swings. She reports that he is a retired high school mathematics teacher, but now has difficulty with simple calculations. Since his retirement twenty years ago, Bob has not been on a healthy diet and drinks three glasses of whiskey and Coca-Cola every night. He was recently diagnosed with early-stage cirrhosis last month. The doctor believes that Bob is showing early signs of Alzheimer's Disease and prescribes him a medication to treat his symptoms. Based on the diagnosis and Bob's past history, which medication did his doctor most likely prescribe? A) Donepezil B) Rivastigmine C) Galantamine D) Tacrine E) Methacholine
B) Rivastigmine
A microscopic aggregation of epithelioid cells, usually surrounded by a collar of lymphocytes, is referred to as a granuloma. This pattern of inflammation is typically associated with: A) Strong TH17 cell activation and a high-level production of IL-17 B) Strong TH1 cell activation and a high-level production of IFN-γ C) Strong TH17 cell activation and a high-level production of IFN-γ D) Strong TH1 cell activation and a high-level production of IL-17
B) Strong TH1 cell activation and a high-level production of IFN-γ
An autopsy was done on a 32-year old man who died from aortic rupture. The family noted how he would throughout his life be able to contort his limbs in unusual positions and had unusually stretchy skin. What disease did the man most likely die from? A) Kyphoscoliosis EDS B) Vascular EDS C) Classical EDS D) Marfan's Syndrome E) Familial Hypercholesterolemia
B) Vascular EDS
Which of the following sets of values would indicate a trait that is determined entirely by genes? A) CMZ=1.0, CDZ=0.0 B)CMZ=1.0, CDZ=0.5 C) CMZ=0.5, CDZ=0.5 D) CMZ=0.5, CDZ=1.0 E) CMZ=0.0, CDZ=1.0
B)CMZ=1.0, CDZ=0.5
: An 18 year old male is celebrating his high school graduation and decides to drink alcohol for the first time with his friends. After a few beers, he starts to notice some prominent facial flushing, nausea, palpitations and some light headedness. Which of the following alleles has he most likely inherited to cause these symptoms? A. FTO B. ALDH2*2 C. TCF7L2 D. HLA DR3 E. MLH1
B. ALDH2*2
A 23-year-old medical student was studying vigorously for a pharmacology exam that he had the next day. Unfortunately, he had fallen behind in keeping up with the lectures and was only starting the process of reviewing them the night before. As one can imagine, he did not get much sleep and went into the exam forgetting most of what he reviewed the previous night. What is the most likely rationale for why his memory failed him in his time of need? A. It was too much information for anyone to retain in that amount of time B. Ach levels were elevated C. There was not enough Ach traveling through the synaptic cleft D. Cortisol levels were elevated E. His neuronal receptor subtypes underwent changes that altered the functional roles within the central nervous system
B. Ach levels were elevated
A newborn baby is diagnosed with Niemann-Pick Disease, a lysosomal storage disease caused by dysfunction in the transport of cholesterol from the lysosome. During normal cholesterol metabolism, intracellular release of free cholesterol from the lysosome into the cytoplasm activates which of these processes? A. Synthesis of more cholesterol B. Activity of acyl-coenzyme A C. Synthesis of LDL receptors D. Activity of HMG CoA reductase
B. Activity of acyl-coenzyme A
A 45 years old male visits a clinic with chief complaints of abdominal cramping, constipation, inadequate emptying of the bladder and appetite loss. He was prescribed a muscarinic receptor agonist that will treat urinary retention and inadequate emptying of the bladder as in postoperative urinary retention. Which of the following drugs was he prescribed? A. Carbachol B. Bethanechol C. Methacholine D. Cevimeline E. Pilocarpine
B. Bethanechol
Darrell came to the clinic with proteinuria, hematuria, hypertension, along with edema in his legs. Later tests show that he has been diagnosed with glomerulonephritis. Which cytokines are most responsible for increasing vascular permeability? A. C4a & C4b B. C5a & C3a C. C4b & C3a D. C5b & C4a E. C5a & C3b
B. C5a & C3a
A 10-year-old boy was brought to the physician's office by his parents to discuss the results of recent tests. Based on his lipid panel and family history the boy has been diagnosed with familial hypercholesterolemia. The physician has decided to prescribe the boy a medication that will bind to bile acids causing the liver to produce more. What has he prescribed? A. Ezetimibe B. Colesevelam C. Iomitapide D. Simvastatin E. Gemfibrozil
B. Colesevelam
A 24-year-old female presents to her physician with complaints of blurred and double vision, droopy eyelids, and muscle weakness. She says that it gets worse throughout the day. Her physician performs the ice pack test by placing the ice pack on the droopy eyelid for 2 minutes, and the patient's symptoms improved. Which of the following drugs would be avoided for treatment of the most likely diagnosis? A. Pyridostigmine B. D-penicillamine C. Prednisone D. Azathioprine E. Beta-blockers
B. D-penicillamine
A 45-year old man is brought to the clinic by his daughter who notes that he's had uncontrollable sweating, salivation, diarrhea and vomiting since he started farming again. If the doctor is to assume the patient is suffering from exposure to insecticides, which of the following could be of help to this patient? A. Atropine B. Pralidoxime C. Botulinum toxin D. Edrophonium E.Isoflurophate
B. Pralidoxime
A 65-year-old female presents to the neurologist with symptoms of mild generalized fatigue and asymmetric ptosis. Her condition improves five minutes after IV administration of an acetylcholinesterase inhibitor. Serum antibody testing is positive for AchR ab and a chest CT reveals no thymoma. Which treatment is the best choice to provide long term relief to the patient? A. Prednisone B. Pyridostigmine C. Atropine D. Thymectomy E. Plasmapheresis and IVIG
B. Pyridostigmine
A 58 y/o post-menopausal woman makes an appointment to see her Primary Care Physician due to recently noticing yellow spots near her eyelids and joints. Her PCP decides to run a couple tests, including a lipid panel. The lab results show elevated triglyceride plasma levels as well as elevated cholesterol plasma levels. Her PCP recommends further genetic testing to confirm a diagnosis, which the patient agrees with. Genetic testing reveals that the patient has defects in the gene that encodes for apolipoprotein E and the patient is diagnosed with dysbetalipoproteinemia. What is the most appropriate therapy to treat this condition? A. A fat-free diet and avoidance of alcohol and glucocorticoid therapy B. Decrease intake of fat and cholesterol, weight loss, omission of alcohol, and fibrates C. Lifestyle modification and use of Cholestyramine D. Lomitapide and exercise E. Diet rich in cholesterol and use of Ezetimibe
B. Decrease intake of fat and cholesterol, weight loss, omission of alcohol, and fibrates
Which of the following describes the clinical manifestation of a patient with a familial null mutation in the apoB100 protein? A. Decreased hepatic clearance of LDLs resulting in higher triglyceride levels B. Decreased binding with LDL receptors causing an overall increase in cholesterol C. Decreased formation of LDLs resulting in increased cholesterol levels D. Increased binding with LDL receptors causes an overall decrease in circulating LDLs E. Increased dissolution of LDLs resulting in more free floating cholesterol
B. Decreased binding with LDL receptors causing an overall increase in cholesterol
A student is researching the effects of receptor activation, focusing on receptors associated with the heart. She notes that when these receptors are antagonized, heart conduction velocity increases. Which of the following physiological changes will be observed upon stimulation of these receptors with an agonist? A. Increased cAMP B. Decreased cAMP C. Increased IP3 D. Decreased IP3 E. Influx of Na, Efflux of K
B. Decreased cAMP
A 42-year-old female presents to the emergency department with sharp upper left quadrant abdominal pain and abdominal swelling. The patient has no major history of medical conditions or surgeries and has not had any recent dietary changes. After an unremarkable physical exam, EKG and MRI, the patient is found to have elevated amylase and triglyceride levels however her other labs and exams were unremarkable. What is the most likely etiology of the patient's medical condition? A. Over production of hepatic VLDL B. Deficiency of Apolipoprotein C-11 C. Impaired binding of LDL particles to LDL receptors D. Excessive fatty acid intake E. Decreased clearance of lipoproteins due to Apo E2 dysfunction
B. Deficiency of Apolipoprotein C-11
A middle-aged man comes into the ER complaining of fatigue and muscle weakness. When conducting his history and physical, it is noted that he was previously prescribed atorvastatin (most potent one) along with another drug. A diagnosis of rhabdomyolysis (due to increased creatine kinase levels) is established. What other drug, when prescribed with a statin, can cause this condition? A. Bile acid resin B. Fibrate C. Ezetimibe D. Glucocorticoid E. PCSK9 inhibitor
B. Fibrate
During the recent Covid-19 outbreak, a recovered patient donates blood in an attempt to provide antibodies to sick patients within the hospital. One patient undergoes a blood transfusion and begins presenting with symptoms typical of acute serum sickness. What is the pathogenic sequence typically seen in this form of a response? A. Accumulation of Compliment → Formation of a MAC → Lysis of cells due to penetrations made by the MAC B. Formation of immune complexes → Deposition of immune complexes → Inflammation and Tissue Injury C. CD8+ T-Cells activated → CD8+ T-Cells destroy antigen presenting cells D. Antibodies reactive to Acetylcholine target the motor end plate → Leads to block of NM transmission E. Sensitization with IgE → Systemic IgE release upon 2nd encounter
B. Formation of immune complexes → Deposition of immune complexes → Inflammation and Tissue Injury
A 30-year-old female is pregnant with her second child and comes in for a routine checkup with her OBGYN. The mother has O-negative blood and is carrying a fetus with A-positive blood. Usually the doctor would give injections of Rh immune globulin to destroy the fetal erythrocytes in the mother's bloodstream before anti-Rh antibodies are created. However, in this case the doctor was not as concerned about giving her the injections. What is the most likely explanation for this? A. Only Rh+ mothers carrying an Rh- fetus need Rh immune globulin injections. B. Her anti-A antibodies quickly destroy the fetal RBCs before she can form anti-Rh antibodies. C. Rh immune globulin injections are only required during a female's third pregnancy. D. Her anti-B antibodies quickly destroy the fetal RBCs before she can form anti-Rh antibodies.
B. Her anti-A antibodies quickly destroy the fetal RBCs before she can form anti-Rh antibodies.
A 28 year old female is pregnant with her second child. She reports to her OBGYN for a routine check up. The physician informs her that she will need a Rhogam shot to protect her baby from erythroblastosis fetalis, a dangerous condition that can cause fetal anemia, spontaneous abortion or still birth. The woman states she did not receive any injections like this with her first pregnancy. What explanation correctly explains why the mother requires Rhogam this pregnancy? A. Her first child was Rh- and she is Rh- and the doctor wants to make sure the baby will also be Rh-. B. Her first child was Rh+ and after delivery she developed antibodies against the Rh antigen and the doctor wants to prevent the mother's antibodies from attacking the new fetus. C. Her first child was Rh+ and after the delivery she developed antigens against the Rh antibody and the doctor wants to prevent the mother's antigens from attacking the new fetus. D. Her first child was Rh+ and she is Rh+ and the physician wants to make sure the baby will also be Rh+. E.Her first child was Rh- and this child is Rh+ so therefore she will need the Rhogam.
B. Her first child was Rh+ and after delivery she developed antibodies against the Rh antigen and the doctor wants to prevent the mother's antibodies from attacking the new fetus.
A 58-year-old male presents to the clinic with complaints of excessive pain localized to the site of where he had received a booster vaccination two days prior. The patient states that he began to notice the pain "a few hours" after receiving the vaccine but decided to come in when the pain would not subside. Upon examination, the painful area is slightly elevated, red, and warm to the touch. Suspecting a localized type III sensitivity reaction, the physician performs a biopsy and the histological assessment reveals necrosis. What is the likely etiology and type of necrosis present? A. Immune complexes have precipitated in the vessel wall, causing liquefactive necrosis. B. Immune complexes have precipitated in the vessel wall, causing fibrinoid necrosis. C. Antigens from the vaccine have bound to endothelial cell membranes, leading to immune cells targeting those endothelial cells for apoptosis, causing liquefactive necrosis. D. Antigens from the vaccine have bound to endothelial cell membranes, leading to immune cells targeting those endothelial cells for apoptosis, causing fibrinoid necrosis. E. Trauma from the needle has damaged the subcutaneous vasculature, causing a localized ischemic area, resulting in fibrinoid necrosis
B. Immune complexes have precipitated in the vessel wall, causing fibrinoid necrosis.
A patient presents to his primary care physician complaining of progressive stiffness, swelling, redness, and pain in his joints that has been developing over the past year. Heart function is found to be normal, with no myocarditis reported. The physician diagnoses the patient with an autoimmune disease that leads to the destruction of articular cartilage. What would be the most likely explanation for the principal mechanism of tissue injury see in the patient? A. Arthus Reaction B. Inflammation mediated by TH17 cytokines C. Antibody cross reaction after a streptococcal infection D. Protease mediated disruption of intercellular adhesions E. Neutralization of intrinsic factor and a decreased absorption of vitamin B12
B. Inflammation mediated by TH17 cytokines
A 40 year old man recently had a lipid panel done, which indicated increased levels of LDL-C. Prior to administration of therapy, he had a liver function test done, and his physician put him on a class of drug to modify his lipid metabolism. He came back a few weeks after starting the drug with complaints of muscle pain. A liver function test was done, and it showed increased levels of AST and ALT. Which class of drugs did his physician most likely prescribe? A. Fibrates B. Inhibitors of cholesterol synthesis C. Inhibitors of VLDL secretion D. Inhibitors of bile acid absorption E. Inhibitors of cholesterol absorption
B. Inhibitors of cholesterol synthesis
A 50 year old patient with a history of smoking five packs of cigarettes daily comes to the clinic complaining of shortness of breath and wheezing. Spirometry and chest x-ray results suggest COPD. Which of the following is the best treatment for the patient? A. Pancuronium B. Ipratropium C. Atropine D. beta adrenergic antagonist E. Methacholine
B. Ipratropium
Familial Hypercholesterolemia is a genetic disorder that is caused by a defect on chromosome 19. What is the one of the specific protein/Gene that is mutated and what is the result of this mutation? A. HFN-1, Decreased Serum Potassium B. LDL Receptor (LDLR), Elevated LDL levels C. HFN-1, Increased Serum potassium D. LDL Receptor (LDLR), Decreased LDL levels E. ATP binding Cassette 1 (ABC-1) , Increased HDL levels
B. LDL Receptor (LDLR), Elevated LDL levels
A patient is given methacholine in order to confirm a diagnosis of asthma. Upon administration of the drug, the patient's heart rate slows down. This effect on the heart is due to the drug's interaction with which of the following receptors? A. M1 B. M2 C. M3 D. M4 E. M5
B. M2
A patient comes into the office with symptoms of diarrhea, salivation, bradycardia and emesis after taking a trip to Yellow Stone park. The doctor prescribes the patient atropine and after an hour the pt begins to feel better. What receptors are mainly going to be affected if we are looking at the heart and smooth muscle of GI? A. M2; M1 B. M2; M3 C. M5; Nm D. Nm; Nn E. M3; M2
B. M2; M3
A 14-year old male is brought to the primary care office by his mother. She is concerned about a possible genetic condition because her son is extremely tall and thin, with long arms, legs, fingers and toes. She also explains to the physician that her son has very flexible joints and scoliosis. What genetic disorder does the patient display and what is the mode of inheritance? A. Phenylketonuria; Autosomal Recessive B. Marfan Syndrome; Autosomal Dominant C. Marfan Syndrome; X-linked Dominant D. Familial Hypercholesterolemia; Autosomal Dominant E. Cystic Fibrosis; Autosomal Recessive
B. Marfan Syndrome; Autosomal DominantC. Marfan Syndrome; X-linked Dominant
A 28 year old woman and her 32 year old husband are expecting their second child. During their previous pregnancy with their first child the couple discovered that they have different RH blood types. Consequently, the mother is routinely injected with Rh immune globulin during and after her pregnancy to destroy any fetal erythrocytes in her blood and prevent the production of maternal anti-Rh antibodies. Which of the following statement's regarding RH incompatibility is false? A. Rh incompatibility is caused by the mating between an Rh-positive man and an Rh-negative woman. B. Maternal anti-Rh antibodies are produced in response to Rh positive red blood cells that stem from the Paternal circulation. C. There are usually no difficulties with the first Rh-incompatible child D. After being sensitized by Rh-positive red blood cells, anti-Rh antibodies produced by the mother can cross the placenta barrier and attack the fetus's blood. E. Without treatment the condition can lead to spontaneous abortion, destruction of red blood cells in the neonate, jaundice, cerebral damage and intellectual deficits.
B. Maternal anti-Rh antibodies are produced in response to Rh positive red blood cells that stem from the Paternal circulation.
A 19-year-old male is brought to the ED via EMS. The patient's friends told EMS that they dared the patient to run into a neighbor's cornfield and do a dance for Tik-Tok. They said that after returning from the field, the patient suddenly began vomiting and had trouble breathing. EMS states that on their arrival the patient was profusely sweating and was noticeably confused. Having seen this before, the EMS suspects organophosphate poisoning from pesticides and immediately administers the drug-of-choice for treatment of such symptoms. Which of the following conditions is contraindicated for use with the aforementioned "drug-of-choice?" A. Acute liver disease B. Narrow-angle glaucoma C. Aortic aneurysm D. Preexisting gallbladder disease E. Severe renal impairment
B. Narrow-angle glaucoma
A 42-year-old male is brought into the emergency department by his wife. The patient appears short of breath. He is vomiting, sweating profusely, notable sialorrhea, and his wife informs the healthcare personnel that these symptoms began a while after he completed some landscaping work. Upon further examination the patient's pupils are constricted, and tears are streaming down his face. The patient's vitals are taken, and significant bradycardia is noted. In the short period of time the patient has been in the emergency department he has started to convulse, and it is feared he may fall into a coma. The diagnosis of organophosphate poisoning is made, and the physician administers atropine along with another medication to treat his symptoms. Which of the following is most likely the desired outcome of the agent that was administered along with atropine? A. Competitive antagonism of acetylcholine B. Recovery of endogenous enzymatic function C. Inhibition of the release of neurotransmitters D. Stabilization of the offending agent-substrate complex E. Displacement of the offending agent
B. Recovery of endogenous enzymatic function
A 45-year-old woman presents to the clinic with complaints that her face and cheeks have grown red and itchy following the start of a new medication prescribed to reduce her triglyceride and LDL cholesterol levels. Her physician informs her that this is a well-known side effect of this particular drug and that it can be easily prevented. What is the most likely mechanism of action of this newly prescribed drug? A. Inhibits HMG-CoA reductase, the rate-limiting enzyme in cholesterol synthesis B. Reduces free fatty acid release from adipose tissue and increases plasma residence time for apoAI C. Regulates nuclear transcription factors including SREBP-1c and PPARa to reduce triglyceride biosynthesis and increase fatty acid oxidation D. Agonist of peroxisome proliferator-activated receptor a (PPARa) E. Decreases cholesterol transport from micelles into enterocytes by inhibiting uptake through brush border protein NPCL1
B. Reduces free fatty acid release from adipose tissue and increases plasma residence time for apoAI
Bill, a 70-year-old man, is diagnosed with high cholesterol. His doctor wants to prescribe the most effective and potent drug for lowering LDL levels but after looking at Bill's medical history, realizes that Bill is on a lot of other medications as well and doesn't want this drug to compete for the same metabolizing enzyme as the others. What first line of treatment should Bill's physician prescribe? A. Atorvastatin B. Rosuvastatin C. Fluvastatin D. Pravastatin E. Lovastatin
B. Rosuvastatin
A 57-year-old female presents to her primary care physician with concerns of a lump in her breast. A biopsy of her breast tissue confirms malignant breast cancer that spread from her axillary lymph nodes on the right side. Genetic testing is conducted and it is discovered that the patient has inherited a mutation in a gene that predisposed her to breast cancer. Which of the following genes is NOT likely to be the cause of her cancer? A. TP53 B. SMAD C. BRCA1 D. CHK2 E. PTEN
B. SMAD
22-year-old patient presents to her primary care physician with concern over her ongoing symptoms of nausea whenever she is boarding a flight. The patient states that she has been experiencing motion sickness from a young age but is seeking help now that she has a job that requires her to travel often. Which drug is the most appropriate to administer and select whether it is a tertiary or quaternary amine. A. Ipratropium; quaternary amine B. Scopolamine; tertiary amine C. Methscopolamine; quaternary amine D. Scopolamine; quaternary amine E. Ipratropium; tertiary amine
B. Scopolamine; tertiary amine
A 50 year old female presents to her physician with complaints of droopy eyelids and significant muscle weakness, as she is unable to walk without assistance. Before starting treatment, her physician would like to perform a diagnostic test to determine the cause of muscle weakness. What is the drug of choice for this test? A. Long acting acetylcholinesterase inhibitor B. Short acting acetylcholinesterase inhibitor C. Botulinum toxin D. Muscarinic receptor agonist E. Nicotinic receptor agonist
B. Short acting acetylcholinesterase inhibitor
A patient undergoing gastric bypass surgery was given Vecuronium, an intermediate-duration nicotinic receptor antagonist, which acts as a nondepolarizing neuromuscular blocker in order to induce flaccid paralysis during the procedure. After the procedure is finished, what drug could be given to accelerate the recovery from this blockade? A. Pancuronium B. Sugammadex C. Ipratropium D. Vesamicol E. Atropine
B. Sugammadex
A 35-year-old male presents to the emergency department with complaints of hemoptysis, chest pain with breathing and coughing, unintentional weight loss, fever, and fatigue. After performing a chest x-ray, doctors diagnose him with tuberculosis. The pattern of inflammation displayed in this infection is associated with CD4+ T cell differentiation into what subset and is induced by which cytokine? A. TH17 and IL-1 B. TH1 and IL-12 C. TH17 and IL-6 D. TH1 and IL-2 E. TH17 and IL-23
B. TH1 and IL-12
A 30-year-old male patient presents to the physician's office with a concern that he has familial hypercholesterolemia. He mentioned that he has tendon xanthomas and his family has a history of familial hypercholesterolemia. His lipid panel comes back with LDL levels of 350 mg/dL (high). His physician says that he has a Class II mutation in his LDL receptor gene which is causing his familial hypercholesterolemia. The physician explains to the patient that his LDL receptor is not removing the LDL from the circulation properly. How should the physician explain to the patient specifically what a class II mutation is? A. The LDL receptor does not migrate to cell surface where the coated pits are located so they cannot carry LDL into the cell. B. The LDL receptor is produced but in the endoplasmic reticulum it is altered and is eventually degraded. C. The mutation causes no LDLR protein to be formed. D. The LDL receptor cannot separate from the LDL that is taken up from circulation and therefore the LDL receptor cannot return to the cell surface and must be degraded. E. The LDL receptor can go to the surface of the cell but it cannot bind circulating LDL.
B. The LDL receptor is produced but in the endoplasmic reticulum it is altered and is eventually degraded.
A mother brings her 10-year-old son into Urgent care after he broke out in hives and had difficulty breathing at a birthday party. The mother was not present at the party and thinks her son may have accidentally ingested some food he is allergic to. The doctor administers epinephrine and monitors the patient. If the reaction were to proceed, what would be the main mediator present 12 hours later? A. Vasoactive amines B. cytokines C. Lipid mediators D. Th2 cells E. None of the above
B. cytokines
A 42-year-old male patient presents to the emergency department with dull abdominal pain that starts after he consumes a meal. His laboratory results reveal elevated serum AST and ALT, and normal serum Bilirubin. When asked about his current medications, he claims to be on Statins for his high cholesterol. What do the lab results most likely suggest about the patient's condition? A. These results are normal for a healthy adult male. B. These results are abnormal for a healthy male adult, but consistent with his medication. C. The patient is experiencing hepatotoxicity, as an adverse effect of his medication. D. The patient is not taking his medication regularly. E. The patient is abusing alcohol, causing alcoholic cirrhosis.
B. These results are abnormal for a healthy male adult, but consistent with his medication.
1. A patient has a follow-up appointment with his doctor after experiencing a life-threatening incident a few days ago. The patient explains to his doctor that he was outside throwing a football with his son when a bee stung him on his arm. The patient experienced a weak pulse, hives, and difficulty breathing. Which of the following descriptions does NOT apply to this type of hypersensitivity? A. Th2 & IL-4/IL-13-producing Tfh cells and IgE production is stimulated by an allergen. IgE binds to Fc receptors on mast cells and subsequent exposure to the allergen activates the mast cells to secrete mediators. B. This is T-cell mediated and is a delayed type hypersensitivity C. Some of the mediators released by mast cells include histamine and products of arachidonic acid metabolism. D. Another example of this type of hypersensitivity is asthma. E. Can be treated with epinephrine which will increase cardiac output, inhibit bronchial smooth cell contraction, and cause vascular smooth muscle cell contraction.
B. This is T-cell mediated and is a delayed type hypersensitivity
Which of the following is a correct pairing of function and formation of an apoB containing lipoprotein: A. Transport VLDL, synthesized in kidney B. Transport dietary lipids, formed in the intestines C. Transport dietary lipids, formed in thymus D. Transport VLDL, formed in intestines E. Transport dietary lipids, formed in pancreas
B. Transport dietary lipids, formed in the intestines
A 24 year old female is at the rheumatologist's office. Her physician explains to her that she is diagnosed with an autoimmune disease where her body produces Antibodies to non collagenous proteins in basement membranes of kidney glomeruli. She learns her disease is called Goodpasture syndrome. What type of hypersensitivity is this disease? A. Type I B. Type II C. Type III D. Type IV E. Arthus reaction
B. Type II
A 14-year-old male has an Autosomal Dominant genetic disorder that affects the COL3A1 gene. This gene is responsible for steps in collagen synthesis. He has thin skin that bruises easy, small joint hyperextensibility, and is at risk for arterial rupture. Which type of collagen is affected in this disorder? A. Type IV B. Type III C. Type I D. Type II E. Type V
B. Type III
A 45 year old female with a history of hypertension and Marfan syndrome presents to her doctor's office after her daughter was also diagnosed with Marfan syndrome. The patient recently got her results from a genetic testing kit she ordered off the internet. She found out that both her and her daughter have the same gene mutation leading to Marfan. She was surprised because her daughter's symptoms are different from her own, despite having the same genes. Which of the following is likely occuring? A. Incomplete penetrance B. Variable expressivity C. X-linked recessive disorder D. Autosomal recessive disorder E. X-linked dominant disorder
B. Variable expressivity
Tianna, a 26 year old female, has been diagnosed with vitamin D-resistant rickets. She is 1 of seven siblings, of which 2/3 of her sisters also have the disorder, and none of her brothers have the disorder. What is the transmission pattern of the disorder, and which parent did the affected siblings most likely inherit the disorder from? A. X-linked recessive; mother B. X-linked Dominant; father C. Autosomal Dominant ; father D. Autosomal recessive; mother E. X-linked dominant; mother
B. X-linked Dominant; father
A patient has been described as having an "unwavering but ineffective" immune response to bacteria and fungi. Most noticeably, he has an extraordinary number of circulating macrophages yet he suffers from pneumonia, liver abscesses, and nodular inflammatory granulomas. What genetic mutation does this patient most likely suffer from? A. X-linked; Negates a macrophage's ability to ingest pathogens B. X-linked; Negates a macrophage's ability to degrade pathogens C. Autosomal recessive; Negates a macrophage's ability to ingest pathogens D. Autosomal recessive; Negates a macrophage's ability to degrade pathogens E. Autosomal dominant; Negates a macrophage's ability to fuse lysosomes with phagocytic vesicles
B. X-linked; Negates a macrophage's ability to degrade pathogens
A 26-year-old presents to their PCP after noticing her vision has become slightly blurred and that it has become more difficult to lift the laundry basket at home. She feels that her arms have become weak noodles even though she hasn't skipped any gym sessions in recent months. The physician notes ptosis of the right eye upon examination and suspects Myasthenia Gravis. What diagnostic test would be most sensitive and indicative of this diagnosis? A. positive ice pack test B. single fiber electromyograph (SFEMG) C. nerve conduction velocity test (NVC) D. muscle biopsy Positron Emission Tomography and Computed-Tomography (PET-CT)
B. single fiber electromyograph (SFEMG)
During a routine visit to his primary care physician Alexander mentions that he has purchased Norwegian Cruise Line tickets for him and his family. With the ship scheduled to sail next month Alexander is nervous that he may experience motion sickness during the trip. His doctor prescribes a common antiemetic that can be applied as a transdermal patch. What is the mechanism of action of this drug? A.) Muscarinic receptor agonist B.) Muscarinic receptor antagonist C.) Nicotinic receptor agonist D.) Nicotinic receptor antagonist E.) Gag reflex inhibitor
B.) Muscarinic receptor antagonist
What are 2 genes involved in DNA repair that predispose women to breast cancer if mutated?
BRCA1 and BRCA2
Why is the assembly, secretion and metabolism of chylomicrons collectively referred to as the Exogenous pathway of lipoprotein metabolism?
Bc the triglyceride component of the chylomicrons originate primarily from the diet
cholesterol balance
Because cholesterol is converted by the liver to bile acids and secreted unmodif ed into bile, overall cholesterol balance depends on the disposition o both cholesterol and bile acids. Most bile acid molecules are not lost in the eces a ter participating in cholesterol transport and at digestion; instead, they are taken up and recycled by high-a f nity transport proteins in the distal ileum. Bile acids enter the portal circulation and are transported back to the liver, where they are cleared rom the blood by hepatocytes with high f rst-pass e f ciency. Bile acids are then re-secreted into bile. The process o recycling bile acids between the liver and intestine is re erred to as enterohepatic circulation . The enterohepatic circulation is highly e f cient, allowing 5% o secreted bile acids to be lost in the eces. However, because bile acids are secreted in such large amounts, the small ractional loss o bile acids amounts to about 0.4 grams per day. Considering that cholesterol is the substrate or bile acid synthesis, ecal bile acids represent a source o cholesterol loss rom the body. Sensitive nuclear hormone receptors within the liver are capable o detecting the rate o loss o bile acids into the eces. These receptors tightly regulate transcription o bile acid synthetic genes. As a result, the liver synthesizes precisely the amount o bile acids that is su f cient to replace what is lost in the feces. In addition to the 1.2 grams o cholesterol that are secreted into bile each day, the average American diet contributes approximately 0.4 grams each day to intestinal cholesterol. There ore, dietary cholesterol represents only a minor fraction (25%) of the total (i.e., biliary and dietary) cholesterol that passes through the intestine. The extent to which intestinal cholesterol is absorbed appears to be genetically regulated. Each individual absorbs a fixed percentage of intestinal cholesterol. In the population, percentages range from as low as 20% to more than 80%. For example, when an average individual absorbs 50% o intestinal cholesterol, this will amount to half of the 1.6 grams (i.e., 1.2 grams o biliary cholesterol plus 0.4 grams o dietary cholesterol), and the other hal (0.8 grams) will be lost in the eces. Combined with a loss o 0.4 grams per day o cholesterol in the form o fecal bile acids, this yields a total cholesterol loss rom the body o 1.2 grams each day. Taking into account intestinal absorption o dietary cholesterol and reabsorption o biliary cholesterol, total body cholesterol synthesis is approximately 0.8 grams per day (i.e., cholesterol synthesis fecal loss o cholesterol bile acids dietary cholesterol intake). Thus, the amount o endogenous cholesterol synthesis is about two old greater than the amount consumed in the average diet
intravascular maturation of HDL
Because disk-shaped pre- -HDL particles are relatively ine f cient at removing excess cholesterol rom cell membranes, these particles must mature into spherical particles in the plasma. HDL maturation occurs as a result o the activity o two distinct circulating proteins (Fig. 20-9A, B). Lecithin:cholesterol acyltransferase (LCAT) binds pre erentially to disk-shaped HDL and converts cholesterol molecules within the particle to cholesteryl esters. This is accomplished by transesterif cation o a atty acid rom a phosphatidylcholine molecule on the sur ace o the HDL to the hydroxyl group o a cholesterol molecule. The reaction also creates a lysophosphatidylcholine molecule, which dissociates rom the particle and binds to serum albumin. Because they are highly insoluble, cholesteryl esters migrate into the core o the HDL particle. The development o a hydrophobic core converts the pre- beta-HDL to a spherical alpha -HDL particle. The second important protein that contributes to HDL maturation in the plasma is phospholipid transfer protein (PLTP). PLTP trans ers phospholipids rom the sur ace coat o apoB-containing remnant particles to the sur ace coat o HDL. During LPL-mediated lipolysis o apoB-containing lipoproteins, the particles become smaller as triglycerides are removed rom the core. This leaves a relative excess o phospholipids on the sur ace o the particle. Because phospholipids are highly insoluble and cannot otherwise dissociate rom a particle, PLTP removes excess phospholipids and thereby maintains the appropriate sur ace concentration or the shrinking core. By trans erring phospholipids to the sur ace o HDL, PLTP also replaces the molecules that are consumed by the LCAT reaction. This allows the core o HDL to continue to enlarge.
Figure 20.10: Biliary lipid secretion
Biliary lipid secretion. A. Within hepatocytes, a portion of cholesterol is converted to bile acids. This process is rate-limited by cholesterol 7 -hydroxylase, which is expressed only in hepatocytes. Cholate is the most abundant bile acid synthesized by the human liver. B. Within the canalicular (apical) membranes, an ATP-dependent pump ABCB11 drives the secretion of bile acids out of the cell against a concentration gradient. Bile acids then stimulate the activities of two other proteins, ABCB4 and a heterodimer of ABCG5 and ABCG8 (ABCG5/G8), to secrete phospholipids and cholesterol, respectively, into bile. Within bile, the interactions among bile acids, phospholipids, and cholesterol result in the formation of micelles
How is cholesterol eliminated once it is delivered to the liver by the process of reverse cholesterol transport?
Biliary secretion
What is the function of Bile acid sequestrants? Examples?
Bind bile acids in the intestinal lumen and increase fecal excretion eg: Cholestyramine, Colestipol and Colesevelam Subsequently, more LDL-cholesterol is used by the liver to synthesis bile acids The decrease in cellular cholesterol pools upregulates LDL receptors and decreases the amount of LDL-cholesterol in the circulation. Mild increases in HDL-cholesterol are also seen with this agent as a result of increased intestinal HDL formation Treatment is associated with a reduction in the incidence of CHD
What is the MOA of ACh receptor antagonists?
Bind directly to the agonist site and competitively blocking stimulation of the receptor by endogenous ACh or exogenously administered receptor agonists
What increases during B cell response to a foreign? What do B cells become when the mature?what do they secrete?
Binding affinity of immunoglobulins for the invading pathogen Plasma cells Antibodies
bipolar disorder
Bipolar disorder, also known as manic- depressive disorder, is characterized by extreme mood swings and emotional instability. The prevalence of the disorder in the general population is approximately 0.5% to 1%, but it rises to 5% to 10% among those with an affected first-degree relative. A study based on the Danish twin registry yielded concordance rates of 79% and 24% for MZ and DZ twins, respectively, yielding a heritability estimate of approximately 72% (other studies have yielded even higher heritability estimates—up to 90%). The corresponding concordance rates for major depressive disorder (sometimes termed unipolar depression) were 54% and 19%. Thus, it appears that bipolar disorder is more strongly influenced by genetic factors than is major depressive disorder. As with schizophrenia, many linkage and genome-wide association studies have been carried out to identify genes contributing to bipolar disorder. These studies have implicated many chromosome regions in multiple population samples. Some of the best-replicated results are for genes that encode voltage-gated calcium channels. Ion-channel modulating drugs are frequently used as mood stabilizers, giving these results functional plausibility. Many of the genes associated with bipolar disorder are also associated with schizophrenia
What is blood pressure regulated by?
Blood pressure regulation is a highly complex process that is influenced by many physiological systems, including various aspects of kidney function, cellular ion transport, vascular tone, and heart function
Where do immune complexes often deposit?
Blood vessels including vessels through which plasma is filtered at high pressure immune complex diseases tend to be systemic and often manifest as widespread vasculitis, arthritis, and nephritis.
What occurs in Autoimmunity?
Body's immune system attacks its own normal cells eg: Type I diabetes is characterized by T cell infiltration of the pancreas and subsequent T cell destruction of the insulin producing beta cells
How does Botulinum toxin cause paralysis? What does it inhibit the release of?
Botulinum toxin blocks voluntary and autonomic cholinergic neuromuscular junctions by binding irreversibly to the presynaptic nerve endings where it inhibits the release of acetylcholine.
Botulism - Def, Epid, patholgoy
Botulism is a rare, potentially lethal, paralytic illness caused by the neurotoxin produced by the anaerobic, sporeforming bacterium Clostridium botulinum. Botulinum toxin blocks voluntary and autonomic cholinergic neuromuscular junctions by binding irreversibly to the presynaptic nerve endings where it inhibits the release of acetylcholine. Human forms of the disease include foodborne botulism most commonly caused by homecanned food, wound botulism with most cases occurring among "black tar" heroin users, and infant botulism occurring usually in the second month of life due to intestinal colonization. Outbreaks of foodborne botulism occur in prison inmates due to ingestion of pruno, an alcoholic drink made illicitly in prison. About 145 botulism cases are reported each year in the United States; approximately 15% are foodborne, 65% are infant, and 20% are wound botulism.
Breast cancer
Breast cancer is the second most commonly diagnosed cancer (after skin cancer) among women, affecting approximately 12% of American women who live to age 85 years or older. Invasive breast cancer was diagnosed in approximately 230,000 American women in 2014. Although the breast cancer mortality rate has been declining for 25 years, about 40,000 women in the United States die from this disease each year. Breast cancer was formerly the leading cause of cancer death among women, but it has been surpassed by lung cancer. Breast cancer can also occur in men, with a lifetime prevalence that is roughly 100 times lower than that of women (2400 cases diagnosed in 2014). The familial aggregation of breast cancer has been recognized for centuries, having been described by physicians in ancient Rome. If a woman has one affected first-degree relative, her risk of developing breast cancer doubles. The risk increases further with additional affected relatives, and it increases if those relatives developed cancer at a relatively early age (before 50 years of age). Several genes are now known to predispose women to developing hereditary breast cancer. Most important among these are BRCA1 and BRCA2, two genes involved in DNA repair (see Chapter 11). Germline mutations in the TP53 and CHK2 genes can cause Li-Fraumeni syndrome, which also predisposes to breast cancer. Cowden disease, a rare autosomal dominant condition that includes multiple hamartomas and breast cancer, is caused by mutations in the PTEN tumor suppressor gene (see Chapter 11). Ataxia-telangiectasia, an autosomal recessive disorder caused by defective DNA repair, includes breast cancer in its presentation. Mutations in the MSH2 and MLH1 DNA repair genes, which lead to hereditary nonpolyposis colorectal cancer (HNPCC), also confer an increased risk of breast cancer. Despite the significance of these genes, it should be emphasized that more than 90% of breast cancer cases are not inherited as mendelian diseases A number of environmental factors are known to increase the risk of developing breast cancer. These include nulliparity (never bearing children), bearing the first child after 30 years of age, a high-fat diet, alcohol use, and estrogen replacement therapy.
How can Electrodiagnostic testing help diagnose/confirm LEMS?
By demonstrating reduced CMAP amplitudes in distal hand muscles CMAP facilitation of at least 100% after 10" maximal voluntary contraction or high frequency RNS (posttetanic facilitation); and CMAP decrement greater than 10% with low frequency RNS Repetitive nerve stimulation: 1- Low frequency stimulation: Decrement of 20% 2- High frequency stimulation/post exercise: Increase 100% This is key to differentiate from Myasthenia gravis as in MG there is no increase in CMAP
In contrast to cytotoxic T cells which destroy infected cells directly, how do Helper T cells (Th) respond to the presence of pathogens?
By secreting Cytokines
How do helper T cells stimulate development of other components of the immune system such as Cytotoxic T cells and B cells?
By secreting cytokines
How does the liver eliminate cholesterol?
By secreting unesterified cholesterol into the bile or by converting cholesterol to bile acids
What does complement activation generate? What is its net result?
By-products mainly C3b and C4b which are deposited on the surfaces of the cells and recognized by phagocytes that express receptors for these proteins Phagocytosis of the opsonized cells and their destruction
Jessica, a 23-year-old recent college graduate, has just received her first job offer. However, before she starts the job, she is told she needs to undergo an employee health screening that includes a PPD test. After she receives her injection, she sees reddening and induration of the injection site about 8-12 hours later. The next day, she realizes the redness has disappeared. The PPD test displays a delayed type hypersensitivity, what typical characteristics can be seen morphologically? A) Fibrinoid necrosis B) Lines of Zahn C) Accumulation of mononuclear cells, producing perivascular "cuffing" D) Granular lumpy deposits of immunoglobulin and complement E) Diffuse thickening of the capillary walls from immune complex deposition
C) Accumulation of mononuclear cells, producing perivascular "cuffing"
A 28-year-old male presents to the clinic with complaints of weakness, fatigue, and dark-colored urine since taking a prophylactic antimalarial drug, primaquine, for his trip to Senegal last week. He appears jaundice and pale, and his blood test results show elevated reticulocyte count. Which of the following most likely explains the patient's presentation? A) A defect in the structure of hemoglobin has left the patient more susceptible to malaria-induced hemolytic anemia. B) A deletion in the genes encoding hemoglobin has led to reduced amounts of structurally normal alpha-globin chains, causing an adverse reaction to primaquine. C) Administration of the drug has revealed a specific enzyme deficiency that under normal conditions does not cause disease. D) Defective mRNA processing has led to reduced amounts of structurally normal beta-globin chains, causing an adverse reaction to primaquine. E) Deficiency in the enzyme glucose-6 phosphate-dehydrogenase has left the patient more susceptible to malaria-induced hemolytic anemia.
C) Administration of the drug has revealed a specific enzyme deficiency that under normal conditions does not cause disease.
A 35 year old female is rushed to the emergency department after fainting following some light afternoon gardening. She presented with symptoms of nausea, vomiting, diarrhea, sweating and hypersalivation. When asked about her history, she mentioned that she fainted shortly after administering some Pesticide. The attending physician at this point suspects that she is suffering from organophosphate poisoning. Given her history and presentation, what drug should be prescribed to alleviate her symptoms? A) Pyridostigmine B) Methacholine C) Atropine D) Donepezil E) Ipratropium
C) Atropine
A 26-year-old female presents to the emergency department appearing to be in labor. After a successful parturition, a muscarinic agonist is given to the mother, as she is having complaints of urinary retention. Which of the following drugs is most likely given to the patient to increase the chance of voiding? A) Carbachol B) Acetylcholine C) Bethanechol D) Methacholine E) Pilocarpine
C) Bethanechol
A 4-month-old male was brought to his PCP for his checkup. His parents were concerned because he seemed smaller than other babies his age, suffered from frequent bouts of diarrhea and had persistent eczema. After taking a thorough history, the physician began to suspect a disease that results in a deficiency of regulatory T cells, so she ordered genetic testing. Testing revealed the patient had IPEX, which is caused by a mutation in which of the following genes? A) RAG1, RAG2 B) HLA-DQA1 C) FOXP3 D) HLA-DRB1 E) FBN1
C) FOXP3
What happens after the MHC 1- peptide complex binds to receptors on the T cell surface?
Destroys the infected cell Cytotoxic T lymphocytes or killer T lymphocytes
A 55 year old male presents to his doctor's office for a yearly physical examination. During the history, the patient reveals he has a family history of heart diseases and so the doctor orders a lipid panel. The results of the lipid panel showed total cholesterol levels of greater than 250 mg/dL, triglycerides levels greater than 175 mg/dL, HDL cholesterol levels less than 35 mg/dL, high apo B (>120 mg/dL) and a low ratio of LDL-cholesterol to apo B100 (<1.2). Further examination revealed that the patient's liver synthesizes VLDL in excessive amounts as well as the absence of any Xanthomas or xanthelasmas on the skin. Based on the information what type of disease is the patient most likely suffering from? A) Familial Defective Apolipoprotein B100 B) Polygenic hypercholesterolemia C) Familial combined Hyperlipoproteinemia D) Elevated Lipoprotein(a) E) Lipoprotein Lipase (LPL) deficiency
C) Familial combined Hyperlipoproteinemia
A 10-year-old male is brought to his primary care physician by his mother for a routine physical. The mother is also concerned about the appearance of some hard lesions that have appeared on her son's arms and legs. She states that the boy's father and uncle both passed away at an early age due to heart attacks and had trouble managing their cholesterol. After obtaining a thorough medical and family history, the physician prescribes lovastatin. What is the physician's main concern in terms of the progression of the patient's condition? A) Formation of xanthoma B) Aortic dissection C) Myocardial infarction D) Pulmonary emphysema E) Childhood obesity
C) Myocardial infarction
An 80-year-old male was recently diagnosed with early stages of Alzheimer's disease. His family was informed that the patient is lacking an important neurotransmitter called Acetylcholine which helps with memory. The patient was then prescribed an acetylcholinesterase inhibitor called Donepezil to help increase his acetylcholine levels. If we are strictly speaking on the mechanisms at work what would be the rate limiting factor for acetylcholine? A) Acetylcholinesterase B) Acetyl CoA C) Na+/choline cotransporter D) Choline acetyltransferase (ChAT) E) Donepezil
C) Na+/choline cotransporter
A 47-year-old male decides to visit his family doctor after expressing shortness of breath and a bit of discomfort in his chest recently. The doctor orders a test for his total cholesterol levels as well as his HDL levels which come out to be 215 mg/dL and 30 mg/dL, respectively. If the patient was diagnosed with congenital heart disease (CHD), what pharmacological therapy is used to help treat CHD? A) Diuretics B) Calcium Channel Blockers (CCBs) C) Statin D) AT1 receptor blocker (ARBs) E) Central sympathoplegics
C) Statin
A 45-year-old woman is very insecure about the line on her forehead. She presents to her dermatologist with questions about receiving an injection to smooth out this line. What is the mechanism of action that causes this injection to have a paralysis-inducing effect on facial lines and wrinkles? A) The injection blocks the high-affinity transporter for choline and prevents the uptake of choline required for acetylcholine (ACh) synthesis. B) The injection blocks the ACh-H+ antiporter that transports ACh into vesicles, preventing the storage of ACh. C) The injection degrades SNAP-25 and prevents synaptic vesicle fusion with the axon terminal membrane. D) The injection binds to and inhibits acetylcholinesterase which allows ACh to accumulate at the synaptic cleft. E) The injection activates receptor-associated channels and produces depolarization at the neuromuscular junction which induces paralysis
C) The injection degrades SNAP-25 and prevents synaptic vesicle fusion with the axon terminal membrane.
A 7 year old female presents to a general practitioner with her mother. She complains of excessive thirst, frequent bed-wetting, excessive hunger, and fatigue. She is diagnosed with type I diabetes and the physician prescribes exogenous insulin along with family counseling to manage the disease. Which of the following regarding the patient is also correct? A) The patient's disease is caused by a mutation in HFN1a B) The patient partially produces insulin C) The patient's disease is also considered an autoimmune disease D) The patient is insulin resistant E) The patient's disease accounts for more than 90% of all diabetes cases
C) The patient's disease is also considered an autoimmune disease
A newborn baby was diagnosed with albinism, a disease with very pale, almost white, skin and hair. This disease is caused by a deficiency in the skin pigment melanin. This deficiency of melanin results from an error in what enzyme? A) Galactose-1-uridyltransferase B) Tyrosine Kinase C) Tyrosinase D) Hexokinase E) DNA polymerase
C) Tyrosinase
George is currently being treated for myasthenia gravis. His doctor prescribed him pyridostigmine. One of the adverse effects of this drug is that it produces bradycardia in the heart. Which mechanism most directly responsible for the production of bradycardia due to use of this drug. A) an increase in acetylcholine causes active βγ receptors to activate potassium channels depolarizing the neurons in the heart B) an decrease in acetylcholine causes active βγ receptors to activate potassium channels hyperpolarizing the neurons in the heart C) an increase in acetylcholine causes active βγ receptors to activate potassium channels hyperpolarizing the neurons in the heart D ) an decrease in acetylcholine causes active βγ receptors to activate sodium channels depolarizing the neurons in the heart E) an increase in acetylcholine causes active βγ receptors to activate sodium channels hyperpolarizing the neurons in the heart
C) an increase in acetylcholine causes active βγ receptors to activate potassium channels hyperpolarizing the neurons in the heart
A 28-year-old woman, pregnant with twins, is referred by her OB/GYN to a geneticist because she is concerned about passing on her "bad genes" to her children. She was addicted to heroin since experimenting with it in high school, but has been sober for the last 3 years. She wants to know if genetics plays a bigger role than lifestyle in the chances of her children having a heroin addiction. Given that the concordance rate is 0.75 and 0.55 for monozygotic and dizygotic twins, respectively, what proportion is heroin addiction determined by the environment? A. 20% B. 40% C. 60% D. 0% E. 100%
C. 60%
A 67-year-old male presents to the emergency department with elevated levels of total plasma cholesterol as well as LDL cholesterol. The physician notes that the patient has normal concentrations of triglycerides and suspects he may have primary hypercholesterolemia. Which of the following would be the least likely cause of this condition? A. familial defective apoB100 B. gain of function mutations in PCSK9 C. Activation of peroxisome proliferator activated receptor alpha (PPAR-alpha) by fibrates D. Polygenic hypercholesterolemia E. Familial combined hyperlipidemia
C. Activation of peroxisome proliferator activated receptor alpha (PPAR-alpha) by fibrates
A geneticist is researching a new inherited disease using family history data and other large data sets. They are having a difficult time determining if the disease is multifactorial or a single-gene disease with reduced penetrance. What extrapolations from the data would indicate the condition is of multifactorial inheritance? A. The recurrence risk stays the same regardless of the number of affected siblings B. The recurrence risk has little to do with the severity of genetic expression C. Affected individuals that belong to the sex that is considered least likely to acquire the disease are more likely to pass it on D. There is not a rapid decrease in recurrence risk as the relationship becomes more remote along the family tree E. The prevalence of the disease is unrelated to the recurrence risk
C. Affected individuals that belong to the sex that is considered least likely to acquire the disease are more likely to pass it on
A 28-year-old female patient comes in to the hospital to see her OB-GYN at 16 weeks of gestation. After performing an ultrasound examination and finding elevated levels of alpha-fetoprotein in maternal serum, the physician is concerned that the child may have a neural tube defect. Which of the following should the physician order next to confirm the diagnosis of a neural tube defect? A. Beta hCG test B. Chorionic Villus Sampling C. Amniocentesis D. Fetal Echocardiogram E. Cell Free Fetal DNA testing
C. Amniocentesis
A 50-year-old female presents to her physician's clinic after completing lab work that reveals elevated lab values. Her physician diagnoses her with Graves disease. What is the cause of this disease? A. Opsonization and phagocytosis of platelets B. Immune complex formation in blood vessels C. Antibody-mediated stimulation of TSH receptors D. Immediate release of histamine after exposure to an allergen E. Excessive T cell activation that targets myelin proteins
C. Antibody-mediated stimulation of TSH receptors
A 62-year-old man presents to his primary care physician with complaints of cutaneous flushing. He was recently prescribed a new drug after his lipid panel showed very low HDL and mildly elevated LDL and triglyceride levels and thinks the symptoms could be associated with the new drug therapy. What drug should the physician administer to be taken concurrently to reduce the flushing? A. Lovastatin B. Fenofibrate C. Aspirin D. Cholestyramine E. Tylenol
C. Aspirin
Madeline, a 23 year old female, recently had her father diagnosed with stage 4 colorectal cancer. She is concerned because her paternal grandfather was diagnosed with stage 2 colorectal cancer in the past. Due to these events she wants to know the risks that she will also develop the disease later in life. Assume that Madeline's father is the proband and that colorectal cancer is more common in men. Which of the following factors is NOT part of the criteria used to define multifactorial inheritance? A. Since Madeline has more than one family member affected, she is at greater risk for developing the disease B. Since Madeline's father has been diagnosed with stage 4 colorectal cancer, it is more likely that she will later develop the disease C. Assuming colorectal cancer is more common among men, the risk of Madeline being affected is higher because she is female. D. Since both Madeline's father and grandfather both had colorectal cancer, her risks do not decrease E. The recurrence risk for Madeline's sister is equal to the square root of the prevalence of the disease in the population
C. Assuming colorectal cancer is more common among men, the risk of Madeline being affected is higher because she is female.
A 16-year-old male presents to the emergency room vomiting and experiencing intermittent seizures. His respiratory rate is 8 breaths per minute with a blood pressure of 150/95 mm Hg. Intravenous phenytoin is administered, and the seizures resolve. Profuse vomiting prevents intubation as the patient's respiratory drive continues to fall. The friend who called 911 informed paramedics that the patient tried simultaneously using a highly concentrated nicotine vape with three nicotine patches on his arm. What drug can be administered to resolve the vomiting and other associated symptoms in this case? A. Pralidoxime B. Methacholine C. Atropine D. Donepezil E. Pyridostigmine
C. Atropine
A 58-year-old woman is admitted to the hospital with shallow breathing, wheezing, profuse rhinorrhea, lacrimation, ocular pain and diminished vision. She tells the physician that the symptoms started when she was in her garden spraying flowers with an insecticide containing carbaryl (a reversible cholinesterase inhibitor). Which of the following drugs would be the most appropriate to treat the patient's symptoms? A. Albuterol B. Physostigmine C. Atropine D. Glycopyrrolate E. Neostigmine
C. Atropine
An 11-month-old boy was rushed to the ED by his mom due to progressive pneumonia. Mom indicated that the baby is up to date with vaccines. She also mentioned that starting at 6 months, her son has recurrent upper respiratory infection. Analysis of lymphocyte cells include decreased in T cell, normal B cell, and decreased in Natural Killer cell. The immunodeficiency is most likely the result of: A. Autosomal recessive mutation that leads to deficient expression of MHC class I B. Autosomal recessive mutation in which lack of recombinase activity impairs VDJ recombination C. Autosomal recessive mutation in which the patient lacks JAK3 genes D. X-linked recessive in which mutation in BTK genes leads to recurrent bacterial infections E. X-linked recessive in which disorder involves deficiency of platelets and progressive T-cell dysfunction
C. Autosomal recessive mutation in which the patient lacks JAK3 genes
A 20-year-old male presents to his primary care physician to ask about growths he started to notice on his elbow. Upon further evaluation and diagnostic testing, he has been diagnosed with familial hypercholesterolemia. It is later determined that he has a class III mutation. Which of the following choices is associated with the abnormal function of the mutant protein? A. Synthesis B. Transport C. Binding D. Clustering E. Recycling
C. Binding
A medical student obtained permission from her local hospital to conduct a study on bipolar disorder using their twin registry. Her results yielded concordance rates of 85% for monozygotic twins (MZ) and 35% for dizygotic twins (DZ). How should she interpret these results? A. Bipolar disorder is strongly influenced by genetic factors in MZ twins, but not DZ twins. B. Bipolar disorder is strongly influenced by environmental factors in both MZ and DZ twins. C. Bipolar disorder is strongly influenced by genetic factors in both MZ and DZ twins. D. Bipolar disorder is strongly influenced by genetic factors in DZ twins, but not MZ twins. Bipolar disorder is not influenced by genetic factors in both MZ and DZ twins.
C. Bipolar disorder is strongly influenced by genetic factors in both MZ and DZ twins.
A patient in presents in the clinic with an infection that elicits an allergic response, most likely due to a multicellular parasite. There is high levels of IL-4, IL-5, IL-6, and IL-13. What receptor can be found on the cell that secretes these cytokines? A. LFA3 B. CD40 C. CD28 D. B7 E. ICAM-1
C. CD28
When an antigen presenting cell encounters a foreign peptide it will transport it to its surface and presents the foreign peptide to T lymphocytes by costimulatory molecules. Which of the following receptor proteins are involved in this process? A. HLA-A and IgG B. ICAM and Rh factor C. CD28 and MHC-II D. IL-7 and HLA-B TNF-beta and IL-13
C. CD28 and MHC-II
A couple bring in their adopted 2-year-old son to the pediatrician. He has had numerous infections since he came to live with them; and he always seems to have a runny nose. The physician runs a couple of tests to confirm that the boy has an X-linked recessive immunodeficiency disorder where his phagocytes are not killing the bacteria and fungi that they ingest. What was the boy diagnosed with? A. Bare lymphocyte syndrome B. Wiskott-Aldrich syndrome C. Chronic granulomatous disease D. HyperIgE deficiency E. Chediak-Higashi syndrome
C. Chronic granulomatous disease
VLDL and chylomicrons are ApoB-containing lipoproteins that function to deliver triglycerides to muscle tissue for ATP biogenesis and to adipose tissue for storage. Each lipoprotein is synthesized in a specific organ, determining the size of the ApoB molecule attached, and act on specific triglycerides. Which of the following correctly matches the lipoprotein to its site of synthesis, ApoB protein attached and triglyceride it acts on? A. VLDL - formed in the intestine, contains ApoB48 and transports synthesized endogenous triglycerides B. VLDL - Formed in the liver, contains ApoB100 and transports triglycerides consumed in the diet C. Chylomicrons - formed in the intestine, contains ApoB48 and transports triglycerides from the diet D. Chylomicrons - formed in the intestine, contains ApoB100 and transports triglycerides from the diet E. VLDL - formed in the liver, contains ApoB100 and transports triglycerides from the diet
C. Chylomicrons - formed in the intestine, contains ApoB48 and transports triglycerides from the diet
A 15-year-old girl has genetic testing done and it is found that she has Familial Hypercholesterolemia just like her father. It is determined that she has a mutation that affects the binding domain of the LDL receptor, so the receptor reaches the cell surface but fails to bind LDL. What class of mutations would this be considered? A. Class I B. Class II C. Class III D. Class IV E. Class V
C. Class III
A 31 year old male presents to the clinic for a physical exam. His blood pressure is 140/90. His father and uncle died from a heart attack. A lipid panel shows that his LDL greatly exceeds normal values for his age range. What is the mechanism of action of the most likely first line treatment for this patient? A. Activates PPAR, increasing LPL expression which decreases apolipoprotein CII B. Binding of bile salts and steroids in small intestine, forming insoluble polymer preventing bile salt absorption C. Competitive inhibition of HMG-CoA reductase D. Inhibition of acetylcholinesterase by binding to receptor active site E. Bind apoB100 mRNA preventing transcription and reducing apoB levels
C. Competitive inhibition of HMG-CoA reductase
A 35 year old teacher presents to her primary care provider with a diffuse rash on her right arm with blistering, central epidermal necrosis, and surrounding inflammation. She states that she went on a field trip with students from her 3rd grade class two days prior, to study plants in a nearby National Park. After which, her symptoms began gradually. Her provider deemed that she was experiencing an inflammatory reaction mediated by TH1 cytokines, and diagnosed the patient with a T cell-mediated disease. What was the likely diagnosis considering the patient's recent history and presenting symptoms? A. Rheumatoid Arthritis B. Psoriasis C. Contact sensitivity D. Arthus reaction E. Systemic Lupus Erythematosus
C. Contact sensitivity
An 18-year-old woman presents with fatigue, joint pain, and a facial rash. Examination shows a butterfly-shaped erythema affecting the cheeks and bridge of the nose, as well as peripheral edema to the ankles. Urinalysis reveals abnormal levels of protein and erythrocytes. The physician suspects an autoimmune disorder involving genes encoding heterodimeric molecules that are primarily expressed on only the surfaces of APCs. Which locus is associated with this disease? A. DRB1, located in the Class I region of chromosome 6p B. B27, located in the Class I region of chromosome 6p C. DRB1, located in the Class II region of chromosome 6p D. B27, located in the Class II region of chromosome 6p E. B27, located in the Class III region of chromosome 6p
C. DRB1, located in the Class II region of chromosome 6p
A 50-year-old woman with severe hypercholesterolemia is treated with a combination of atorvastatin and Drug X to control high LDL and low HDL levels. With this drug combination, it is important that the patient be monitored closely for signs of myopathy. What is the mechanism of action of Drug X? A. Decrease enterohepatic circulation by binding to bile acids via up regulation of 7α hydroxylase B. Decrease VLDL secretion by inhibiting apoB mRNA translation and lipid transfer via MTP binding C. Decrease free fatty acid release and increase apoAI plasma residence time by inhibiting adipocyte HSL activity D. Decrease cholesterol transport from micelles into enterocytes by inhibiting uptake through NPC1L1 E. Decrease triglyceride biosynthesis and increase fatty acid oxidation by regulating SREBP-1c and PPARα
C. Decrease free fatty acid release and increase apoAI plasma residence time by inhibiting adipocyte HSL activity
Joe is a 35-year-old coach with familial hypercholesterolemia, presenting with atherosclerosis. He is prescribed a medication that targets the rate limiting step in cholesterol synthesis to manage his condition and reduce the risk of further progression. Kim is 50-year-old nurse with a genetic disorder that puts her at an increased risk of a stroke. Despite having a normal lipid metabolism, Kim is prescribed the same medication as Joe. What it the pleiotropic effect characteristic to this drug that provides a therapeutic effect to some like Kim? A. the Decrease of alanine transaminase levels in the serum B. Decrease endothelial response to Nitrous oxide C. Decreased levels of tissue factor D. Increased levels of prothrombin E. None of the above.
C. Decreased levels of tissue factor
A 17-year old comes in for a routine physical. He presents with abdominal obesity and hypertension. He has a family history of high cholesterol and coronary artery disease. Blood work reveals increased fasting triglycerides and total cholesterol. LDL is normal and HDL is low. What is the most likely cause of his presentation? A. Familial hypercholesterolemia B. Familial hypertriglyceridemia C. Familial Combined Hyperlipidemia D. Dysbetalipoproteinemia E. Familial defective apoB100
C. Familial Combined Hyperlipidemia
A patient comes to your clinic to get an annual physical. On physical, everything seems normal and you tell them to get a blood test. After getting the blood test report you see the patient has elevated cholesterol. What enzyme is most likely responsible for the increase in cholesterol? A. Phenylalanine hydroxylase B. Hexosaminidase C. HMG-CoA reductase D. Dystrophin E. Rb protein
C. HMG-CoA reductase
A 22-year-old male visits his primary care physician with complaints of rhinitis, sinusitis, and itchy eyes. The patient states that he noticed his symptoms started when he went outside to play some football with his little brother, who wanted to go outside and enjoy the nice spring weather. The physician diagnoses the patient with seasonal allergies and prescribes Claritin. The production of what antibody mediates this type of hypersensitivity. A. IgG B. IgM C. IgE D. IgA E. IgD
C. IgE
A 10-year old boy presents with hepatomegaly, pancreatitis, and eruptive xanthomas. A lipid panel comes back with profound hypertriglyceridemia. After further testing, the physician suggests the boy may have familial lipoprotein lipase deficiency. How is this disorder diagnosed? A. A lipid panel and family history are sufficient to diagnose familial LPL deficiency B. Infusion of plasma containing normal apoCII will reduce triglyceride levels C. Infusion of heparin will increase lipase activity in the plasma D. If the lipid panel shows in increase in LDL, HDL, and triglycerides
C. Infusion of heparin will increase lipase activity in the plasma
A 30-year-old woman is coming in for her lipid panel. It is shown that LDL makes up 75% of her total plasma cholesterol. What is the reasoning behind why LDL makes up the most of the total cholesterol panel? A. LDL has ApoB100 B. LDL has ApoCIII C. LDL lacks ApoE D. LDL lacks ApoB100 E. LDL has ApoE
C. LDL lacks ApoE
A 12-year-old female presents to the clinic with her father for an annual check-up appointment about her type I diabetes. Since the last appointment she has taken on the responsibility of checking and maintaining her diabetes. During the appointment she admits to having less control of her diabetes over the last year. After checking her blood sugar and lipid levels her physician sees that her TG level is around 1,000 mg/dL. A genetic test confirms that she has a mutation in a gene that results in the deficiency of LPL synthesis/function. Which of the following disorders will she be diagnosed with? A. Apolipoprotein C-II deficiency B. Elevated Lipoprotein(a) C. Lipoprotein Lipase (LPL) deficiency D. Familial hypertriglyceridemia E. Familial Defective Apolipoprotein B100
C. Lipoprotein Lipase (LPL) deficiency
A 45-year-old male went to a genetic counselor to get a referral for a prostate cancer screening. The screening was recommended due to a familial genetic predisposition for the disease. In fact, the counselor had the patient undergo a genome scan and was able to identify a polymorphism in chromosome 8q24. What genetic expression is altered biologically? A. MLH1 DNA repair genes B. CHK2 gene C. MYC oncogene D. KCNQ1 E. BRCA2
C. MYC oncogene
Which of the following is most correct about the immune system? A. The innate immune system does not use immunoglobulins, and it produces memory cells to fight off future infections. B. The innate immune system uses immunoglobulins, and it does not produce memory cells to fight off future infections. C. Mature B lymphocytes secrete circulating antibodies. D. Mature B lymphocytes carry-out cytotoxic functions. E. T lymphocytes develop in the bone marrow, while B lymphocytes develop in the thymus.
C. Mature B lymphocytes secrete circulating antibodies.
A 24 year old female presents with a characteristic butterfly rash on her face. Additional workup has revealed that she has anti-DNA antibodies circulating in her blood. Furthermore, she complains of joint pain and fatigue. Which of the following conditions is of the same category of hypersensitivity? A. Goodpasture syndrome B. Asthma C. Polyarteritis nodosa D. Psoriasis E. Rheumatoid arthritis F. D and E
C. Polyarteritis nodosa
Joe E., a 57-year-old male, presented to the emergency room after he was found unconscious in his prison cell. The inmates told the guards he was complaining of having a dry mouth, trouble swallowing, blurry vision and muscle weakness. They also told the guards that while he was hanging out with his friend Carole B. yesterday, she gave him some pruno, an alcoholic drink made illicitly in prison. Which of the following is the best choice of treatment for Joe? A. Symptomatic treatment with 3,4-DAP 5 to 10 mg every 3 to 4 hours B. Edrophonium chloride (Tensilon) C. Prompt mechanical ventilation, parenteral feeding and equine antitoxin D. A combination of an anti-muscarinic agent, AChE reactivator, and diazepam E. Pyridostigmine 60 mg every 4 hours
C. Prompt mechanical ventilation, parenteral feeding and equine antitoxin
Which one of the following diseases is a type IV hypersensitivity reaction associated with CD4+ T cell-mediated inflammation? A. Serum sickness B. Type I diabetes mellitus C. Rheumatoid arthritis D. Goodpasture syndrome E. Myasthenia Gravis
C. Rheumatoid arthritis
Mark enters the emergency room complaining of a dry mouth despite drinking a gallon of water. When taking the history and physical, it is learned that Mark also displays weakness in his lower extremities except on the weekends when he plays soccer. What cancer is associated with the disease Mark has? A. Pheochromocytoma B. Thymoma C. Small Cell Lung Cancer D. Rhabdomyosarcoma E. Prostate Cancer
C. Small Cell Lung Cancer
A newborn baby was diagnosed with Phenylketonuria during a routine checkup with their pediatrician two days after birth. Phenylketonuria is associated with what type of molecular lesion, and how will this lesion impact the primary enzyme involved in manifestation of the disease? A. Missense mutation, leading to impaired enzyme inhibitor secretion from liver to serum B. Point mutation, leading to abnormal enzyme with reduced activity C. Splice-site mutation, leading to reduced enzyme production D. Deletions, leading to reduced enzyme production E. Deletions, leading to nonfunctional or misfolded enzyme
C. Splice-site mutation, leading to reduced enzyme production
The innate and adaptive immune systems work together to efficiently remove a pathogen from the body, even though they have different mechanisms of doing so. Which of the following is true regarding the differences between the innate and adaptive immune systems? A. The adaptive immune system reacts very quickly to a foreign pathogen since it is a nonspecific response. B. The innate immune system reacts after being stimulated by the adaptive immune system and can make memory cells to mount a greater and faster response should the pathogen be encountered again. C. The innate immune system reacts very quickly to a foreign pathogen since it is a nonspecific response and stimulates the adaptive immune system, which is a delayed, but specific response, to make memory cells should the pathogen be encountered again. D. The adaptive immune system can still respond to an infection even if the innate immune system does not stimulate it first. E. The innate immune system can cause a massive, specific response to a pathogen after a couple days of "learning" the characteristics of that pathogen.
C. The innate immune system reacts very quickly to a foreign pathogen since it is a nonspecific response and stimulates the adaptive immune system, which is a delayed, but specific response, to make memory cells should the pathogen be encountered again.
A patient in presents in the clinic and is diagnosed with Severe Combined Immunodeficiency (SCID) due to a defective Recombinase enzyme. What is the mechanism by which these enzymes work in the immune system? A. They add additional nucleotides to ends of the VDJ sequences in an antibody's DNA. B. They add additional nucleotides to the ends of VDJ sequences in an antibody's RNA. C. They delete all but 1 V, D, and J segment in an antibody's DNA so it is structurally diverse. D. They delete all but 1 V, D, and J segment in an antibody's mRNA so it is structurally diverse. They help with somatic hypermutation resulting in affinity maturation
C. They delete all but 1 V, D, and J segment in an antibody's DNA so it is structurally diverse.
A patient with myasthenia gravis is prescribed neostigmine. One month after prescription, the patient returns to the clinic with progressive muscle weakness. You suspect undertreatment with the medication thus you administer edrophonium. What do you expect to happen? A. Long-term worsening of muscle weakness B No change in muscle weakness C. Transient improvement of muscle weakness D. Transient worsening of muscle weakness
C. Transient improvement of muscle weakness
A 35 year old woman presents to the clinic with a goiter. She was previously diagnosed with Graves Disease by another physician and wants more information and a second opinion on treatment options. The physician describes that Grave's Disease is a __________ caused by ______________. A. Type I Hypersensitivity; IgE production against an inappropriate antigen leading to mast cell degranulation B. Type II Hypersensitivity; Antibody mediated opsonization and phagocytosis of Erythrocytes C. Type II Hypersensitivity; Antibody mediated stimulation of TSH receptors D. Type III Hypersensitivity; Streptococcal cell wall antigens cross reacting with self antigen leading to formation and deposition of immune complexes Type III Hypersensitivity; Antibodies forming immune complexes with foreign serum
C. Type II Hypersensitivity; Antibody mediated stimulation of TSH receptors
A 45-year-old female comes in for a chief complaint of a rash that appears on her cheeks and nose after going outside. She states that her fingers and knees have been hurting and have been progressively getting worse over the course of six months. The physician notices proteinuria and a positive antinuclear antibody test (ANA). The patient is diagnosed with systemic lupus erythematous (SLE). SLE represents which of the following type of hypersensitivity reactions? A. Immediate type hypersensitivity B. Type II hypersensitivity C. Type III systemic hypersensitivity D. Type III local hypersensitivity E. Delayed type hypersensitivity
C. Type III systemic hypersensitivity
26-year-old female has been feeling fatigued for a couple of months and finally goes in to see a primary care physician after she finds blood in her urine and a strange rash on that goes from her left cheek, over her nose and onto the right cheek. After many tests, the physician concludes that the patient has systemic lupus erythematous. What is type of hypersensitivity is this disease and the pathologic immune mechanism? A. Type I; Th2 cells, IgE, mast cells, eosinophils B. Type II; IgM, IgG, antibodies against cell surface or extra cellular matrix antigens C. Type III; immune complexes of circulation antigens and IgM or IgG antibodies deposited in vascular basement membrane D. Type IV; CD4+ T-cells, CD8+ T cells E. None of the above correctly match the hypersensitivity with the mechanism.
C. Type III; immune complexes of circulation antigens and IgM or IgG antibodies deposited in vascular basement membrane
A 21-year-old female presents to the clinic with complaints of an erythematous rash on her cheeks, nose, and forehead. Symptoms started a couple days following her 21st birthday. Patient says she has not consumed alcohol since the night of her birthday and has started using a new face wash and shampoo she received as gifts. Suspecting a hypersensitivity, which type most likely coincides with the onset of the symptoms? A. Type III B. Type I C. Type IV D. Type II E. Type V
C. Type IV
A mother, with her 6-month-old son, presents to the clinic with the chief complaint that her son continuously experiences both severe otitis media, and well as pneumonia. The mother explains that for the first couple months of his life, he seemed perfectly happy and healthy, but now is constantly experiencing these infections, which have not been made better by any OTC medications. The doctor explains that her son has a rare genetic disorder where he does not produce B cells and is therefore unable to fight infections. What is the mode of inheritance and mutation responsible for her son's presentation? A. Autosomal dominant, mutation in BTK gene B. Autosomal recessive, mutation in RAG 1 or RAG2 C. X-linked recessive, mutation in BTK gene D. X-linked recessive, mutation in gene encoding subunit of cytochrome b E. Autosomal recessive, mutation in TAP2 gene
C. X-linked recessive, mutation in BTK gene
A 1-month-old infant is brought in by their parents for a follow up visit and to discuss abnormal gastrointestinal movements. Lipid panels, blood work and an MRI reveal an enlarged fatty liver and early atherosclerosis with hypertriglyceridemia. A differential diagnosis of Wolman disease is suspected with genetic testing to confirm. Assuming the differential is correct, what component of the lipid metabolism is being altered? A. decreased apoE2 B. increased HDL C. decreased LAD D. decreased apoC2 E. decreased LPL
C. decreased LAD
What do patients who suffer from Familial hypercholesterolemia develop? What happens it remains untreated in homozoygous
CHD, aortic stenosis due to atherosclerosis of the aortic root, and tendon xanthomas (often in the Achilles tendon). patients with homozygous familial hypercholesterolemia typically die of myocardial infarction before 20 years of age.
CI AND SIDE EFFECTS: fluvastatin
CI: Active liver disease, Pregnancy and lactation, Serum transaminase elevations, Hypersensitivity to drug SIDE EFFECTS: Myopathy, rhabdomyolysis, hepatotoxicity, dermatomyositis, systemic lupus erythematosus, Abdominal pain, constipation, arthralgia
CI AND SIDE EFFECTS: lovastatin
CI: Active liver disease, Pregnancy and lactation, Serum transaminase elevations, Hypersensitivity to drug SIDE EFFECTS: Myopathy, rhabdomyolysis, hepatotoxicity, dermatomyositis, systemic lupus erythematosus, Abdominal pain, constipation, arthralgia
CI AND SIDE EFFECTS: simvastatin
CI: Active liver disease, Pregnancy and lactation, Serum transaminase elevations, Hypersensitivity to drug SIDE EFFECTS: Myopathy, rhabdomyolysis, hepatotoxicity, dermatomyositis, systemic lupus erythematosus, Abdominal pain, constipation, arthralgia
CI AND SIDE EFFECTS: rusovastatin
CI: Active liver disease, Pregnancy and lactation, Serum transaminase elevations, Hypersensitivity to drug SIDE EFFECTS: Myopathy, rhabdomyolysis, hepatotoxicity, dermatomyositis, systemic lupus erythematosus, Abdominal pain, constipation, arthralgia, pancreatitis, acute renal failure
CI AND SIDE EFFECTS: pravastatin
CI: Active liver disease, Pregnancy and lactation, Serum transaminase elevations, Hypersensitivity to drug SIDE EFFECTS: Myopathy, rhabdomyolysis, hepatotoxicity, dermatomyositis, systemic lupus erythematosus, Abdominal pain, constipation, arthralgia, pancreatitis, musculoskeletal pain, upper respiratory infection
CI AND SIDE EFFECTS: pitavastatin
CI: Active liver disease, Pregnancy and lactation, Serum transaminase elevations, Hypersensitivity to drug, concomitant use of cylcosporine, SIDE EFFECTS:Myopathy, rhabdomyolysis, hepatotoxicity, dermatomyositis, systemic lupus erythematosus, Abdominal pain, constipation, arthralgia,
CI and Side Effects: pancuronium
CI: Drug interactions. Cholinesterase inhibiters + NDNMB (Reverses the effects). Inhaled aesthestics + NDNMB (sensitize nm junction to effects). Aminoglycoside (inhibits Ach release, prolonged nm blockade). CI in neonates. SIDE EFFECTS: anaphylaxis, hypertension, respiratory failure, prolonged muscle weakness
CI AND SIDE EFFECTS: mipomersen
CI: FDA approved for HoFH and requires physician certification and risk evelauation and mitigation strategy (REMS); hepatic impairment; serum transaminase elevations, SIDE EFFECTS: Steatosis of liver, anti-glomerular basement membrane tubulointerstitial nephritis, angioedema, cancer Mild increase in liver transaminases, increased hepatic fat, injection site reaction, flu-like symptoms, headache, fatigue, mild gastrointestinal distress, reduced vitamin E levels
CI AND SIDE EFFECTS: lomitapide
CI: FDA approved for HoFH and requires physician certification and risk evelauation and mitigation strategy (REMS); hepatic impairment; serum transaminase elevations; use of CYP3A4 inhibitors, SIDE EFFECTS: Steatosis of liver, anti-glomerular basement membrane tubulointerstitial nephritis, angioedema, cancer Mild increase in liver transaminases, increased hepatic fat, injection site reaction, flu-like symptoms, headache, fatigue, mild gastrointestinal distress, reduced vitamin E levels, chest pain, weight loss
CI and Side Effects: Hemicholinium-3
CI: N/A (experimental). SIDE EFFECTS: N/A (experimental)
Contraindications(CI) and Side Effects: vesamicol
CI: N/A (experimental). SIDE EFFECTS: N/A (experimental)
With sustained activation macrophages often undergo a morphologic transformation into what?
Epithelioid cells large epithelium-like cells with abundant cytoplasm
CI AND SIDE EFFECTS: gemfibrozil
CI: Preexisting gallbladder disease, Hepatic dysfunction, Severe renal impairment, Nursing mothers, Hypersensitivity to drug, Concomitant use with repaglinide or simvastatin SIDE EFFECTS: Elevated liver function tests, myopathy when co-administered with a statin, abdominal discomfort
CI AND SIDE EFFECTS: Niacin
CI: active liver disease, active peptic ulcer, arterial bleeding, hypersensitivity to niacin, SIDE EFFECTS: Cutaneous flushing/pruritis, hyperglycemia, hyperuricemia, hepatotoxicity, rhabdomyolysis
CI and Side Effects: succinylcholine
CI: drug interactions: inhaled anesthetics, & aminoglycoside/tetracyclines. Hyperkalemia - avoid in Pts w/ severe skeletal muscle trauma, cardiac arrest, and pediatrics, Carty: Hypersensitivity to succinylcholine, Personal or family history of malignant hyperthermia, Skeletal muscle myopathies, Upper motor neuron injury, Extensive denervation of skeletal muscle SIDE EFFECTS: Cardiac arrhythmia, cardiac arrest, malignant hyperthermia, muscle pain, increased intraocular pressure, hyperkalemia, rhabdomyolysis, anaphylaxis, renal failure, respiratory depression
CI and Side Effects: atropine
CI: hypersensitivity to atropine, narrow-angle glaucoma, reflux esophagitis, Ulcerative Colitis, paralytic ileus, or obstructive GI disease, unstable cardiovascular status, myasthenia gravis, SIDE EFFECTS: "Atropine toxicity: consusion, blurred vision, flsuhed skin, dry mucous/no sweat, hyperthermia. (treat w/ physostigmine). cardiac arrthymia, MI, anaphylaxis, delirum, hallucinations, seizure, glaucoma, pulmonary edema" Carty: Cardiac arrhythmia, myocardial infarction, anaphylaxis, delirium, hallucinations, seizure, glaucoma, pulmonary edema, Tachycardia, dry mucous membranes, flushing, gastrointestinal upset, xerostomia, dizziness, headache, blurred vision, urinary retention, impotence
CI and Side Effects: botulinum toxin
CI: hypersensitivity to botulinum toxin; infection at the proposed injection site, SIDE EFFECTS: "cardiac arrhythmia, syncope, hepatoxicity, anaphylaxis. also: injection site pain, dyspepsia, dysphagia, muscle weakness, neck pain, eyelid ptosis, fever"
CI and Side Effects: donepezil
CI: hypersensitivity to drug SIDE EFFECTS: Cardiac arrthymia
CI AND SIDE EFFECTS: cholestyramine
CI: hypersensitivity to drug, Hypertriglyceridemia, Complete biliary obstruction SIDE EFFECTS: Increase in triglyceride levels, bloating, dyspepsia, flatulence, bleeding diathesis secondary to vitamin K deficiency
CI and Side Effects:pilocarpine
CI: hypersensitivity to drug, acute iritis, narrow-angle glaucoma, asthma. SIDE EFFECTS: DUMBBEELSS; retinal detachment, pulmonary edema
CI AND SIDE EFFECTS: colesevelam
CI: hypersensitivity to drug, hypertriglyceridemia, complete biliary obstruction, SIDE EFFECTS: Pancreatitis, heart disease, Increase in triglyceride levels, bloating, dyspepsia, flatulence, bleeding diathesis secondary to vitamin K deficiency
CI and Side Effects: physostigmine
CI: hypersensitivity to drug, mechanical intestinal or urinary obstruction, cardiovascular disease, asthma, diabetes, gangrene, concomitant choline ester or depolarizing neuromuscular blocker use; SIDE EFFECTS: "seizure, bronchospasm, cardiac arrhythmia, bradycardia, cardiac arrest. also: hypotension or hypertension, salivation, lacrimation, diaphoresis, vomiting, diarrhea, miosis, frequent urination
CI and Side Effects: pyridostigmine
CI: hypersensitivity to drug, mechanical intestinal or urinary obstruction. SIDE EFFECTS: "seizure, bronchospasm, cardiac arrhythmia, bradycardia, cardiac arrest. also: hypotension or hypertension, salivation, lacrimation, diaphoresis, vomiting, diarrhea, miosis, frequent urination "
CI AND SIDE EFFECTS: ezetimibe
CI: hypersensitivity to ezetimibe, concomitant use with a statin in pregnancy or while nursing, active liver disease, unexplained hepatic transaminase elevations, SIDE EFFECTS: Hepatitis, myopathy, rhabdomyolysis, Arthralgia, myalgia
CI and Side Effects: methacholine
CI: hypersensitivity to methacoline, recent MI or stroke, aortic aneurysm, uncontrolled HTN, low baseline pulmonary function tests SIDE EFFECTS: DUMBBEELSS; dyspnea, light-headedness, headache, pruritus, throat irritation DUMBEELSS: Diarrhea (Diaphoresis), Urination, Miosis, Bronchospasm (secretion) Bradycardia, Excite skeletal muscle and CNS (Emesis), Lacrimation, Lethargy, Salivate
CI AND SIDE EFFECTS: omega-3 fatty acids (EPA and DHA)
CI: hypersensitivity to omega-3 fatty acids, SIDE EFFECTS: increased low-density lipoprotein cholesterol, GI discomfort
CI and Side Effects: scopolamine
CI: hypersensitivity to scopolamine, chronic lung disease, hepatic or renal impairment, narrow-angle glaucoma, prostatic hypertrophy, pyloric obstruction, SIDE EFFECTS: "glaucoma, drug induced psychosis. also: somnolence, xerostermia, blurred vision"
CI and Side Effects:bethanechol
CI: hypersenstivity to drug, asthma, bradycardia, hypotension, CAD, epilepsy, compromised integrity of GI or bladder wall, hyperthyroidism, parkinsonism, peptic ulcer, SIDE EFFECTS: DUMBBEELSS; seizure, asthma exacerbation
CI and Side Effects: ipratropium
CI: hypersenstivity to ipratropium, SIDE EFFECTS: "hypersenstivity reaction, stroke, MI, bronchospasm. also: abnormal taste in mouth, xerostermia, bronchitis, sinusitus"
CI AND SIDE EFFECTS: fenofibrate
CI: prexisting gallbladder disease, hepatic dysfunction, severe renal impairment, nursing mothers, hypersentivity to drug, SIDE EFFECTS: Elevated liver function tests, myopathy when co-administered with a statin (shared adverse effects); pancreatitis, serum creatinine elevation, Abdominal discomfort
Clinical commentary
CLINICAL COMMENTARY 9-1 The Immune Response as a Molecular Arms Race The vast majority of the pathogens that assault the human body are destroyed by our immune system. Consequently, there is strong natural selection for pathogens that can evade immune surveillance and destruction. These microorganisms often have high mutation rates, and their numbers are huge. Thus, despite their biological simplicity, viruses and other pathogens have evolved some very clever ways of overcoming the immune response. Our immune systems, in turn, are constantly creating new ways to deal with pathogenic ingenuity. Three examples of this molecular "arms race" are discussed here. Cytomegalovirus (CMV) is a common infectious agent that can produce mononucleosis, hemolytic anemia, pneumonitis, congenital infections, and thrombocytopenia (a decrease in the number of platelets). Cells infected with CMV are the targets of destruction by cytotoxic T cells. However, some CMV strains (as well as other viruses and tumor cells) can evade T-cell detection by down-regulating the expression of class I molecules on the surfaces of infected cells. Without viral peptide presentation by class I molecules, the cytotoxic T cells are blind to the presence of CMV, and they do not destroy the infected cell. At this point, the cell would normally become the target of natural killer cells, which attack cells that lack MHC class I molecules on their surfaces. But CMV has devised a way to outwit natural killer cells, too. The virus encodes a cell-surface protein that resembles class I molecules closely enough so that the natural killer cells mistake the viral protein for a true class I molecule. The viral protein is also sufficiently different from a true class I MHC molecule that it does not trigger destruction by the more finely tuned cytotoxic T cell. In this way, CMV can often avoid destruction by both T cells and natural killer cells. An important immunological challenge for placental mammals is pregnancy, in which placental cells express foreign class I MHC molecules derived from the father. Ordinarily, such cells would be quickly destroyed by the mother's cytotoxic T cells. To avoid this, placental cells down-regulate the expression of class I MHC molecules on their surfaces. As with viral downregulation of class I molecules, this lack of MHC class I expression leaves the placental cells liable to destruction by the mother's natural killer cells. In this case, the cells are saved from destruction by presenting HLA-G molecules on their surface. This relatively nonvariant MHC molecule does not stimulate a T-cell response, which is limited to presentation of HLA-A, -B, and -C molecules. It does inhibit the natural killer cell, which has HLA-G receptors on its surface. The fetus, like CMV, has thus devised ways of avoiding destruction by both T cells and natural killer cells. A third example of the molecular arms race is given by one of the most feared infectious agents of modern times, the human immunodeficiency virus (HIV). Some strains of this virus gain entry into macrophages and helper T cells via a cell-surface receptor, CC chemokine receptor 5 (CCR5). Once inside the cell, HIV inserts its genetic material into the nucleus and takes advantage of the cell's machinery to replicate itself (HIV is a retrovirus, a type of virus that is discussed further in Chapter 13). Helper T cells are a critical component of the body's immune system, and their destruction by HIV leads to severe secondary immunodeficiency. Persons who are homozygous for a 32-bp deletion of the CCR5 gene lack the CCR5 receptor and are thus remarkably resistant to HIV infection. Among those who are heterozygous for this deletion, progression to AIDS symptoms after seroconversion is slowed by 2 to 4 years. This deletion is especially common in northeastern European populations, where the gene frequency reaches 0.20. It is virtually absent in Asian and African populations. Analysis of linkage disequilibrium in the chromosomal region that contains CCR5 indicates that the deletion arose in European populations only 700 to 2000 years ago. Because HIV appeared in humans only a few decades ago, the high gene frequency in northeastern Europe cannot be due to HIV and must be due to another selective force or perhaps to genetic drift. Considering the age of the deletion, it is possible that it underwent positive selection because it provided resistance to pathogens, such as smallpox, that ravaged European populations in the past. Clearly, there would be strong selection in favor of this mutation in populations now heavily exposed to HIV, and the deletion would eventually rise in frequency in these populations. Even now, knowledge of the consequences of this deletion may help accelerate the medical arms race against HIV, where genetic modification of CCR5 is being tested in HIV clinical trials (see Chapter 13).
What adverse effects can acute toxicity with nicotine, often due to ingestion of cigarettes and insecticides, cause?
CNS hyperexcitation (seizures progressing to coma and respiratory arrest), skeletal muscle depolarization blockade (respiratory arrest), and cardiovascular abnormalities (hypertension and arrhythmias)
Cancer
Cancer is the second leading cause of death in the United States, although it is estimated that it might soon surpass heart disease as the leading cause of death. It is well established that many major types of cancer (e.g., breast, colon, prostate, ovarian) cluster strongly in families. This is due both to shared genes and shared environmental factors. Although numerous cancer genes have been isolated, environmental factors also play an important role in causing cancer by inducing somatic mutations. In particular, tobacco use is estimated to account for one third of all cancer cases in developed countries, making it the most important known cause of cancer. Diet (i.e., carcinogenic substances and the lack of "anticancer" components such as fiber, fruits, and vegetables) is another leading cause of cancer and may also account for as much as one third of cancer cases. It is estimated that approximately 15% of worldwide cancer cases are caused primarily by infectious agents (e.g., human papilloma virus for cervical cancer, hepatitis B and C for liver cancer). Because cancer genetics was the subject of Chapter 11, we confine our attention here to genetic and environmental factors that influence susceptibility to some of the most common cancers.
What are the most life threatening features of Marfan syndrome?
Cardiovascular lesions 2 most common lesions: 1- Mitral valve prolapse 2-Dilation of the ascending aorta due to cystic medionecrosis (greater importance) Histologically the changes in the media are virtually identical to those found in cystic medionecrosis not related to Marfan syndrome excessive TGF-β signaling in the adventitia may also contribute to aortic dilation. Weakening of the media predisposes to an intimal tear, which may initiate an intramural hematoma that cleaves the layers of the media to produce aortic dissection.
What are Bile acid sequestrants?
Catonic polymer resins that bind noncovalently to negatively charged bile acids in the small intestine
The Non-neuronal cholinergic system (NNCS)
Cells o various non-neuronal tissues and organs produce ACh (non-neuronal ACh [NN-ACh]). NN-ACh acts in an autocrine and paracrine ashion on nicotinic and muscarinic ACh receptors that are expressed on neighboring ACh-producing or e ector cells. The NNCS regulates physiologic processes including cell growth, adhesion, migration, and di erentiation. Dys unction o the NNCS may contribute to the pathogenesis o disease in several organ systems, including the skin (e.g., atopic dermatitis, pemphigus, psoriasis, vitiligo) and the urinary (e.g., overactive bladder syndrome), gastrointestinal (e.g., gastroesophageal re ux disease, peptic ulcer disease, pancreatitis), immune (e.g., Sjogren's syndrome, rheumatoid arthritis, sepsis), pulmonary (e.g., asthma, chronic obstructive pulmonary disease, cystic f brosis), musculoskeletal (e.g., osteoporosis, tendinosis), reproductive (e.g., sperm dysmotility), and cardiovascular systems (e.g., atherosclerosis). For some conditions, anticholinergic drugs are already standard therapeutic choices. However, or most organ systems, the role o the NNCS remains to be urther delineated, and specif c therapeutic interventions targeted at the NNCS remain to be developed.
HDL mediated cholesterol efflux from cells
Cellular cholesterol e ux is the mechanism by which excess insoluble cholesterol molecules are removed rom cells. This occurs when unesterif ed cholesterol is trans erred rom the plasma membrane o cells to an HDL particle. The mechanism o cholesterol e ux varies depending on the cell type and the type o HDL particle. Lipid-poor pre- HDL particles, apoAI, and apoAII can promote cholesterol e ux by interacting with ABCA1. This process is not only important in HDL ormation by the liver but is also a mechanism or removing excess cholesterol rom cells within the subendothelial space and or protecting macrophages rom cholesterol-induced cytotoxicity. Spherical HDL very e f ciently stimulates cholesterol e ux by several di erent mechanisms. First, the interaction o apoAI on HDL with SR-BI on the plasma membrane promotes cholesterol e ux. Second, macrophages express not only ABCA1 and SR-BI but also ABCG1, which also mediates cholesterol e ux to spherical HDL. Finally, spherical HDL particles may promote cholesterol e ux in the absence o binding to a specif c cell sur ace protein. Although cholesterol has very low monomeric solubility, it can dissociate in appreciable amounts and di use short distances through the plasma to acceptor particles that are enriched with phospholipids on their sur aces. Quantitatively, e f ux to spherical HDL particles accounts or most o the removal o excess cholesterol rom cells . This capacity o HDL to remove cellular cholesterol is enhanced by the activities o LCAT and PLTP, which prevent the sur ace coat o the particle rom becoming saturated with cholesterol.
What enzyme expressed only in hepatocytes catalyzes the rate limiting step in the catabolism of cholesterol to bile acids?
Cholesterol 7alpha-hydroxylase (CYP7A1) Bile acids, unlike cholesterol, are highly soluble in water. Moreover, bile acids are biological detergents that promote the formation fo micelles
What does excess LDL result in the formation of ? What areas can they deposit and what are they called?
Cholesterol plaques 1- Arteries- Atheroma 2- Skin and tendon- Xanthoma 3- Eyelids- Xanthelasma 4- Iris- corneal arcus Metabolic disorders such as diabetes and obesity are often associated with vascular disease and hypertriglyceridemia, and the atherogenic impact of other elements in these disorders is difficult to separate from the effect of hypertriglyceridemia. However, in several population-based studies, abnormal triglyceride levels correlated with increased risk for CHD.
Intro
Cholinergic pharmacology is centered on the properties o the f rst identif ed neurotransmitter, acetylcholine (ACh) . The unctions o cholinergic pathways are diverse; generally, they involve the neuromuscular junction (NMJ), the autonomic nervous system, the central nervous system (CNS), and the non-neuronal cholinergic system (NNCS). In the neuronal cholinergic systems, ACh acts as a neurotransmitter at the NMJ, at autonomic ganglia, at terminal synapses o parasympathetic postganglionic f bers and a ew sympathetic postganglionic f bers, and in the CNS. Many non-neuronal cells also express ACh receptors and can thereby serve as e ector cells or both neuronally and non-neuronally released ACh. Despite the many physiologic actions o ACh, the current therapeutic uses or cholinergic and anticholinergic drugs are limited by the ubiquitous nature o cholinergic pathways, and thus, by the inherent di f culty o e ecting a specif c pharmacologic intervention without inducing adverse e ects. Nonetheless, medications with somewhat targeted cholinomimetic and anticholinergic actions are in widespread clinical use or their e ects on the brain (especially cognition and behavior), neuromuscular junction, heart, eyes, lungs, and genitourinary and gastrointestinal tracts. Other chapters that discuss applications o cholinergic pharmacology are Chapter 18, Pharmacology o Analgesia;
What are the enzymes responsible for the degradation of Acetylcholine collectively known as? What are the 2 types?
Cholinesterases 1- Acetylcholinesterase (AChE) 2- Butyrylcholinesterase (BuChE)
What are the symptoms occur as a result of Pyloric stenosis? How can it be treated?
Chronic vomiting, constipation, weight loss and electrolyte imbalance Can be corrected by surgery or sometimes resolves spontaneously
What is marked hypertriglyceridemia (>1000 mg/dL) associated with? which is characterized by what?
Chylomicronemia syndrome 1- Pancreatitis 2- Xanthoma
What 2 factors is there abundant evidence for that they increase the risk of CAD? With 2 factors decrease the risk for CAD?
Cigarette smoking and Obesity Exercise and a diet low in saturated fats The approximate 60% reduction in age-adjusted mortality due to CAD and stroke in the United States since 1950 is usually attributed to a decrease in the percentage of adults who smoke cigarettes, decreased consumption of saturated fats, improved medical care, and increased emphasis on healthy lifestyle factors such as exercise.
How are Immune complexes deposited?
Circulating antigen-antibody complexes are deposited in various tissues complexes that are of medium size, formed in slight antigen excess, are the most pathogenic Organs where blood is filtered at high pressure to form other fluids, like urine and synovial fluid, are sites where immune complexes become concentrated and tend to deposit; hence, immune complex disease often affects glomeruli and joints
Studies for the genetic determinants of Coronary artery disease have been focused on what?
Circulating lipoprotein levels
A 13 year old girl has recently just become the world's youngest female olympian to win a gold medal on her floor routine for the United States of America. While watching the Olympics at home on TV, a family practitioner from Millcreek Community Hospital notices that her joints are hypermobile, there are lots of bruises on her arms and legs, and she has slight scoliosis. The physician is suspicious that the girl has Ehlers-Danlos Syndrome. What gene is most likely mutated? A) COL3A1 B) COL5A1 C) COL2A2 D) COL1A1 E) COL2A1
D) COL1A1
The rate of reverse cholesterol transport is said to have an inverse correlation to plasma HDL levels and Cardiovascular Disease in humans. The regulation of such involves several important organs that are responsible for the synthesis, storage, and excretion of cholesterol. What enzyme is involved in the rate-limiting step of the catabolic process in which cholesterol is converted to bile acids and in what cell type does this conversion take place in? A) CYP450, hepatocytes B) CYP3A4, hepatocytes C) CYP7A1, endothelial cells of the gallbladder D) CYP7A1, hepatocytes E) CYP1A7, enterocytes
D) CYP7A1, hepatocytes
A patient has recently undergone testing and it is discovered that the patient has a primary immunodeficiency disorder in which phagocytes can ingest bacteria and fungi, but are unable to kill them. What is the most common cause of this immunodeficiency disorder? A) A defect in TAP2 B) A lack of class 1 MHC C) Loss of function mutation in the in the gene WAS D) Caused by an X-linked mutation E) None of the above
D) Caused by an X-linked mutation
A 32-year-old male comes in for a chief complaint of joint pain in his knees and hips. He states the pain has been progressively getting worse over the course of 2 years. Urinalysis reveals urine that turns black when it is exposed to air. The patient is diagnosed with a metabolic disorder that results from a defect in homogentisate oxidase. Which of the following modes of inheritance regarding this disorder is true? A) It is likely to appear in each generation B) Males will be hemizygous C) It requires one affected allele for the disease to occur D) Consanguinity may be a factor E) At least one parent will be affected
D) Consanguinity may be a factor
A 53-year-old male complained of difficulty finding regularly used items in his home, agitation and moodiness, and jumbled speech. As a result he is given a prescription for an acetylcholinesterase inhibitor and has been accidently taking two times the prescribed dosage for two weeks. What are some symptoms are expected to see as a result of this? A) Miosis, arteriole contraction, and bradycardia B) Mydriasis, urinary retention, and xerostomia C) Arteriole vasodilation, cycloplegia and xerostomia D) Diaphoresis, diarrhea, and erection E) Tachycardia, bronchoconstriction and anhidrosis
D) Diaphoresis, diarrhea, and erection
A 14 year old patient comes into the office and complains about some type of yellow growth on the his Achilles tendon. The doctor looks at it and decides to order a lipid profile test for the patient. The physician gets the results and observes that there is elevated total cholesterol (600-1000mg/dL ;normal>300mg/dL), elevated LDL-cholesterol (550-950mg/dL; normal>250mg/dL) but normal triglycerides and HDL-cholesterol lvls. The physician begins to discuss ways to help possibly treat this concerning problem through a combination diet w/ drug therapies that control cholesterol since if left untreated can lead to myocardial infarction. What disease is most likely the cause of these problems? A) Familial Dysbetalipoproteinemia B) Familial Hypertriglyceridemia C) Polygenic Hypercholesterolemia D) Familial Hypercholesterolemia E) Lipoprotein Lipase Deficiency
D) Familial Hypercholesterolemia
A 10 year old female presents to the pediatricians office with her mother who explains that she has had increased thirst, frequent urination along with unintended weight loss. The mother also mentions that her daughter has had an increased appetite despite the weight loss. The physician believes she has Type 1 diabetes. Which MHC associated Locus is associated with Type 1 Diabetes? A. B27 B. DRB1 C. A3 D. DQB1 E. B53
D. DQB1
A 42-year-old female comes to the family physician office after discussing possible familial complications. Her mother passed away a year ago due to chronic heart disease, which has also been the case for her mother's sister as well as her grandmother. A lipid panel shows elevated triglyceride and low HDL levels, while all other lipid tests show normal levels. Further testing revealed increased hepatic VLDL. She was told by a previous doctor to avoid drinking alcoholic beverages until further notice. What familial lipid disorder might this patient have? A) Familial hypertriglyceridemia B) Familial dysbetalipoproteinemia C) Familial combined hyperlipoproteinemia D) Familial defective apo B100 E) Familial hypercholesterolemia
D) Familial defective apo B100
A normal immunoglobulin molecule consists of two identical heavy chains and two identical light chains. Which chain and region determines the major class to which an immunoglobulin belongs? A) Heavy chain variable region B) Light chain variable region C) Light chain constant region D) Heavy chain constant region E) An immunoglobulin class can be determined by any chain and region.
D) Heavy chain constant region
Pam states she experiences allergies very frequently. Allergies are mediated by which of the following immunoglobulin? A) IgG B) IgA C) IgD D) IgE E) IgM
D) IgE
A 38-year-old male recently diagnosed with familial hypercholesterolemia was prescribed Atorvastatin to control his cholesterol levels. After about a month of diet modification, lifestyle changes, and consistent use of the statin, his physician finds insufficient reduction of LDL plasma levels. His physician therefore prescribes an adjunct, orally-administered drug that exclusively acts at the brush border within the intestines. What is the method of action of this drug? A) It inhibits Microsomal Triglyceride-Transfer Protein, decreasing the assembly of chylomicrons B) It inhibits Niemann-Pick C1-like 1 protein, increasing the absorption of cholesterol C) It inhibits Proprotein Convertase Subtilisin/Kexin Type 9, increasing the amount of LDL receptors D) It inhibits Niemann-Pick C1-like 1 protein, decreasing the absorption of cholesterol E) It potentiates Niemann-Pick C1-like 1 protein, decreasing the absorption of cholesterol
D) It inhibits Niemann-Pick C1-like 1 protein, decreasing the absorption of cholesterol
A 19-year-old male patient presents to his doctor's office for his yearly check-up. During the physical exam it is noted that the patient displays xanthomas on his skin and xanthelasma on his eyelids. The physician decides to order a lipid panel which later shows total elevated cholesterol of 800 mg/dL and LDL-cholesterol of 600 mg/dL. Triglyceride and HDL levels are normal. This disease presentation is most likely due to: A) Overproduction of VLDL B) apo C-II deficiency C) LPL deficiency D) LDL Receptor mutation E) Low affinity apo E2
D) LDL Receptor mutation
An 8 year old boy is sent to the school nurse around 3 PM for a large swollen area on his left arm. The boy claims this started off as a small itchy bump that progressively grew over the past 6 hours. Around that time, the boy was in art class painting with some brand-new paint that the school had just bought but he had used at home once before. He accidentally set his left arm in some paint during class. What is the most likely cause of the boy's presentation at the nurse's office? A) Type II Hypersensitivity reaction B) Type III Hypersensitivity reaction C) Immediate hypersensitivity reaction D) Late-Phase reaction E) Anaphylaxis
D) Late-Phase reaction
A 19-year-old female presents to her cardiologist's office for a testing follow up due to her recent diagnosis of familial hypercholesterolemia. Test results indicate that she has the rare, homozygous form of this autosomal dominant disease. The cardiologist recommends a low-fat, low-cholesterol diet in conjunction with drug therapy of lovastatin and a drug which binds to apo B mRNA, inhibiting its production. Which of the following was she prescribed? A) Rosuvastatin B) Ezetimibe C) Colestipol D) Mipomersen E) Lomitapide
D) Mipomersen
A 39-year-old female presents to her family physician with concerns about Alzheimer Disease. Her mother was diagnosed with early onset Alzheimer Disease at the age of 60, and now she wants to know her likelihood of developing the disease. The physician orders a genetic test. Mutation in what gene(s) results in a greater proportion of long amyloid products in the brain? A) Mutation in GABRG2 B) Mutation in HNF4a C) Mutation in MYBPC D) Mutation in PS1 and PS2 E) Mutation in TP53
D) Mutation in PS1 and PS2
Jared is a 47 year old male who presents to his primary care physician after experiencing some adverse effects with a new medication that is supposed to help his hyperlipidemia. He notes that he experiences flushing and pruritis shortly after taking his medicine. His physician recommends taking a NSAID before taking his medicine as well as switching Jared to a timed-release form of this medication. What drug was Jared most likely taking? A) Ezetimibe B) Lomitapide C) Gemfibrozil D) Niacin E) Omega 3 Fatty Acids
D) Niacin
An 8-month-old male is taken to a small clinic in a developing country by his mother for a follow-up appointment after having presented with mild deformities in the bones of the legs, the skull, and the pelvis 2 months prior. Original history had revealed that the patient's diet may not have contained sufficient amounts of vitamin D, which contributed to the physician suspecting Rickets as a possible cause. After the first appointment, the mother was given vitamin D supplements to give to the child. The mother now states that she gave the child the supplements as directed, but the symptoms are now even worse, and his legs are beginning to appear bowed. The physician then orders a vitamin D blood test, which comes back as normal. Still suspecting Rickets, the physician orders a genetic screening. Assuming the physician's suspicion of Rickets is correct, what type of mutation would likely be seen and what type of protein would it affect? A) Large deletions affecting vitamin D transport proteins on the intestinal lumen. B) Point mutations affecting lysosomal enzymes. C) Point mutations affecting vitamin D transport proteins on the intestinal lumen. D) Point mutations affecting the vitamin D receptor. E) Large deletions affecting the vitamin D receptor.
D) Point mutations affecting the vitamin D receptor.
A 24-year-old patient presented to the clinic with nasal congestion, itchy throat and dry cough. In addition, the stress of social-distancing and being quarantined by her family for 2 weeks against her will caused her to form wrinkles beneath her eyes. After confirming it was only seasonal allergies, the dermatologist recommended an injection which will help clear up and prevent further formation of wrinkles. What is the mechanism of action of the recommended injection? A. Inhibition of high affinity choline transporter B. Inhibition of Ach-H antiport C. Reversible inactivation of AchE D. Degradation of Snap-25 E. Irreversible inactivation of AchE
D. Degradation of Snap-25
A 63-year old woman follows up with her PCP to discuss her nonfasting lipid panel results done at a free wellness clinic. The woman is concerned that she might need medication because she is a smoker, and she has a "high number" for her LDL and cholesterol. Based on current guidelines, what should be the PCP's next course of action if the patient's results are as follows: Total Cholesterol 240 mg/dL HDL 39 mg/dL LDL 140 mg/dL [Optimal values: Total Cholesterol <200 mg/dL, HDL >60mg/dL, LDL <100mg/dL] A) Place patient on a cholesterol absorption inhibitors B) Begin a high fat diet to increase the HDL levels since they are "good" fats C) Place patient on a HMG-CoA inhibitors D) Retest the patient with a fasting lipid panel before making any changes to their lifestyle E) Advise the patient to come and get rescreened in 7 years when she is 70
D) Retest the patient with a fasting lipid panel before making any changes to their lifestyle
A 29-year-old male is preparing for his honeymoon cruise in the Bahamas. He has a history of severe motion sickness and mentions this problem to his primary care physician. To prevent the motion sickness from occurring on his cruise, his physician prescribes a transdermal patch that acts as a muscarinic antagonist. Which of the following medications was most likely prescribed? A) Methacholine B) Donepezil C) Ipratropium D) Scopolamine E) Pilocarpine
D) Scopolamine
A 35-year-old female presents to her physician's office for her annual physical. She appears in overall good health and mentions she is nervous for her upcoming vacation. She mentions that she is planning to go on a cruise for her best-friend's bachelorette party but gets extremely motion sick even in the car. She asks if there is any sort of medication or remedy that would be able to alleviate her nausea from motion sickness. What medication could the physician prescribe as an antiemetic? A) Atropine B) Ipratropium C) Hemicholinium-3 D) Scopolamine E) Botulinum Toxin
D) Scopolamine
A 58-year-old female presents to the Emergency Room after experiencing pain in her joints. She stated that she had also experienced a fever in the past week. During the exam, the physician notes lymph node enlargement. After further workups, the physician makes the diagnosis of arthritis. Based on this description and knowledge of pathogenesis, which of the following diseases does arthritis fall under and which is the correct progression of arthritis? A) Local Immune Complex Disease; formation of immune complexes leads to deposition of immune complexes leads to inflammation & tissue injury B) Systemic Immune Complex Disease; formation of immune complexes leads to inflammation & tissue injury leads to deposition of immune complexes C) Local Immune Complex Disease; localized area of tissue necrosis resulting from acute immune complex vasculitis D) Systemic Immune Complex Disease; formation of immune complexes leads to deposition of immune complexes leads to inflammation & tissue injury E) Systemic Immune Complex Disease; Inflammation & tissue injury leads to formation of immune complexes leads to deposition of immune complexes
D) Systemic Immune Complex Disease; formation of immune complexes leads to deposition of immune complexes leads to inflammation & tissue injury
A 32 year-old-female presents to her OB-GYN as she recently found out she is pregnant with her first child. Her physician states it is important to start therapy for future pregnancies as her blood type is A- and the father's is B+. However, aside from other risk factors, she is unlikely to experience any complications in her current pregnancy without therapy. Why is this true? A) The father is heterozygous (Dd) for the Rh gene. Only homozygous dominant (DD) fathers can cause erythroblastosis fetalis in an Rh (-) mother. B) The mother has formed anti-Rh antibodies, but it is unable to cross the placenta. C) The relationship between a mother's and fetus's Rh factor is not important. D) The mother has not yet formed anti-Rh antibodies E) Even if the fetus is Rh (+), the mother's immune system will not produce antibodies during delivery as it is capable of recognizing the blood as self.
D) The mother has not yet formed anti-Rh antibodies
A 63-year-old male presents with progressive muscle weakness and fatigue. Physical examination exhibits ptosis and difficulties with swallowing. He is diagnosed with Myasthenia Gravis and put on a drug designed for chronic treatment. What is the most likely mechanism of this newly prescribed medication? A. Agonist at muscular nicotinic receptor B. Degradation of SNAP-25 C. HMG-CoA Reductase inhibitor D. Acetylcholinesterase inhibitor E.Agonist at M1 receptors
D. Acetylcholinesterase inhibitor
A medical student is contacting agencies for research in adoption studies to estimate the genetic contribution to multifactorial traits. He looked into an adopted monozygotic twin that was adopted by a newlywed couple that recently have been diagnosed with schizophrenia. History of the adopted twins show that their biological parents have schizophrenia, but their adopted parents do not. Although the medical student is aware that schizophrenia is seen in 8% to 10% of adopted children whose natural parents had it, as opposed to 1% of unaffected parents, he feels he should take precaution with his interpretations. What is/are factors that could exaggerate the apparent influence of biological inheritance? A. Prenatal environmental influences could have long-lasting effects on an adopted child B. Children are sometimes adopted after they are several years old, ensuring that some nongenetic influences have been imparted by the natural parents C. adoption agencies sometimes try to match the adoptive parents with the natural parents in terms of attributes such as socioeconomic status D. All the above E. Both A and B
D. All the above
After presenting with high cholesterol, a patient is prescribed a drug that acts by inhibiting the rate limiting enzyme in cholesterol synthesis. Which of the following is NOT a pleiotropic effect of this drug? A. Decreased Inflammation B. Decreased Thrombosis C. Reversal of endothelial dysfunction D. Destabilization of atherosclerotic plaques E. Decrease in acute-phase reactants
D. Destabilization of atherosclerotic plaques
A 58-year-old male presents to his cardiologist after recovering from an MI. This was his first MI and told his physician about his concerns for prevention. The physician prescribes him a medication used for secondary prevention of a myocardial infarction. He also explains to the patient the other beneficial effects of the medication including decreased inflammation and thrombosis, reversal of endothelial dysfunction and improved stability of atherosclerotic plaques. Which drug did the physician prescribe? A. An anti-hypertensive drug that dilates vascular smooth muscle B. An antilipidemic drug that reduces cholesterol absorption by the small intestine C. An anti-hypertensive drug that suppresses the sympathetic nervous system D. An antilipidemic drug that inhibit HMG-CoA reductase E. An antilipidemic drug that bind to and activate PPAR𝛂
D. An antilipidemic drug that inhibit HMG-CoA reductase
A 24-year-old male presents to the clinic saying that his legs have been feeling weak and wobbly lately and that he has been having trouble walking. He also claims that he has been trying to drink more water recently because his mouth always feels dry and he hasn't been sweating as much as he used to so he thinks he must be really dehydrated. Physical examination reveals decreased tendon reflexes and slow pupillary reflexes. What is most likely causing this patients symptoms? A. Antibodies against the postsynaptic acetylcholine receptors B. An organophosphate compound used as a pesticide C. A neurotoxin produced by an anaerobic spore-forming bacterium D. Antibodies against the P/Q type voltage-gated calcium channel E. A demyelinating polyradiculoneuropathy associated with antiganglioside antibodies
D. Antibodies against the P/Q type voltage-gated calcium channel
A 28-year-old female is 27 weeks pregnant with her second child. The mother is Rh-, while her first and second child are both Rh+. The mother is given RhoGAM to prevent her anti-Rh antibodies from crossing the placenta and causing damage to her second child. Which type of hypersensitivity reaction is being prevented by the RhoGAM? A. Type I hypersensitivity B. T cell-mediated hypersensitivity C. Type IV hypersensitivity D. Antibody-mediated hypersensitivity E. Immediate hypersensitivity
D. Antibody-mediated hypersensitivity
A 24 year old male is seen by his primary care physician. He has a history of connective tissue disorder affecting the production of fibrillin-1. While discussing the complications of his disorder, the physician takes extra time to explain the most serious outcomes. The physician likely spoke at great length about which of the following conditions? A. Stroke B. Blindness C. Brittle bones D. Aortic dissection E. Liver failure
D. Aortic dissection
A 68-year-old male presents to his primary care physician with complaints of mild cognitive impairment, dementia and memory loss. The patient notes that his symptoms have gradually increased over the past 2 years. After the workup, the patient is diagnosed with late-onset Alzheimer Disease. A defect in which of the following genes is most commonly associated with late-onset Alzheimer Disease? A. Presenilin 1 (PS1) B. Presenilin 2 (PS2) C. Amyloid-β precursor protein (APP) D. Apolipoprotein E (APOE) E. BRACA1
D. Apolipoprotein E (APOE)
A patient comes into your office for a checkup, family history reveals that his father and grandfather both have died from myocardial infarctions. Upon further examination, you noticed around his Achilles area that it is quite enlarged and swollen. After doing some genetic testing and see that this person has a mutation that encodes for the LDL receptors. Which of the following is correct about the disorder this patient has? A. Autosomal Recessive Disorder B. X-linked Disorder C. Y- linked Disorder D. Autosomal Dominant Disorder E. Mitochondrial Disorder
D. Autosomal Dominant Disorder
Over the course of a typical week in a pediatrics practice, a physician saw two different toddlers who had recently been diagnosed with Neurofibromatosis Type 1. The first toddler's parents noticed several brownish spots on his back and decided to seek answers. The second toddler's diagnosis was made after she was diagnosed with multiple skin tumors early in her life. What is the inheritance pattern of Neurofibromatosis Type 1 and what is the explanation for the unique phenomenon described? A. Autosomal Recessive Disorder, Incomplete Penetrance B. Autosomal Dominant Disorder, Incomplete Penetrance C. Autosomal Recessive Disorder, Variable Expressivity D. Autosomal Dominant Disorder, Variable Expressivity E. X Linked Disorder, Codominance
D. Autosomal Dominant Disorder, Variable Expressivity
A 25 year old male is preoperatively administered the intravenous general anesthetic propofol before undergoing a laparoscopic cholecystectomy. The surgery was performed without complication and the patient stayed in the hospital for two days postoperatively during recovery. In the day following the procedure the patient described to the physician that he was straining to pass urine and his bladder felt full. What agent did the physician use to relieve the patient's symptoms and what is its mechanism of action? A. Atropine, Muscarinic receptor antagonist B. Carbachol, Muscarinic receptor agonist C. Scopolamine, Muscarinic receptor antagonist D. Bethanechol, Muscarinic receptor agonist E. Ipratropium, Muscarinic receptor antagonist
D. Bethanechol, Muscarinic receptor agonist
A researcher has started a project looking into the genetic mutations involved with Ehlers-Danlos. The researcher is particularly interested in the set of knockout mice that has symptoms of severe joint hypermobility, mild skin changes, scoliosis, and bruising. The researcher states this type of Ehlers-Danlos is inherited in an autosomal dominant pattern and causes a defect in the conversion of procollagen to collagen leading to structurally abnormal pro-alpha1 chains. What gene is most likely knocked out in these mice? A. COL5A1 B. COL3A1 C. COL9A3 D. COL1A1 E. COL5A2
D. COL1A1
1. A 46-year-old male presents to his primary care office secondary to worsening muscle weakness and fatigue. The patient is eventually diagnosed with Myasthenia Gravis and is started on the acetylcholinesterase inhibitor Pyridostigmine. What CNS effect is most likely to be observed in the patient due to the effect of this medication? A. Disturbance or decrease in quality of REM sleep B. Decreased attentiveness during wakefulness C. Inhibition of the Nucleus Basalis of Meynert D. Difficulty with memory recall secondary to decreased slow-wave sleep and inhibition of memory consolidation. E. Decreased activity of autoreceptors and subsequent increased ACh release.
D. Difficulty with memory recall secondary to decreased slow-wave sleep and inhibition of memory consolidation.
Over the past few months, Donna has been experiencing progressively worsening muscle weakness that is exacerbated by exercise and improves with rest. While visiting with her doctor, she is also noted to have drooping of both eyelids. Her doctor decides to administer a drug in order to see if her symptoms improve. Which drug would the doctor use to confirm the most likely diagnosis? A. Physostigmine B. Donepezil C. Bethanechol D. Edrophonium E. Pyridostigmine
D. Edrophonium
A 12-year old boy got stung by a bee. The boy soon afterward couldn't breathe. The boy also faints. What is most true in this situation? A. Mast cells have not released any cytokines B. Widespread vasoconstriction has occurred C. Mast cells have IgM antibodies attached to them D. Epinephrine is a good treatment for the boy's symptoms E. A type II hypersensitivity reaction has occurred
D. Epinephrine is a good treatment for the boy's symptoms
A 50-year-old male with medical history of diabetes mellitus and premature CHD presents to his primary care physician for an annual physical exam. On exam, the physician notes xanthomas involving the palmer crease. The physician orders a lipid panel which indicates elevated levels of total cholesterol, triglycerides, and LDL. In order to reach a definitive diagnosis, the physician orders genetic testing which reveals the patient has an apo E2 mutation, indicating there is a lower affinity of apo E2 for the LDL receptor. What lipid disorder is the patient diagnosed with? A. Familial hypercholesterolemia B. Familial defective apolipoprotein B100 C. Familial combined hyperlipoproteinemia (FCHL) D. Familial dysbetalipoproteinemia E. Polygenic hypercholesterolemia
D. Familial dysbetalipoproteinemia
A 38-year-old female presents to her primary care physician with complaints of flu-like symptoms. Laboratory testing shows she has elevated liver enzymes, and her history is significant for diagnosis of a lipid disorder. The patient was recently prescribed a medication that inhibits apo B production in an effort to treat her dyslipidemia. The physician worries that her current presentation may be the manifestation of adverse effects from the newly prescribed medication. Which of the following is most likely the diagnosis that warranted treatment with this medication? A. Polygenic hypercholesterolemia B. Familial combined hyperlipoproteinemia C. Apolipoprotein C-II deficiency D. Homozygous familial hypercholesterolemia E. Lipoprotein lipase deficiency
D. Homozygous familial hypercholesterolemia
A couple presents to the clinic. They state that they want to have children, but they wanted to screen for possible genetic diseases since they both have a family history of sickle cell anemia. A few weeks later they get the results back. The man is heterozygous for this allele, and the woman is unaffected. Which of the following is true? A. If they had children, they have a 50% chance of having this disease B. If they had children, all females but no males would have this disease C. Multiple genes affect this disease, so it would be hard to tell the likelihood of passing on this condition D. If they had children 50% would be carriers E. The inheritance pattern of this disease is similar to that of marfan syndrome
D. If they had children 50% would be carriers
A 38-year-old woman presents to urgent care following scraping her foot on a rusty nail. The attending physician recommends a tetanus booster vaccine to mitigate the potentially dangerous effects of the tetanus toxin. Following the injection, the patient notices localized painful swelling at the site of injection and is diagnosed with cutaneous vasculitis when she returns to urgent care. What type of hypersensitivity does the patient have to the tetanus vaccine? A. Antibody-mediated diseases B. T cell-mediated C. Immediate hypersensitivity D. Immune complex-mediated diseases E. Delayed-type hypersensitivity
D. Immune complex-mediated diseases
The leading cause of death in the United States and most Western countries is cardiovascular disease. Much of the prevalence of cardiovascular disease can be attributed to? A. Decreased blood concentrations of low-density lipoprotein particles. B. Decreased blood concentrations of lipoproteins rich in triglycerides. C. Increased blood concentrations of high-density lipoprotein particles. D. Increased blood concentrations of low-density lipoprotein particles. E. None of the above.
D. Increased blood concentrations of low-density lipoprotein particles.
After moving away from home to attend college, Brian picked up a habit of smoking cigarettes and drinking occasionally on the weekends. He went to his local dentist as he had begun to experience pain in his teeth as well as cracked lips and altered sense of taste. The dentist informed Brian that his pain was the result of tooth decay caused by a lack of salivary production in his mouth, which was also causing his other symptoms. Which of the following medications would the dentist prescribe to treat the xerostomia? A. Cevimeline B. Carbachol C. Bethanechol D. Pilocarpine E. Methacholine
D. Pilocarpine
Gemfibrozil and fenofibrate are a class of lipid lowering drugs effective in lowering plasma triglyceride levels. They do so by a series of activation steps that ultimately leads to increase in the transcription of genes and proteins involved in lipid metabolism. What are the main downstream effects that give the clinical result of decreased plasma triglycerides? A. Decreased muscle expression of LPL, decreased hepatic expression of apolipoprotein CIII, and increased hepatic oxidation of fatty acids. B. Decreased muscle expression of LPL, decreased hepatic expression of apoCIII, and decreased hepatic oxidation of fatty acids. C. Increased muscle expression of LPL, increased hepatic expression of apoCIII, and increased hepatic oxidation of fatty acids. D. Increased muscle expression of LPL, decreased hepatic expression of apoCIII, and increased hepatic oxidation of fatty acids. E. Decreased muscle expression of LPL, increased hepatic expression of apoCIII, and increased hepatic oxidation of fatty acids.
D. Increased muscle expression of LPL, decreased hepatic expression of apoCIII, and increased hepatic oxidation of fatty acids.
A 46-year-old male presents to the emergency department with persistent swelling and pain over the muscles of his lower extremities. The patient's history is unremarkable, except for the fact that his primary care physician started him on "something for my cholesterols" about 1 week prior. The emergency department physician believes the patient has some form of rhabdomyolysis or myositis. What is the mechanism of action of the drug his primary care physician most likely started him on? A. Binding of bile acids to increase fecal excretion B. Direct reduction of VLDL production C. Inhibition of microsomal triglyceride transfer protein D. Inhibition of the rate-limiting step of cholesterol synthesis E. Antagonism of muscarinic acetylcholine receptors
D. Inhibition of the rate-limiting step of cholesterol synthesis
A 6-month-old infant comes to the ER displaying a floppy and weak posture, very high-pitched cry, and loss of appetite. The baby is unable to hold her head up. When asked about the baby's feeding history, the mother said she normally gives her formula. Two days ago, she mixed in honey to soothe the sore throat the baby had. Which of the following best describes the mechanism of the causative agent causing these symptoms? A. Inhibition of acetylcholinesterase (AChE) causing an accumulation of acetylcholine at the cholinergic receptor site and continuous stimulation of cholinergic fibers. B. Reversible binding to postsynaptic nerve endings to inhibit binding of acetylcholine to autonomic cholinergic receptors in neuromuscular junctions C. Antibodies against P/Q type voltage-gated calcium channel leading to reduced Ca2+ influx into presynaptic nerve terminal resulting in decreased acetylcholine release D. Irreversible binding to the presynaptic nerve endings to inhibit release of acetylcholine blocking voluntary and autonomic cholinergic neuromuscular junction E. Antibodies against postsynaptic acetylcholine receptors in neuromuscular junction
D. Irreversible binding to the presynaptic nerve endings to inhibit release of acetylcholine blocking voluntary and autonomic cholinergic neuromuscular junction
Sia, a world-renowned singer, was recently diagnosed with Ehlers-Danlos Syndrome (EDS). Which of the following types of EDS is inherited in an autosomal recessive fashion? A. Classic (I and II) B. Hypermobility (III) C. Vascular (IV) D. Kyphoscoliosis (VI) E. Arthrochalasia (VIIa/VIIb)
D. Kyphoscoliosis (VI)
A study is being done with participants from families where complications of atherosclerotic cardiovascular disease and tendinous xanthomas occurred before the age of 30. Some individuals were observed to have early atheroma formation. The affected individuals benefit from treatment with agents that inhibit HMG-CoA reductase. These affected individuals are most likely to have a mutation in a gene encoding a cell surface receptor for which of the following? A. Insulin B. Cortisol C. Leptin D. LDL cholesterol E. TGFa
D. LDL cholesterol
A 12-year-old boy has trouble breathing due to bronchial smooth muscle constriction after 30 minutes of playing soccer outside. What cell type is responsible for releasing the factors that directly cause this reaction? A. TH1 cells B. TH2 cells C. Memory B cells D. Mast cells E. Natural Killer cells
D. Mast cells
An 8-year-old male patient presents to his pediatrician complaining of shortness of breath since he has recently joined a soccer team. He complains that less than half way into practice he has coughing spells that he cannot control and must sit out the rest of practice. His pediatrician asked his mom if he has had similar symptoms prior to his increase in exercise. The mom said that sometimes during the night she can hear him coughing and wheezing in his sleep. The pediatrician suspects that the patient is suffering from asthma. What drug should the pediatrician use to diagnose his asthma? A. Bethanechol B. Ipratropium C. Pilocarpine D. Methacholine E. Succinylcholine
D. Methacholine
A couple comes into the clinic to get genetic testing done, as they are planning to start a family soon. When the results come back, it is indicated that the male has a mutation in a gene that elevates his LDL, resulting in Familial Hypercholesterolemia. Which of the following genes is LEAST likely to have a related mutation? A. LDLR B. APOB C. PCSK9 D. PS1 E. All of the above can contribute to Familial Hypercholesterolemia
D. PS1
A 29 year old male presents to the Emergency Department. The patient states they were out hiking near the Wintergreen Gorge and decided to ingest an abundance of wild mushrooms growing near the water. The Resident Physician on duty proceeds to administer medication for the mushroom poisoning, however accidentally administers too much. Noticing the Resident's mistake, the Attending Physician should begin symptomatic treatment and to administer an IV of which of the following drugs? A. Bethanechol B. Atropine C. Succinylcholine D. Physostigmine E. Scopolamine
D. Physostigmine
A 32-year-old female with medical history of a recent diagnosis of Sjögren's syndrome presents to her primary care physician with a chief complaint of severe dry mouth. She states she has been increasing her daily water intake to mitigate the dry mouth though it has not been effective. On physical exam, the physician notes dry mucous membranes. Which of the following muscarinic receptor agonists will the physician most likely prescribe the patient to treat her dry mouth? A. Methacholine B. Bethanechol C. Carbachol D. Pilocarpine E. Acetylcholine
D. Pilocarpine
Lab results for 39-year-old female show elevated levels of total plasma cholesterol and LDL cholesterol, with normal triglyceride concentrations. After genetic testing, she was diagnosed with familial hypercholesterolemia. She informed her physician that she is a heavy drinker. Which statin drug should be prescribed for treatment of this patient? A. Lovastatin B. Simvastatin C. Atorvastatin D. Pravastatin E. Fluvastatin
D. Pravastatin
The liver is the primary organ responsible for the removal of LDL particles from circulation. Which of the following is not a homeostatic mechanism associated with the uptake of LDL by its receptor on a hepatocyte? A. Inhibition of HMG-CoA reductase B. Activation of ACAT C. Increase endocytosis of LDL receptor D. Removal and transfer of ApoE to HDL E. Decrease in synthesis of cholesterol
D. Removal and transfer of ApoE to HDL
A 17-year-old female presents to a new primary care physician for a routine physical. She states that the reason she needs a physical is that she will be going off to college in a few months. The physician starts by taking the patient's medical history, during which the patient states she suffers from Ehlers Danlos syndrome, specifically Kyphoscoliosis. Aside from hypermobile joints and scoliosis, what is the doctor most concerned about regarding the patient's health? A. Rupture of colon B. Rupture of large arteries C. Joint dislocations D. Retinal detachment E. Diaphragmatic hernia
D. Retinal detachment
A 42-year-old male presents to the ER with painless, swollen lymph nodes on the neck and axilla, persistent fatigue, fever, and unexplained weight loss. A biopsy of the swollen lymph nodes and a CT scan confirm the diagnosis of Stage II Non-Hodgkin Lymphoma. The patient begins chemotherapy with a combination regimen of cyclophosphamide, doxorubicin, vincristine, and prednisone and the monoclonal antibody, rituximab. A few weeks later, the patient returns with major complaints of nausea, vomiting, and loss of appetite. What medication can the attending physician prescribe to alleviate the patient's side effects of chemotherapy? A. Tropicamide B. Trihexyphenidyl C. Bethanechol D. Scopolamine E. Tiotropium
D. Scopolamine
An 11-year-old boy has an allergic reaction to a bee sting. His mother brings him into the doctor's office and says that she doesn't understand why this is happening because he has been stung before. The doctor then explains that upon first exposure, which of the following processes occur? A. Arthus reaction B. Anaphylaxis C. Late-phase reaction D. Sensitization E. Complement activation
D. Sensitization
Which of the following is NOT a mechanism for generating antibody diversity in T cell receptors? A. Multiple germline gene segments B. VDJ somatic recombination C. Junctional diversity D. Somatic hypermutation E. All of the above mechanisms are used
D. Somatic hypermutation
A 23-year-old female presents to clinic with several abnormalities throughout her body. The physician suspects a relative common condition and orders several lab tests to confirm his diagnosis. Which of the following conditions could the patient be suffering from if she was diagnosed with a Type III, Immune Complex-Mediated Hypersensitivity? A. Goodpasture syndrome B. Myasthenia gravis C. Inflammatory bowel disease D. Systemic lupus erythematosus E. Type 1 diabetes mellitus
D. Systemic lupus erythematosus
A 26-year-old female visits her primary care physician with complaints of tremor, anxiety, heat intolerance, and significant weight loss over the past three months. Upon physical examination, proptosis of the eyes and a prominent goiter are noted. The primary care physician suspects that the patient is suffering from Graves' disease. Presence of an autoantibody that targets which of the following would help support this diagnosis? A. Acetylcholine Receptors B. GpIIb/IIIa Integrin C. Intrinsic Factor D. TSH Receptors E. Erythrocyte Membrane Proteins
D. TSH Receptors
A 7-month old boy was taken to his pediatrician with the parents requesting genetic testing. The infant has a history of recurrent bacterial, viral and fungal infections since birth. The doctor diagnosed the child with severe combined immune deficiency (SCID), an X-linked recessive mutation. He explained that the lack of T cells and antibodies in the child leaves him susceptible to infections. Which of the following is correct regarding the child's genetic diagnosis? A. Granulomas will soon develop because of the inability for phagocytes to kill ingested microbes. B. The allelic mutations will result in autoimmunity in the patient. C. Immunoglobulins cannot mature due to the lack of activation-induced deaminase enzyme (AIRE). D. The absence of B and T cells arise from mutations in RAG1 or RAG2 genes in VDJ recombination. E. The lack of thymic development of T cells leaves the patient susceptible to viral infections
D. The absence of B and T cells arise from mutations in RAG1 or RAG2 genes in VDJ recombination.
A researcher is conducting a study on how moderate to severe asthmatic patients respond to different steroid combinations over a 3-year study and is looking to recruit kids from the ages of 10-15 years old. The research will need to give each participant a drug that will induce bronchoconstriction to measure the lung function first, in order to make sure that the participants fit the criteria of the study. Which of the following drugs may the researcher use to reverse the bronchoconstriction that the offending agent will cause in the patients that have poorly controlled asthmatics? A. Methacholine with oxygen B. Bethanechol with oxygen C. Succinylcholine with glucocorticoids D. Tiotropium with glucocorticoids E. Pilocarpine with oxygen
D. Tiotropium with glucocorticoids
A 21 year old male was camping with his college biology class for a field study assignment. While setting up camp, he brushed up against some shrubs. The following day, he started to develop blisters and a rash around his legs and arms. While heading to his professor for help, he got stung by a bee and began feeling lightheaded and exhibiting shortness of breath. Which of the following hypersensitivities is he experiencing? A. Type 1 & 2 B. Type 2 & 3 C. Type 3 & 4 D. Type 4 & 1 E. Only Type 1
D. Type 4 & 1
A 26-year-old male was hiking when he was bit by a snake. In the ED he was given a serum antibody preparation via injection. The injected antibodies start forming immune complexes with the snake venom antigen and depositing in the blood vessels. He was diagnosed with serum sickness. Which type of hypersensitivity reaction has occurred? A. Type II B. Type I C. Type IV D. Type III E. Delayed Type Hypersensitivity
D. Type III
Acetylcholine has multiple effects on the Central Nervous System (CNS). Which one of these effects is NOT modulated by Acetylcholine? A.) Arousal and attention B.) Cognitive function C.) Memory processing D.) Touch sensation E.) Pain modulation
D.) Touch sensation
Twelve hours after going on a hike through dense foliage, a 40-year-old man notices a slightly raised and tender irregular reddish rash on one forearm that was not covered by clothing. This rash gradually increases in intensity for 2 days and then fades away after two weeks. Which of the following forms of immunologic hypersensitivity is most likely demonstrated in this patient? A. Type I Hypersensitivity B. Type II Hypersensitivity C. Type III Hypersensitivity D. Type IV Hypersensitivity E. Type V Hypersensitivity
D. Type IV Hypersensitivity
A new student is enrolling at LECOM and is given a packet of medical documents to fill out. The student is required to take a PPD skin test to detect for a mycobacterial infection. This is testing for what type of hypersensitivity? A. Type I hypersensitivity B. Type II hypersensitivity C. Type III hypersensitivity D. Type IV hypersensitivity E. Immediate hypersensitivity
D. Type IV hypersensitivity
When considering the uptake of LDL by LDL receptors in the liver, lysosomal acid lipase (LAL) plays a significant role in hydrolyzing and releasing unesterified cholesterol and fatty acids from LDL particles. These two products, unesterified cholesterol and fatty acids, then affeJoct three functions of major pathways of cholesterol synthesis and storage: inhibition of HMG-CoA reductase, activation of acetyl-CoA cholesterol acyltransferase (ACAT), and downregulation of LDL receptor expression. Which of the follow provides an appropriate explanation of a factor that leads to the development of foam cell plaque formation in blood vessels? A. Macrophages reject all forms of cholesterol, causing LDL particles to accumulate on the surface of these cells and the endothelium B. Plasma LDL particles are spontaneously reduced by plasma enzymes, bind to ICAM and VCAM, and are endocytosed by endothelial cells C. Scavenger receptors on mononuclear phagocytic cells are downregulated by cholesterol leading increased cholesterol synthesis inside the cell via HMG-CoA reductase D. Unlike LDL receptors in the liver, scavenger receptors on mononuclear phagocytic cells of the endothelium are not downregulated in the presence of cholesterol accumulation E. LDL particles inhibit HMG-CoA in endothelial cells and force the cells to absorb cholesterol from the LDL particle, leading to accumulation of cholesterol inside enlarged endothelial cells
D. Unlike LDL receptors in the liver, scavenger receptors on mononuclear phagocytic cells of the endothelium are not downregulated in the presence of cholesterol accumulation
A 23-year-old male with a known diagnosis of neurofibromatosis 1 is at a conference for individuals affected by the same disease. He meets up with another individual who has very significant neurofibromas and states that he has skeletal abnormalities as well that affect his quality of life, while the 23-year-old is only covered in café-au-lait spots. The range of seen symptoms between the two is an example of which of the following? A. Incomplete penetrance B. Mendelian trait C. Codominance D. Variable expressivity E. Genetic heterogeneity
D. Variable expressivity
A couple has four children, two sons and two daughters. The daughters both have the same disorder as their father, but the sons are unaffected. The physician explains it's because they have an x-linked dominant condition. What type of disorder might the daughters have? A. Diabetes insipidus B. agammaglobulinemia C. glucose-6-phosphate dehydrogenase deficiency D. Vitamin D- resistant rickets
D. Vitamin D- resistant rickets
A researcher is examining the effects of a combination of drugs on a population of mice. One of the drugs used is Hemicholinium. How does this drug restrict the availability of acetylcholine (ACh) at presynaptic cholinergic neurons? A: Blocks the reuptake of ACh from the synaptic cleft B: Blocks the transfer of ACh into storage vesicles C: Inhibits the release of ACh into the synaptic cleft D: Inhibits the synthesis of ACh E: Inhibits binding of SNARE-complex proteins
D: Inhibits the synthesis of ACh
The tuberculin reaction is an intracutaneous injection of purified protein derivative, a protein-containing the antigen of tubercle bacillus. If an individual is sensitized, reddening and induration of the site appears within several hours and will slowly subside. Which type of hypersensitivity reaction is occurring, and which cells are responsible for this reaction? A: Type II hypersensitivity; CD4+ T cells B: Type IV hypersensitivity; CD8+ T cells C: Type III hypersensitivity; Macrophages D: Type IV hypersensitivity; CD4+ T cells E: Type III hypersensitivity; Macrophages
D: Type IV hypersensitivity; CD4+ T cells
A 23-year-old male was working on a landscaping crew on a hot sunny day. He was on the riding mower listening to his headphones when all of a sudden, he felt a painful sting from a bee on his right shoulder. Considering he is in the MMS program at LECOM, he knew that these antigens from the sting would cause exposure of allergens within the body to his immune system. What would be a primary response to getting stung by a bee from his immune system? A: we would see heavy vasoconstriction within the body due to vasoactive amines B: we would see smooth muscle relaxation within the body due to vasoactive amines C: we would see tissue destruction due to the complement pathway proteins D: we would see vasodilation via prostaglandins within the body E: we would see decreased vascular leakage due to an IgG antibody mediated response
D: we would see vasodilation via prostaglandins within the body
Examples of Major Histocompatability Complex and Disease associations
DISEASE MHC (HLA) ASSOCIATED LOCUS (LOCI)* APPROXIMATE RELATIVE RISK† Type 1 diabetes DQB1, DQA1 10 Ankylosing spondylitis B27 90 Narcolepsy DR2 and DQA1, DQB1 >100 Celiac disease DQA1, DQB1 10 Rheumatoid arthritis DRB1, DQA1 5 Myasthenia gravis C, DR3, DR7 2.5 Multiple sclerosis DRB1 4 Systemic lupus erythematosus DRB1 6 Hemochromatosis A3 20 Malaria B53 0.6059 Graves disease DR3 5 Psoriasis C 13 Abacavir (anti-HIV drug) hypersensitivity B57 ≈1000
Prevalent Rates of Common congenital malformations in persons of European descent
DISORDER APPROXIMATE PREVALENCE PER 1000 BIRTHS Cleft lip/palate 1.0 Club foot 1.0 Congenital heart defects 4.0-8.0 Hydrocephaly 0.5-2.5 Isolated cleft palate 0.4 Neural tube defects 1.0-3.0 Pyloric stenosis 3.0
What test can be done to confirm the diagnosis of Familial defective apolipoprotein B100?
DNA analysis but genetic diagnosis is not necessary to initiate therapy
Drugs to be avoided or used with caution in MG
DRUGS TO BE AVOIDED • D-penicillamine and alpha-interferon should not be used in myasthenic patients because they can cause Myasthenia Gravis (MG) • Botulinum toxin treatment should be avoided as it blocks NMT DRUGS TO USE ONLY WITH CAUTION AND MONITOR FOR EXACERBATION OF MG SYMPTOMS • Selected antibiotics, particularly aminogylcosides, telithromycin (Ketek) and ciprofloxacin (many other antibiotics have been reported to increase weakness in occasional patients with MG) • Magnesium, magnesium salts contained in some laxatives and antacids • Neuromuscular blocking agents such as succinylcholine and vecuronium should only be used by an anesthesiologist familiar with MG • Quinine, quinidine or procainamide • Betablockers (propranolol; timolol maleate eyedrops) • Calcium channel blockers • Iodinated contrast agents
What is special about of DZ twin studies?
DZ twins provide a convient comparision: their environmental differences should be similar to those of MZ twins but their genetic differences are as great as those between siblings
disorders of HDL metabolism
Decreased HDL cholesterol is an independent risk actor or development o atherosclerosis and cardiovascular disease. Numerous rare genetic de ects in HDL metabolism have been identif ed, including de ects in apoAI, ABCA1, and LCAT. Each o these de ects results in decreased levels o HDL, or which no e ective treatments are currently available. More commonly, low HDL is associated with visceral obesity and insulin resistance. Elevated concentrations o HDL occur in the setting o aerobic activity, alcohol consumption, estrogen use, and corticosteroid therapy. Recently, reductions in CETP activity have been characterized as a relatively common genetic cause o increased HDL levels. The increased plasma HDL concentration associated with decreased CETP activity has been attributed to a decrease in the trans er o cholesterol rom HDL to remnant particles. Although it might be assumed that the increased HDL levels would be cardioprotective, this is not always observed. Decreased CETP activity may increase the risk o atherogenesis in some cases, whereas in others, it appears to be cardioprotective. Additional research will be necessary be ore the role o CETP polymorphisms in lipid metabolism and cardiovascular disease risk can be identif ed. Genetic variations in hepatic lipase and endothelial lipase can also lead to increased HDL.
What does topical application of AChE inhibitor to the cornea of the eye result in?
Decreased intraocular pressure by facilitating the outflow of aqueous humor
What is the evidence for diminished inflammation with statin therapy?
Decreases in acute phase reactants which are plasma proteins that are increased during inflammatory states and may play a role in the destabilization of atherosclerotic plaques best characterized acute phase reactant is C-reactive protein (CRP)
What does Niacin decreases levels of? increase?
Decreases: 1- Plasma LDL-cholesterol 2- Triglyceride Increases: 1- HDL cholesterol
Characteristics of Plasma lipoproteins
Density (g/mL) 0.95 0.95-1.006 1.006-1.019 1.019-1.063 1.063-1.210 Diameter (nm) 75-1,200 30-80 25-35 18-25 5-12 Total lipid (% wt) 98 90 82 75 67 Composition, % dry weight Protein 2 10 18 25 33 Triglycerides 83 50 31 9 8 Unesterif ed cholesterol and cholesteryl esters 8 22 29 45 30 Phospholipids (% wt lipid) 7 18 22 21 29 Electrophoretic mobilitya None Pre- or Pre- Plasma hal -li e 1 hour 30-60 minutes 30 minutes 2-4 days 2-5 days Major apolipoproteins B48, AI, AIV, E, CI, CII, CIII B100, E, CI, CII, CIII B100, E, CI, CII, CIII B100 AI, AII, CI, CII, CIII, E
What does treatment of patients with an antibody specific for CD20(surface protein of mature B cells) result in?
Depletion of the B cells and may be useful for treating some antibody-mediated disorders Other approaches being tried for inhibiting the production of autoantibodies include treating patients with antibodies that block CD40 or its ligand and thus inhibit helper T cell-dependent B cell activation and antibodies to block cytokines that promote the survival of B cells and plasma cells. There is also interest in inducing tolerance in cases in which the autoantigens are known.
What occurs in Rheumatic fever?
Deposition of antistreptococcal antibodies in the heart triggers an inflammatory disease called rheumatic fever, Can lead to acute heart failure or slow scarring of valves and late-onset heart failure.
What is the likely cause that patients with Autosomal dominant disorders have unaffected parents?
Development of new mutations involving either the egg or sperm from which they were derived Their siblings are neither affected nor at increased risk for disease development commonly occurs in older fathers
What is the only current use of Methacholine?
Diagnosis of Asthma in this application the bronchial hyperreactivity that is characteristic of asthma causes an exaggerated bronchoconstriction response to parasympathomimetics
What is essential for the treatment of marked hypertriglyceridemia?
Dietary restriction of fat (<10%) Other factors such as carbohydrate and alcohol intake can also increase the synthesis of triglyceride. Restriction of alcohol intake to 1 or 2 servings per week and adherence to a low-fat, high-fiber diet will improve hypertriglyceridemia. Exercise has been shown to increase LPL activity. Even a single exercise session can reduce triglycerides and increase HDL. The impact of exercise on LDL is less clear. With low- to moderateintensity exercise regimens, clearance of VLDL particles increases LDL production. However, this effect is not seen with highintensity exercise programs. A decrease in LDL-cholesterol occurs with high-intensity exercise, and this effect is independent of weight loss.
What do the pharmacokinetic differences among the different statins result from?
Differential cytochrome P450 metabolism Lovastatin, simvastatin, and atorvastatin are metabolized by CYP3A4 cytochrome P450-mediated pathways metabolize fluvastatin and pitavastatin Pravastatin and rosuvastatin are not metabolized via the cytochrome P450 pathway.
What occurs in liability distriubtion?
Diseases do not follow bell shaped distribution, they appear to be either present or absent in individuals A commonly used explanation is an underlying liability distribution in which people on the low end of the distribution have little chance of developing the disease (ie: they have few of the alleles or environmental factors that would cause the disease) Those who are closer to the high end of the distribution have more of the diseases causing alleles and environmental factors and are more likely to develop the disease
What are Congenital malformations?
Diseases that are present at birth appx 2% of newborns present with them- 1/50 live births sibling recurrence risks for most of these disorders range from 1-5%
What is Marfan syndrome? prevalence? mode of inheritance?
Disorder of connective tissues, manifested principally by changes in the skeleton, eyes and cardiovascular system Its prevalence is estimated to be 1 in 5000. Approximately 70% to 85% of cases are familial and transmitted by autosomal dominant inheritance.
What does overall cholesterol balance depend on?
Disposition of both cholesterol and bile acids Most bile acid molecules are not lost in the feces after participating in cholesterol transport and at digestion; instead, they are taken up and recycled by high-affinity transport proteins in the distal ileum
What second generation AChE inhibitor is indicated for the treatment of Alzheimer's dementia in the mild, moderate and severe stages?
Donepezil
What is a women's risk for cancer if she has affected first-degree relatives?
Doubles The risk increases further with additional affected relatives, and it increases if those relatives developed cancer at a relatively early age (before 50 years of age).
Certain genetically determined enzyme deficiencies are unmasked only after exposure of the affected individual to certain?
Drugs eg of drug induced injury is associated with deficiency of G6PD: Under normal conditions glucose-6 phosphate-dehydrogenase (G6PD) deficiency does not result in disease, but on administration, for example, of the antimalarial drug primaquine, a severe hemolytic anemia results
Inhibitors of Acetylcholine synthesis storage and release
Drugs that inhibit the synthesis, storage, or release o ACh have only recently begun to have clinical use (Fig. 10-1). Hemicholinium-3 blocks the high-a f nity transporter or choline and thus prevents the uptake o choline required or ACh synthesis. Vesamicol blocks the ACh-H antiporter that transports ACh into vesicles, thereby preventing the storage o ACh. Both o these compounds are utilized only in research settings, however. Botulinum toxin A , produced by Clostridium botulinum , degrades SNAP-25 and thus prevents synaptic vesicle usion with the axon terminal (presynaptic) membrane. This paralysis-inducing property is currently used in the treatment o several diseases associated with increased muscle tone, such as torticollis, achalasia, strabismus, blepharospasm, and other ocal dystonias. Botulinum toxin is also approved or cosmetic treatment o acial lines or wrinkles and is used to treat various headache and pain syndromes (e.g., by intrathecal delivery into the spinal uid). Because it degrades a protein common to the synaptic vesicle usion machinery in multiple types o nerve terminals, botulinum toxin has a general e ect on the release o many di erent neurotransmitters, not just ACh
What is the reason that LDL cholesterol accounts for appx 65-75% of the total plasma cholesterol?
Due to the lack of apoE, LDL particles are relatively weak ligands or the LDL receptor As a result, the half- life of LDL in the circulation is markedly prolonged (2-4 days) The uptake of LDL-C by the LDL receptor also appears to be promoted by sortilin
How does Ach affect memories and what role does sleep play?
During awake states, ACh prevents interference with initial learning in the hippocampus by suppressing retrieval of previously stored memories (to prevent them from interfering with new encoding), but release of this suppression is necessary to allow consolidation o new memories. During sleep (in particular, during SWS), lower ACh levels are required or proper consolidation of newly acquired memories because stronger excitatory feedback transmission is needed to reactivate memories or consolidation within neocortical brain areas. There ore, it may be use ul to remember to sleep, as sleep is needed to remember, or at least to remember better
Diagnosis
Dyslipidemia is defined by a total cholesterol, triglyceride, or LDL level greater than the 90th percentile or an HDL level lower than the 10th percentile for the general population. Because chylomicrons are present in plasma for up to 10 hours after a meal, fasting total cholesterol, triglyceride, and lipoprotein assessments are required for diagnosis. It is advisable to confirm dyslipidemia with two separate determinations. Total cholesterol, triglyceride, and HDL levels can be measured directly; VLDL and LDL levels usually are calculated. If the triglyceride concentration is lower than 400 mg/dL, then VLDL is calculated by dividing the triglyceride level by 5. LDLcholesterol is estimated by subtracting VLDL and HDL from the total cholesterol. VLDL and LDL cannot be determined if triglyceride levels are greater than 400 mg/dL. In that case, the lipoprotein abnormality can be identified by inspecting the serum. When the triglyceride level exceeds 350 mg/dL, the serum is cloudy. After refrigeration, a white surface layer depicts excess chylomicrons, whereas a dispersed, opaque infranatant reflects a VLDL dysfunction Current guidelines recommend selective screening of children who have a family history of lipoprotein abnormality or premature vascular disease and adults who have an increased risk for CHD. The U.S. Preventive Services task force recommended universal screening starting at 35 years of age for men and at 45 years of age for women; there is a paucity of data supporting long-term benefits from screening of younger individuals. Either a fasting or a nonfasting total cholesterol and HDL measurement can be the initial screen. If the total cholesterol value is greater than 200 mg/dL or the HDL value is less than 40 mg/dL, then a repeat fasting lipid panel is required. If the total cholesterol value is less than 200 mg/dL and the HDL value is greater than 40 mg/dL, then retesting is recommended every 5 years. Individuals with CHD, risk factors for CHD, or CHD equivalents (i.e., symptomatic carotid artery disease, peripheral arterial disease, abdominal aortic aneurysm, or diabetes) should be screened more frequently based on risk assessment, as shown in Table 69-4. CHD risk factors include age (men >45 years, women >55 years), family history of premature CHD (affected male first-degree relative <55 years or female first-degree relative <65 years of age), smoking, hypertension, and low HDL (<40 mg/dL). HDL concentrations higher than 60 mg/dL are cardioprotective. Overall, the level of evidence to support screening is fair (level B) but increases with age, male gender, and CHD (level A).
clinical presentation
Dyslipidemia plays a significant role in the development of atherosclerosis. Increased incidence of CHD with high LDL- and low HDL-cholesterol is well documented. Excess LDL results in the formation of cholesterol plaques that deposit in arteries (atheroma), skin and tendon (xanthomas), eyelids (xanthelasma), and iris (corneal arcus). The impact of triglycerides on vascular disease is less clear. Metabolic disorders such as diabetes and obesity are often associated with vascular disease and hypertriglyceridemia, and the atherogenic impact of other elements in these disorders is difficult to separate from the effect of hypertriglyceridemia. However, in several population-based studies, abnormal triglyceride levels correlated with increased risk for CHD. Marked hypertriglyceridemia (>1000 mg/dL) is associated with the chylomicronemia syndrome, characterized by pancreatitis and xanthomas.
A 59 year old male visits his local clinic. He has palpitation, difficulty breathing and general fatigue. After asking about his family history, the physician suspects he might have familial hypertrophic cardiomyopathy. A mutation in what gene would increase the risk of developing hypertrophic cardiomyopathy? A) Factor V B) KCNQ1 C) LDLR D) APC E) B-Myosin heavy chain
E) B-Myosin heavy chain
A 50-year old man presents to his primary care physician for routine blood work. His lipid panel comes back showing elevated triglycerides. After discussing treatment options, the patient decided that he does not want to take any medications but would still like to reduce his triglyceride levels. Which of the following lifestyle modifications would the physician most likely recommend to the patient? A) Increasing intake of carbohydrates B) Increasing intake of trans-unsaturated fats C) Increasing intake of alcohol D) Decreasing intake of soluble fibers E) Decreasing intake of saturated fats
E) Decreasing intake of saturated fats
Claudia, 16 years old, comes into her primary care clinic with a primary complaint of chest pain during exercise, specifically running. The physician does a physical exam and notices fatty deposits on her elbow. Her physician suspects a particular disease due to these xanthomas. What is the most likely diagnosis, and what is the mechanism of the disease? A) Familial hypercholesterolemia; an increase in LDL receptors B) Familial hypertriglyceridemia; a decrease in LDL receptors C) Familial hypercholesterolemia; a decrease in plasma LDL D) hypertriglyceridemia; an increase in plasma LDL E) Familial hypercholesterolemia; a decrease in LDL receptors
E) Familial hypercholesterolemia; a decrease in LDL receptors
Elizabeth is a 19-year-old college student who presents to her primary care physician for an annual physical exam. Elizabeth has no current medical complaints. She is a type I diabetic and states that her blood glucose levels are well-controlled. She admits to drinking every weekend at the fraternity parties on campus. She is currently sexually active and would like her doctor to prescribe her birth control. Her doctor advises her to minimize the alcohol consumption and does not prescribe her birth control due to an underlying medical condition. When Elizabeth was 5-years-old, she was diagnosed with an autosomal dominant disorder which is characterized by the overproduction of hepatic VLDL and increased catabolism of HDL. What condition does Elizabeth suffer from? A) Familial hypercholesterolemia B) Polygenic hypercholesterolemia C) Familial dysbetalipoproteinemia D) Apolipoprotein C-II deficiency E) Familial hypertriglyceridemia
E) Familial hypertriglyceridemia
Antibody-antigen reactions may cause disease by destroying cells, triggering inflammation, or interfering with normal functions. In some cases, antibodies directed against cell surface receptors dysregulate function without causing cell injury or inflammation. Which of the following diseases has antibody mediated stimulation of cell function as its mechanism? And what is its target antigen? A) Myasthenia Gravis; TSH Receptor B) Autoimmune Hemolytic Anemia; Rh blood group antigens C) Graves Diseases; Acetylcholine Receptor D) Myasthenia Gravis; Acetylcholine Receptor E) Graves Disease; TSH Receptor
E) Graves Disease; TSH Receptor
A 55-year-old woman was diagnosed with myasthenia gravis last year. She was prescribed pyridostigmine as the first line drug choice to treat her symptoms. Which of the following drugs has a similar mechanism of action as the drug prescribed? A) Methacholine B) Pancuronium C) Scopolamine D) Pilocarpine E) Physostigmine
E) Physostigmine
A twelve week old infant is brought to the emergency department by his parents who have noticed their child has had difficulty gaining weight and tends to vomit after feeding. The physician orders blood work and imaging suspecting a possible gastrointestinal blockage. An electrolyte imbalance and narrowed pyloric canal was noted. The physician informs the parents that she suspects pyloric stenosis. Upon hearing this, the child's mother looks up the condition on her phone and reads that it "is a multifactorial disease with a 5 to 1 male to female sex ratio and requires a certain amount of risk factors to be present in order for development" . What underlying genetic mechanism does this best represent? A) Concordance B) Variable expressivity C) Autosomal dominance D) Quantitative trait loci E) Threshold of Liability
E) Threshold of Liability
A 34 year old presents to her Obstetrician's office with complaints of mild spotting. She is 27 weeks pregnant with her second child. The physician reviews the patients blood work and notes that she is Rh negative. To help prevent incompatibility resulting in hemolytic disease of the newborn which of the following must be administered? A. Anti-B antibodies B. Rh negative antibodies C. Rh positive antigens D. Anti-A antibodies E. Anti-Rh antibodies
E. Anti-Rh antibodies
Mark, a 16-year-old patient, comes to the clinic with complaints of muscle weakness in the arms and legs, droopy eyelids and mouth. A diagnosis has been made of myasthenia gravis. What cellular dysfunction is displayed in this condition? A. TNF is causing inflammation in the joints. B. NK cells are attacking the optic nerve. C. The overstimulation of acetylcholine receptors at the neuromuscular junction. D. Antibodies are stimulating thyroid cells causing hyperthyroidism. E. Antibodies are blocking the acetylcholine receptors at the neuromuscular junction
E. Antibodies are blocking the acetylcholine receptors at the neuromuscular junction
Alzheimer's disease is responsible for about 60-70% of cases of progressive cognitive impairment among the elderly. Allelic variations in which gene product is implicated in the more common late-onset Alzheimer's disease? A. RET1 B. PSN1/PSN2 C. APP D. MODY E. Apolipoprotein E
E. Apolipoprotein E
An 18 year old patient presents to a new PCP with back pain. Upon physical examination, mild skin changes and scoliosis were observed, as well as bruising and sever joint hypermobility. The patient reports that they had genetic testing done and were confirmed to have a defect on their COL1A1 gene. What classification of Ehlers-Danlos Syndrome does the patient have? A. Classic Type B. Hypermobility C. Vascular D. Kyphoscoliosis E. Arthrochalasia
E. Arthrochalasia
A patient comes into the clinic with complaints of neck pain and is unable to rotate his head. Upon examination the physician notes that the patient's head is tilted to one side and the muscles of the neck and shoulders were tender. The physician diagnoses the patient with torticollis. What is the drug and the mechanism of action of that drug that will help the patient? A. Methacholine; Agonist at the muscarinic receptor B. Pyridostigmine; Acetylcholinesterase inhibitor: C. Botulinum toxin: Agonist at the nicotinic receptor. D. Scopolamine: Antagonist at muscarinic receptor. E. Botulinum Toxin: Inhibition of acetylcholine release.
E. Botulinum Toxin: Inhibition of acetylcholine release.
A student is conducting an experiment to understand the effects of acetylcholine on muscarinic receptors. Which of the following effects could the student observe if the muscarinic receptors are stimulated by acetylcholine? A. Decreased sweat gland secretions B. Ciliary muscle relaxation C. Bronchodilation D. Decreased gastrointestinal secretions E. Bradycardia
E. Bradycardia
Two couples come in for genetic counseling. Both couples have previously had a child with a genetic birth defect. It is determined that Couple #1 child's birth defect is a result of a multifactorial disease and Couple #2 is from a single gene disease. What is the chance that Couple #1 and Couple #2 will have another child with the same birth defects? A. Couple #1: Same chance, and Couple #2: Increased chance B. Couple #1: Decreased chance, and Couple #2: Same chance C. Couple #1: Same chance, and Couple #2: Same chance D. Couple #1: Increased chance, and Couple #2: Decreased chance E. Couple #1: Increased chance, and Couple #2: Same chance
E. Couple #1: Increased chance, and Couple #2: Same chance
A 24-year-old female is brought to the emergency department secondary to an anaphylactic reaction after ingesting a cookie containing peanut butter. Patients mother states that she has a known peanut allergy. During her stay, she was given epinephrine and antihistamines to alleviate her symptoms. She was kept for observation, but eight hours later suddenly develops worsening shortness of breath and coughing. Her current presentation is due to recruitment of which of the following cell types? A. Mast cells and epithelial cells B. Eosinophils and mast cells C. Eosinophils and epithelial cells D. Neutrophils and mast cells E. Eosinophils and neutrophils
E. Eosinophils and neutrophils
A 54-year-old patient visited her family physician after her kids, Ross and Rosa, said they were worried about her weight. She told her doctor her weight began to rise after she was self-isolated and quarantined many years ago. A lipid panel reveals a fasting triglyceride level 213mg/dL, 205 mg/dL LDL, and 39 mg/dL HDL level. The physician recommended her to adjust to a healthier diet and exercise regimen. On top of that the physician also recommended a drug that directly interferes with the synthesis of cholesterol and told her to avoid taking blood thinners. What is the mechanism of action of the prescribed drug? A. Reduce VLDL production in the liver B. Decrease influx of free fatty acid into liver C. Intestinal lumen cholesterol transport inhibitor D. Brush-border cholesterol transport inhibitor E. HMG-CoA reductase inhibitor
E. HMG-CoA reductase inhibitor
Maturity-onset diabetes of the young (MODY) is an autosomal dominant disease that typically presents in patients before the age of 25. These patients have underlying mutations in genes expressed in pancreatic beta cells, which result in beta cell abnormalities. Studies have shown that the most common mutation seen in MODY patients is in a gene that encodes glucokinase, an enzyme involved in the conversion of glucose to G6P in the pancreas. Other mutations have been identified in any of five genes that encode transcriptions factors involved in pancreatic development and/or insulin regulation. Which Mendelian subtype of MODY is implicated in a patient with a known mutation in Hepatocyte nuclear factor-1b (HNF1b)? A. MODY 1 B. MODY 2 C. MODY 3 D. MODY 4 E. MODY 5
E. MODY 5
Peter, a 28-year-old male, decided that he wanted to compete on a reality television dating show, in hopes of finding his future wife. While filming the show, the producers decided that they wanted some footage of Peter on a boat in the middle of the ocean. Realizing that he would probably become extremely nauseous during filming, Peter placed a specific transdermal patch behind his ear to prevent any motion sickness he might experience on the boat. What type of drug did Peter most likely utilize? A. Nicotinic Receptor Antagonist B. Acetylcholinesterase Inhibitor C. Nicotinic Receptor Agonist D. Muscarinic Receptor Agonist E. Muscarinic Receptor Antagonist
E. Muscarinic Receptor Antagonist
A 58-year-old female presents to the ER with dysarthria, loss of balance, and unilateral facial droop. The attending physician also notes that the patient can only raise and hold up one arm when asked to lift both of them. Her son was also present and reported that the patient prefers to stay seated for extended periods of time due to her osteoarthritis, that she is on captopril to treat her elevated blood pressure, and that she had previously smoked for 30 years. He also mentions that his grandfather (the patient's father) had very similar symptoms before passing away several years ago. While the patient's condition is being treated, the attending physician orders a genetic test and discovers a mutation in a particular clotting factor that confers resistance to inactivation by protein C. What mutation is the patient suffering from? A. Mutations in BRCA1, BRCA2 genes B. Mutation in SMAD4 gene C. Mutation in the NOTCH3 gene D. Mutation in the WNK1, WNK4 genes E. Mutation in the factor V Leiden allele
E. Mutation in the factor V Leiden allele
A 53-year-old male presents to the emergency department with facial paralysis, ptosis, and generalized weakness. Before treatment is provided, the physician orders a dose of Edrophonium as a diagnostic measure. If the patient's symptoms show improvement, which of the following medications would most likely be prescribed to treat the patient? A. Scopolamine B. Methacholine C. Pilocarpine D. Donepezil E. Pyridostigmine
E. Pyridostigmine
Jerry is a 37-year-old male who goes to his doctor for some routine tests and lab work. All of his tests come back clean except that he has low HDL. Which of the following is the correct mechanism of action of the drug that Jerry was most likely prescribed? A. HMG-CoA reductase inhibitor B. Bind to and activate peroxisome proliferator activated receptor a (PPARa) C. Bind to bile acids and prevent enterohepatic circulation D. Inhibits apoB translation E. Reduces free fatty acid release from adipose tissue and increases plasma residence time for apoAI
E. Reduces free fatty acid release from adipose tissue and increases plasma residence time for apoAI
A 45-year-old truck driver presents to the clinic for routine checkup. Significant lipid panel include LDL-C of 299 mg/dL and an HDL of 54mg/dL. He also complained of sore throat and productive cough for the past two weeks. The clinician prescribed appropriate medications including erythromycin. Which drug is most likely prescribed to address his elevated LDL-C? A. Niacin B. Gemfibrozil C. Fluvastatin D. Pravastatin E. Rosuvastatin
E. Rosuvastatin
A 24-year-old female comes into the emergency room with complaints of ptosis, double vision and a dry mouth. When taking her history, she states that she thought her mouth was dry due to her consistent heroin use, but the ocular symptoms are new. On examination, she begins to have descending paralysis. What is this patient suffering from? A. Infant Botulism B. Lambert-Eaton Myasthenic Syndrome C. Drug Overdose D. Foodborne Botulism E. Wound Botulism
E. Wound Botulism
What is administered for treatment of Botulism?
Equine antitoxin- within the first 24 hours may arrest the progression of paralysis and decrease the duration of illness. Prompt intensive care support with mechanical ventilation and parenteral feeding Children less than 12 months old should not be fed honey because it can contain Clostridium botulinum.
A 35-year-old female presents to the emergency department presenting with fatigable weakness of her respiratory muscles, droopy eyelids, and limb weakness. The physician suspects she is suffering from Myasthenia Gravis and gives her medication to aid with her symptoms. Which of the following treatments should the physician avoid in this patient given her symptoms? A. Pyridostigmine B. Plasmapheresis and IVIG C. Prednisone D. Azathioprine E. botulinum toxin treatment
E. botulinum toxin treatment
Sarah is at her local health fair where she learns about the dangers of a high fat diet. Sarah learns that she may have an accumulation of cholesterol-rich macrophages. What are potential complications from accumulation of cholesterol-rich macrophages? A. decreased accumulation of foam cells B. decreased local inflammatory response C. non-reactive aldehyde intermediates D. aids in liver secretion of unesterified cholesterol into the bile E. destabilized atherosclerotic plaques
E. destabilized atherosclerotic plaques
Classification of the 6 types of Ehlers-Danlos syndromes
EDS Type* Clinical Findings Inheritance Gene Defects Classic (I/II) Skin and joint hypermobility, atrophic scars, easy bruising Autosomal dominant COL5A1, COL5A2 Hypermobility (III) Joint hypermobility, pain, dislocations Autosomal dominant Unknown Vascular (IV) Thin skin, arterial or uterine rupture, bruising, small joint hyperextensibility Autosomal dominant COL3A1 Kyphoscoliosis (VI) Hypotonia, joint laxity, congenital scoliosis, ocular fragility Autosomal recessive Lysyl hydroxylase Arthrochalasia (VIIa,b) Severe joint hypermobility, skin changes (mild), scoliosis, bruising Autosomal dominant COL1A1, COL1A2 Dermatosparaxis (VIIc) Severe skin fragility, cutis laxa, bruising Autosomal recessive Procollagen N-peptidase
What accounts for the majority of the removal of excess cholesterol from cells?
Efflux to spherical HDL particles
What is the process of recycling bile acids between the liver and intestine referred to as? What occurs in it?
Enterohepatic circulation 1- Bile acids enter the portal circulation and are transported back to the liver, where they are cleared from the blood by hepatocytes with high first-pass efficiency. 2- Bile acids are then re-secreted into bile enterohepatic circulation is highly efficient: allows less than 5% of secreted bile acids to be lost in feces
What leukocyte accumulates in the bronchial mucosa in asthmatic patients?
Eosinophils excessive secretion of mucus occurs in the airways, and the bronchial smooth muscle becomes hypertrophied and hyperreactive to various stimuli
What does the liver synthesize in excessive amounts in Familial combined hyperlipoproteinemia?
Excess VLDL VLDL is hydrolyzed by LPL to produce LDL Mutations in the LPL gene affecting its expression or function can decrease the efficiency of VLDL catabolism. Dysfunction of LPL is observed in one third of patients with FCHL
What is the anticholinergic effect of HDL attributed to the removal of?
Excess cholesterol from tissue sites and other proteins
What occurs in Cellular cholesterol efflux?
Excess insoluble cholesterol molecules are removed from cells This occurs when unesterified cholesterol is transferred from the plasma membrane of cells to an HDL particle
What occurs in lysosomal storage diseases?
Excessive accumulation of complex substrates within the lysosomes as a result of deficiency of degradative enzymes is responsible for a group of diseases
What are Superantigens?
Excessive polyclonal T cell activation by certain microbial toxins produced by some bacteria and viruses can lead to production of large amounts of inflammatory cytokines, causing a syndrome similar to septic shock. These toxins are called superantigens because they stimulate large numbers of T cells. Superantigens bind to invariant parts of T cell receptors on many different clones of T cells, regardless of antigen specificity, thereby activating these cells.
What is Bipolar disorder characterized by?
Extreme mood swings and emotional instability AKA manic-depressive disorder The prevalence of the disorder in the general population is approximately 0.5% to 1%, but it rises to 5% to 10% among those with an affected first-degree relative.
What is an example of a Cholesterol absorption inhibitor? What is its MOA?
Ezetimibe Decreases cholesterol transport from micelles into enterocytes by selectively inhibiting cholesterol uptake through the brush border protein NPC1L1 (Niemann-Pick C1-Like 1) - ezetimibe reduces intestinal cholesterol absorption by about 50%, without reducing the absorption of triglycerides or fat-soluble vitamins
What is an example of a Cholesterol absorption inhibitor? How do they work?
Ezetimibe Function by interfering with the transport of cholesterol at the intestinal brush border
A 33-year-old man is currently being treated for dyslipidemia. After beginning the medication, he complains of an uncomfortable itching sensation and reddening of the skin. He is prescribed aspirin to mitigate these adverse effects. Which of the following is an additional adverse effect that can be caused by the medication being used to treat his dyslipidemia? A. Fat malabsorption B. Bleeding C. Arrhythmia D. Exacerbation of gout E. Impaired insulin sensitivity F. D and E
F. D and E
What is Familial combined hyperlipidemia (FCHL)? treatment?
FCHL is a common disease associated with moderately elevated concentrations of fasting triglycerides and total cholesterol and reduced concentrations of HDL cholesterol - These patients o ten present with other features of the metabolic syndrome , including abdominal obesity, glucose intolerance, and hypertension Combination therapy that includes addition of a fibrate or niacin may be necessary to normalize triglyceride and LDL cholesterol concentrations , as well as to increase HDL cholesterol
Comparison of the major features of Type I and Type II diabetes Mellitus
FEATURE TYPE 1 DIABETES TYPE 2 DIABETES Age of onset Usually <40 yr Usually >40 yr Insulin production None Partial Insulin resistance No Yes Autoimmunity Yes No Obesity Not common Common MZ twin concordance 0.35-0.50 0.90 Sibling recurrence risk 1-6% 15-40
Receptor Mediated Endocytosis
FIG 12-7 The process of receptor-mediated endocytosis. 1, The low-density lipoprotein (LDL) receptors, which are glycoproteins, are synthesized in the endoplasmic reticulum of the cell. 2, They pass through the Golgi apparatus to the cell surface, where part of the receptor protrudes outside the cell. 3, The circulating LDL particle is bound by the LDL receptor and localized in cell-surface depressions called coated pits (so named because they are coated with a protein called clathrin). 4, The coated pit invaginates, bringing the LDL particle inside the cell. 5, Once inside the cell, the LDL particle is separated from the receptor, taken into a lysosome, and broken down into its constituents by lysosomal enzymes. 6, The LDL receptor is recirculated to the cell surface to bind another LDL particle. Each LDL receptor goes through this cycle approximately once every 10 minutes, even if it is not occupied by an LDL particle. 7, Free cholesterol is released from the lysosome for incorporation into cell membranes or metabolism into bile acids or steroids. Excess cholesterol can be stored in the cell as a cholesterol ester or removed from the cell by association with high-density lipoprotein (HDL). 8, As cholesterol levels in the cell rise, cellular cholesterol synthesis is reduced by inhibition of the rate-limiting enzyme, HMG-CoA reductase. 9, Rising cholesterol levels also increase the activity of acyl-coenzyme A:cholesterol acyltransferase (ACAT), an enzyme that modifies cholesterol for storage as cholesterol esters. 10, In addition, the number of LDL receptors is decreased by lowering the transcription rate of the LDL receptor gene itself. This decreases cholesterol uptake.
MOA and Use:pilocarpine
MOA: Cholinomimetic; stimulate muscarinic receptor activity. USE: Treats glaucoma, xerostomia( dry mouth), cevimeline
Structure of lipoprotein particles
FIGURE 20-1. Structure of lipoprotein particles. Lipoproteins are spherical particles (5- 1,000 nm in diameter) that transport hydrophobic molecules, principally cholesterol and triglycerides, as well as at-soluble vitamins. The sur ace o the particle is composed o a monolayer o phospholipid and unesterif ed cholesterol molecules. These polar lipids orm a coating that shields a hydrophobic core o nonpolar triglyceride and cholesteryl esters rom interacting with the aqueous environment o plasma. Lipoproteins contain amphipathic apolipoproteins (also called apoproteins) that associate with the sur ace lipids and hydrophobic core. Apolipoproteins provide structural stability to the lipoprotein particle and act as ligands or specif c cell sur ace receptors or as co actors or enzymatic reactions. In the example shown, a very-low-density lipoprotein (VLDL) particle contains apolipoprotein E, apolipoprotein B100, and apolipoproteins CI, CII, and CIII (shown here as apolipoprotein C).
What does autoantibody production occur as a result of?
Failure of Self tolerance autoantbodies may bind to self antigens in tissues or may form immune complexes with circulating self antigens
Familial combined hyperlipoproteinemia
Familial combined hyperlipoproteinemia (FCHL) is an autosomal dominant polygenic disorder that affects 1% to 2% of the population. Factors such as diet, glucose intolerance, and medications can influence the phenotypic presentation. In FCHL, the liver synthesizes excess VLDL. VLDL is hydrolyzed by LPL to produce LDL. Mutations in the LPL gene affecting its expression or function can decrease the efficiency of VLDL catabolism. Dysfunction of LPL is observed in one third of patients with FCHL. Diminished LPL activity increases circulating VLDL-triglyceride; furthermore, fewer VLDL remnant particles are available for HDL synthesis. Therefore, FCHL needs to be considered in all patients whose total cholesterol level is greater than 250 mg/dL, triglycerides greater than 175 mg/dL, or HDL-cholesterol less than 35 mg/dL. There are no definitive diagnostic tests, but family screening can help confirm the diagnosis. The phenotype of FCHL is variable, with individuals displaying high LDL-cholesterol, high VLDL-triglyceride, or both based on the genetic defect and environmental factors. Patients also typically have high apo B (>120 mg/dL) and a low ratio of LDL-cholesterol to apo B100 (<1.2). They accumulate small dense LDL particles, which are thought to be atherogenic and contribute to premature CHD. Xanthomas or xanthelasmas are not a feature of this disorder. Affected individuals require a low-fat, low-cholesterol diet plus multiple lipid-lowering drugs to achieve target goals. Fibric acid derivatives, which hydrolyze the triglyceride core of VLDL particles and increase LDL production, are recommended for treatment of the hypertriglyceridemia. Patients with FCHL often additionally require a statin or niacin to lower their LDLcholesterol level.
What is an example of a disorder associated with defects in receptor proteins ie is a receptor disease?
Familial hypercholesterolemia
What does the mutation in the gene that encodes LDL (Apo B/E) receptor result in?
Familial hypercholesterolemia Impairment in LDL receptor synthesis or function decreases the clearance of LDL and increases circulating LDL levels, resulting in cholesterol plaque formation.
A defect in receptor mediated transport system is exemplified by what condition? What occurs in this condition?
Familial hypercholesterolemia reduced synthesis or function of LDL receptors leads to defective transport of LDL into the cells and secondarily to excessive cholesterol synthesis by complex intermediary mechanisms In cystic fibrosis the transport system for chloride ions in exocrine glands, sweat ducts, lungs, and pancreas is defective. By mechanisms not fully understood, impaired chloride transport leads to serious injury to the lungs and pancreas
Genetic disorders of lipid metabolism
Familial hypercholesterolemia Mutation in the gene that encodes LDL receptor Elevated TC and LDL Familial defective apolipoprotein B100 Impaired binding of LDL to LDL receptor due to a defect in apo B100 protein Elevated TC and LDL Elevated plasma Lp(a) Increased binding of LDL to apolipoprotein(a) Elevated Lp(a) Polygenic hypercholesterolemia Increased binding of apo E4-containing lipoprotein to LDL receptor resulting in downregulation of the LDL receptor Elevated TC and LDL Familial combined hyperlipoproteinemia Polygenic disorder associated with increased hepatic VLDL production, resulting in increased LDL and decreased HDL production; some individuals have a mutation in the LPL gene that affects expression and function of LPL Elevated TC, LDL, and TG Low HDL Familial dysbetalipoproteinemia Lower affinity of apo E2 for LDL receptor Elevated TG, TC, and LDL Lipoprotein lipase deficiency Mutation in the LPL gene Elevated TG Apolipoprotein C-II deficiency Decrease in activation of LPL due to a deficiency of apo CII Elevated TG Familial hypertriglyceridemia Overproduction of hepatic VLDL and increased catabolism of HDL Elevated TG Low HDL
disorders associated with defects in receptor proteins - familial hypercholesterolemia
Familial hypercholesterolemia is a "receptor disease" that is the consequence of a mutation in the gene encoding the receptor for LDL, which is involved in the transport and metabolism of cholesterol. As a consequence of receptor abnormalities there is a loss of feedback control and elevated levels of cholesterol that induce premature atherosclerosis, leading to a greatly increased risk of myocardial infarction. Familial hypercholesterolemia is one of the most frequently occurring Mendelian disorders. Heterozygotes with one mutant gene, representing about 1 in 500 individuals, have from birth a two-fold to three-fold elevation of plasma cholesterol level, leading to tendinous xanthomas and premature atherosclerosis in adult life (Chapter 11). Homozygotes, having a double dose of the mutant gene, are much more severely affected and may have fivefold to six-fold elevations in plasma cholesterol levels. Skin xanthomas and coronary, cerebral, and peripheral vascular atherosclerosis may develop in these individuals at an early age. Myocardial infarction may occur before age 20 years. Large-scale studies have found that familial hypercholesterolemia is present in 3% to 6% of survivors of myocardial infarction
Familial hypertriglyceridemia
Familial hypertriglyceridemia is an autosomal dominant disorder that is characterized by overproduction of hepatic VLDL. The exact defect or mutation is unknown. Secondary factors that increase VLDL, such as diabetes, alcohol ingestion, and estrogen therapy, appear to exacerbate this condition. Low HDL associated with familial hypertriglyceridemia is related to increased catabolism. Individuals with this condition have hypertriglyceridemia (200 to 500 mg/dL) and low HDL-cholesterol (<35 mg/dL) at presentation. This diagnosis is considered in individuals who have a family and personal history of hypertriglyceridemia, CHD, and normal LDL levels. Cloudy infranatant after overnight refrigeration of plasma identifies a disorder of VLDL metabolism. Treatment starts with management of secondary factors that may exacerbate the condition. Dietary fat restriction (<10% of calories) and drug therapy with fish oil, niacin, and fibric acid derivates should be initiated if target goals are not achieved.
What are the features included in the clinical presentation of Myasthenia gravis?-2 types?
Fluctuating, fatigable weakness in either: 1-Ocular MG: Isolated to the ocular muscles or 2- Generalized MG: ocular as well as limb, bulbar and respiratory muscles
What does LPL mediated lipolysis liberate?
Free fatty acids and glycerol The free fatty acids are then taken up by the neighboring parenchymal cells The expression level and intrinsic activity of LPL in muscle and adipose tissue are regulated according to the fed/fasting state, allowing the body to direct the delivery of fatty acids preferentially to muscle during fasting and to adipose after a meal
nature and nurture- disentangling the effects of genes and environment
Family members share genes and a common environment. Family resemblance in traits such as blood pressure therefore reflects both genetic and environmental commonality ("nature" and "nurture," respectively). For centuries, people have debated the relative importance of these two types of factors. It is a mistake, of course, to view them as mutually exclusive. Few traits are influenced only by genes or only by environment. Most are influenced by both. Determining the relative influence of genetic and environmental factors can lead to a better understanding of disease etiology. It can also help in the planning of public health strategies. A disease in which hereditary influence is relatively small, such as lung cancer, may be prevented most effectively through emphasis on lifestyle changes (avoidance of tobacco). When a disease has a relatively larger hereditary component, as in breast cancer, examination of family history should be emphasized in addition to lifestyle modification. In the following sections, we review two research strategies that are often used to estimate the relative influence of genes and environment: twin studies and adoption studies. We then discuss methods that aim to delineate the individual genes responsible for multifactorial diseases
What is required to diagnose hypertriglyceridemia? What triglyceride level is classified as abnormal? borderline? normal?
Fasting lipid panel TG level higher than 200 mg/dL Borderline triglyceride levels range from 150 to 200 mg/dL, normal values are lower than 150 mg/dL A diet and exercise program is recommended for all individuals with abnormal triglyceride levels (level C). However, pharmacologic treatments to reduce triglyceride levels may be considered if fasting levels are higher than 200 mg/dL, especially if the individual is at risk for CHD or pancreatitis
What can bile acid sequestrants decrease the absorption of?
Fat soluble vitamins bleeding due to vitamin K deficiency has occasionally been reported They can also bind certain co-administered drugs, such as digoxin and warfarin, and thereby lower the bioavailability of the co-administered agents. This interaction can be eliminated by administering the bile acid sequestrant at least 1 hour be ore or 4 hours after other drugs. Colesevelam is more selective and appears to avoid this problem.
What does QT (LQT) syndrome predispose affected people to?
Fatal cardiac arrhythmias caused by either inherited mutations or by exposure to drugs that block potassium channels
What do bile acids help transport?
Fats Approximately 50% of the cholesterol and 97% of the bile acid entering the lumen is reabsorbed back into the circulation. The reabsorbed cholesterol regulates cholesterol and LDL receptor synthesis Many cells in the body, including liver parenchymal cells, synthesize cholestero
What is the high affinity receptor for IgE called? What does it consist of?
FcεRI consists of three polypeptide chains, one of which binds the Fc portion of the ε heavy chain very strongly, the other 2 chains are signaling proteins
Is the risk for Pyloric stenosis higher when the proband is male or female?
Female this reflects the concept that females, having a higher liability threshold, must be exposed to more disease-causing factors than males in order to develop the disease Thus, a family with an affected female must have more genetic and environmental risk factors, producing a higher recurrence risk for pyloric stenosis in future offspring
What do Omega-3 fatty acids constitute 30-50% of? 85% of? side effects?
Fish oil supplements Prescribed pharmacologic preparations (ie: Lovaza and Vascepa) In clinical trials, both Lovaza and Vascepa 4 g/day lowered triglyceride levels by 45%. Fish oil supplements seem to be a reasonable, cost-effective means to reduce triglyceride levels; side effects include eructation (burping), taste perversion, and dyspepsia.
What occurs following the brief period of excitation(manifested by widespread fasciculations) ? What are the 2 reasons why it occurs?
Flaccid paralysis 1- Open cholinergic channels maintain the cell membrane in a depolarized state, effecting inactivation of voltage gated sodium channels so that they cannot open to support further action potentials 2- Agonist bound nAChRs spontaneously desensitize, preventing, preventing their opening and response to any subsequently delivered, additional agonist
What do lower leptin levels lead to?
Increased apetite
The effects of muscarinic receptor activation occurs through what proteins?
G proteins results in latency (at least 100-250 ms associated with muscarinic responses to receptor activation. (In contrast, nicotinic receptor channels have latencies on the order o 5 ms.)
Lipoprotein genes known to contribute to coronary heart disease risk
GENE CHROMOSOME LOCATION FUNCTION OF PROTEIN PRODUCT Apolipoprotein A-I 11q HDL component; LCAT cofactor Apolipoprotein A-IV 11q Component of chylomicrons and HDL; may influence HDL metabolism Apolipoprotein C-III 11q Allelic variation associated with hypertriglyceridemia Apolipoprotein B 2p Ligand for LDL receptor; involved in formation of VLDL, LDL, IDL, and chylomicrons Apolipoprotein D 2p HDL component Apolipoprotein C-I 19q LCAT activation Apolipoprotein C-II 19q Lipoprotein lipase activation Apolipoprotein E 19q Ligand for LDL receptor Apolipoprotein A-II 1p HDL component LDL receptor 19p Uptake of circulating LDL particles Lipoprotein(a) 6q Cholesterol transport Lipoprotein lipase 8p Hydrolysis of lipoprotein lipids Hepatic triglyceride lipase 15q Hydrolysis of lipoprotein lipids LCAT 16q Cholesterol esterification Cholesterol ester transfer protein 16q Facilitates transfer of cholesterol esters and phospholipids between lipoproteins
What is Polygenic hypercholesterolemia?
General term that has been used to categorize the majority of patients with hypercholesterolemia who have no defined genetic cause or the disorder -may be the result of complex gene-environment interactions, multiple uncharacterized genetic susceptibilities, or variant LDL particles such as small dense LDL and lipoprotein(a) [Lp(a)].
What 3 genes can half of early onset Alzheimer's disease be attributed to?
Genes that affect: 1- Beta-amyloid deposition 2- Presenilin 1 (PS1) 3- Presenilin 2 (PS2) Gain-of-function mutations in PS1 or PS2 affect the cleavage of APP such that amyloid-producing forms of it accumulate excessively and are deposited in the brain This is thought to be a primary cause of AD. Mutations in PS1 typically result in especially early onset of AD, with the first occurrence of symptoms in the fifth decade of life.
What are the most commonly mutated genes that lead to hypertrophic cardiomyopathy?
Genes that encode: 1- Beta- myosin heavy chain (35% of familial cases) 2- Myosin binding protein C (20% of cases) 3- Troponin T (15% of cases)
Alterations in Structure, Function, or Quantity of Nonenzyme Proteins
Genetic defects resulting in alterations of nonenzyme proteins often have widespread secondary effects, as exemplified by sickle cell disease. The hemoglobinopathies, sickle cell disease being one, all of which are characterized by defects in the structure of the globin molecule, best exemplify this category. In contrast to the hemoglobinopathies, the thalassemias result from mutations in globin genes that affect the amount of globin chains synthesized. Thalassemias are associated with reduced amounts of structurally normal α-globin or β-globin chains (Chapter 14). Other examples of genetic disorders involving defective structural proteins include collagen, spectrin, and dystrophin, giving rise to osteogenesis imperfecta (Chapter 26), hereditary spherocytosis (Chapter 14), and muscular dystrophies (Chapter 27), respectively.
What is the rate limiting step in Cholesterol synthesis pathway? What occurs if it is inhibited?
HMG-CoA reductase Drugs that inhibit HMG-CoA reductase decrease cholesterol biosynthesis and cellular cholesterol pools
Inhibition of what enzyme decreases intracellular pools and subsequently increases uptake of LDL cholesterol from the circulation?
HMG-CoA reductase target of statins
What is an example of a condition that has mild manifestations of immediate hypersensitivity? What do reactions occur in response to?
Hay fever- allergic rhinitis and sinusitis Inhaled allergens eg: as a protein of ragweed pollen Mast cells in the nasal mucosa produce histamine, and Th2 cells produce IL-13, and these two mediators cause increased production of mucus. Late phase reactions may lead to more prolonged inflammation.
What is the function of TAP1 and TAP2 proteins?
Help transport peptides into the endoplasmic reticulum, where they initially form complexes with class 1 molecules before migrating to cell surface
When associated with foreign peptides and after binding to T cell receptors what do MHC II molecules stimulate?
Helper T cell activity
What stimulates B lymphocytes and other types of T lymphocytes to respond to infections more effectively? What lymphocytes can directly kill infected cells?
Helper T lymphocytes Cytotoxic T lymphocytes
What is seen in people Heterozygote for Factor V Leiden? Homozygosity?
Heterozygosity for this allele, which is seen in approximately 5% of whites, produces a sevenfold increase in the risk of venous thrombosis (clots). In homozygotes, the risk increases to 100-fold. However, the evidence for an association between the factor V Leiden allele and stroke is inconsistent.
What occurs in individuals that are Heterozygotes for familial hypercholesterolemia?
Heterozygotes with one mutant gene, representing about 1 in 500 individuals have from birth a two-fold to three-fold elevation of plasma cholesterol level, leading to tendinous xanthomas and premature atherosclerosis in adult life
Heterozygotes vs Homozygotes in familial hypercholesterolemia
Heterozygous individuals (1 in 500 in the United States) have elevated total plasma cholesterol concentrations from birth throughout life, with adult levels averaging 275-500 mg/dL (normal is less than 200 mg/dL) Homozygous FH is a much more severe but rare disorder (1 in 1 million in the United States) that is characterized by the absence of functional LDL receptors. This leads to very high plasma cholesterol concentrations (700-1,200 mg/dL) and cardiovascular disease that presents clinically prior to the age of 20
What state are Autosomal dominant disorders manifested in? Are both males and females affected?
Heterozygous state so at least one parent of an index case is usually affected Yes, both males and females are affected and both can transmit the condition
What do IgE antibodies that are produced in response to an allergen bind to ?
High affinity Fc receptors specific for epsiolon heavy chain that are expressed on mast cells
National cholesterol education program adult treatment panel III guidelines
High risk: CHD or CHD risk equivalents (10-year risk 20%) 100 mg/dL; optional goal 70 mg/dL 100 mg/dL 100 mg/dL Moderately high risk: 2 risk factors (10-year risk 10-20%) 130 mg/dL 130 mg/dL 130 mg/dL (consider drug options if 100-129 mg/dL) Moderate risk: 2 risk factors (10-year risk 10%) 130 mg/dL 130 mg/dL 160 mg/dL Low risk: 0-1 risk factor 160 mg/dL 160 mg/dL 190 mg/dL (consider drug options if 160-189 mg/dL
What is the major amine secreted by mast cells? What effects does it have?
Histamine 1- dilation of small blood vessels 2- Increase in vascular permeability 3- Stimulates the transient contraction of smooth muscles
What effect do increased fat stores have?
Increased fat stores lead to elevated leptin levels which produces satiety and a loss of apetite
types of hypersensitivity reactions
Hypersensitivity reactions are classified on the basis of the principal immunologic mechanism that is responsible for tissue injury and disease (Fig. 11-1). We prefer the more informative descriptive designations rather than numeric terms, so these descriptors are used throughout this chapter • Immediate hypersensitivity, or type I hypersensitivity, is a type of pathologic reaction that is caused by the release of mediators from mast cells. This reaction most often depends on the production of immunoglobulin E (IgE) antibody against environmental antigens and the binding of IgE to mast cells in various tissues. • Antibodies other than IgE that are directed against cell or tissue antigens can damage these cells or tissues or can impair their function. These diseases are said to be antibody mediated and represent type II hypersensitivity. • Antibodies against soluble antigens may form complexes with the antigens, and the immune complexes may deposit in blood vessels in various tissues, causing inflammation and tissue injury. Such diseases are called immune complex diseases and represent type III hypersensitivity. • Some diseases result from the reactions of T lymphocytes, often against self antigens in tissues. These T cell-mediated diseases represent type IV hypersensitivity. This classification scheme is useful because it distinguishes the mechanisms of immune- mediated tissue injury. In many human immunologic diseases, however, the damage may result from a combination of antibody-mediated and T cell-mediated reactions, so it is often difficult to classify these diseases neatly into one type of hypersensitivity.
In addition to family history and specific genes what factors are known to increase the risk of stroke?
Hypertension, Obesity, atherosclerosis, diabetes and smoking
What are omega-3 fatty acids useful for treating patients with?
Hypertriglyceridemia effective at reducing plasma triglycerides by up to 50%
What is a major form of Cardiomyopathy? What is it characterized by?
Hypertrophic cardiomyopathy thickening (hypertrophy) of portions of the left ventricle seen in as many as 1/500 adults
When converted to remnants following exchange with apolipoproteins with HDL what are these more dense particles called?
IDL (Intermediate density lipoprotein) Because IDL contain apoE, a fraction of these particles (approximately 50%) may be cleared by the liver via remnant receptor pathways However, the remainder are converted to LDL by hepatic lipase, which further hydrolyzes triglycerides in the core of IDL.
What is the principal macrophage activating cytokine? What is its source?
IFN-gamma (interferon gamma) TH1
What cytokines stimulate the differentiation of T cells to Th17 subset
IL-1, IL-6, IL-23
What Cytokine induces differentiation of CD4 T cells to Th1 subset?
IL-12
What cytokine does Th17 and Th22 secrete respectively?
IL-17 IL-22
What cytokines do activated Th17 cells secrete? What do these cytokines collectively recruit? what do they promote?
IL-17 and IL-22, chemokines and other cytokines Neutrophils and Monocytes Inflammation Th17 cells also produce IL-21 which amplifies the Th17 response
What are the 2 cytokines secreted by Th2 cells or Tfh cells? What do they stimulate?
IL-4 and IL-13 Stimulate B lymphocytes to switch to IgE producing plasma cells
What does a mutation in clotting factor V- Factor V Leiden allele cause?
Increased resistance to activated protein C and thus produces an increased susceptibility to clotting
How is VLDL levels calculated?
If the triglyceride concentration is lower than 400 mg/dL, then VLDL is calculated by dividing the triglyceride level by 5. Total cholesterol, triglyceride and HDL levels can be measured directly
What is an Immediate hypersensitivity reaction (Type I)? What does it cause?
IgE antibody and mast cell mediated reaction to certain antigens causes rapid vascular leakage and mucosal secretions, often followed by inflammation.
activation of mast cells and secretion of mediators
IgE antibody produced in response to an allergen binds to high-affinity Fc receptors, specific for the ε heavy chain, that are expressed on mast cells (see Fig. 11-2). Thus, in an atopic individual, mast cells are coated with IgE antibody specific for the antigen(s) to which the individual is allergic. This process of coating mast cells with IgE is called sensitization, because coating with IgE specific for an antigen makes the mast cells sensitive to activation by subsequent encounter with that antigen. In normal individuals, by contrast, mast cells may carry IgE molecules of many different specificities, because many antigens may elicit small IgE responses, and the amount of IgE specific for any one antigen is not enough to cause immediate hypersensitivity reactions upon exposure to that antigen. Mast cells are present in all connective tissues, especially under epithelia, and they are usually located adjacent to blood vessels. Which of the body's mast cells are activated by cross-linking of allergen-specific IgE often depends on the route of entry of the allergen. For example, inhaled allergens activate mast cells in the submucosal tissues of the bronchus, whereas ingested allergens activate mast cells in the wall of the intestine The high-affinity receptor for IgE, called FcεRI, consists of three polypeptide chains, one of which binds the Fc portion of the ε heavy chain very strongly, with a Kd of approximately 10−11 M. (The concentration of IgE in the plasma is approximately 10-9 M, which explains why even in normal individuals, mast cells are always coated with IgE bound to FcεRI.) The other two chains of the receptor are signaling proteins. The same FcεRI is also present on basophils, which are circulating cells with many of the features of mast cells, but the role of basophils in immediate hypersensitivity is not as well established as the role of mast cells When mast cells sensitized by IgE are exposed to the allergen, the cells are activated to secrete their mediators (Fig. 11-4). Mast cell activation results from binding of the allergen to two or more IgE antibodies on the cell. When this happens, the FcεRI molecules that are carrying the IgE are cross-linked, triggering biochemical signals from the signal-transducing chains of FcεRI. The signals lead to three types of responses in the mast cell: rapid release of granule contents (degranulation), synthesis and secretion of lipid mediators, and synthesis and secretion of cytokines. The most important mediators produced by mast cells are vasoactive amines and proteases stored in and released from granules, newly generated and secreted products of arachidonic acid metabolism, and cytokines (see Fig. 11-4). These mediators have different actions. The major amine, histamine, causes the dilation of small blood vessels, increases vascular permeability, and stimulates the transient contraction of smooth muscles. Proteases may cause damage to local tissues. Arachidonic acid metabolites include prostaglandins, which cause vascular dilation, and leukotrienes, which stimulate prolonged smooth muscle contraction. Cytokines induce local inflammation (the latephase reaction, described next). Thus, mast cell mediators are responsible for acute vascular and smooth muscle reactions and inflammation, the hallmarks of immediate hypersensitivity. Cytokines produced by mast cells stimulate the recruitment of leukocytes, which cause the late-phase reaction. The principal leukocytes involved in this reaction are eosinophils, neutrophils, and Th2 cells. Mast cell-derived tumor necrosis factor (TNF) and IL-4 promote neutrophil- and inflammation. Chemokines produced by mast cells and by epithelial cells in the tissues also contribute to leukocyte recruitment. Eosinophils and neutrophils liberate proteases, which cause tissue damage, and Th2 cells may exacerbate the reaction by producing more cytokines. Eosinophils are prominent components of many allergic reactions and are an important cause of tissue injury in these reactions. These cells are activated by the cytokine IL-5, which is produced by Th2 cells, innate lymphoid cells, and mast cells.
Deposition of what antibodies may activate the complement system by the classical pathway?
IgG or IgM
What immunoglobin isotype do immature B lymphocytes produce?
IgM but as they mature they can rearrange genes of heavy chain to form others - Class switching
Immediate hypersensitivity
Immediate hypersensitivity is an IgE antibody- and mast cell-mediated reaction to certain antigens that causes rapid vascular leakage and mucosal secretions, often followed by inflammation. Disorders in which IgEmediated immediate hypersensitivity is prominent are also called allergy, or atopy, and individuals with a propensity to develop these reactions are said to be atopic. Immediate hypersensitivity may affect various tissues and may be of varying severity in different individuals. Common types of allergies include hay fever, food allergies, bronchial asthma, and anaphylaxis. Allergies are the most frequent disorders of the immune system, estimated to affect 10% to 20% of people, and the incidence of allergic diseases has been increasing in industrialized societies. The sequence of events in the development of immediate hypersensitivity reactions begins with the activation of Th2 and IL-4-secreting follicular helper T (Tfh) cells, which stimulate the production of IgE antibodies in response to an antigen, binding of the IgE to specific Fc receptors of mast cells, then on subsequent exposure to the antigen, cross-linking of the bound IgE by the antigen, and release of mast cell mediators (Fig. 11-2). Some mast cell mediators cause a rapid increase in vascular permeability and smooth muscle contraction, resulting in many of the symptoms of these reactions (Fig. 11-3). This vascular and smooth muscle reaction may occur within minutes of reintroduction of antigen into a previously sensitized individual, hence the name immediate hypersensitivity. Other mast cell mediators are cytokines that recruit neutrophils and eosinophils to the site of the reaction over several hours. This inflammatory component is called the late-phase reaction, and it is mainly responsible for the tissue injury that results from repeated bouts of immediate hypersensitivity. With this background, we proceed to a discussion of the steps in immediate hypersensitivity reactions.
clinical symptoms and therapy
Immediate hypersensitivity reactions have diverse clinical and pathologic features, all of which are attributable to mediators produced by mast cells in different amounts and in different tissues • Some mild manifestations, such as allergic rhinitis and sinusitis, which are common in hay fever, are reactions to inhaled allergens, such as a protein of ragweed pollen. Mast cells in the nasal mucosa produce histamine, and Th2 cells produce IL-13, and these two mediators cause increased production of mucus. Latephase reactions may lead to more prolonged inflammation. • In food allergies, ingested allergens trigger mast cell degranulation, and the released histamine causes increased peristalsis, resulting in vomiting and diarrhea. • Bronchial asthma is most often a form of respiratory allergy in which inhaled allergens (often undefined) stimulate bronchial mast cells to release mediators, including leukotrienes, which cause repeated bouts of bronchial constriction and airway obstruction. In chronic asthma, large numbers of eosinophils accumulate in the bronchial mucosa, excessive secretion of mucus occurs in the airways, and the bronchial smooth muscle becomes hypertrophied and hyperreactive to various stimuli. Some cases of asthma are not associated with IgE production, although all are caused by mast cell activation. In some patients, asthma may be triggered by cold or exercise; how either of these causes mast cell activation is unknown. • The most severe form of immediate hypersensitivity is anaphylaxis, a systemic reaction characterized by edema in many tissues, including the larynx, accompanied by a fall in blood pressure and bronchoconstriction. Some of the most frequent inducers of anaphylaxis include bee stings, injected or ingested penicillin-family antibiotics, and ingested nuts or shellfish. The reaction is caused by widespread mast cell degranulation in response to the systemic distribution of the antigen, and it is life threatening because of the sudden fall in blood pressure and airway obstruction.
How is the innate immune system activated?
Immune cell receptor recognition of general features that are detected on pathogens but not the host eg: gram positive bacteria produce peptidoglycans
What type of disease is Systemic lupus Erytheomatosus?
Immune complex disease
What type of disease is Serum sickness?
Immune complex disease seen in subjects who repeatedly received animal serum for the treatment of infections
What are common sites where immune complexes are deposited and thus immune complex mediated diseases occur?
Immune complex disease tend to be systemic but they preferentially occur in: 1- Kidney - Glomerulonephritis 2- Joints- Arthritis 3- Small blood vessels - Vasculitis
Immunodeficiency diseases
Immunodeficiency disease results when one or more components of the immune system (e.g., T cells, B cells, MHC, complement proteins) are missing or fail to function normally. Primary immunodeficiency diseases are caused by abnormalities in cells of the immune system and are usually produced by genetic alterations. To date, more than 100 different primary immunodeficiency syndromes have been described, and it is estimated that these diseases affect at least 1 in 10,000 persons. Secondary immunodeficiency occurs when components of the immune system are altered or destroyed by other factors, such as radiation, infection, or drugs. For example, the human immunodeficiency virus (HIV), which causes acquired immunodeficiency syndrome (AIDS), attacks macrophages and helper T lymphocytes, central components of the immune system. The result is increased susceptibility to a multitude of opportunistic infections B-cell immunodeficiency diseases render the patient especially susceptible to recurrent bacterial infections, such as Streptococcus pneumoniae. An important example of a B-cell immunodeficiency is X-linked agammaglobulinemia (XLA). Patients with this disorder, the overwhelming majority of whom are male, lack B cells completely and have no IgA, IgE, IgM, or IgD in their serum. Because IgG crosses the placenta during pregnancy, infants with XLA have some degree of humoral immunity for the first several months of life. However, the IgG supply is soon depleted, and the infants develop recurrent bacterial infections. They are treated with large amounts of γ-globulin. XLA is caused by mutations in a gene (BTK) that encodes a B-cell tyrosine kinase necessary for normal B-cell maturation. Mutations in the genes that encode the immunoglobulin heavy and light chains can cause a form of autosomal recessive B-cell immunodeficiency. T-cell immunodeficiency diseases directly affect T cells, but they also affect the humoral immune response, because B-cell proliferation largely depends on helper T cells. Thus, in the most severe T-cell defects, affected patients develop severe combined immune deficiency (SCID) and are susceptible to many opportunistic infections, including Pneumocystis jiroveci (a protozoan that commonly infects AIDS patients). Without bone marrow transplants, these patients usually die within the first several years of life. About half of SCID cases are caused by X-linked recessive mutations in a gene that encodes the γ chain found in six different cytokine receptors (those of interleukins 2, 4, 7, 9, 15, and 21). Lacking these receptors, T cells and natural killer cells cannot receive the signals they need for normal maturation. These receptors all interact with an intracellular signaling molecule called Jak3. As might be expected, persons who lack Jak3 as a result of autosomal recessive mutations in the JAK3 gene experience a form of SCID that is very similar to the X-linked form just described. About 15% of SCID cases are caused by adenosine deaminase (ADA) deficiency, an autosomal recessive disorder of purine metabolism that results in a buildup of metabolites that are toxic to B and T cells. This type of SCID, as well as the X-linked form, can be treated by bone marrow transplantation, and some cases are being treated successfully with gene therapy (see Chapter 13). SCID can also arise from mutations in RAG1 or RAG2, two of the genes involved in VDJ recombination and the proper formation of T-cell and B-cell receptors. These mutations produce a combined B-cell and T-cell immunodeficiency, although normal natural killer cells are produced. Other examples of SCID are given in Table 9-3. Several immune system defects result in lymphocytes that lack MHC molecules on their surfaces. These are collectively termed bare lymphocyte syndrome, one form of which is caused by mutations in the TAP2 gene. TAP2 encodes a protein that helps to transport peptides to the endoplasmic reticulum, where they are bound by class I MHC molecules. A defect in the TAP2 protein destabilizes the class I MHC molecules so that they are not expressed on the cell surface. Because exposure to MHC molecules is necessary for normal T-cell development in the thymus, bare lymphocyte syndrome results in a severe reduction in the number of functional T and B cells. Bare lymphocyte syndrome can also be caused by defects in several different transcription factors that bind to promoters in the class II MHC region. The result is a lack of class II MHC molecules on APCs, a deficiency of helper T cells, and a consequent lack of antibody production. Chronic granulomatous disease (CGD) is a primary immunodeficiency disorder in which phagocytes can ingest bacteria and fungi but are then unable to kill them. This brings about a persistent cellular immune response to the ingested microbes, resulting in the formation of the granulomas (nodular inflammatory lesions containing macrophages) for which the disease is named. These patients develop pneumonia, lymph node infections, and abscesses in the skin, liver, and other sites. The most common cause of CGD is an X-linked mutation, but there are also at least three autosomal recessive genes that can cause CGD. The gene that causes X-linked CGD was the first disease-causing gene to be isolated through positional cloning. It encodes a subunit of cytochrome b, a protein that the phagocyte requires for a burst of microbe-killing oxygen metabolism. Multiple defects in the various proteins that make up the complement system have been identified. Most of these are inherited as autosomal recessive disorders, and most result in increased susceptibility to bacterial infections. Finally, a number of syndromes include immunodeficiency as one of their features. One example is the DiGeorge sequence (see Chapter 6), in which a lack of thymic development leads to T-cell deficiency. Wiskott-Aldrich syndrome is an X-linked recessive disorder that involves deficiencies of platelets and progressive T-cell dysfunction. It is caused by loss-of-function mutations in a gene (WAS) whose protein product is expressed in hematopoietic cells, where it relays signals from the cell surface to the cytoskeleton. Wiskott-Aldrich syndrome, like SCID, has been successfully treated with gene therapy (see Chapter 13). Immunodeficiency is also seen in several syndromes that involve DNA instability (e.g., ataxia telangiectasia, Bloom syndrome, Fanconi anemia; see Chapter 3). Primary immunodeficiency diseases involve intrinsic defects of immune response cells (e.g., B cells, T cells, MHC, complement system, or phagocytes) and are usually caused by genetic alterations. Secondary immunodeficiency disorders, of which AIDS is an example, are caused by external factors. Immunodeficiency is also seen in a number of genetic syndromes, including several DNA instability disorders
In patients in whom symptoms are not adequately controlled with 3,4- DAP and pyrdiostigmine what drugs can be used?
Immunomodulation with Prednisone, azathioprine or mycophenolate mofetil is used
What is Autism spectrium disorder characterized by?
Impairments in social interaction and communication as well as restricted, repetitive and stereotyped behaviors and activities ASD is 3-4 times more common in males than in females and is usually diagnosed during the first three years of life Genetic factors play a large role in the etiology of ASD, with twin studies suggesting a heritability of approximately 70%.
What occurs in Classic type of Ehlers danlos syndrome?
In 30% to 50% of these cases, mutations in the genes for type V collagen (COL5A1 and COL5A2) have been detected
What are the most important environmental risk factors for hypertension?
Increased sodium intake, decreased physical activity, psychosocial stress and obesity
CD4 t cell mediated inflammation
In CD4+ T cell-mediated hypersensitivity reactions, cytokines produced by the T cells induce inflammation that may be chronic and destructive. The prototype of T cell-mediated inflammation is delayed-type hypersensitivity (DTH), a tissue reaction to antigens given to immune individuals. In this reaction, an antigen administered into the skin of a previously immunized individual results in a detectable cutaneous reaction within 24 to 48 hours (hence the term delayed, in contrast to immediate hypersensitivity). Both TH1 and TH17 cells contribute to organ-specific diseases in which inflammation is a prominent aspect of the pathology. The inflammatory reaction associated with TH1 cells is dominated by activated macrophages, and that triggered by TH17 cells has a greater neutrophil component. The inflammatory reactions stimulated by CD4+ T cells can be divided into sequential stages: Activation of CD4+ T Cells. As described earlier, naive CD4+ T cells recognize peptides displayed by dendritic cells and secrete IL-2, which functions as an autocrine growth factor to stimulate proliferation of the antigen- responsive T cells. The subsequent differentiation of antigen-stimulated T cells to TH1 or TH17 cells is driven by the cytokines produced by APCs at the time of T-cell activation. In some situations the APCs (dendritic cells and macrophages) produce IL-12, which induces differentiation of CD4+ T cells to the TH1 subset. IFN-γ produced by these effector cells promotes further TH1 development, thus amplifying the reaction. If the APCs produce inflammatory cytokines such as IL-1, IL-6, and a close relative of IL-12 called IL-23, these stimulate differentiation of T cells to the TH17 subset. Some of the differentiated effector cells enter the circulation and may remain in the memory pool of T cells for long periods, sometimes years. Responses of Differentiated Effector T Cells. Upon repeat exposure to an antigen, TH1 cells secrete cytokines, mainly IFN-γ, which are responsible for many of the manifestations of delayed-type hypersensitivity. IFN-γ-activated ("classically activated") macrophages are altered in several ways: their ability to phagocytose and kill microorganisms is markedly augmented; they express more class II MHC molecules on the surface, thus facilitating further antigen presentation; they secrete TNF, IL-1, and chemokines, which promote inflammation (Chapter 3); and they produce more IL-12, thereby amplifying the TH1 response. Thus, activated macrophages serve to eliminate the offending antigen; if the activation is sustained, continued inflammation and tissue injury result. Activated TH17 cells secrete IL-17, IL-22, chemokines, and several other cytokines. Collectively, these cytokines recruit neutrophils and monocytes to the reaction, thus promoting inflammation. TH17 cells also produce IL-21, which amplifies the TH17 response.
secondary hyperlipidemia
In addition to the genetic causes o primary dyslipidemia described above, a number o secondary actors can lead to hyperlipidemia (Table 20-2). For example, alcohol intake increases the synthesis o atty acids, which are then esterif ed to glycerol to orm triglycerides. There ore, excess alcohol consumption can result in increased VLDL production. Hypertriglyceridemia in type 2 diabetes mellitus results from increased VLDL synthesis and secretion and rom reduced chylomicron and VLDL catabolism by LPL. Furthermore, apoCIII levels are increased in association with insulin resistance, and this reduces the catabolism o chylomicrons and VLDL particles. Hypothyroidism is an important and common cause of secondary hyperlipidemia. Any patient with a lipid disorder should be screened or hypothyroidism
mechanisms of tissue injury
In different T cell-mediated diseases, tissue injury is caused by inflammation induced by cytokines that are produced mainly by CD4+ T cells or by killing of host cells by CD8+ CTLs (Fig. 11-11). These mechanisms of tissue injury are the same as the mechanisms used by T cells to eliminate cell-associated microbes. CD4+ T cells may react against cell or tissue antigens and secrete cytokines that induce local inflammation and activate macrophages. Different diseases may be associated with activation of Th1 and Th17 cells. Th1 cells are the source of interferon-γ (IFN-γ), the principal macrophageactivating cytokine, and Th17 cells are responsible for the recruitment of leukocytes, including neutrophils. The actual tissue injury in these diseases is caused mainly by the macrophages and neutrophils. The typical reaction mediated by T cell cytokines is delayed-type hypersensitivity (DTH), so called because it occurs 24 to 48 hours after an individual previously exposed to a protein antigen is challenged with the antigen (i.e., the reaction is delayed). The delay occurs because it takes several hours for circulating effector T lymphocytes to home to the site of antigen challenge, respond to the antigen at this site, and secrete cytokines that induce a detectable reaction. DTH reactions are manifested by infiltrates of T cells and blood monocytes in the tissues, edema and fibrin deposition caused by increased vascular permeability in response to cytokines produced by CD4+ T cells, and tissue damage induced by leukocyte products, mainly from macrophages that are activated by the T cells (Fig. 11-12). DTH reactions often are used to determine if people have been previously exposed to and have responded to an antigen. For example, a DTH reaction to a mycobacterial antigen, PPD (purified protein derivative), is an indicator of a T cell response to the mycobacteria. This is the basis for the PPD skin test, used to detect past or active mycobacterial infection. CD8+ T cells specific for antigens on host cells may directly kill these cells. CD8+ T cells also produce cytokines that induce inflammation, but they are usually not the major sources of cytokines in immune reactions. In many T cell-mediated autoimmune diseases, both CD4+ T cells and CD8+ T cells specific for self antigens are present, and both contribute to tissue injury.
activation of th2 cells and production of IgE antibody
In individuals who are prone to allergies, exposure to some antigens results in the activation of Th2 and Tfh cells and the production of IgE antibody (see Fig. 11-2). Most individuals do not mount strong Th2 responses to environmental antigens. For unknown reasons, when some individuals encounter certain antigens, such as proteins in pollen, certain foods, insect venoms, or animal dander, or if they are treated with certain drugs such as penicillin, there is a strong Th2 response. Immediate hypersensitivity develops as a consequence of the activation of Th2 cells in response to protein antigens or chemicals that bind to proteins. Antigens that elicit immediate hypersensitivity (allergic) reactions often are called allergens. Any atopic individual may be allergic to one or more of these antigens. It is not understood why only a small subset of common environmental antigens elicit Th2-mediated reactions and IgE production, or what characteristics of these antigens are responsible for their behavior as allergens. Two of the cytokines secreted by Th2 cells or by Tfh cells activated by the same antigen are IL-4 and IL-13. These cytokines stimulate B lymphocytes to switch to IgE-producing plasma cells. Therefore, atopic individuals produce large amounts of IgE antibody in response to antigens that do not elicit IgE responses in other people. The propensity toward development of IL-4-producing T cells, IgE production, and immediate hypersensitivity has a strong genetic basis; a major known risk for developing allergies is a family history of atopic disease. Many different genes appear to play contributory roles, but the mechanisms by which these genes influence the development of allergies are poorly understood.
Where are VLDL particles formed and what do they transport?
Liver Triglycerides synthesized endogenously
Degradation of Ach
In order or acetylcholine to be use ful or rapid, repeated neurotransmission, there must be a mechanism to limit its duration of action. Degradation of ACh is essential not only to prevent unwanted activation of neighboring neurons or muscle cells but also to ensure proper timing o signaling at the postsynaptic cell. A single receptor molecule is typically capable of distinguishing between two sequential presynaptic release events because degradation o ACh in the synaptic cleft occurs aster than the time course o nAChR activation. Enzymes collectively known as cholinesterases are responsible or degrading acetylcholine. The two types o cholinesterase, AChE and butyrylcholinesterase ( BuChE , also known as pseudocholinesterase or nonspecif c cholinesterase), are distributed widely throughout the body. AChE is indispensable or the degradation o ACh and is capable of hydrolyzing about 4 x 10^ 5 molecules of ACh per enzyme molecule per minute; its turnover time of 150 micosec makes it one o the most e f cient hydrolytic enzymes known. AChE is concentrated on the postsynaptic membrane, and the choline that it rees there is efficiently transported back into the presynaptic terminal. BuChE has a secondary role in ACh degradation; the enzyme can hydrolyze ACh but at rates much slower than that o AChE. Evidence suggests that BuChE may be involved in early neural development as a co-regulator o ACh and may also be involved in the pathogenesis o Alzheimer's disease. Because o its central importance to cholinergic transmission, a class o drugs known as acetylcholinesterase inhibitors has been designed to target AChE.
LEMS - diagnosis
In paraneoplastic LEMS the prognosis is determined by the underlying cancer. The presence of LEMS in patients with small cell lung cancer (SCLC) is associated with longer survival from the malignancy. Nonparaneoplastic LEMS, when optimally treated, has an excellent prognosis and normal life expectancy, although patients may continue to experience various degrees of muscle weakness.
cellular dysfunction
In some cases, antibodies directed against cell surface receptors impair or dysregulate function without causing cell injury or inflammation (Fig. 6-16C). For example, in myasthenia gravis, antibodies reactive with acetylcholine receptors in the motor end plates of skeletal muscles block neuromuscular transmission and therefore cause muscle weakness. The converse (i.e., antibody-mediated stimulation of cell function) is the basis of Graves disease. In this disorder, antibodies against the thyroid-stimulating hormone receptor on thyroid epithelial cells stimulate the cells, resulting in hyperthyroidism.
Pathology
In the intestinal lumen, dietary triglycerides and cholesterol esters are hydrolyzed by pancreatic lipase to produce glycerol, FFA, and free cholesterol. Formation of micelles enables the absorption of glycerol and FFA into the intestinal cell. The transport of free cholesterol is mediated by a cholesterol gradient that exists between the lumen and the intestinal cell. Within the cell, glycerol combines with three fatty acid chains to form triglycerides, and cholesterol is esterified to form cholesterol esters. Chylomicrons are formed from triglycerides (85% of chylomicron mass) and cholesterol esters assembled with surface lipoproteins. Chylomicrons enter into the circulation and acquire more surface apolipoproteins such as apo C-II and apo E from high-density lipoprotein (HDL) particles (Fig. 69-1). Apo C-II activates lipoprotein lipase (LPL), which is located on the capillary endothelium. LPL hydrolyzes the core chylomicron triglycerides to release FFA, which function as an energy source. Excess fatty acids are stored in adipose tissue or utilized in hepatic lipoprotein synthesis. The triglyceride-poor chylomicron remnant is then cleared from the circulation by hepatic LDL receptors. These receptors are activated by apo E, which is located on the surface of chylomicrons. Very-low-density lipoproteins (VLDL) are synthesized by the liver (see Fig. 69-1). FFA and cholesterol obtained from the circulation or synthesized by the liver are incorporated into VLDL particles. Any condition that increases the flux of FFA to the liver, such as poorly controlled diabetes, will increase VLDL production. The liver assembles triglycerides (55% of VLDL mass), cholesterol (20%), and surface apolipoproteins to form VLDL particles. Apo C-II, the cofactor for LPL, hydrolyzes the triglyceride core of VLDL particles to generate VLDL remnant or intermediate-density lipoprotein (IDL). The IDL, depleted of triglycerides (25%), can be cleared from the circulation by apo E-mediated LDL receptors, or it can be hydrolyzed further to form low-density lipoproteins (LDL). LDL particles are triglyceride poor (5% of LDL mass) and consist mostly of cholesterol esters (60%) and apolipoproteins. Apo B100 on the surface of LDL binds LDL receptors and facilitates LDL clearance from the circulation. Internalized LDL-cholesterol is used to synthesize hormones, produce cell membranes, and store energy In the liver, LDL-cholesterol is used to synthesize bile acids (see Fig. 69-1), which are secreted into the intestinal lumen along with free cholesterol. Bile acids help transport fat. Approximately 50% of the cholesterol and 97% of the bile acid entering the lumen is reabsorbed back into the circulation. The reabsorbed cholesterol regulates cholesterol and LDL receptor synthesis Many cells in the body, including liver parenchymal cells, synthesize cholesterol (Fig. 69-2). Acetate is converted to 3-hydroxy3-methylglutaryl-coenzyme A (HMG-CoA). HMG-CoA reductase converts HMG-CoA to mevalonic acid, which is then converted to cholesterol through a series of steps. HMG-CoA reductase is the rate-limiting step in the cholesterol synthesis pathway. Drugs that inhibit this enzyme decrease cholesterol biosynthesis and cellular cholesterol pools. Internalization of LDL particles into cells is regulated by negative feedback (see Fig. 69-2). A negative cholesterol balance increases the expression of LDL receptors and subsequent uptake of cholesterol from the circulation. A positive cell cholesterol balance suppresses LDL receptor expression and decreases uptake of LDL-cholesterol into cells. Circulating LDL then enter macrophages and other tissues via scavenger receptors. Because the scavenger receptors are not regulated, these cells accumulate excess intracellular cholesterol, resulting in the formation of foam cells and atheromatous plaques The anti-atherogenic effect of HDL is attributed to the removal of excess cholesterol from tissue sites and other lipoproteins. HDL is synthesized in the liver and intestine (see Fig. 69-1). Excess phospholipids, cholesterol, and apolipoproteins on remnant chylomicrons, VLDL, IDL, and LDL, are transferred to HDL particles and thus increase HDL mass. Apo A-I, a surface lipoprotein on HDL particles, mobilizes cholesterol from intracellular pools and accepts cholesterol released during lipolysis of triglyceride-rich lipoproteins. It also activates lecithin-cholesterol acyltransferase (LCAT), an enzyme that esterifies cholesterol. These cholesterol esters move the hydrophilic HDL surface to the hydrophobic HDL core. Cholesterol ester transfer protein (CETP) transfers core HDL cholesterol esters to other lipoproteins such as VLDL. These lipoproteins deliver cholesterol to peripheral sites for hormone and cell membrane synthesis Defects in the production or removal of lipoproteins results in dyslipidemia. Both genetic and acquired conditions have been implicated in the pathogenesis of lipid disorders (Tables 69-2 and 61-3). These are discussed later in the chapter.
Familial Defective Apolipoprotein B100
In this autosomal dominant disorder, a defect in the apo B100 protein results in impaired binding of LDL particles to the LDL receptor. The disorder affects as many as 1 in 750 Caucasians with hypercholesterolemia. The clinical presentation is similar to familial hypercholesterolemia, with elevated total cholesterol and LDL-cholesterol levels associated with premature CHD and tendon xanthomas. However, the homozygous and heterozygous clinical forms of familial defective apo B100 are milder than familial hypercholesterolemia, because apo E-mediated clearance of remnant particles is still functional. Total cholesterol concentration ranges from 350 to 550 mg/dL in the homozygous and 200 to 350 mg/dL in the heterozygous disorder. DNA analysis can identify the apo B100 gene mutation and confirm the diagnosis, but genetic diagnosis is not necessary to initiate therapy. A low-cholesterol, low-fat diet in combination with a statin, bile acid resin, and/or niacin is recommended to lower cholesterol levels to target ranges.
CD8 t cell mediated cytotoxicity
In this type of T cell-mediated reaction, CD8+ CTLs kill antigen-expressing target cells. Tissue destruction by CTLs may be an important component of some T cell-mediated diseases, such as type 1 diabetes. CTLs directed against cell surface histocompatibility antigens play an important role in graft rejection, to be discussed later. They also play a role in reactions against viruses. In a virus-infected cell, viral peptides are displayed by class I MHC molecules and the complex is recognized by the TCR of CD8+ T lymphocytes. The killing of infected cells leads to the elimination of the infection, but in some cases it is responsible for cell damage that accompanies the infection (e.g., in viral hepatitis). Tumor-associated antigens are also presented on the cell surface, and CTLs are involved in the host response to transformed cells (Chapter 7). The principal mechanism of T cell-mediated killing of targets involves perforins and granzymes, preformed mediators contained in the lysosome-like granules of CTLs. CTLs that recognize the target cells secrete a complex consisting of perforin, granzymes, and other proteins which enters target cells by endocytosis. In the target cell cytoplasm, perforin facilitates the release of the granzymes from the complex. Granzymes are proteases that cleave and activate caspases, which induce apoptosis of the target cells (Chapter 2). Activated CTLs also express Fas ligand, a molecule with homology to TNF, which can bind to Fas expressed on target cells and trigger apoptosis. CD8+ T cells also produce cytokines, notably IFN-γ, and are involved in inflammatory reactions resembling DTH, especially following virus infections and exposure to some contact sensitizing agents.
Influence of fibrates on lipid metabolism
In uence o f brates on lipid metabolism. Fibrates have several benef cial e ects on lipid metabolism, all o which appear to be secondary to the activation o the transcription actor PPAR . PPAR activation by f brates increases hepatic synthesis o apoAI and apoAII, which leads to increased plasma HDL-cholesterol concentrations. PPAR activation also down-regulates hepatic synthesis o apoCIII and increases lipoprotein lipase expression in muscle vascular beds. The decrease in apoCIII, an inhibitor o lipoprotein lipase, combines with the increase in lipoprotein lipase expression to increase atty acid uptake and atty acid oxidation in muscle cells. PPAR also increases atty acid oxidation in hepatocytes. The combined e ects o these metabolic changes are decreased plasma triglyceride concentrations and increased plasma HDL cholesterol. Because o decreased hepatic atty acid and triglyceride synthesis (not shown), LDLcholesterol concentrations also decrease modestly
What do cholesterol absorption inhibitors increase utilization of and decrease levels of?
Increase cholesterol utilization Decrease LDL-cholesterol levels Despite reductions in LDL-cholesterol, improvements in cardiovascular events and mortality have not been reported with treatment
What is the main effect of AChE inhibitor on the Gastrointestinal (GI) system? What AChE is used for relief of abdominal distention?
Increase in smooth muscle motility due to enhancement of ganglionic transmission at Auerbach's plexus eg: Neostigmine is used for relief of abdominal distention
What effect do Chemotactic agents C3a and C5a have?
Increase vascular permeability
Influence of Niacin on Lipid metabolism
Inf uence o niacin on lipid metabolism. Niacin lowers triglyceride and LDL levels while increasing HDL. Activation by niacin o a G proteincoupled receptor on adipocytes results in decreased hormone-sensitive lipase activity in adipose tissue, which decreases the f ux o ree atty acids to the liver. The decreased ree atty acid f ux reduces hepatic triglyceride synthesis and limits VLDL synthesis. Because LDL is derived rom VLDL, the decreased VLDL synthesis decreases plasma concentrations o LDL cholesterol. Niacin also increases the hal -li e o apoAI, an important apolipoprotein in HDL. The increased apoAI levels directly increase levels o plasma HDL and may also augment reverse cholesterol transport, delivery o cholesterol rom HDL to the liver, and excretion o cholesterol in the bile.
What is the resultant injury due to when antibodies deposit in fixed tissues such as basement membranes and extracellular matrix?
Inflammation
Intravenous metabolism of apoB containing lipoproteins
Intravascular metabolism of apoB-containing lipoproteins. After secretion, chylomicrons and VLDL particles are activated for lipolysis when they encounter HDL particles in the plasma and acquire the exchangeable apolipoprotein apoCII. When chylomicrons and VLDL circulate into capillaries of muscle or adipose tissue, apoCII promotes binding of the particle to lipoprotein lipase, which is bound to the surface of endothelial cells. Lipoprotein lipase mediates hydrolysis of triglycerides, but not cholesteryl esters, from the core of the lipoprotein particle. The resulting fatty acids are taken up into muscle or adipose tissue.
hypercholesterolemia
Isolated hypercholesterolemia is characterized by elevated levels o total plasma cholesterol and LDL cholesterol, with normal concentrations o triglycerides. The causes o primary hypercholesterolemia are amilial hypercholesterolemia, amilial de ective apoB100, gain-o - unction mutations in PCSK9, amilial combined hyperlipidemia (FCHL), and, most commonly, polygenic hypercholesterolemia Familial hypercholesterolemia (FH) is an autosomal dominant disease involving de ects in the LDL receptor. Mutations in the gene encoding the LDL receptor result in one o our molecular de ects: lack o receptor synthesis, ailure to reach the plasma membrane, de ective LDL binding, and ailure to internalize bound LDL particles. Heterozygous individuals (1 in 500 in the United States) have elevated total plasma cholesterol concentrations rom birth throughout li e, with adult levels averaging 275-500 mg/dL (normal, 200 mg/dL). Clinical eatures include tendon xanthomas (caused by intracellular and extracellular accumulation o cholesterol) and arcus corneae (deposition o cholesterol in the cornea). Homozygous FH is a much more severe but rare disorder (1 in 1 million in the United States) that is characterized by the absence o unctional LDL receptors. This leads to very high plasma cholesterol concentrations (700-1,200 mg/dL) and cardiovascular disease that presents clinically prior to the age o 20. Heterozygotes or FH respond well to statins and other LDL-lowering drugs that up-regulate LDL receptor density on the cell sur ace. In the introductory case, Jake was most likely heterozygous or FH. Because homozygotes lack unctional LDL receptors, until recently, the only e ective treatment has been plasmapheresis with immunoadsorption o LDL particles. However, the development o molecules that inhibit PCSK9, MTP, or apoB synthesis now show promise toward a complementary treatment to reduce the severe elevations in LDL-C observed in patients with homozygous FH. An autosomal recessive orm o hypercholesterolemia has also been described in which a de ective molecular adaptor protein that participates in LDL receptor internalization leads to a phenotype similar to that o FH. Familial defective apoB100 is an autosomal dominant disorder in which mutations in the apoB100 protein lead to decreased a f nity o the LDL particle or LDL receptors. Due to decreased catabolism o LDL, cholesterol concentrations in amilial de ective apoB100 can be similar to those in patients with FH. Gain-o - unction mutations in PCSK9 have been identif ed in amilies with clinical eatures similar to FH; the pathophysiology o this disorder re ects increased PCSK9 unction and decreased LDL receptor expression on cell sur aces. Familial combined hyperlipidemia is characterized by di erent combinations o hyperlipidemia in di erent amilies (see below); one presentation is elevated LDL cholesterol. Polygenic hypercholesterolemia is a general term that has been used to categorize the majority o patients with hypercholesterolemia who have no def ned genetic cause or the disorder. Polygenic hypercholesterolemia may be the result o complex gene-environment interactions, multiple uncharacterized genetic susceptibilities, or variant LDL particles such as small dense LDL and lipoprotein(a) [Lp(a)]. Further research into genetic predispositions or hypercholesterolemia will be necessary in order to identi y clear etiologies or the majority o patients with hypercholesterolemia.
What is the epidemiology of Colorectal cancer?
It is estimated that 1 in 20 Americans will develop colorectal cancer, and roughly one third of those with this cancer will die from it. With approximately 135,000 new cases and 50,000 deaths in the United States each year, colorectal cancer is second only to lung cancer in the total number of annual cancer deaths. The risk of colorectal cancer in people with one affected first-degree relative is two to three times higher than that of the general population. As with breast cancer, most colorectal cancer cases (>90%) are not inherited as mendelian conditions and are likely to be caused by a complex interaction of inherited and somatic genetic alterations and environmental factors. The latter risk factors include a lack of physical activity and a high-fat, lowfiber diet.
multifactorial versus single gene inheritance
It is important to clarify the difference between a multifactorial disease and a single-gene disease in which there is locus heterogeneity. In the former case, a disease is caused by the simultaneous influence of multiple genetic and environmental factors, each of which has a relatively small effect. In contrast, a disease with locus heterogeneity, such as osteogenesis imperfecta, requires only a single mutation to cause it. Because of locus heterogeneity, a single mutation at either of two or more loci can cause disease; some affected persons have one mutation while others have the other mutation In some cases, a trait may be influenced by the combination of both a single gene with large effects and a multifactorial background in which additional genes and environmental factors have small individual effects (Fig. 12-4). Imagine that variation in height, for example, is caused by a single locus (termed a major gene) and a multifactorial component. Individuals with the AA genotype tend to be taller, those with the aa genotype tend to be shorter, and those with Aa tend to be intermediate. But additional variation is caused by other factors (the multifactorial component). Thus, those with the aa genotype vary in height from 130 cm to about 170 cm, those with the Aa genotype vary from 150 cm to 190 cm, and those with the AA genotype vary from 170 to 210 cm. There is substantial overlap among the three major genotypes because of the influence of the multifactorial background. The total distribution of height, which is bell-shaped, is caused by the superposition of the three distributions about each genotype. Many of the diseases to be discussed later can be caused by a major gene and/or multifactorial inheritance. That is, there are subsets of the population in which diseases such as colon cancer, breast cancer, or heart disease are inherited as single-gene disorders (with additional variation in disease susceptibility contributed by other genetic and environmental factors). These subsets usually account for only a small percentage of the total number of disease cases. It is nevertheless important to identify the responsible major genes, because their function can provide important clues to the pathophysiology and treatment of the disease. Multifactorial diseases can be distinguished from singlegene disorders caused by mutations at different loci (locus heterogeneity). Sometimes a disease has both singlegene and multifactorial components.
What 2 ions is nAChR equally permeable to?
K+ and Na+ also has a small permeability to Calcium channels when open, these channels produce a net inward current that depolarizes the postsynaptic cell. Stimulation o multiple nAChRs may depolarize the cell sufficiently to generate action potentials and to open voltage-dependent calcium channel
key concepts
KEY CONCEPTS Pathogenesis of Diseases Caused by Antibodies and Immune Complexes ■ Antibodies can coat (opsonize) cells, with or without complement proteins, and target these cells for phagocytosis by phagocytes (macrophages), which express receptors for the Fc tails of lgG and for complement proteins. The result is depletion of the opsonized cells. ■ Antibodies and immune complexes may deposit in tissues or blood vessels, and elicit an acute inflammatory reaction by activating complement, with release of breakdown products, or by engaging Fc receptors of leukocytes. The inflammatory reaction causes tissue injury. ■ Antibodies can bind to cell surface receptors or other essential molecules and cause functional derangements (either inhibition or unregulated activation) without cell injury.
MOA and Use:bethanechol
MOA: Cholinomimetic; stimulate muscarinic receptor activity. USE: treat urinary retention, & postoperative abdominal distention, gastic atony, gastoparesis, adynamic ileus
Key concepts
KEY CONCEPTS Transmission Patterns of Single-Gene Disorders ■ Autosomal dominant disorders are characterized by expression in heterozygous state; they affect males and females equally, and both sexes can transmit the disorder. ■ Enzyme proteins are not affected in autosomal dominant disorders; instead, receptors and structural proteins are involved. ■ Autosomal recessive diseases occur when both copies of a gene are mutated; enzyme proteins are frequently involved. Males and females are affected equally. ■ X-linked disorders are transmitted by heterozygous females to their sons, who manifest the disease. Female carriers usually are protected because of random inactivation of one X chromosome
What do fibrates also lower levels of (modestly)?
LDL The lower LDL levels result from a PPAR-alpha- induced shift in hepatocyte metabolism toward f atty acid oxidation
Approximately 7% of the body's cholesterol circulates in the plasma, predominantly in what form?
LDL the amount of plasma cholesterol is influenced by its synthesis and catabolism, and the liver plays a crucial role in both these processes
The leading cause o death in the United States and most Western countries, can be attributed to elevated blood concentrations of?
LDL (low density lipoprotein) particles as well as lipoproteins that are rich in triglycerides decreased concentrations of high-density lipoproteins (HDL) also predispose to atherosclerotic disease.
What is the only receptor capable of clearing significant amounts of LDL from the plasma?
LDL receptor
What does Apo B100 (on the surface of LDL) bind to facilitate clearance of from circulation?
LDL receptors LDL
LEMS- clinical presentation
LEMS should be suspected whenever the triad of muscle weakness, dry mouth, and decreased or absent reflexes is present. Patients have fluctuating weakness and fatigability of proximal limb and trunk muscles, with the lower limbs more severely affected than the upper ones. Difficulty walking is a common symptom. Dysphagia, dysarthria, and ocular symptoms (ptosis, blurred vision, and diplopia) are less common than in MG. Tendon reflexes are hypoactive or absent and may increase following short exercise of the muscle. Autonomic manifestations (dry mouth, impotence, decreased sweating, orthostatic hypotension, and slow pupillary reflexes) occur in 75% of patients.
What does germline mutations in TP53 and CHK2 cause? What does it predispose to?
Li-Fraumeni syndrome Breast cancer
What should be the initial step in the management of hyperlipidemia?
Lifestyle modification Restricting the dietary intake of fat lowers total cholesterol by approximately 15% and LDL cholesterol by 25%
Lifestyle modification
Lifestyle modification should be the initial step in the management of hyperlipidemia (see Table 69-6). Restricting the dietary intake of fat lowers total cholesterol by approximately 15% and LDL cholesterol by 25%. Low-fat diets that limit saturated fat content promote LDL receptor expression and increase the uptake of LDL-cholesterol from the circulation. By contrast, saturated fat downregulates hepatic LDL receptors and increases circulating LDL. Because unsaturated fats (polyunsaturated and monounsaturated) generally do not have this effect, they are the preferred form of fat intake. However, polyunsaturated fats containing fatty acids with a trans rather than cis double bond configuration (trans-fatty acids) increase plasma cholesterol levels similarly to saturated fat Limiting the intake of saturated and trans-unsaturated fatty acids requires appropriate calorie substitutions. Increasing carbohydrate content to achieve this goal can increase the hepatic synthesis of triglyceride. Dietary substitution with soluble fibers (e.g., oat bran) has been recommended, because these fibers have a limited effect on triglyceride levels. They also bind bile acids in the gut and thereby decrease cholesterol levels. Other polyunsaturated fats, such as omega-3 fatty acids, are cardioprotective. They are abundant in fatty fish, flaxseed oil, canola oil, and nuts. They reduce VLDL production, inhibit platelet aggregation, and decrease CHD. Even two servings per week of fatty fish such as salmon can be beneficial. Dietary restriction of fat (<10%) is essential for the treatment of marked hypertriglyceridemia. Other factors such as carbohydrate and alcohol intake can also increase the synthesis of triglyceride. Restriction of alcohol intake to 1 or 2 servings per week and adherence to a low-fat, high-fiber diet will improve hypertriglyceridemia. Exercise has been shown to increase LPL activity. Even a single exercise session can reduce triglycerides and increase HDL. The impact of exercise on LDL is less clear. With low- to moderateintensity exercise regimens, clearance of VLDL particles increases LDL production. However, this effect is not seen with highintensity exercise programs. A decrease in LDL-cholesterol occurs with high-intensity exercise, and this effect is independent of weight loss.
What is the Arthus reaction?
Localized area of tissue necrosis resulting from acute immune complex vasculitis, usually elicited in the skin The reaction can be produced experimentally by intracutaneous injection of antigen in a previously immunized animal that contains circulating antibodies against the antigen. As the antigen diffuses into the vascular wall, it binds the preformed antibody, and large immune complexes are formed locally. These complexes precipitate in the vessel walls and cause fibrinoid necrosis, and superimposed thrombosis worsens the ischemic injury.
diabetes mellitus
Like the other disorders discussed in this chapter, the etiology of diabetes mellitus is complex and not fully understood. Nevertheless, progress is being made in understanding the genetic basis of this disorder, which is the leading cause of adult blindness, kidney failure, and lower-limb amputation and a major cause of heart disease and stroke. An important advance has been the recognition that diabetes mellitus is actually a heterogeneous group of disorders, all characterized by elevated blood sugar. We focus here on the three major types of diabetes, type 1 (formerly termed insulin-dependent diabetes mellitus, or IDDM), type 2 (formerly termed non- insulin-dependent diabetes mellitus, or NIDDM), and maturity-onset diabetes of the young (MODY).
What is the goal of therapies for antibody-mediated diseases? What drugs are used? What treatment can be used in severe cases?
Limit inflammation and its injurious consequences with drugs like: Corticosteroids In severe cases Plasmapheresis is used to reduce levels of circulating antibodies or immune complexes
Are T cell mediated autoimmune diseases limited to a few organs or systemic?
Limited to a few organs are not systemic
Intro
Lipids are insoluble or sparingly soluble molecules that are essential or membrane biogenesis and maintenance o membrane integrity. They also serve as energy sources, hormone precursors, and signaling molecules. In order to acilitate transport through the relatively aqueous environment o the blood, nonpolar lipids, such as cholesteryl esters and triglycerides, are packaged within lipoproteins. Increased concentrations o certain lipoproteins in the circulation are associated strongly with atherosclerosis. Much o the prevalence o cardiovascular disease (CVD), the leading cause o death in the United States and most Western countries, can be attributed to elevated blood concentrations o cholesterol-rich low-density lipoprotein (LDL) particles as well as lipoproteins that are rich in triglycerides. Epidemiologically, decreased concentrations o high-density lipoproteins (HDL) also predispose to atherosclerotic disease. The major contributors to lipoprotein abnormalities appear to be Western diets combined with sedentary li estyles, but a limited number o genetic causes o hyperlipidemia have also been identif ed. The role o genetics in common orms o hyperlipidemia is the subject o intense study utilizing cutting-edge genomic approaches. It is apparent that genes modi y both the sensitivity o individuals to adverse dietary habits and li estyles and the response o individuals to lipid-lowering therapies This chapter highlights the biochemistry and physiology o cholesterol and lipoproteins, with an emphasis on the role o lipoproteins in atherogenesis, and the pharmacologic interventions that can ameliorate hyperlipidemia. Abundant clinical outcomes data have proven that morbidity and mortality rom cardiovascular disease can be reduced by the use o lipid-lowering drugs.
Definition of Epidemiology
Lipids such as free fatty acids (FFA), cholesterol, and triglycerides are hydrophobic molecules that bind proteins for transport. Nonesterified FFA travel as anions complexed to albumin. Esterified complex lipids are transported in lipoprotein particles. Lipoproteins have a hydrophobic core (cholesteryl esters and triglycerides) and an amphiphilic surface monolayer (phospholipids, unesterified cholesterol, and apolipoproteins). Ultracentrifugation separates lipoproteins into five classes based on their density (Table 69-1). Proteins on the surface of lipoproteins (i.e., apolipoproteins) activate enzymes and receptors that guide lipid metabolism. Defects in the synthesis and catabolism of lipoproteins result in dyslipidemia. Prevalence of dyslipidemia in the United States is approximately 20% and varies with the population studied. An estimated 70% of individuals with premature coronary heart disease (CHD) have dyslipidemia. In clinical trials, treatment of dyslipidemia improved both CHD and all-cause mortality rates. Two classes of lipids, triglyceride and cholesterol, play a significant, yet modifiable, role in the pathogenesis of atherosclerosis and therefore are the focus of this chapter.
What does Apo C-II activate? What role does the activated product play?
Lipoprotein lipase (LPL)-located in capillary endothelium LPL hydrolyzes the core chylomicron triglycerides to release FFA, which function as an energy source
In order to be transported through the blood what are nonpolar lipids packaged within?
Lipoproteins eg: of nonpolar lipids- cholesterol esters and triglycerides
biochem and physiology of cholesterol and lipoprotein metabolism
Lipoproteins are macromolecular aggregates that transport triglycerides and cholesterol in the blood. Circulating lipoproteins can be di erentiated on the basis o density, size, and protein content (Table 20-1). As a general rule, larger, less dense lipoproteins have a higher percentage composition o lipids; chylomicrons are the largest and least dense lipoprotein subclass, whereas HDLs are the smallest lipoproteins, containing the lowest lipid content and the highest proportion o protein Structurally, lipoproteins are microscopic spherical particles ranging rom 5 to 1,000 nm in diameter. Each lipoprotein particle consists o a monolayer o polar, amphipathic lipids that surrounds a hydrophobic core. Each lipoprotein particle also contains one or more types o apolipoprotein (Fig. 20-1). The polar lipids that comprise the sur ace coat are unesterif ed cholesterol and phospholipid molecules arranged in a monolayer. The hydrophobic core o a lipoprotein contains cholesteryl esters (cholesterol molecules linked by an ester bond to a atty acid) and triglycerides (three atty acids esterif ed to a glycerol molecule). Apolipoproteins (also re erred to as apoproteins ) are amphipathic proteins that intercalate into the sur ace coat o lipoproteins. In addition to stabilizing the structure o lipoproteins, apolipoproteins engage in biological unctions. They may act as ligands or lipoprotein receptors or may activate enzymatic activities in the plasma. The apolipoprotein composition determines the metabolic ate o the lipoprotein. For example, each LDL particle contains one apolipoprotein B (apoB) 100 molecule, which is a ligand or the low-density lipoprotein receptor (discussed below); in turn, binding o LDL to the LDL receptor promotes cholesterol uptake into cells. From a metabolic perspective, lipoprotein particles can be divided into lipoproteins that participate in the delivery o triglyceride molecules to muscle and at tissue (the apoB-containing lipoproteins, chylomicrons, and VLDL and lipoproteins that are involved primarily in cholesterol transport ( HDL and the remnants o apoB-containing lipoproteins). HDL also serves as a reservoir or exchangeable apolipoproteins in the plasma, including apoAI, apoCII, and apoE. The ollowing discussion presents each lipoprotein class in the context o its unction
What is the only organ in the body that has the ability to eliminate cholesterol?
Liver
Where does HDL formation mainly occur?
Liver
Appx 70% of plasma is cleared by?
Liver Eventhough Although many cell types, including fibroblasts, lymphocytes, smooth muscle cells, hepatocytes, and adrenocortical cells, possess high-affinity LDL receptors
What are Very low density lipoproteins (VLDL) synthesized by?
Liver Free fatty acid (FFA) and cholesterol obtained from the circulation or synthesized by the liver are incorporated into VLDL particles Any condition that increases the flux of FFA to the liver, such as poorly controlled diabetes, will increase VLDL production
What type of disease is Arthus reaction? What does it result in the formation of?
Localized immune complex reaction Immune complexes at the site of antigen injection and a local vasculitis induced by subcutaneous administration of a protein antigen to a previously immunized animal a small percentage of vaccine recipients who have previously been vaccinated or already have antibodies against the vaccine antigen, a painful swelling that develops at the injection site represents a clinically relevant Arthus reaction.
What drug is approved for use in patients with Homozygous familial hypercholesterolemia (HoFH)? Adverse effects?
Lomitapide At therapeutic doses, lomitapide reduces LDL-C by 30-50% in these patients. Gastrointestinal distress due to at malabsorption, reductions in plasma vitamin E levels, and transaminase elevations that are correlated with increased hepatic at content
What are the preferred drugs of choice for treatment of chronic Myasthenia gravis?
Longer acting AChE inhibitors: Pyridostigmine, Neostigmine and Ambenonium
What occurs as a result of aortic lesions in Marfan syndrome?
Loss of connective tissue support in the mitral valve leaflets makes them soft and billowy, creating a so-called floppy valve great majority of deaths are caused by rupture of aortic dissections, followed in importance by cardiac failure.
Figure 5.6
Low-density lipoprotein (LDL) metabolism and the role of the liver in its synthesis and clearance. Lipolysis of very-low-density lipoprotein (VLDL) by lipoprotein lipase in the capillaries releases triglycerides, which are then stored in fat cells and used as a source of energy in skeletal muscles. See text for explanation of abbreviations used.
What is the primary goal for the therapy to treat Elevated Lp(a)? What drugs are used?
Lower LDL levels with agents such as Statins If LDL goals cannot be achieved, then Lp(a)-lowering therapy with niacin and neomycin may be considered.
Elevated Plasma lipoprotein (a)
Lp(a) is a specialized form of LDL that is assembled extracellularly from apolipoprotein(a) and LDL. Lp(a), when present at elevated levels, interferes with fibrinolysis by competing with plasminogen. This leads to decreased thrombolysis and increased clot formation. Lp(a) also binds macrophages, promoting foam cell formation and atherosclerotic plaques. Screening should be considered in individuals who have a family or personal history of premature CHD without dyslipidemia and in those for whom cholesterol-lowering therapy has failed. The diagnosis can be made by documenting Lp(a) levels higher than 30 mg/dL in a patient with premature CHD. The primary goal of therapy is to lower LDL levels with agents such as statins. If LDL goals cannot be achieved, then Lp(a)-lowering therapy with niacin and neomycin may be considered.
What 2 muscarinic receptors are coupled to Gi (adenylyl cyclase inhibition and K+ channel activation)?
M2 and M4 Gi --> even numbers
Examples of Mendelian subtypes of Complex disorders
MENDELIAN SUBTYPE PROTEIN (GENE) CONSEQUENCE OF MUTATION Heart Disease Familial hypercholesterolemia LDL receptor (LDLR) Elevated LDL level Apolipoprotein B (APOB) Elevated LDL level Proprotein convertase subtilisin/kexin type 9 (PCSK9) Gain-of-function mutations elevate LDL; loss-of-function mutations reduce LDL Tangier disease ATP-binding cassette 1 (ABC1) Reduced HDL level Familial dilated cardiomyopathy Cardiac troponin T (TNNT2) Reduced force generation by sarcomere Titin (TTN) Destabilized sarcomere Cardiac β-myosin heavy chain (MYH7) Reduced force generation by sarcomere β-Sarcoglycan (SGCB) Destabilized sarcolemma and signal transduction δ-Sarcoglycan (SGCD) Destabilized sarcolemma and signal transduction Dystrophin Destabilized sarcolemma in cardiac myocytes Familial hypertrophic cardiomyopathy Cardiac β-myosin heavy chain (MYH7) Reduced force generation by sarcomere Cardiac troponin T (TNNT2) Reduced force generation by sarcomere Myosin-binding protein C (MYBPC) Sarcomere damage Long QT syndrome Cardiac potassium channel α subunit (LQT1, KCNQ1) Prolonged QT interval on electrocardiogram, arrhythmia Cardiac potassium channel α subunit (LQT2, KCNH2) Prolonged QT interval on electrocardiogram, arrhythmia Cardiac sodium channel (LQT3, SCN5A) Prolonged QT interval on electrocardiogram, arrhythmia Ankyrin B anchoring protein (LQT4, ANK2) Prolonged QT interval on electrocardiogram, arrhythmia Cardiac potassium channel β subunit (LQT5, KCNE1) Prolonged QT interval on electrocardiogram, arrhythmia Cardiac potassium channel subunit (LQT6, KCNE2) Prolonged QT interval on electrocardiogram, arrhythmia Hypertension Liddle syndrome Renal epithelial sodium channel subunits (SCNN1B, SCNN1G) Severe hypertension, low renin and suppressed aldosterone Gordon syndrome WNK1 or WNK4 kinase genes High serum potassium level and increased renal salt reabsorption Glucocorticoid-remediable aldosteronism Fusion of genes that encode aldosterone synthase and steroid 11β-hydroxylase Early-onset hypertension with suppressed plasma renin and normal or elevated aldosterone levels Syndrome of apparent mineralocorticoid excess 11 β-Hydroxysteroid dehydrogenase (11β-HSD2) Early-onset hypertension, low potassium and renin levels, low aldosterone Diabetes MODY1 Hepatocyte nuclear factor-4α (HNF4A) Decreased insulin secretion MODY2 Glucokinase (GCK) Impaired glucose metabolism, leading to mild nonprogressive hyperglycemia MODY3 Hepatocyte nuclear factor-1α (HNF1α) Decreased insulin secretion MODY4 Insulin promoter factor-1 (IPF1) Decreased transcription of insulin gene MODY5 Hepatocyte nuclear factor-1β (HNF1β) β-cell dysfunction leads to decreased insulin secretion MODY6 NeuroD transcription factor (NEUROD1) Decreased insulin secretion Alzheimer Disease Familial Alzheimer disease Amyloid-β precursor protein (APP) Alteration of cleavage sites in amyloid-β precursor protein, producing longer amyloid fragments Presenilin 1 (PS1) Altered cleavage of amyloid-β precursor protein, producing larger proportion of long amyloid fragments Presenilin 2 (PS2) Altered cleavage of amyloid-β precursor protein, producing larger proportion of long amyloid fragments
Examples of Mendelian subtypes of Complex disorders- cont'd
MENDELIAN SUBTYPE PROTEIN (GENE) CONSEQUENCE OF MUTATION Parkinson Disease Familial Parkinson disease (autosomal dominant) α-Synuclein (PARK1, SNCA) Formation of α-synuclein aggregates Familial Parkinson disease (autosomal recessive) Parkin: E3 ubiquityl ligase, thought to ubiquinate α-synuclein (PARK2) Compromised degradation of α-synuclein Familial Parkinson disease (autosomal dominant) Ubiquitin C-hydrolase-L1 (PARK5) Accumulation of α-synuclein Amyotrophic Lateral Sclerosis Familial amyotrophic lateral sclerosis Superoxide dismutase 1 (SOD1) Neurotoxic gain of function Familial amyotrophic lateral sclerosis C9ORF72 Expanded repeat mutation Juvenile amyotrophic lateral sclerosis (autosomal recessive) Alsin (ALS2) Presumed loss of function Epilepsy Benign neonatal epilepsy, types 1 and 2 Voltage-gated potassium channels (KCNQ2 and KCNQ3, respectively) Reduced M current increases neuronal excitability Generalized epilepsy with febrile seizures plus type 1 Sodium channel β1 subunit (SCN1B) Sodium current persistence leading to neuronal hyperexcitability Autosomal dominant nocturnal frontal lobe epilepsy Neuronal nicotinic acetylcholine receptor subunits (CHRNA4 and CHRNB2) Increased neuronal excitability in response to cholinergic stimulation Generalized epilepsy with febrile seizures plus type 3 GABAA receptor (GABRG2) Loss of synaptic inhibition leading to neuronal excitability
MG- Clinical presentation
MG is characterized by fluctuating, fatigable weakness either isolated to the ocular muscles (ocular MG), or involving ocular as well as limb, bulbar, and respiratory muscles (generalized MG). The majority of patients present first with ocular symptoms (blurred vision, double vision, droopy eyelids), but about 15% of cases present with bulbar symptoms first (dysarthria, dysphagia, shortness of breath), or limb weakness. Ptosis is usually asymmetric. Myasthenia crisis is a true neurological emergency that occurs in 15% to 20% of patients and consists of severe dysphagia or respiratory failure requiring ventilator support and/or tube feeding in an ICU setting.
MOA and Use: pyridostigmine
MOA: AchE inhibitor. USE:Reversal of nm blockade, Myasthenia gravis Drug of choice, prophylaxis for organophosphate poisoning, glaucoma, urinary or GI motility agent
MOA and USE: pitavastatin
MOA: Competitively inhibit HMG-CoA reductase (rate limiting step in cholesterol synthesis). USE: hypercholesterolemia, familial hypercholesterolemia, coronary atherosclerosis, prophylaxis for coronary atherosclerosis
MOA and USE: rusovastatin
MOA: Competitively inhibit HMG-CoA reductase (rate limiting step in cholesterol synthesis). USE: hypercholesterolemia, familial hypercholesterolemia, coronary atherosclerosis, prophylaxis for coronary atherosclerosis
MOA and USE: pravastatin
MOA: Competitively inhibit HMG-CoA reductase (rate limiting step in cholesterol synthesis). USE: hypercholesterolemia, familial hypercholesterolemia, coronary atherosclerosis, prophylaxis for coronary atherosclerosis, stroke
MOA and USE: simvastatin
MOA: Competitively inhibit HMG-CoA reductase (rate limiting step in cholesterol synthesis). USE: hypercholesterolemia, familial hypercholesterolemia, coronary atherosclerosis, prophylaxis for coronary atherosclerosis, stroke
MOA and Use: botulinum toxin
MOA: Degrades SNAP-25. Blocks release of Ach. USE: wrinkles, torticollis, achalasia, focal dystonia, hyperhidrosis, headache/pain, strabismus
MOA and USE: fenofibrate
MOA: Fibrate: Activates PPARa (agonist) (Peroxisome proliferator activated receptor alpha) USE: isolated hypertriglyceridemia, dysbetalipoproteinemia, Hypertriglyceridemia with low HDL
MOA and USE: gemfibrozil
MOA: Fibrate: Activates PPARa (agonist) (Peroxisome proliferator activated receptor alpha) USE: isolated hypertriglyceridemia, dysbetalipoproteinemia, Hypertriglyceridemia with low HDL
MOA and Use: vesamicol
MOA: Inhibitor of Acetylcholine release, storage and synthesis. Inhibits VAT (ACh-H+ antiporter) that transports ACh into synaptic vesicles. USE: Inhibit storage of Ach
MOA and Use: Hemicholinium-3
MOA: Inhibitor of Acetylcholine synthesis, storage and release. Block Na/choline cotransporter; prevents uptake of choline required for ACh synthesis. USE: Inhibits synthesis of Ach and research
MOA and USE: cholestyramine
MOA: Inhibitor of bile acid absorption. Large nonabsorbable polymers. Bind to bile acids and preventing enterhepatic circulation. Bind bile acids and steroids in the intesine. Upregulate LDL Receptors. USE: Hypercholesterolemia, ** ↓LDL, ↑TG, slightly ↑HDL, pruritus
MOA and USE: ezetimibe
MOA: Inhibitor of cholesterol absorption. Decrease cholesterol transport from micelles into enterocytes by inhibiting uptake through brush border protein Niemann-Pick C1-Like 1 (NPC1L1) USE: Primary Hypercholesterolemia. ** ↓LDL, ↓TG, slightly ↑ HDL, sitosterolemia, mixed hyperlipidemia
MOA and Use: succinylcholine
MOA: Nicotinic receptor agonist (Nm) (depolarizing); stimulates opening of NAChR causes depol. of cell membrane; perists at the NMJ and activates NAChR channels continuously, which results in inactivation of voltage gated Na channels so they can't open to support further action potentials. Use: induction of neuromuscular blockade in surgery; intubation
MOA and Use: mivacurium
MOA: Non-depolarizizng neuromuscular blocker. Competitive antagonist at nicotinic receptor (Nm). USE: induction of neuromuscular blockade in surgery; intubation
MOA and Use: pancuronium
MOA: Non-depolarizizng neuromuscular blocker. Competitive antagonist at nicotinic receptor (Nm). USE: induction of neuromuscular blockade in surgery; intubation
MOA and USE: niacin
MOA: Reduces free fatty acid release from adipose tissue and increases plasma residence time for apoA1 USE: Isolated low HDL, Low HDL with mildly elevated LDL or Triglyceride, familial combined hyperlipidemia
MOA and Use: atropine
MOA: competitively prevents Ach from binding to muscarinic receptors. USE: GI: anti-spasm. CV: treat breadycardia at low dose. Antidote for: mushroom poisoning, pilocarpine, physostigmine, organophosphate poisoning. Inhibit excessive salivation/mucus secretion.
MOA and USE: lomitapide
MOA: inhibit ApoB lipidation by MTP to decrease VLDL secretion. USE: homozygous familial hypercholesterolemia (HoFH)
MOA and USE: mipomersen
MOA: inhibit ApoB mRNA translation to decrease VLDL secretion. USE: homozygous familial hypercholesterolemia (HoFH)
MOA and USE: omega-3 fatty acids (EPA and DHA)
MOA: regulate nuclear transcription factors including SREBP-1c and PPARa to reduce Triglyceride biosynthesis and increases Fatty Acid oxidation. USE: hypertriglyceridemia
MOA and Use: scopolamine
MOA:Competitively inhibits Ach from binding to muscarinic receptors USE: Prevent motion sickness
mature onset diabetes of the young
MODY, which accounts for 1% to 5% of all diabetes cases, typically occurs before 25 years of age and follows an autosomal dominant mode of inheritance. In contrast to type 2 diabetes, it is not associated with obesity. Studies of MODY pedigrees have shown that about 50% of cases are caused by mutations in the gene that encodes glucokinase, a rate-limiting enzyme in the conversion of glucose to glucose-6-phosphate in the pancreas. Another 40% of MODY cases are caused by mutations in any of five genes that encode transcription factors involved in pancreatic development or insulin regulation: hepatocyte nuclear factor-1α (HNF1α), hepatocyte nuclear factor-1β (HNF1β), hepatocyte nuclear factor-4α (HNF4α), insulin promoter factor-1 (IPF1), and neurogenic differentiation 1 (NEUROD1). Mutations in these genes, all of which are expressed in pancreatic beta cells, lead to beta cell abnormalities and thus to diabetes. Type 1 (insulindependent) and type 2 (non- insulindependent) diabetes both cluster in families, with stronger familial clustering observed for type 2 diabetes. Type 1 has an earlier average age of onset, is HLAassociated, and is an autoimmune disease. Type 2 is not an autoimmune disorder and is more likely to be seen in obese persons. Several genes have been identified that increase susceptibility to type 1 or type 2 diabetes. Most cases of autosomal dominant MODY are caused by mutations in any of six specific genes.
What are the steps in the formation of cholesterol?
Many cells in the body, including liver parenchymal cells, synthesize cholesterol 1- Acetate is converted to 3-hydroxy-3 methylglutaryl-coenzyme A (HMG-CoA) 2- HMG-CoA reductase converts HMG-CoA into Melvonic Acid 3- Melvonic acid is then converted to cholesterol through a series of steps
Clinical syndromes and therapy
Many chronic hypersensitivity disorders in humans are caused by, or are associated with, antibodies against cells and tissues (Fig. 11-9) and immune complexes (Fig. 11-10). The first immune complex disease studied was serum sickness, seen in subjects who repeatedly received animal serum for the treatment of infections. This illness could be re-created in experimental animals. Serum sickness is induced by systemic administration of a protein antigen, which elicits an antibody response and leads to the formation of circulating immune complexes. Systemic lupus erythematosus is another example of a well-studied systemic immune complex disease. A localized immune complex reaction called the Arthus reaction was first studied in experimental animals. It is induced by subcutaneous administration of a protein antigen to a previously immunized animal; it results in the formation of immune complexes at the site of antigen injection and a local vasculitis. In a small percentage of vaccine recipients who have previously been vaccinated or already have antibodies against the vaccine antigen, a painful swelling that develops at the injection site represents a clinically relevant Arthus reaction. Therapy for antibody-mediated diseases is intended mainly to limit inflammation and its injurious consequences, with drugs such as corticosteroids. In severe cases, plasmapheresis is used to reduce levels of circulating antibodies or immune complexes. Some of these diseases respond well to treatment with intravenous IgG (IVIG) pooled from healthy donors. How IVIG works is not known; it may induce the expression of and bind to the inhibitory Fc receptor on myeloid cells and B cells (see Chapter 7, Fig. 7-15), or it may reduce the half-life of pathogenic antibodies by competing for binding to the neonatal Fc receptor in endothelial and other cells (see Chapter 8, Fig. 8-2). Treatment of patients with an antibody specific for CD20, a surface protein of mature B cells, results in depletion of the B cells and may be useful for treating some antibody-mediated disorders. Other approaches being tried for inhibiting the production of autoantibodies include treating patients with antibodies that block CD40 or its ligand and thus inhibit helper T cell-dependent B cell activation and antibodies to block cytokines that promote the survival of B cells and plasma cells. There is also interest in inducing tolerance in cases in which the autoantigens are known.
clinical syndromes and therapy
Many organ-specific autoimmune diseases in humans are believed to be caused by T cells, based on the identification of these cells in lesions and similarities with animal models in which the diseases are known to be T cell mediated (Fig. 11-13). These disorders typically are chronic and progressive, in part because T cell reactions tend to be prolonged and often selfperpetuating, and because the inciting antigens, such as tissue antigens or proteins expressed by resident microbes, are often never cleared. Also, tissue injury causes release and alteration of self proteins, which may result in reactions against these newly encountered proteins. This phenomenon has been called epitope spreading to indicate that the initial immune response against one or a few self antigen epitopes may expand to include responses against many more self antigens. Chronic inflammatory diseases that are initiated by immune reactions are sometimes called immune-mediated inflammatory diseases. The therapy for T cell-mediated hypersensitivity disorders is designed to reduce inflammation and to inhibit T cell responses. The mainstay of treatment of such diseases has been the potent antiinflammatory steroids, but these drugs have significant side effects. The development of more targeted therapies based on understanding of the fundamental mechanisms of these diseases has been one of the most impressive accomplishments of immunology. Antagonists of TNF have proved to be beneficial in patients with rheumatoid arthritis and inflammatory bowel disease by reducing inflammation. Newer agents developed to inhibit T cell responses include drugs that block costimulators such as B7, and antagonists against cytokines or their receptors such as IL-1, IL-6, and IL-17. B cell depletion with anti-CD20 has also been effective in rheumatoid arthritis and multiple sclerosis; it is not clear if this is because antibodies contribute to the diseases or because B cells function as antigen-presenting cells to promote T cell activation. There also is great hope for inducing tolerance in pathogenic T cells, but no successful clinical trials have been reported.
key concepts - marfan and EDS
Marfan Syndrome ■ Marfan syndrome is caused by a mutation in the FBN1 gene encoding fibrillin, which is required for structural integrity of connective tissues and regulation of TGF-β signaling. ■ The major tissues affected are the skeleton, eyes, and cardiovascular system. ■ Clinical features may include tall stature, long fingers, bilateral subluxation of lens, mitral valve prolapse, aortic aneurysm, and aortic dissection. ■ Clinical trials with drugs that inhibit TGF-β signaling such as angiotensin receptor blockers are ongoing, as these have been shown to improve aortic and cardiac function in mouse models. Ehlers-Danlos Syndromes ■ There are six variants of Ehlers-Danlos syndromes, all characterized by defects in collagen synthesis or assembly. Each of the variants is caused by a distinct mutation involving one of several collagen genes or genes that encode other ECM proteins like tenascin-X. ■ Clinical features may include fragile, hyperextensible skin vulnerable to trauma, hypermobile joints, and ruptures involving colon, cornea, or large arteries. Wound healing is poor.
marfan syndrome
Marfan syndrome is a disorder of connective tissues, manifested principally by changes in the skeleton, eyes, and cardiovascular system. Its prevalence is estimated to be 1 in 5000. Approximately 70% to 85% of cases are familial and transmitted by autosomal dominant inheritance. The remainder are sporadic and arise from new mutations.
biochemical and molecular basis of single-gene mendelial disorders
Mendelian disorders result from alterations involving single genes. The genetic defect may lead to the formation of an abnormal protein or a reduction in the output of the gene product. Virtually any type of protein may be affected in single-gene disorders and by a variety of mechanisms (Table 5-4). To some extent the pattern of inheritance of the disease is related to the kind of protein affected by the mutation. For this discussion, the mechanisms involved in single-gene disorders can be classified into four categories: (1) enzyme defects and their consequences; (2) defects in membrane receptors and transport systems; (3) alterations in the structure, function, or quantity of nonenzyme proteins; and (4) mutations resulting in unusual reactions to drugs
marfan syndrome pathogenesis
Marfan syndrome results from an inherited defect in an extracellular glycoprotein called fibrillin1. There are two fundamental mechanisms by which loss of fibrillin leads to the clinical manifestations of Marfan syndrome: loss of structural support in microfibril rich connective tissue and excessive activation of TGF-β signaling. Each of these is discussed below. • Fibrillin is the major component of microfibrils found in the extracellular matrix (Chapter 1). These fibrils provide a scaffolding on which tropoelastin is deposited to form elastic fibers. Although microfibrils are widely distributed in the body, they are particularly abundant in the aorta, ligaments, and the ciliary zonules that support the lens; these tissues are prominently affected in Marfan syndrome. Fibrillin occurs in two homologous forms, fibrillin-1 and fibrillin-2, encoded by two separate genes, FBN1 and FBN2, mapped on chromosomes 15q21.1 and 5q23.31, respectively. Mutations of FBN1 underlie Marfan syndrome; mutations of the related FBN2 gene are less common, and they give rise to congenital contractural arachnodactyly, an autosomal dominant disorder characterized by skeletal abnormalities. Mutational analysis has revealed more than 600 distinct mutations of the FBN1 gene in individuals with Marfan syndrome. Most of these are missense mutations that give rise to abnormal fibrillin-1. These can inhibit polymerization of fibrillin fibers (dominant negative effect). Alternatively, the reduction of fibrillin content below a certain threshold weakens the connective tissue (haploinsufficiency). • While many clinical manifestations of Marfan syndrome can be explained by changes in the mechanical properties of the extracellular matrix resulting from abnormalities of fibrillin, several others such as bone overgrowth and myxoid changes in mitral valves cannot be attributed to changes in tissue elasticity. Recent studies indicate that loss of microfibrils gives rise to abnormal and excessive activation of transforming growth factor-β (TGF-β), since normal microfibrils sequester TGF-β and thus control the bioavailability of this cytokine. Excessive TGF-β signaling has deleterious effects on vascular smooth muscle development and it also increases the activity of metalloproteases, causing loss of extracellular matrix. This schema is supported by two sets of observations. First, in a small number of individuals with clinical features of Marfan syndrome (MFS2), there are no mutations in FBN1 but instead gain-of-function mutations in genes that encode TGF-β receptors. Second, in mouse models of Marfan syndrome generated by mutations in Fbn1, administration of antibodies to TGF-β prevents alterations in the aorta and mitral valves.
What is responsible for the appearance of xanthomas and contribution to the pathogenesis of premature atherosclerosis?
Marked increase in the scavenger receptor-mediated traffic of LDL cholesterol into the cells of the mononuclear phagocyte system and possibly the vascular walls
What occurs in the late phase reaction? What is it responsible for?
Mast cell mediators are cytokines that recruit neutrophils and eosinophils to the site of the reaction over several hours. the tissue injury that results from repeated bouts of immediate hypersensitivity
What increases the chances of graft or transplant tolerance in humans?
Matching of the donors and recepients MHC Class I alleles
What is the most frequent cytogenetic abnormality seen in persons with Autism spectrum disorder?
Maternally transmitted duplication of chromosome 15q11-q13- the region that is deleted in Prader-Willi and Angelman syndromes A 600 kb duplication or deletion of 16p11.2 is seen in about 1% of ASD cases.
Mechanism of LDL lowering by statins
Mechanism o LDL lowering by statins. Statins competitively inhibit HMG-CoA reductase, the enzyme that catalyzes the rate-limiting step in cholesterol biosynthesis. Decreased cellular cholesterol concentrations lead to protease activation and cleavage of the sterol regulatory element binding protein (SREBP), which is a transcription factor that normally resides in the cytoplasm. The cleaved SREBP diffuses into the nucleus, where it binds to sterol response elements (SRE), leading to up-regulation of LDL receptor gene transcription. This leads to increased cellular LDL receptor expression. This promotes uptake of LDL particles and results in reduced LDL-cholesterol concentrations in the plasma.
Key concepts
Mechanisms of T Cell-Mediated Hypersensitivity Reactions ■ Cytokine-mediated inflammation: CD4+ T cells are activated by exposure to a protein antigen and differentiate into TH1 and TH17 effector cells. Subsequent exposure to the antigen results in the secretion of cytokines. IFN-γ activates macrophages to produce substances that cause tissue damage and promote fibrosis, and IL-17 and other cytokines recruit leukocytes, thus promoting inflammation. ■ The classical T cell-mediated inflammatory reaction is delayed type hypersensitivity. ■ T cell-mediated cytotoxicity: CD8+ cytotoxic T lymphocytes (CTLs) specific for an antigen recognize cells expressing the target antigen and kill these cells. CD8+ T cells also secrete IFN-γ
What complex does Complement activation on cells lead to? What does it do?
Membrane attack complex Disrupts membrane integrity by drilling holes through the lipid bilayer, thereby causing osmotic lysis of the cells(kills it) This mechanism of killing is probably effective only with cells that have thin cell walls, such as Neisseria bacteria.
What ensures a rapid response to a subsequent infection by the same pathogen?
Memory T cells
What are the outcome of mouse studies with leptin?
Mice with loss-of-function mutations in the leptin gene have uncontrolled appetites and become obese When injected with leptin, these mice lose weight. leptin participates in important interactions with other components of appetite control, such as neuropeptide Y, as well as α-melanocyte-stimulating hormone and its receptor, the melanocortin-4 receptor (MC4R). Several genomewide association studies have demonstrated an association between an intronic variant in the brain-expressed FTO gene and obesity in whites. Homozygosity for this variant, which is seen in about 16% of whites, confers increased
Formation of what enables the absorption of glycerol and FFA into intestinal cells? What is the transport of free cholesterol mediated by?
Micelles A cholesterol gradient that exists between the lumen and the intestinal cell
What is the selectivity of action of most agents/drugs dependent on?
Pharmacodynamic and pharmacokinetic factors eg: differences in receptor binding affinity, bioavailability, tissue localization, and resistance to degradation.
Pharmacologic classes and agents
Pharmacologic manipulation o cholinergic transmission has met with only limited success because the complex actions o ACh make it di f cult to obtain selective e ects. For example, many cholinergic agents are capable o both stimulating and blocking cholinergic receptors through a mechanism known as depolarizing blockade (see below). There ore, only a relatively small raction o the many cholinergic and anticholinergic agents discovered over the past century are used in clinical practice. These drugs are used primarily or (1) modulation o gastrointestinal motility, (2) xerostomia (dry mouth), (3) glaucoma, (4) motion sickness and antiemesis, (5) neuromuscular diseases such as myasthenia gravis and Eaton-Lambert syndrome, (6) acute neuromuscular blockade and reversal during surgery, (7) ganglionic blockade during aortic dissection, (8) dystonias (e.g., torticollis), headache, and pain syndromes, (9) reversal o vagal-mediated bradycardia, (10) mydriasis, (11) bronchodilation in chronic obstructive pulmonary disease, (12) bladder spasms and urinary incontinence, (13) cosmetic e ects on skin lines and wrinkles, and (14) treatment o Alzheimer's disease and other neurodegenerative dementias. Slight variations in the pharmacologic properties o individual cholinergic and anticholinergic agents are responsible or their large di erences in therapeutic utility. The relative selectivity o action o the most use ul agents depends on both pharmacodynamic and pharmacokinetic actors, including inherent di erences in receptor binding a f nity, bioavailability, tissue localization, and resistance to degradation. These variations, in turn, derive rom the molecular structure and charge o the drug. The structure o pirenzepine , or example, allows the drug to bind M 1 muscarinic receptors (located in autonomic ganglia) with higher a f nity than M 2 and M 3 receptors (located at parasympathetic end organs). As a result, the drug's predominant e ect at clinically used doses is ganglionic blockade (see Table 10-1). Similarly, the addition o a methyl group to acetylcholine yields methacholine , which is more resistant to degradation by AChE and, hence, possesses a longer duration o action. Charged agents such as muscarine generally do not cross membrane barriers. The absorption o such drugs through both the gastrointestinal (GI) mucosa and the blood-brain barrier is signif cantly impaired, unless specif c carriers are available to transport the drug; there ore, such drugs typically have little or no e ect on the CNS. In contrast, lipophilic agents have excellent CNS penetration. As one example, the high CNS penetration of physostigmine makes this drug the agent of choice or treating the CNS effects o anticholinergic overdose The ollowing discussion is ordered mechanistically. For each class o drugs, the selectivity o individual agents within the class is used as a basis to explain the therapeutic uses of each agent.
What type of trait is blood pressure?
Multifactorial trait There is a correlation between parents' blood pressures (systolic and diastolic) and those of their children, and there is good evidence that this correlation is due in part to genes. But blood pressure is also influenced by environmental factors, such as diet and stress
Muscarinic receptors
Muscarinic cholinergic transmission occurs mainly at autonomic ganglia, at end organs innervated by the parasympathetic division o the autonomic nervous system, and in the CNS. As G protein-coupled receptors, muscarinic receptors transduce signals across the cell membrane and interact with GTP-binding proteins. Because the e ects o muscarinic receptor activation occur through the actions o these G proteins, there is a latency o at least 100-250 ms associated with muscarinic responses to receptor activation. (In contrast, nicotinic receptor channels have latencies on the order o 5 ms.) Activation o G proteins by agonist binding to muscarinic receptors may have several di erent e ects on cells. These include inhibition o adenylyl cyclase (via G i ) and stimulation o phospholipase C (via G q/11 ), both mediated by an subunit o the G protein. (See Chapter 1, Drug-Receptor Interactions, or a discussion o these signaling mechanisms.) Muscarinic activation also modulates ion channels via the subunit o a G protein. The predominant e ect o such mAChR stimulation is to increase the opening o specif c potassium channels (G protein-modulated inwardly recti ying K channels, or GIRKs), thereby hyperpolarizing the cell. The subunit o the G i protein binds to the channel and enhances its probability o being open. Five distinct cDNAs or muscarinic receptors, denoted M 1 -M 5 , have been isolated and detected in human cells. These receptor types orm two unctionally distinct groups. M 1 , M 3 , and M 5 are coupled to G proteins responsible or the stimulation o phospholipase C. M 2 and M 4 , on the other hand, are coupled to G proteins responsible or adenylyl cyclase inhibition and K channel activation. The receptors o each unctional group can be distinguished based on their responses to pharmacologic antagonists (Table 10-1). Generally, M 1 is expressed in cortical neurons and autonomic ganglia, M 2 in cardiac muscle, and M 3 in smooth muscle and glandular tissue. Because stimulation o M 1 , M 3 , and M 5 receptors acilitates excitation o the cell, while stimulation o M 2 and M 4 receptors suppresses cellular excitability, there is a predictable correlation between the receptor subtype and the e ect o ACh on the cell. The various muscarinic receptor subtypes account or much o the diversity in cellular responses to mAChR agonists
Lipoprotein lipase deficiency
Mutations in the LPL gene resulting in deficiency of LPL synthesis or function lead to increased circulating chylomicron and VLDL particles and severe hypertriglyceridemia. Homozygous LPL deficiency is rare. It manifests in childhood with triglyceride levels higher than 1000 mg/dL. Heterozygous LPL deficiency occurs in 2% to 4% of the population and usually requires a precipitating factor, such as uncontrolled diabetes or estrogen therapy, to manifest the phenotype. These individuals have moderate hypertriglyceridemia (250 to 750 mg/dL) that can increase to levels greater than 1000 mg/dL with secondary factors. This can result in the chylomicronemia syndrome, which is characterized by marked hypertriglyceridemia (>1000 to 2000 mg/dL), pancreatitis, eruptive xanthomas, lipemia retinalis, and hepatosplenomegaly. Visual inspection demonstrates lipemic plasma. After refrigeration for 12 hours, a creamy top layer (increased chylomicrons) or turbid plasma infranatant (increased VLDL), or both, can be demonstrated. Documentation of diminished LPL activity confirms the diagnosis. A diet low in fat (<10% of total calories or 20 to 25 g/day) is the primary treatment. Secondary factors such as uncontrolled diabetes and alcohol use should be addressed, and VLDL-lowering agents (e.g., fibric acid derivatives, niacin) may be needed to prevent severe hypertriglyceridemia.
Familial Hypercholesterolemia
Mutations in the gene that encodes the LDL (apo B/E) receptor result in familial hypercholesterolemia. Impairment in LDL receptor synthesis or function decreases the clearance of LDL and increases circulating LDL levels, resulting in cholesterol plaque formation. These plaques deposit in the arteries (atheroma), skin or tendons (xanthoma), eyelids (xanthelasma), and iris (corneal arcus). The homozygous form of this autosomal dominant disorder is rare. Affected individuals present early in life with elevated levels of total cholesterol (600 to 1000 mg/ dL) and LDL-cholesterol (550 to 950 mg/dL). Triglyceride and HDL-cholesterol levels are normal. These patients develop CHD, aortic stenosis due to atherosclerosis of the aortic root, and tendon xanthomas (often in the Achilles tendon). If the condition remains untreated, patients with homozygous familial hypercholesterolemia typically die of myocardial infarction before 20 years of age. The heterozygous form of familial hypercholesterolemia affects 1 in every 500 individuals, with the partial receptor defect resulting in cells that display half the normal number of fully functional LDL receptors. These individuals have less strongly elevated concentrations of total cholesterol (>300 to 600 mg/dL) and LDL-cholesterol (250 to 500 mg/dL) than do those with the homozygous form. Premature CHD (before 45 years of age in men and 55 years in women) and tendon xanthomas are characteristic clinical findings. Although familial hypercholesterolemia can be established by identifying one of the many gene mutations in the LDL receptor or by demonstrating diminished LDL receptor function, the diagnosis of familial hypercholesterolemia usually is made on the basis of clinical features. Elevated total cholesterol (>300 mg/ dL) and LDL-cholesterol (>250 mg/dL) in an individual with a personal or family history of premature CHD and tendon xanthomas identifies patients at risk for familial hypercholesterolemia. Treatment requires a low-fat (<20% of total calories), low cholesterol (<100 mg/day) diet in combination with drug therapy. Usually, patients with familial hypercholesterolemia require multiple agents to lower cholesterol levels to the target range. In patients who do not tolerate the medications or who have limited receptor function, liver transplantation to provide functional receptors, ileal bypass surgery to decrease gastrointestinal absorption of bile acids, or LDL apheresis to remove excess LDL may be considered. Both lomitapide and mipomersen may also be considered as adjuvant therapy.
transmission patterns of single gene disorders
Mutations involving single genes typically follow one of three patterns of inheritance: autosomal dominant, autosomal recessive, and X-linked. The general rules that govern the transmission of single-gene disorders are well known; only a few salient features are summarized. Singlegene disorders with nonclassic patterns of inheritance are described later.
enzyme defects and their consequences
Mutations may result in the synthesis of an enzyme with reduced activity or a reduced amount of a normal enzyme. In either case, the consequence is a metabolic block. Figure 5-5 provides an example of an enzyme reaction in which the substrate is converted by intracellular enzymes, denoted as 1, 2, and 3, into an end product through intermediates 1 and 2. In this model the final product exerts feedback control on enzyme 1. A minor pathway producing small quantities of M1 and M2 also exists. The biochemical consequences of an enzyme defect in such a reaction may lead to three major consequences: • Accumulation of the substrate, depending on the site of block, may be accompanied by accumulation of one or both intermediates. Moreover, an increased concentration of intermediate 2 may stimulate the minor pathway and thus lead to an excess of M1 and M2. Under these conditions tissue injury may result if the precursor, the intermediates, or the products of alternative minor pathways are toxic in high concentrations. For example, in galactosemia, the deficiency of galactose1-phosphate uridyltransferase (Chapter 10) leads to the accumulation of galactose and consequent tissue damage. Exces sive accumulation of complex substrates within the lysosomes as a result of deficiency of degradative enzymes is responsible for a group of diseases generally referred to as lysosomal storage diseases • An enzyme defect can lead to a metabolic block and a decreased amount of end product that may be necessary for normal function. For example, a deficiency of melanin may result from lack of tyrosinase, which is necessary for the biosynthesis of melanin from its precursor, tyrosine, resulting in the clinical condition called albinism. If the end product is a feedback inhibitor of the enzymes involved in the early reactions (in Fig. 5-5 it is shown that the product inhibits enzyme 1), the deficiency of the end product may permit overproduction of intermediates and their catabolic products, some of which may be injurious at high concentrations. A prime example of a disease with such an underlying mechanism is the Lesch-Nyhan syndrome (Chapter 26). • Failure to inactivate a tissue-damaging substrate is best exemplified by α1-antitrypsin deficiency. Individuals who have an inherited deficiency of serum α1-antitrypsin are unable to inactivate neutrophil elastase in their lungs. Unchecked activity of this protease leads to destruction of elastin in the walls of lung alveoli, leading eventually to pulmonary emphysema (Chapter 15).
Intro Myasthania gravis -Definition, Epidemiology and Pathology
Neuromuscular junction diseases are caused by abnormal neuromuscular transmission of the action potential from the nerve terminal to the muscle, and they can be autoimmune (myasthenia gravis, Lambert Eaton Syndrome), hereditary (congenital myasthenic syndromes), or toxic (botulism, organophosphate intoxication Myasthenia gravis (MG) is a rare autoimmune disease caused by antibodies against the postsynaptic acetylcholine receptors (AChR Ab) in the neuromuscular junction. All ages are affected but incidence is higher in women younger than 40, and in men older than 50. Prevalence is approximately 20 in 100,000. Transient neonatal MG occurs in about 12% of newborns of myasthenic mothers and is caused by transplacental passive transfer of antibodies from the mother to the fetus. Thymoma is found in 10% of patients with MG and thymic hyperplasia is present in 65%
Are Xanthomas or Xanthelasmas a feature of FCH?
No
Can VLDL and LDL levels be determined if the triglyceride levels are greater than 400 mg/dL?
No In that case, the lipoprotein abnormality can be identified by inspecting the serum. When the triglyceride level exceeds 350 mg/dL, the serum cholesterol is cloudy. After refrigeration: a white surface layer depicts excess chylomicrons, whereas a dispersed, opaque infranatant refects a VLDL dysfunction
Does somatic mutation occur in genes that encode T cells?
No Their diversity is created by the same mechanisms that produce immunoglobulin diversity
What receptors within the liver are capable of detecting the rate of loss of bile acids into the feces?
Nuclear hormone receptors These receptors tightly regulate transcription of bile acid synthetic genes. As a result, the liver synthesizes precisely the amount of bile acids that is sufficient to replace what is lost in the eces
Pathophysiology
Numerous studies have demonstrated a def nitive link between elevated plasma lipid concentrations and the risk o cardiovascular disease. Increased risk o cardiovascular mortality is most closely linked to elevated levels o LDL cholesterol and decreased levels o HDL cholesterol. In addition, hypertriglyceridemia represents an independent risk actor. The risk is urther increased when hypertriglyceridemia is associated with low HDL-cholesterol concentrations, even i LDL-cholesterol concentrations are normal. From a clinical perspective, the dyslipidemias can be divided into hypercholesterolemia, hypertriglyceridemia, mixed hyperlipidemia, and disorders o HDL metabolism The causes o hyperlipidemia are multi actorial. These include well-def ned monogenic diseases and the contributions o genetic polymorphisms, as well as less well-def ned gene- environment interactions. For many individuals, elevated cholesterol may be the consequence o a diet high in saturated at and cholesterol superimposed on a susceptible genetic prof le. The ollowing section describes the major genetic predispositions or hyperlipidemia. This is ollowed by a brie overview o the secondary causes o hyperlipidemia. It is important to appreciate that the decision to treat elevated cholesterol concentrations is based on estimations o the risk o cardiovascular disease. Current clinical practice does not incorporate genetic causes o hyperlipidemia into these calculations. As common genetic predispositions to dyslipidemia and the contributions o these predispositions to cardiovascular disease become better understood, lipid-lowering therapies may one day be tailored toward individual genetic susceptibilities
What are risk factors for Coronary artery disease (CAD)?
Obesity, cigarette smoking, hypertension, elevated cholesterol level, and positive family history (more than one affected first degree relative) a person with family history for CAD is twice (2X) as likely to suffer from CAD
Where is SR-BI highly expressed?
On the sinusoidal plasma membranes of hepatocytes
Where are the majority of LDL receptors expressed? What does this then mean?
On the surface of hepatocytes As a result, the liver is primarily responsible or the removal of LDL particles from the circulation.
biliary lipid secretion
Once cholesterol is delivered to the liver by the process o reverse cholesterol transport, it is eliminated by biliary secretion. An essential step occurs when a raction o the cholesterol is converted to bile acids (Fig. 20-10A). Cholesterol 7 -hydroxylase (CYP7A1), an enzyme expressed only in hepatocytes, catalyzes the rate-limiting step in the catabolism o cholesterol to bile acids. Bile acids, unlike cholesterol, are highly soluble in water. Moreover, bile acids are biological detergents that promote the ormation o micelles (Fig. 20-10B). These macromolecular aggregates, which are rich in phospholipids derived rom hepatocyte membranes, solubilize cholesterol in bile or transport rom the liver to the small intestine. In this way, micelles serve as a unctional counterpart to HDL particles in plasma Bile ormation begins when bile acids are pumped into bile by the action o a canalicular membrane transport pump known as ABCB11 (Fig. 20-10B). In turn, these bile acids stimulate the biliary secretion o phospholipids and cholesterol. Phospholipid and cholesterol secretion are mediated by two additional transporters: ABCB4 or phospholipids and a heterodimer o ABCG5 and ABCG8 or cholesterol. Large amounts o bile acids, phospholipids, and cholesterol are secreted into bile at approximate rates o 24, 11, and 1.2 grams each day, respectively. These molecules comprise the biliary lipids, which are stored in the gallbladder during asting. The stimulus o a atty meal leads to gallbladder contraction, which propels its contents into the small intestine. As described above, bile acilitates the digestion and absorption o ats, in addition to promoting the elimination o endogenous cholesterol.
What is the most clinically used alkaloid? What is it used to treat?
Pilocarpine Xerostomia (dry mouth) also treats Glaucoma
Organophosphate poisoning
Organophosphorus compounds (OPs) are used as pesticides and developed for chemical warfare. Exposure to even small amounts of an OP can be fatal and death is usually caused by respiratory failure. OPs cause inhibition of acetylcholinesterase (AChE) accumulation of acetylcholine at the cholinergic receptor sites, producing continuous stimulation of cholinergic fibers throughout the nervous system. A combination of an antimuscarinic agent (e.g., atropine), AChE reactivator, such as one of the pyridinium oximes (i.e., pralidoxime, trimedoxime, obidoxime, and HI6), and diazepam are used for the treatment of OP poisoning in humans.
What are dietary triglycerides and cholesterol esters hydrolyzed by? what is produced?
Pancreatic lipase Glycerol, Free fatty acid (FFA) and Cholesterol
What is an example of a very long acting Selective nicotinic receptor antagonist? Clinical use?
Pancuronium neuromuscular blockade during surgical procedures
What do Fibrates bind to and activate?
Peroxisome proliferator activated receptor alpha (PPARalpha) a nuclear receptor expressed in hepatocytes, skeletal muscle, macrophages, and the heart
Opsonization and phagocytosis
Phagocytosis is largely responsible for depletion of cells coated with antibodies. Cells opsonized by IgG antibodies are recognized by phagocyte Fc receptors, which are specific for the Fc portions of some IgG subclasses. In addition, when IgM or IgG antibodies are deposited on the surfaces of cells, they may activate the complement system by the classical pathway. Complement activation generates by-products, mainly C3b and C4b, which are deposited on the surfaces of the cells and recognized by phagocytes that express receptors for these proteins. The net result is phagocytosis of the opsonized cells and their destruction (Fig. 6-16A). Complement activation on cells also leads to the formation of the membrane attack complex, which disrupts membrane integrity by "drilling holes" through the lipid bilayer, thereby causing osmotic lysis of the cells. This mechanism of killing is probably effective only with cells that have thin cell walls, such as Neisseria bacteria. Antibody-mediated destruction of cells may occur by another process called antibody-dependent cellular cytotoxicity (ADCC). Cells that are coated with IgG antibody are killed by a variety of effector cells, mainly NK cells and macrophages, which bind to the target by their receptors for the Fc fragment of IgG, and cell lysis proceeds without phagocytosis. The contribution of ADCC to common hypersensitivity diseases is uncertain. Clinically, antibody-mediated cell destruction and phagocytosis occur in the following situations: (1) transfusion reactions, in which cells from an incompatible donor react with and are opsonized by preformed antibody in the host (Chapter 14); (2) hemolytic disease of the newborn (erythroblastosis fetalis), in which there is an antigenic difference between the mother and the fetus, and IgG antierythrocyte antibodies from the mother cross the placenta and cause destruction of fetal red cells (Chapter 10); (3) autoimmune hemolytic anemia, agranulocytosis, and thrombocytopenia, in which individuals produce antibodies to their own blood cells, which are then destroyed (Chapter 14); and (4) certain drug reactions, in which a drug acts as a "hapten" by attaching to plasma membrane proteins of red cells and antibodies are produced against the drug-protein complex.
VLDL contain triglycerides that are assembled by the liver using what?
Plasma fatty acids derived from the adipose tissue or synthesized de novo For this reason, the assembly, secretion, and metabolism of VLDL are often referred to as the endogenous pathway of lipoprotein metabolism.
How is severe weakness associated with Lambert Eaton MS treated?
Plasmapheresis or IVIG
What is it called when a single mutant gene can lead to many end effects?
Pleitropism eg: Sickle cell anemia In this hereditary disorder not only does the point mutation in the gene give rise to HbS, which predisposes the red cells to hemolysis, but also the abnormal red cells tend to cause a logjam in small vessels, inducing, for example, splenic fibrosis, organ infarcts, and bone changes
What type of a trait is height?
Polygenic multifactorial trait the loci underlying variation in a quantitative trait such as height are termed quantitative trait loci
What are Nicotinic receptors primarily involved as in the CNS? What are the Muscarinic receptors primarily involved as?
Presynaptic heteroreceptors that modulate the release of other NTs such as Glutamate Autoreceptors- modulate the release of ACh
What is the prevalence of Pyloric stenosis? More common in males or females?
Prevalence among whites is about 3/1000 live births. It is much more common in males than females, affecting 1/200 males and 1/1000 females It is thought that this difference in prevalence reflects two thresholds in the liability distribution; a lower one in males and a higher one in females (see Fig. 12-2). A lower male threshold implies that fewer disease-causing factors are required to generate the disorder in males.
What is Hypertriglyceridemia?
Primary hypertriglyceridemia is characterized by high plasma triglyceride concentrations (200-500 mg/dL or higher) when measured following an overnight fast. normal: 150 mg/dL
What is Botulism? What is it caused by? Which is produced by ?
Rare, potentially lethal paralytic illness Neurotoxin produced by the anaerobic, spore forming bacteria Clostridium botulinum
What are the enzymes that carry out somatic (VDJ) recombination?
Recombinases which are encoded by Rag enzymes (RAG1 and RAG2)
What is the goal of the therapy for T cell mediated hypersensitivity disorders ? What type of drugs are used?
Reduce inflammation and to inhibit T cell responses Potent anti-inflammatory steroids (have potent side effects)
hypertriglyceridemia
Primary hypertriglyceridemia is characterized by high plasma triglyceride concentrations (200-500 mg/dL or higher; normal, 150 mg/dL), when measured ollowing an overnight ast. Three major etiologies o hypertriglyceridemia have been identif ed: amilial hypertriglyceridemia, amilial lipoprotein lipase (LPL) def ciency, and apoCII def ciency. Familial combined hyperlipidemia can also present with isolated hypertriglyceridemia. More commonly, hypertriglyceridemia develops with age, weight gain, obesity, and diabetes and is an important component of the metabolic syndrome Familial hypertriglyceridemia is a common autosomal dominant disorder characterized by hypertriglyceridemia with normal LDL-cholesterol concentrations. HDL cholesterol is o ten reduced. Although the underlying de ect in this disorder is unknown, it is hypothesized to be a de ect in bile acid metabolism, leading to increased hepatic production o triglyceride-rich VLDL. A strong amily history o premature coronary heart disease is usually absent. Management is generally with exercise and diet. I that approach is unsuccess ul at reducing triglyceride concentrations below 500 mg/dL, a f brate should be considered. Drug therapy should be initiated i triglycerides exceed 1,000 mg/dL Familial lipoprotein lipase def ciency is an autosomal recessive disorder caused by the absence o active LPL. This condition may be diagnosed by testing the plasma or lipase activity ollowing an in usion o heparin, which competes or binding sites on endothelial cells and dislodges LPL molecules into the plasma. Patients with LPL def ciency exhibit pro ound hypertriglyceridemia, which is characterized by elevated chylomicrons during in ancy and impaired removal o VLDL later in li e. In ants or young adults may present with pancreatitis, eruptive xanthomas, hepatomegaly, and splenomegaly attributable to the accumulation o lipid-laden oam cells. Treatment consists o a at- ree diet and avoidance o substances that increase VLDL production by the liver, such as alcohol and glucocorticoids ApoCII def ciency is a rare genetic disorder with presentation and treatment similar to amilial lipoprotein lipase def ciency. It is caused by def ciency o apoCII, a co actor protein o LPL. It may be distinguished rom LPL def ciency by demonstrating that the triglyceride levels o patients are reduced ollowing in usion o plasma that contains normal apoCII; this does not occur in patients with amilial LPL def ciency. It is now appreciated that mutations in apoAV can present with chylomicronemia and severe hypertriglyceridemia, consistent with an apparent role or apoAV in acilitating the interaction between apoCII and LPL.
Where do B cells and T cells mature respectively?
Primary lymphoid tissues: B cells: Bone marrow T cells: Thymus
As LPL continues to hydrolyze triglycerides from chylomicrons and VLDL, the particles become progressively what?
Progressively depleted of triglycerides and relatively enriched in cholesterol Once approximately 50% of the triglycerides have been removed, the particles lose their affinity for LPL and dissociate from the enzyme
What do helper T cells stimulate?
Proliferation of B cells whose receptors (immunoglobulins) can bind to the foreign particles
Botulism- treatment
Prompt intensive care support with mechanical ventilation and parenteral feeding as needed are crucial in reducing mortality. Timely administration of equine antitoxin within the first 24 hours may arrest the progression of paralysis and decrease the duration of illness. The antitoxin is provided by the CDC through the local health departments. Children less than 12 months old should not be fed honey because it can contain Clostridium botulinum.
What is the plasma protein that regulates LDL receptor activity?
Proprotein convertase subtilisin-like kexin type 9 (PCSK9) 1- PCSK9 is synthesized as a 72-kDa proPCSK9 proprotein that is autocatalytically cleaved in the endoplasmic reticulum to orm the mature protein 2- It then enters the secretory pathway, and with the assistance o sortilin in the trans Golgi network, it is secreted into the plasma 3- PCSK9 then binds to the epidermal growth actor-like repeat A (EGFA) moti o the LDL receptor. This complex is targeted to lysosomes or degradation
What is the second most commonly diagnosed cancer in men (after skin cancer)?
Prostate Cancer Prostate cancer is second only to lung cancer as a cause of cancer death in men, causing more than 27,000 deaths each year. Having an affected first-degree relative increases the risk of developing prostate cancer by a factor of two to three, and the heritability of prostate cancer is estimated to be approximately 40%. The relatively late age of onset of most prostate cancer cases (median age, 72 years) makes genetic analysis especially difficult
prostate cancer
Prostate cancer is the second most commonly diagnosed cancer in men (after skin cancer), with approximately 220,000 new cases annually in the United States. Prostate cancer is second only to lung cancer as a cause of cancer death in men, causing more than 27,000 deaths each year. Having an affected first-degree relative increases the risk of developing prostate cancer by a factor of two to three, and the heritability of prostate cancer is estimated to be approximately 40%. The relatively late age of onset of most prostate cancer cases (median age, 72 years) makes genetic analysis especially difficult. However, loss of heterozygosity (see Chapter 11) has been observed in a number of genomic regions in prostate tumor cells, possibly indicating the presence of genetic alterations in these regions. In addition, genome scans (GWAS) have identified several dozen polymorphisms associated with prostate cancer risk. Several of these are located in chromosome 8q24, which contains polymorphisms associated with several other cancers as well (colon, pancreas, and esophagus). Although the 8q24 region contains no protein-coding genes, it contains enhancer elements that affect expression of the MYC oncogene, located about 250 kb from 8q24. Nongenetic risk factors for prostate cancer may include a high-fat diet. Because prostate cancer usually progresses slowly and because it can be detected by digital examination and by the prostate-specific antigen (PSA) test, fatal metastasis can usually be prevented. Most common cancers have genetic components. Recurrence risks tend to be higher if there are several affected relatives and if those relatives developed cancer at an early age. Specific genes have been discovered that cause inherited colon, breast, and prostate cancer in some families.
What does Glucose-6-phoshphate dehydrogenase deficiency (G6PD) predispose to?
Red cell hemolysis in patients receiving certain types of drugs- expressed mainly in males In the female, a proportion of the red cells may be derived from precursors with inactivation of the normal allele Transmitted on the X chromosome
Biochemical and molecular basis of some mendelian disorders
Protein Type/ Function Example Molecular Lesion Disease Enzyme Phenylalanine hydroxylase Hexosaminidase Splice-site mutation: reduced amount Splice-site mutation or frameshift mutation with stop codon: reduced amount Point mutations: abnormal protein with reduced activity Phenylketonuria Tay-Sachs disease Adenosine deaminase Severe combined immunodeficiency Enzyme inhibitor α1-Antitrypsin Missense mutations: impaired secretion from liver to serum Emphysema and liver disease Receptor Low-density lipoprotein receptor Deletions, point mutations: reduction of synthesis, transport to cell surface, or binding to low-density lipoprotein Familial hypercholesterolemia Vitamin D receptor Point mutations: failure of normal signaling Vitamin D-resistant rickets Transport Oxygen Ion channels Hemoglobin Deletions: reduced amount Defective mRNA processing: reduced amount Point mutations: abnormal structure α-Thalassemia β-Thalassemia Sickle cell anemia Cystic fibrosis transmembrane conductance regulator Deletions and other mutations: nonfunctional or misfolded proteins Cystic fibrosis Structural Extracellular Collagen Deletions or point mutations cause reduced amount of normal collagen or normal amounts of defective collagen Missense mutations Deletion with reduced synthesis Heterogeneous Osteogenesis imperfecta; Ehlers-Danlos syndromes Cell membrane Fibrillin Dystrophin Spectrin, ankyrin, or protein 4.1 Marfan syndrome Duchenne/Becker muscular dystrophy Hereditary spherocytosis Hemostasis Factor VIII Deletions, insertions, nonsense mutations, and others: reduced synthesis or abnormal factor VIII Hemophilia A Growth regulation Rb protein Neurofibromin Deletions Heterogeneous Hereditary retinoblastoma Neurofibromatosis type 1
What do lipids such as Free fatty acids (FFA), cholesterol and Triglycerides bind to for transport?
Proteins all are hydrophobic
What is an asymetric sign of MG?
Ptosis
MG- treatment
Pyridostigmine 30 to 60 mg every 4 hours improves symptoms in most patients with MG; it is used alone to treat purely ocular and generalized cases with only minimal or mild weakness, or in combination with immunosuppressant drugs in patients with more severe manifestations. Prednisone is effective in improving muscle weakness in a short period of time, but longterm use is associated with side effects. Azathioprine and mycophenolate mofetil are used for longterm treatment and as steroidssparing agents. Plasmapheresis and IVIG are used for cases with severe bulbar or generalized weakness, respiratory crisis, and in refractory patients who do not respond to oral immunomodulating medications. Thymoma resection is indicated in all patients with MG and thymoma. Thymectomy is also recommended as an option in patients with nonthymomatous autoimmune MG to increase the probability of remission or improvement. Thymectomy is usually not recommended in patients over age 60. Some medications may exacerbate the symptoms of MG or precipitate the initial signs and symptoms of the disease (Table 1231)
What does the extent of depolarization dependent on?
Quantity of ACh released into the synaptic cleft Release of ACh is quantal in nature; that is, ACh is released in discrete quantities by the presynaptic motor neuron.
What is Cowden disease? What is it caused by?
Rare autosomal dominant condition that includes multiple hamartomas and breast cancer Mutations in the PTEN tumor supressor gene
What occurs in Apolipoprotein C-II deficiency?
Rare autosomal recessive disorder that leads to increased chylomicrons and VLDL particles in the circulation, resulting in severe hypertriglyceridemia
What is Lysosomal acid lipase deficiency (LAL-D)? What is it caused by?
Rare lysosomal storage disorder caused by mutations in the LIPA gene, which encodes lysosomal acid lipase This mutation leads to deficiency of the enzyme, with a corresponding reduction in the ability of LDL-C to be processed normally by hepatocytes
What do P/Q antibodies cause in LEMS?
Reduced Ca+ influx into the presynaptic nerve terminal resulting in decreased acetylcholine release and neuromuscular transmission failure.
What does Chronic serum sickness occur as a result of?
Repeated or prolonged exposure to an antigen This occurs in several diseases, such as systemic lupus erythematosus (SLE), which is associated with persistent antibody responses to autoantigens.
What is the main adverse effect associated with Statin use?
Rhabdomyolysis plasma creatine kinase levels (a marker of muscle injury) are not useful or routine monitoring of statin-treated patients
CD4 T cell-mediated inflammation is the basis of tissue injury in many organ specific and systemic autoimmune diseases such as?
Rheumatoid arthritis and Multiple Sclerosis as well as diseases probably caused by uncontrolled reactions to bacterial commensals, such as inflammatory bowel disease
What are LDL-cholesterol treatment strategies based on?
Risk indicators There is strong evidence that dietary modifications can reduce LDL-cholesterol and triglyceride levels. However, evidence that lifestyle-induced lipid modifications improve cardiovascular outcomes is limited (level C). If target goals are not achieved, then pharmacologic therapy is considered
What the autosomal dominant form of QT (LQT) syndrome known as? What is it caused by?
Roman-Ward syndrome loss of-function mutations in genes that encode potassium channels (such as KCNQ1, KCNH2, KCNE1, KCNE2, or KCNJ2). These mutations delay cardiac repolarization. Romano-Ward syndrome can also be caused by gain-of-function mutations in sodium or calcium channel genes (SCN5A and CACNA1C, respectively), which result in a prolonged depolarizing current
Which of the body's mast cells are activated by cross linking of allergen specific IgE often depends on what?
Route of entry of the allergen eg: inhaled allergens activate mast cells in the submucosal tissues of the bronchus, whereas ingested allergens activate mast cells in the wall of the intestine.
What does SR-BI mediate? which thus promotes?
SR-BI mediates efflux of excess cholesterol from the membrane SR-BI in the liver promotes selective uptake of lipids. the cholesterol and cholesteryl esters of HDL particles are taken up into the hepatocyte in the absence of uptake of apolipoproteins
What do Circulating LDL enter macrophages and other tissues via?
Scavenger receptors Because the scavenger receptors are not regulated, these cells accumulate excess intracellular cholesterol resulting in the formation of foam cells and atherosclerosis plaques
schizophrenia
Schizophrenia is a severe emotional disorder characterized by delusions, hallucinations, retreat from reality, and bizarre, withdrawn, or inappropriate behavior (contrary to popular belief, schizophrenia is not a "split personality" disorder). The lifetime recurrence risk for schizophrenia among the offspring of one affected parent is approximately 8% to 10%, which is about 10 times higher than the risk in the general population. The empirical risks increase when more relatives are affected. For example, a person with an affected sibling and an affected parent has a risk of about 15% to 20%, and a person with two affected parents has a risk of 40% to 50%. The risks decrease when the affected family member is a second- or third-degree relative. Details are given in Table 12-8. On inspection of this table, it may seem puzzling that the proportion of schizophrenic probands who have a schizophrenic parent is only about 5%, which is substantially lower than the risk for other first-degree relatives (e.g., siblings, affected parents, and their offspring). This can be explained by the fact that schizophrenics are less likely to marry and produce children than are other persons. There is thus substantial selection against schizophrenia in the population. Twin and adoption studies indicate that genetic factors are likely to be involved in schizophrenia. Data pooled from multiple twin studies show a 47% concordance rate for MZ twins, compared with only 12% for DZ twins. The concordance rate for MZ twins reared apart, 46%, is about the same as the rate for MZ twins reared together. The risk of developing the disease for offspring of a schizophrenic parent who are adopted by normal parents is about 10%, approximately the same as the risk when the offspring are raised by a schizophrenic biological parent. Dozens of genome scans have been performed in an effort to locate schizophrenia genes. The techniques discussed in Chapter 8 (GWAS, exome sequencing) have identified significant associations between schizophrenia and polymorphisms in more than 100 genomic regions. A number of the genes in these regions encode components of the glutamatergic and dopaminergic neuronal signaling pathways. These findings are biologically plausible because the major drugs used to treat schizophrenia block dopamine receptors
Nicotonic receptor antagonists
Selective nicotinic receptor antagonists are used primarily to produce nondepolarizing (competitive) neuromuscular blockade during surgical procedures. Nondepolarizing neuromuscular junction (NMJ) blockers, such as tubocurare , act by antagonizing nicotinic ACh receptors directly, thus preventing binding o endogenously released ACh and subsequent muscle cell depolarization. This leads to f accid paralysis that is similar in presentation to the paralysis in myasthenia gravis. In selecting a specific agent, the primary consideration is its duration o action—ranging rom very long-lasting agents (d-tubocurarine, pancuronium) to intermediate-duration agents (vecuronium, rocuronium) to rapidly degraded compounds (mivacurium) . Because nicotinic receptors are expressed in autonomic ganglia as well as the NMJ, nondepolarizing blocking agents o ten have variable adverse e ects associated with ganglionic blockade. Both the muscular paralysis and the autonomic blockade can be reversed by administration o AChE inhibitors. A new class o agents, epitomized by sugammadex , can also be used to accelerate the recovery o blockade by vecuronium and rocuronium . These agents act by chelating vecuronium or rocuronium in an inactive complex, which is then slowly cleared rom the circulation. Sugammadex is investigational in the United States In special cases, compounds with relatively selective antagonist activity at nAChRs can be used to induce autonomic blockade. The e ects o autonomic ganglionic blockade are discussed above and are listed in detail in Table 10-2. Most commonly, mecamylamine and trimethaphan are administered when ganglionic blockade is desired. The only current use or these agents is to treat hypertension in patients with acute aortic dissection, because the drugs lower blood pressure while simultaneously blunting the sympathetic re exes that would normally cause a deleterious rise in pressure at the site o the tear
What occurs in Tuberculosis?
T cell mediated immune response develops against protein antigens of Mycobacterium tuberculosis and the response becomes chronic because the infection is difficult to eradicate the resultant granulomatous inflammation causes injury to normal tissues at the site of infection
LEMS- diagnosis and differential diagnosis
Serum antibodies against P/Q VGCCs are found in nearly all cases of paraneoplastic LEMS, and in about 90% of nonparaneoplastic cases. Electrodiagnostic testing can help confirm the diagnosis by demonstrating reduced CMAP amplitudes in distal hand muscles; CMAP facilitation of at least 100% after 10" maximal voluntary contraction or high frequency RNS (posttetanic facilitation); and CMAP decrement greater than 10% with low frequency RNS. Patients diagnosed with LEMS should be screened and monitored with chest CT for lung cancer, especially if they are smokers and over age 50. LEMS and MG can be differentiated with electrodiagnostic and antibody testing.
What do patients develop with the most severe T cell deficits? What are they susceptible to? caused by?
Severe combined immune deficiency (SCID) Opportunistic infections eg: Pneumocystis jiroveci without bone marrow transplants these patients usually die within first several years of life X linked recessive mutations in a gene that encodes gamma chain found in 6 different cytokine receptors (those of interleukins 2,4,7,9, 15 and 21) lacking these receptors T cells and Natural killer cells cannot receive the signals they need for normal maturation persons who lack Jak3 as a result of autosomal recessive mutations in the JAK3 gene experience a form of SCID that is very similar to the X-linked form just described.
What is Niacin the drug of choice for?
Since Niacin is currently the most effective agent available for raising HDL, it may also be the drug of choice or patients with modestly elevated LDL and decreased HDL
What is the most sensitive test in the diagnosis of Myasthenia gravis? What does it reveal?
Single fiber electromyography (SFEMG) Increased jiter and blocking 99% of patients with generalized MG and 97% of those with purely ocular MG when a weak muscle is tested
What is Liddle syndrome? What is it characterized by?
Single gene disorder that causes hypertension Low plasma aldosterone and hypertension caused by mutations that alter the ENaC epithelial sodium channel
What is Gordon Syndrome? What is it characterized by?
Single gene disorder that causes hypertension hypertension, high serum potassium level and increased renal salt reabsorption caused by mutations in the WNK1 or WNK4 kinase genes
What are MHC I composed of?
Single heavy glycoprotein and a single light chain called B2- microglobulin the most important of the class 1 loci are Human leukocyte antigens A,B, C (HLA-A,B,C) on chromosome 6 these result in high MHC variability MHC I are found on the surfaces of nearly all cells and bind with cytotoxic t cell receptors
What are the most striking feature of Marfan syndrome?
Skeletal abnormalities 1- Typically the patient is unusually tall with exceptionally long extremities and long, tapering fingers and toes. 2- The joint ligaments in the hands and feet are lax, suggesting that the patient is double-jointed; typically the thumb can be hyperextended back to the wrist 3- The head is commonly dolichocephalic (long-headed) with bossing of the frontal eminences and prominent supraorbital ridges 4- spinal deformities may appear, including kyphosis, scoliosis, or rotation or slipping of the dorsal or lumbar vertebrae. 5- The chest is classically deformed, presenting either pectus excavatum (deeply depressed sternum) or a pigeon-breast deformity
Inherited deficiences in Protein C and Protein S (both of which are Coagulation inhibitors) are increased with increased risk of what ?
Stroke Because blood clots are a common cause of stroke, it is expected that mutations in genes that encode coagulation factors might affect stroke susceptibility especially in children
Morphology
Skeletal abnormalities are the most striking feature of Marfan syndrome. Typically the patient is unusually tall with exceptionally long extremities and long, tapering fingers and toes. The joint ligaments in the hands and feet are lax, suggesting that the patient is double-jointed; typically the thumb can be hyperextended back to the wrist. The head is commonly dolichocephalic (long-headed) with bossing of the frontal eminences and prominent supraorbital ridges. A variety of spinal deformities may appear, including kyphosis, scoliosis, or rotation or slipping of the dorsal or lumbar vertebrae. The chest is classically deformed, presenting either pectus excavatum (deeply depressed sternum) or a pigeon-breast deformity The ocular changes take many forms. Most characteristic is bilateral subluxation or dislocation (usually outward and upward) of the lens, referred to as ectopia lentis. This abnormality is so uncommon in persons who do not have this disease that the finding of bilateral ectopia lentis should raise the suspicion of Marfan syndrome. Cardiovascular lesions are the most life-threatening features of this disorder. The two most common lesions are mitral valve prolapse and, of greater importance, dilation of the ascending aorta due to cystic medionecrosis. Histologically the changes in the media are virtually identical to those found in cystic medionecrosis not related to Marfan syndrome (Chapter 12). Loss of medial support results in progressive dilation of the aortic valve ring and the root of the aorta, giving rise to severe aortic incompetence. In addition, excessive TGF-β signaling in the adventitia may also contribute to aortic dilation. Weakening of the media predisposes to an intimal tear, which may initiate an intramural hematoma that cleaves the layers of the media to produce aortic dissection. After cleaving the aortic layers for considerable distances, sometimes back to the root of the aorta or down to the iliac arteries, the hemorrhage often ruptures through the aortic wall. Such a calamity is the cause of death in 30% to 45% of these individuals.
some general principals and conclusions
Some general principles can be deduced from the results obtained thus far regarding the genetics of complex disorders. First, the more strongly inherited forms of complex disorders generally have an earlier age of onset (examples include breast cancer, Alzheimer disease, and heart disease). Often, these include subsets of cases in which there is single-gene inheritance. Second, when there is laterality, bilateral forms sometimes cluster more strongly in families (e.g., cleft lip/palate or breast cancer). Third, while the sex-specific threshold model fits some of the complex disorders (e.g., pyloric stenosis, cleft lip/palate, autism, heart disease), it fails to fit others (e.g., type 1 diabetes). There is a tendency, particularly among the lay public, to assume that the presence of a genetic component means that the course of a disease cannot be altered ("If it's genetic, you can't change it"). This is incorrect. Most of the diseases discussed in this chapter have both genetic and environmental components. Thus, environmental modification (e.g., diet, exercise, stress reduction) can often reduce risk significantly. Such modifications may be especially important for persons who have a family history of a disease, because they are likely to develop the disease earlier in life. Those with a family history of heart disease, for example, can often add many years of productive living with relatively minor lifestyle alterations. By targeting those who can benefit most from intervention, genetics helps to serve the goal of preventive medicine In addition, the identification of a specific genetic alteration can lead to more effective prevention and treatment of the disease. Identification of mutations causing familial colorectal cancer can allow early screening and prevention of metastasis. Pinpointing a gene responsible for a neurotransmitter defect in a psychiatric disorder such as schizophrenia could lead to the development of more effective drug treatments. In some cases, such as familial hypercholesterolemia, gene therapy may be useful. It is important for health-care practitioners to make their patients aware of these facts. Although the genetics of common disorders is complex and sometimes confusing, the public health impact of these diseases and the evidence for hereditary factors in their etiology demand that genetic studies be pursued. Substantial progress is already being made. The next decade will undoubtedly witness many advancements in our understanding and treatment of these disorders.
What are Type IV hypersensitivity reactions mediated by?
T cell mediated Diseases result from the reactions of T lymphocytes, often against self antigens in tissues.
What gene is associated with reduced levels of LDL cholesterol (LDL-C)?
Sort1 encodes Sortilin- involved in cellular trafficking of VLDL particles Sortilin facilitates the post-translational degradation of apoB by a lysosome-dependent mechanism.
Acetylcholine plays a role in pain modulation via the inhibition of?
Spinal nociceptive transmission ACh released by the cholinergic neurons is believed to bind to muscarinic ACh receptors located on secondary sensory neurons specific or pain transmission, resulting in suppression of action potential firing in these cells and consequently in analgesia
The combination of what 2 drugs may be most useful in patients with high levels of LDL cholesterol and low levels of HDL cholesterol?
Statin and Niacin co-administration of niacin and a statin could slightly increase the risk of myopathy, such patients should be closely monitored or the development of adverse effects
What is the first line therapy in the management of abnormal LDL-cholesterol levels?
Statins
Ample evidence supports the use of what in the primary and secondary prevention of Coronary heart disease (CHD)?
Statins Treatment effects of statin can be assessed after 1 to 2 months. Additional agents can be considered if target goals are not achieved with maximal drug dosing.
What drugs are used to lower plasma cholesterol by? What is its MOA?
Statins suppress intracellular cholesterol synthesis by inhibiting the enzyme HMG CoA reductase. This in turn, allows greater synthesis of LDL receptors Statins have been widely and successfully used for secondary prevention of ischemic heart disease. They exemplify rational design of drugs based on an understanding of pathophysiology.
inhibitors of cholesterol synthesis
Statins competitively inhibit the activity o HMG-CoA reductase, the rate-limiting enzyme in cholesterol synthesis. Inhibition o this enzyme results in a transient, modest decrease in cellular cholesterol concentration (Fig. 20-11). The decrease in cholesterol concentration activates a cellular signaling cascade culminating in the activation o sterol regulatory element binding protein 2 (SREBP2), a transcription actor that up-regulates expression o the gene encoding the LDL receptor. Increased LDL receptor expression causes increased uptake o plasma LDL and consequently decreases plasma LDL-cholesterol concentration. Approximately 70% o LDL receptors are expressed by hepatocytes, with the remainder expressed by a variety o cell types in the body. Statins have been shown in numerous clinical trials to reduce mortality signif cantly a ter a myocardial in arction. This is re erred to as secondary prevention . Recent studies have also concluded that lowering o LDL with statins can decrease mortality even in the absence of overt cardiovascular disease, which is called primary prevention . Despite these convincing percentage risk reductions in both secondary and primary prevention trials, it should be noted that statin use is associated with a greater absolute risk reduction in secondary prevention; the reason may be that patients in this treatment group have a higher absolute risk o death and there ore display the greatest benef t rom statins. It is also important to note that statins have proven to be e ective in reducing cardiovascular disease risk or highrisk patients (e.g., diabetic patients) with average, or even below average, LDL-cholesterol levels The magnitude o LDL-cholesterol lowering depends on the e f cacy and dose o the statin that is administered. In general, statins reduce LDL-cholesterol concentrations by up to about 60%. Statins increase HDL-cholesterol concentrations by an average o 10% and reduce triglyceride concentrations by up to about 40%, depending on statin dose and degree o hypertriglyceridemia. The e ect o statins on triglyceride levels is mediated by decreased VLDL production and increased clearance o remnant lipoproteins by the liver. The dose-response relationship o statins is nonlinear: the largest e ect occurs with the starting dose. Each subsequent doubling o the dose produces, on average, an additional 6% LDL reduction. This is sometimes re erred to as the "rule of 6s." In addition to reducing LDL-cholesterol concentrations, statins have a number o other pharmacologic consequences. These are collectively re erred to as pleiotropic effects , which include decreased in ammation, reversal o endothelial dys unction, decreased thrombosis, and improved stability o atherosclerotic plaques. Evidence or diminished in ammation with statin therapy includes decreases in acute-phase reactants, which are plasma proteins that are increased during in ammatory states and may play a role in the destabilization o atherosclerotic plaques. The best characterized acutephase reactant is C-reactive protein (CRP). Importantly, a recent large randomized clinical trial has shown that, among patients with a moderate risk o developing cardiovascular disease and with elevated baseline CRP levels, use o a statin reduces cardiovascular morbidity and mortality, even when the patients do not have elevated LDL-cholesterol concentrations. Evidence or reversal o endothelial dys unction with statin therapy includes an improved vasodilatory response o endothelium to nitric oxide. Improved vasodilation could help prevent ischemia. Evidence or decreased thrombosis with statin therapy includes a decrease in prothrombin activation and a decrease in tissue actor production. Because thrombosis is at the root o most acute coronary syndromes, its reduction could contribute to the survival benef t o statins. Finally, plaque stability is enhanced with statin therapy because the f brous cap that overlies the lipid-rich plaque becomes thicker. This e ect may be attributable to decreased macrophage inf ltration and inhibition o vascular smooth muscle proli eration. It is important to emphasize that most o these pleiotropic e ects o statins have been demonstrated only in vitro or in animal models, and their relevance in humans is unclear. Clinical data indicate that the reductions in cardiovascular morbidity and mortality due to statins are primarily attributable to the lowering o LDLcholesterol concentrations in the plasma. Seven statins— lovastatin , pravastatin , simvastatin , fluvastatin , atorvastatin , rosuvastatin, and pitavastatin —are currently approved or use in hypercholesterolemia and mixed hyperlipidemia. They are considered f rst-line therapy or increased LDL levels, and their use is supported by numerous trials showing that statins decrease both cardiovascularrelated and total mortality. Stroke is also reduced. All o the statins are believed to act by the same mechanism. The main di erences are attributable to potency and pharmacokinetic parameters. Among the statins, uvastatin is the least potent, and atorvastatin and rosuvastatin are the most potent. Beyond their capacity to reduce LDL-cholesterol concentrations, the clinical relevance o these potency di erences has not been determined. The pharmacokinetic di erences among the statins result rom di erential cytochrome P450 metabolism. Lovastatin, simvastatin, and atorvastatin are metabolized by CYP3A4, whereas other cytochrome P450-mediated pathways metabolize fluvastatin and pitavastatin. Pravastatin and rosuvastatin are not metabolized via the cytochrome P450 pathway. As explained below, the pathways o statin metabolism have important implications or drug interactions. Statins are generally well tolerated; the incidence o adverse e ects is lower with statins than with any o the other lipid-lowering drug classes. The main adverse e ect is myopathy and/or myositis with rhabdomyolysis. The latter is a very rare complication that occurs primarily at high doses o the most potent statins. There ore, plasma creatine kinase levels (a marker o muscle injury) are not use ul or routine monitoring o statin-treated patients. Certain patients who have inherited a molecular variant o an organic anion transporter responsible or statin uptake may be at higher risk o developing statin-induced myopathy (see Chapter 7, Pharmacogenomics) High-potency statins can also cause increases in serum transaminase levels (i.e., alanine transaminase [ALT] and aspartate transaminase [AST]). In the vast majority o cases, these commonly observed elevations in ALT and AST most likely re ect an adaptive response o the liver to changes in cholesterol homeostasis. True hepatotoxicity is indicated by ALT and AST elevations that are accompanied by elevations in serum bilirubin concentrations I a statin alone is insu f cient to lower LDL to target levels, the statin can be used e ectively in combination with other agents. The combination o a statin with a bile acid sequestrant or cholesterol absorption inhibitor results in additive LDL decreases and is not associated with signif cant drug interactions. The combination o niacin and a statin may be most useful in patients with high levels of LDL cholesterol and low levels of HDL cholesterol. However, because co-administration of niacin and a statin could slightly increase the risk o myopathy, such patients should be closely monitored or the development o adverse effects Fibrates and statins have also been reported to be e f cacious in combination. However, certain f brates inhibit both the transport o statins into the liver and the glucuronidation of statins in the liver, thereby decreasing statin clearance. These agents may there ore raise the plasma statin concentration and increase the risk o rhabdomyolysis. This e ect has been documented or gemfibrozil but does not occur with fenofibrate . Finally, in patients who require LDL lowering and are taking drugs that are metabolized by cytochrome P450—such as certain antibiotics, calcium channel blockers, war arin, and protease inhibitors (see Chapter 4, Drug Metabolism)—a statin that is not metabolized by P450 enzymes is preferable
What can occur if atherosclerosis develops in arteries that supplies the brain?
Stroke
adoption studies
Studies of adopted children are also used to estimate the genetic contribution to a multifactorial trait. Offspring who were born to parents who have a disease but who were adopted by parents lacking the disease can be studied to find out whether the offspring develop the disease. In some cases, these adopted persons develop the disease more often than do children in a comparative control population (i.e., adopted children who were born to parents who do not have the disease). This provides evidence that genes may be involved in causing the disease, because the adopted children do not share an environment with their affected natural parents. For example, schizophrenia is seen in 8% to 10% of adopted children whose natural parent had schizophrenia, whereas it is seen in only 1% of adopted children of unaffected parents. As with twin studies, several precautions must be exercised in interpreting the results of adoption studies. First, prenatal environmental influences could have long-lasting effects on an adopted child. Second, children are sometimes adopted after they are several years old, ensuring that some nongenetic influences have been imparted by the natural parents. Finally, adoption agencies sometimes try to match the adoptive parents with the natural parents in terms of attributes such as socioeconomic status. All of these factors could exaggerate the apparent influence of biological inheritance. Adoption studies provide a second means of estimating the influence of genes on multifactorial diseases. They consist of comparing disease rates among the adopted offspring of affected parents with the rates among adopted offspring of unaffected parents. As with the twin method, several biases can influence these studies These reservations, as well as those summarized for twin studies, underscore the need for caution in basing conclusions on twin and adoption studies. These approaches do not provide definitive measures of the role of genes in multifactorial disease, nor can they identify specific genes responsible for disease. Instead, they provide a preliminary indication of the extent to which a multifactorial disease may be influenced by genetic factors. Methods for the direct detection of genes underlying multifactorial traits are summarized in Box 12-1.
What is Sickle cell anemia caused by?
Substitution of normal hemoglobin (HbA) by hemoglobin S (HbS)
Depending on the site of the block what may accumulate in a biochemical reaction due to enzyme defects?
Substrate accumulation may be accompanied by accumulation of one or both intermediates as well an increased concentration of intermediate 2 may stimulate the minor pathway and thus lead to an excess of M1 and M2 Under these conditions tissue injury may result if the precursor, the intermediates, or the products of alternative minor pathways are toxic in high concentrations eg: Inn galactosemia, the deficiency of galactose1-phosphate uridyltransferase leads to the accumulation of galactose and consequent tissue damage
LEMS- treatment
Symptomatic treatment with 3,4DAP 5 to 10 mg every 3 to 4 hours and up to a maximum daily dose of 80 to 100 mg is most effective in improving muscle strength in patients with LEMS. Side effects at doses up to 60 mg per day are rare. Acral and perioral paresthesias occur within minutes from a dose and resolve in about 15 minutes. It is contraindicated in patients with seizures. 3,4DAP is not currently FDA approved in the United States, but it can be obtained in specialized neuromuscular centers. Pyridostigmine 60 mg every 4 hours is also used to improve symptoms. In patients in whom symptoms are not adequately controlled with 3,4DAP and pyridostigmine, immunomodulation with prednisone, azathioprine, or mycophenolate mofetil is used. Severe weakness is treated with plasmapheresis or IVIG. The underlying cancer should be treated.
What does the increase in intracellular Ca2+ facilitate the binding of? What does this complex then mediate?
Synaptotagmin to the SNARE complex proteins Vescile membrane attachment and fusion, the result is that contents of the vesicle are released in discrete quanta into the synaptic cleft
Common Autosomal dominant disorders
System Disorder Nervous Huntington disease Neurofibromatosis Myotonic dystrophy Tuberous sclerosis Urinary Polycystic kidney disease Gastrointestinal Familial polyposis coli Hematopoietic Hereditary spherocytosis von Willebrand disease Skeletal Marfan syndrome* Ehlers-Danlos syndrome (some variants)* Osteogenesis imperfecta Achondroplasia Metabolic Familial hypercholesterolemia* Acute intermittent porphyria
What is a key risk factor for heart disease, stroke and kidney disease?
Systemic hypertension it is estimated that hypertension is responsible for appx half of all cardiovascular mortality
Immune complexe diseases
Systemic lupus erythematosus Polyarteritis nodosa Poststreptococcal glomerulonephritis DNA, nucleoproteins, others In some cases, microbial antigens (e.g., hepatitis B virus surface antigen); most cases unknown Streptococcal cell wall antigen(s) Nephritis, arthritis, vasculitis Nephritis Immune complex disease Antibody specificity Clinicopathologic manifestations Vasculitis Serum sickness (clinical and experimental) Arthus reaction (experimental) Various protein antigens Various protein antigens Cutaneous vasculitis
Since T cells are never secreted from cells how are they activated?
T cell activation requires the presentation of foreign peptide along with MHC molecule
What is Type I diabetes characterized by? How is treated?
T-cell infiltration of the pancreas and destruction of the insulin-producing beta cells, usually (though not always) manifests before 40 years of age Patients with type 1 diabetes must receive exogenous insulin to survive. In addition to T-cell infiltration of the pancreas, autoantibodies are formed against pancreatic cells, insulin, and enzymes such as glutamic acid decarboxylase These findings, along with a strong association between type 1 diabetes and the presence of several human leukocyte antigen (HLA) class II alleles, indicate that this is an autoimmune disease
What is the most significant gene identified to be linked to Type II diabetes?
TCF7L2 encodes a transcription factor involved in secreting insulin A significant association has also been observed between type 2 diabetes and a common allele of the gene that encodes peroxisome proliferator-activated receptor-γ (PPAR-γ)- This receptor is the target of thiazolidinediones [TZDs], a class of drugs commonly used to increase insulin sensitivity in those with type 2 diabetes. Variation in KCNJ11, which encodes a potassium channel necessary for glucose-stimulated insulin secretion, confers an additional 20% increase in type 2 diabetes susceptibility
What is the treatment for Rheumatoid arthritis and inflammatory bowel disease?
TNF antagonists B cell depletion with anti-CD20 has also been effective in rheumatoid arthritis and multiple sclerosis
Local Immune Complex Disease (Arthus Reaction)
The Arthus reaction is a localized area of tissue necrosis resulting from acute immune complex vasculitis, usually elicited in the skin. The reaction can be produced experimentally by intracutaneous injection of antigen in a pre viously immunized animal that contains circulating
Major Histocompatibility Complex (MHC)- class 1,2,3 genes
The MHC, which includes a series of more than 200 genes that lie in a 4-Mb region on the short arm of chromosome 6 (Fig. 9-8), is commonly classified into three groups: class I, class II, and class III. The class I MHC molecule forms a complex with foreign peptides that is recognized by receptors on the surfaces of cytotoxic T lymphocytes. Class I presentation is thus essential for the cytotoxic T-cell response. Some viruses evade cytotoxic T-cell detection by down-regulating the expression of MHC class I genes in the cells they infect. Class I MHC molecules are composed of a single heavy glycoprotein chain and a single light chain called β2microglobulin (Fig. 9-9A). The most important of the class I loci are labeled human leukocyte antigens A, B, and C (HLAA, -B, and -C).† Each of these loci has dozens or hundreds of alleles, resulting in a high degree of class I MHC variability among individuals. The class I region spans 1.8 Mb and includes a number of additional genes and pseudogenes (genes that are similar in DNA sequence to coding genes but that have been altered so that they cannot be transcribed or translated). The class I molecules were first discovered in the 1940s by scientists who were experimenting with tissue grafts in mice. When the class I alleles in donor and recipient mice differed, the grafts were rejected. This is the historical basis for the term major histocompatibility complex. In humans, matching of the donor's and recipient's class I alleles increases the probability of graft or transplant tolerance. Because grafts and transplants are a relatively new phenomenon in human history, the MHC obviously did not evolve to effect transplant rejection. Instead, T cells, when confronted with foreign MHC molecules on donor cells, interpret these as foreign peptides and attack the cells The class I MHC molecules are encoded by the highly polymorphic HLA-A, -B, and -C loci on chromosome 6. In addition to presenting foreign peptides on the surfaces of infected cells, they can also bring about transplant rejection when non-self MHC molecules stimulate cytotoxic T cells. Whereas the class I MHC molecules are found on the surfaces of nearly all cells and can bind with cytotoxic T-cell receptors, the class II MHC molecules ordinarily are found only on the surfaces of the immune system's APCs (e.g., phagocytes and B lymphocytes). When associated with foreign peptides, they stimulate helper T cell activity after binding to the T cells' receptors, as described previously. The class II molecules are heterodimers consisting of an α and a β chain, each of which is encoded by a different gene located on chromosome 6 (see Fig. 9-9B). In addition to the genes in the major class II groups (HLA-DP, -DQ, and -DR), this region includes genes that encode peptide transporter proteins (TAP1 and TAP2) that help to transport peptides into the endoplasmic reticulum, where they initially form complexes with class I molecules before migrating to the cell surface. Class II MHC molecules are heterodimers encoded by genes on chromosome 6. They present peptides on the surfaces of antigenpresenting cells. These peptides, in conjunction with class II MHC molecules, are bound by receptors on helper T cells. Like the major class I MHC loci, the major class II loci are highly polymorphic, expressing hundreds of different alleles. Indeed, the MHC loci are, as a class, the most polymorphic loci known in humans. Each MHC allele encodes a molecule with slightly different binding properties: some variants bind peptide from a given pathogen more effectively than others do.‡ Consequently, a person who expresses a greater variety of MHC molecules has a better chance of dealing effectively with diverse infectious organisms. For example, someone who is homozygous for each of the major class I loci (A, B, and C) expresses only three different class I MHC molecules in each cell, whereas someone who is heterozygous for each of these loci expresses six different class I MHC molecules in each cell and can cope more successfully with pathogenic diversity (many thousands of MHC molecules are expressed on a typical cell's surface). A higher degree of polymorphism in the general population increases the chance that any individual in the population is heterozygous. For example, HIVinfected persons who are heterozygous for the HLA-A, HLA-B, and/or HLA-C loci have longer survival times than those who are homozygous at these loci. In addition, greater MHC polymorphism in a population decreases the chance that an infectious pathogen can spread easily through the population. Thus, the high degree of polymorphism in MHC genes is thought to be the result of natural selection for allelic variation. In some cases, specific MHC alleles are known to produce proteins that are effective against specific pathogens. For example, the HLA-B53 allele was shown to have a strong protective effect against severe malaria in the population of Gambia, and the HLA-DRB1*13:02 allele protects against hepatitis B infection in the same population. These alleles produce MHC molecules that have higher-affinity binding of the infectious agents. Like the class I MHC genes, class II genes are highly polymorphic. This increases the ability of individuals and populations to respond to a wide variety of pathogens. Both class I and class II MHC molecules guide T-cell receptors (cytotoxic and helper, respectively) to specific cells. T-cell receptors recognize peptides only in combination with MHC molecules on cell surfaces, a phenomenon known as MHC restriction. Not all components of the immune system are MHC restricted. The complement system, for example, does not require direct interaction with MHC molecules. Some virus-infected cells and tumor cells take advantage of MHC restriction: they suppress the expression of MHC molecules on their surfaces in an attempt to evade detection by T cells (Clinical Commentary 9-1). Fortunately, natural killer cells are activated by the absence, rather than the presence, of MHC molecules on cell surfaces. This activation is mediated by an important and diverse family of receptors found on the surfaces of natural killer cells, killer cell immunoglobulin-like receptors (KIR). These receptors inhibit natural killer cells when they bind to MHC class I molecules on the surfaces of normal cells but activate them when MHC class I molecules are absent. The class III MHC region spans 680 kb and contains at least 36 genes, only some of which are involved in the immune response. Among the most important of these are the genes encoding the complement proteins. The genes encoding the immunoglobulins, the T-cell receptors, KIR, and the class I and class II MHC proteins all share similar DNA sequences and structural features. Thus, they are members of a gene family, like the globin genes, the color vision genes, and the collagen genes described in earlier chapters. Table 9-1 provides a summary of the major genes of the immune system and their chromosome locations. It is important to emphasize that the class I and class II MHC molecules differ greatly among individuals, but each cell within an individual has the same class I and class II molecules (this uniformity is necessary for recognition by T cells). In contrast, after VDJ recombination the T-cell receptors and immunoglobulins differ from cell to cell within individuals, allowing the body to respond to a large variety of different infectious agents. The immunoglobulin, T-cell receptor, KIR, and MHC genes are members of a gene family. Whereas immunoglobulins and T-cell receptors vary between cells within an individual, MHC molecules vary between individuals.
What are hypersensitivity reactions classified on the basis of?
The Principal immunologic mechanism that is responsible for tissue injury and disease
T cell receptors
The T-cell receptors are similar in many ways to the immunoglobulins, or B-cell receptors. Like the immunoglobulins, T-cell receptors must be able to bind to a large variety of peptides derived from invading organisms. Unlike immunoglobulins, however, T-cell receptors are never secreted from the cell, and T-cell activation requires the presentation of foreign peptide along with an MHC molecule. Approximately 90% of T-cell receptors are heterodimers composed of an α and a β chain, and approximately 10% are heterodimers composed of a γ and a δ chain (Fig. 9-7). A given T cell has a population of either α-β receptors or γ-δ receptors Most of the mechanisms involved in generating immunoglobulin diversity—multiple germline gene segments, VDJ somatic recombination, and junctional diversity—are also important in generating T-cell receptor diversity. However, somatic hypermutation does not occur in the genes that encode the T-cell receptors. This is thought to help avoid the generation of T cells that would react against the body's own cells (an autoimmune response, which is discussed later in the chapter). T-cell receptors are similar in function to B-cell receptors (immunoglobulins). Unlike immunoglobulins, however, they can bind to a foreign peptide only when it is presented by an MHC molecule. Their diversity is created by the same mechanisms that produce immunoglobulin diversity, with the exception of somatic hypermutation.
Etiology of antibody-mediated diseases
The antibodies that cause disease most often are autoantibodies against self antigens and less commonly are specific for foreign (e.g., microbial) antigens. The production of autoantibodies results from a failure of self-tolerance. In Chapter 9 we discussed the mechanisms by which self-tolerance may fail, but why this happens in any human autoimmune disease is still not understood. Autoantibodies may bind to self antigens in tissues or may form immune complexes with circulating self antigens Two of the best-described diseases caused by antibodies produced against microbial antigens are rare, late sequelae of streptococcal infections. After such infections, some individuals produce antistreptococcal antibodies that cross-react with an antigen in heart tissues. Deposition of these antibodies in the heart triggers an inflammatory disease called rheumatic fever, which can lead to acute heart failure or slow scarring of valves and late-onset heart failure. Other individuals make antistreptococcal antibodies that deposit in kidney glomeruli, causing an inflammatory process called poststreptococcal glomerulonephritis that can lead to renal failure. Some immune complex diseases are caused by complexes of antimicrobial antibodies and microbial antigens. This may occur in patients with chronic infections with certain viruses (e.g., the hepatitis virus) or parasites (e.g., malaria).
What is the synthesis of Ach limited by?
The availability of Choline substrate which depends on uptake of choline into the neuron note: Choline acetyltransferase is not the rate limiting step of Ach synthesis
Inhibitors of bile acid absorption
The bile acid sequestrants are cationic polymer resins that bind noncovalently to negatively charged bile acids in the small intestine. The resin-bile acid complex cannot be reabsorbed in the distal ileum and is excreted in the stool. Decreased bile acid reabsorption by the ileum partially interrupts enterohepatic bile acid circulation, causing hepatocytes to up-regulate 7 -hydroxylase, the rate-limiting enzyme in bile acid synthesis (Fig. 20-10A). The increase in bile acid synthesis decreases hepatocyte cholesterol concentration, leading to increased expression o the LDL receptor and enhanced LDL clearance rom the circulation. The e ectiveness o bile acid sequestrants in clearing LDL rom the plasma is partially o set by concomitant up-regulation o hepatic cholesterol and triglyceride synthesis, which stimulates the production o VLDL particles by the liver. As a result, bile acid sequestrants may also raise triglyceride levels and should be used with caution in patients with hypertriglyceridemia. The three available bile acid sequestrants are cholestyramine , colesevelam , and colestipol . These drugs possess similar e f cacy, causing up to 28% reductions in LDL levels at therapeutic concentrations. In order to maximize the binding o these agents to bile acids, drug administration is timed so that the drugs are present in the small intestine a ter a meal (i.e., a ter gallbladder emptying). Because bile acid sequestrants are not absorbed systemically, they have little potential or serious toxicity. However, signif cant bloating and dyspepsia o ten limit patient adherence. Bile acid sequestrants can decrease absorption o at-soluble vitamins, and bleeding due to vitamin K def ciency has occasionally been reported. They can also bind certain co-administered drugs, such as digoxin and war arin, and thereby lower the bioavailability o the co-administered agents. This interaction can be eliminated by administering the bile acid sequestrant at least 1 hour be ore or 4 hours a ter other drugs. Colesevelam is more selective and appears to avoid this problem. Because o the demonstrated clinical e f cacy and tolerability o statins, bile acid sequestrants have been relegated to second-line agents or lipid reduction. Currently, bile acid sequestrants are used mainly or treatment o hypercholesterolemia in young ( 25 years old) patients and in patients or whom statins alone do not provide su f cient plasma LDL reduction. Some experts pre er bile acid sequestrants or young patients (such as patients with amilial hypercholesterolemia) because these agents are not absorbed and are generally considered sa e or long-term use. However, other experts pre er to use a statin or initial therapy in children.
T cell mediated type IV hypersensitivity
The cell-mediated type of hypersensitivity is caused by inflammation resulting from cytokines produced by CD4+ T cells and cell killing by CD8+ T cells (Fig. 6-18). CD4+ T cell-mediated hypersensitivity induced by environmental and self antigens is the cause of many chronic inflammatory diseases, including autoimmune diseases (Table 6-5). CD8+ cells may also be involved in some of these autoimmune diseases and may be the dominant effector cells in certain reactions, especially those that follow viral infections.
Clinical examples of CD4 t cell mediated inflammatory reactions
The classic example of DTH is the tuberculin reaction, which is produced by the intracutaneous injection of purified protein derivative (PPD, also called tuberculin), a protein-containing antigen of the tubercle bacillus. In a previously sensitized individual, reddening and induration of the site appear in 8 to 12 hours, reach a peak in 24 to 72 hours, and thereafter slowly subside. Morphologically, delayed-type hypersensitivity is characterized by the accumulation of mononuclear cells, mainly CD4+ T cells and macrophages, around venules, producing perivascular "cuffing" (Fig. 6-19). In fully developed lesions, the venules show marked endothelial hypertrophy, reflecting cytokinemediated endothelial activation. With certain persistent or nondegradable antigens, such as tubercle bacilli colonizing the lungs or other tissues, the infiltrate is dominated by macrophages over a period of 2 or 3 weeks. With sustained activation, macrophages often undergo a morphologic transformation into epithelioid cells, large epithelium-like cells with abundant cytoplasm. A microscopic aggregation of epithelioid cells, usually surrounded by a collar of lymphocytes, is referred to as a granuloma (Fig. 6-20A). This pattern of inflammation, called granulomatous inflammation (Chapter 3), is typically associated with strong TH1-cell activation and high-level production of cytokines such as IFN-γ (Fig. 6-20B). It can also be caused by indigestible foreign bodies, which activate macrophages without eliciting an adaptive immune response Contact dermatitis is a common example of tissue injury resulting from DTH reactions. It may be evoked by contact with urushiol, the antigenic component of poison ivy or poison oak, and presents as a vesicular dermatitis. It is thought that in these reactions, the environmental chemical binds to and structurally modifies some self proteins and peptides derived from these modified proteins are recognized by T cells and elicit the reaction. Chemicals may also modify HLA molecules, making them appear foreign to T cells. The same mechanism is responsible for most drug reactions, among the most common immunologic reactions of humans. In these reactions, the drug (often a reactive chemical) alters self proteins, including MHC molecules, and the "neoantigens" are recognized as foreign by T cells, leading to cytokine production and inflammation. These often manifest as skin rashes. CD4+ T cell-mediated inflammation is the basis of tissue injury in many organ-specific and systemic autoimmune diseases, such as rheumatoid arthritis and multiple sclerosis, as well as diseases probably caused by uncontrolled reactions to bacterial commensals, such as inflammatory bowel disease
Intro
The concept that the immune system is required for defending the host against infections has been emphasized throughout this book. However, immune responses are themselves capable of causing tissue injury and disease. Injurious, or pathologic, immune reactions are called hypersensitivity reactions. An immune response to an antigen may result in sensitivity to challenge with that antigen, and therefore hypersensitivity is a reflection of excessive or aberrant immune responses. Hypersensitivity reactions may occur in two situations. First, responses to foreign antigens (microbes and noninfectious environmental antigens) may cause tissue injury, especially if the reactions are repetitious or poorly controlled. Second, the immune responses may be directed against self (autologous) antigens, as a result of the failure of self-tolerance (see Chapter 9). Responses against self antigens are termed autoimmunity, and disorders caused by such responses are called autoimmune diseases. This chapter describes the important features of hypersensitivity reactions and the resulting diseases, focusing on their pathogenesis. Their clinicopathologic features are described only briefly and can be found in other medical textbooks. The following questions are addressed: • What are the mechanisms of different types of hypersensitivity reactions? • What are the major clinical and pathologic features of diseases caused by these reactions, and what principles underlie treatment of such diseases?
MG-Diagnosis and Differential diagnosis
The diagnosis of MG is based on a combination of clinical history, physical examination, and confirmatory tests. The ice pack test is a simple and relatively sensitive test to differentiate ptosis caused by MG from other causes of ptosis. In this test an ice pack is applied to the ptotic eye for 2 minutes and an improvement of 2 mm or more in ptosis supports MG. Edrophonium chloride (Tensilon) is a shortacting acethylcholinesterase inhibitor administered IV to demonstrate symptom improvement in patients with MG. A positive Tensilon test is defined as an unequivocal improvement in strength in an affected muscle after 2 to 5 minutes from administration of 2 mg incremental doses up to 10 mg. Atropine should be available during g a tensilon test because bradychardia and hypotension are possible side effects. Edrophonium testing can be positive in other disorders. Electrodiagnostic testing with 3 Hz repetitive nerve stimulation (RNS) demonstrates a compound muscle action potential (CMAP) decrement more than10% in about 50% to 75% of patients with generalized MG, but is abnormal in less than 50% of patients with purely ocular symptoms. Single fiber electromyography (SFEMG) is the most sensitive test in the diagnosis of MG and reveals increased jitter and blocking in 99% of patients with generalized MG, and in 97% of those with purely ocular MG when a weak muscle is tested. SFEMG is usually available only in specialized EMG laboratories. Serum antibody testing for AchR Ab (binding antibody) is positive in about 80% of patients with generalized MG, and 50% of patients with purely ocular symptoms. Anti MuSK antibody is detected in a portion of seronegative patients, usually men. Chest CT should be performed to rule out thymoma. Thyroid function should be evaluated because thyroid disease is commonly associated with MG. Electrodiagnostic and serum antibody testing help with differentiating MG from motor neuron disease, LambertEaton myasthenic syndrome (LEMS), and GuillainBarre syndrome (GBS).
Botulism- Clinical presentation
The disease is characterized by symmetric descending flaccid paralysis starting with blurred or double vision, ptosis, dysphagia, dry mouth, dysarthria, and muscle weakness. Symptoms usually start 18 to 36 hours after ingesting contaminated food. Botulism should be suspected in any infant with poor feeding and sucking, constipation, dilated pupils, weak cry, poor tone, and respiratory distress. Sensory examination and mental status are normal.
other complex disorders
The disorders discussed in this chapter represent some of the most common multifactorial disorders and those for which significant progress has been made in identifying genes. Many other multifactorial disorders are being studied as well, and in some cases specific susceptibility genes have been identified. These include, for example, Parkinson disease, hearing loss, multiple sclerosis, amyotrophic lateral sclerosis, epilepsy, asthma, inflammatory bowel disease, and some forms of blindness (see Table 12-6 and Table 8-2 in Chapter 8).
Relationship of Insulin gene and Type I diabetes
The insulin gene, which is located on the short arm of chromosome 11, is another logical candidate for type 1 diabetes susceptibility. Polymorphisms within and near this gene have been tested for association with type 1 diabetes. Intriguingly, a strong risk association is seen with allelic variation in a VNTR polymorphism located just 5′ of the insulin gene Differences in the number of VNTR repeat units might affect transcription of the insulin gene (possibly by altering chromatin structure), which would result in variation in susceptibility
etiology of t cell mediated diseases
The major causes of T cell-mediated hypersensitivity reactions are autoimmunity and exaggerated or persistent responses to environmental antigens. The autoimmune reactions usually are directed against cellular antigens with restricted tissue distribution. Therefore, T cell- mediated autoimmune diseases tend to be limited to a few organs and usually are not systemic. Examples of T cell-mediated hypersensitivity reactions against environmental antigens include contact sensitivity to chemicals (e.g., various therapeutic drugs and substances found in plants such as poison ivy). Tissue injury also may accompany T cell responses to microbes. For example, in tuberculosis, a T cell-mediated immune response develops against protein antigens of Mycobacterium tuberculosis, and the response becomes chronic because the infection is difficult to eradicate. The resultant granulomatous inflammation causes injury to normal tissues at the site of infection Excessive polyclonal T cell activation by certain microbial toxins produced by some bacteria and viruses can lead to production of large amounts of inflammatory cytokines, causing a syndrome similar to septic shock. These toxins are called superantigens because they stimulate large numbers of T cells. Superantigens bind to invariant parts of T cell receptors on many different clones of T cells, regardless of antigen specificity, thereby activating these cells.
omega 3 fatty acids
The omega-3 atty acids eicosapentaenoic acid (EPA) and docosahexaenoic acid (DHA), also re erred to as f sh oils, are e ective at reducing plasma triglycerides by up to 50% in patients with hypertriglyceridemia. The likely mechanism o triglyceride lowering involves regulation o nuclear transcription actors, including SREBP-1c and PPAR , to cause reduced triglyceride biosynthesis and increased atty acid oxidation in the liver. Omega-3 atty acids are available over the counter as nutritional supplements in the orm o atty acid ethyl esters. Lovaza® , a prescription-strength orm o omega-3 atty acids, has also become available. Lovaza® is enriched (84%) in EPA and DHA, whereas most dietary supplements contain 13-63% f sh oils. The recommended dose o Lovaza® is 4 grams once a day. Omega-3 atty acids are generally added to therapy when plasma triglyceride concentrations exceed 500 mg/dL. The in uence o omega-3 atty acid use on clinical outcomes is uncertain.
Systemic immune disease part 2
The pathogenesis of systemic immune complex disease can be divided into three phases (Fig. 6-17). 1. Formation of immune complexes. The introduction of a protein antigen triggers an immune response that results in the formation of antibodies, typically about a week after the injection of the protein. These antibodies are secreted into the blood, where they react with the antigen still present in the circulation and form antigenantibody complexes. 2. Deposition of immune complexes. In the next phase the circulating antigen-antibody complexes are deposited in various tissues. The factors that determine whether immune complex formation will lead to tissue deposition and disease are not fully understood, but the major influences seem to be the characteristics of the complexes and local vascular alterations. In general, complexes that are of medium size, formed in slight antigen excess, are the most pathogenic. Organs where blood is filtered at high pressure to form other fluids, like urine and synovial fluid, are sites where immune complexes become concentrated and tend to deposit; hence, immune complex disease often affects glomeruli and joints 3. Inflammation and tissue injury. Once immune complexes are deposited in the tissues, they initiate an acute inflammatory reaction. During this phase (approximately 10 days after antigen administration), clinical features such as fever, urticaria, joint pains (arthralgias), lymph node enlargement, and proteinuria appear. Wherever complexes deposit the tissue damage is similar. The mechanisms of inflammation and injury were discussed above, in the discussion of antibodymediated injury. The resultant inflammatory lesion is termed vasculitis if it occurs in blood vessels, glomerulonephritis if it occurs in renal glomeruli, arthritis if it occurs in the joints, and so on. It is clear that complement-fixing antibodies (i.e., IgG and IgM) and antibodies that bind to leukocyte Fc receptors (some subclasses of IgG) induce the pathologic lesions of immune complex disorders. The important role of complement in the pathogenesis of the tissue injury is supported by the observations that complement proteins can be detected at the site of injury and, during the active phase of the disease, consumption of complement leads to a decrease in serum levels of C3. In fact, serum C3 levels can, in some cases, be used to monitor disease activity. If the disease results from a single large exposure to antigen, such as acute serum sickness, the lesions tend to resolve as a result of catabolism of the immune complexes. A form of chronic serum sickness results from repeated or prolonged exposure to an antigen. This occurs in several diseases, such as systemic lupus erythematosus (SLE), which is associated with persistent antibody responses to autoantigens. In many diseases, the morphologic changes and other findings suggest immune complex deposition but the inciting antigens are unknown. Included in this category are membranous glomerulonephritis and several vasculitides.
metabolism of apoB containing lipoproteins
The primary unction o apoB-containing lipoproteins is to deliver atty acids in the orm o triglycerides to muscle tissue or use in ATP biogenesis and to adipose tissue or storage. Chylomicrons are ormed in the intestine and transport dietary triglycerides, whereas VLDL particles are ormed in the liver and transport triglycerides that are synthesized endogenously. The metabolic li espan o apoB-containing lipoproteins can be divided into three phases: assembly, intravascular metabolism, and receptor-mediated clearance. This is a convenient categorization because pharmacologic agents are available that in uence each phase.
Morphology
The principal morphologic manifestation of immune complex injury is acute vasculitis, associated with necrosis of the vessel wall and intense neutrophilic infiltration. The necrotic tissue and deposits of immune complexes, complement, and plasma protein appear as a smudgy eosinophilic area of tissue destruction, an appearance termed fibrinoid necrosis (see Fig. 2-15). When deposited in the kidney, the complexes can be seen on immunofluorescence microscopy as granular lumpy deposits of immunoglobulin and complement and on electron microscopy as electron-dense deposits along the glomerular basement membrane antibodies against the antigen. As the antigen diffuses into the vascular wall, it binds the preformed antibody, and large immune complexes are formed locally. These complexes precipitate in the vessel walls and cause fibrinoid necrosis, and superimposed thrombosis worsens the ischemic injury
Botulism- Diagnosis
The proportion of patients with botulism who die has fallen from 50% to between 3% and 5% in the past 50 years. Recovery of muscle strength may take several months. Mortality in untreated botulism is 60%.
Prognosis of Botulism
The proportion of patients with botulism who die has fallen from 50% to between 3% and 5% in the past 50 years. Recovery of muscle strength may take several months. Mortality in untreated botulism is 60%.
What does the overall effect of Ganglionic blockade depend on?
The relative predominance of sympathetic and parasympathetic tone at the various end organs For example the heart is influenced at rest primarily by the parasympathetic system, whose tonic effect is a slowing of the heart rate. Thus, blockade of autonomic ganglia that innervate the heart by moderate to high doses of the antimuscarinic agent atropine results in blockade of vagal slowing o the sinoatrial node and hence in relative tachycardia . Blood vessels, in contrast, are innervated only by the sympathetic system. Because the normal e ect o sympathetic stimulation is to cause vasoconstriction, ganglionic blockade results in vasodilation.
Figure 12.8-
The renin-angiotensin-aldosterone system. ↑, Increased; ↓, decreased; AT1, angiotensin type II receptor 1. (Modified from King RA, Rotter JI, Motulsky AG, eds. The Genetic Basis of Common Diseases. New York: Oxford University Press; 1992
clinical symptoms and therapy 2
The therapy for immediate hypersensitivity diseases is aimed at inhibiting mast cell degranulation, antagonizing the effects of mast cell mediators, and reducing inflammation (Fig. 11-6). Common drugs include antihistamines for hay fever, agents that relax bronchial smooth muscles in asthma, and epinephrine in anaphylaxis. In diseases with inflammation as an important pathologic component, such as asthma, corticosteroids are used to inhibit inflammation. Many patients benefit from repeated administration of small doses of allergens, called desensitization or allergen-specific immunotherapy. This treatment may work by changing the T cell response away from Th2 dominance or the antibody response away from IgE, by inducing tolerance in allergen-specific T cells, or by stimulating regulatory T cells (Tregs). Before concluding the discussion of immediate hypersensitivity, it is important to address the question of why evolution has preserved an IgE antibody- and mast cell-mediated immune response whose major effects are pathologic. There is no definitive answer to this puzzle, but immediate hypersensitivity reactions likely evolved to protect against pathogens or toxins. It is known that IgE antibody and eosinophils are important mechanisms of defense against helminthic infections, and mast cells play a role in innate immunity against some bacteria and in destroying venomous toxins.
obesity
The worldwide prevalence of obesity is increasing rapidly among adults and children. Approximately 70% of American adults and 60% of British adults are overweight (body mass index [BMI] >25),§ and about half of these overweight persons are obese (BMI >30). Although obesity itself is not a disease, it is an important risk factor for several common diseases, including heart disease, stroke, type 2 diabetes, and cancers of the prostate, breast, and colon As one might expect, there is a strong correlation between obesity in parents and obesity in their children. This could easily be ascribed to common environmental effects: parents and children usually share similar diet and exercise habits. However, there is good evidence for genetic components as well. Four adoption studies each showed that the body weights of adopted persons correlated significantly with their natural parents' body weights but not with those of their adoptive parents. Twin studies also provide evidence for a genetic effect on body weight, with most studies yielding heritability estimates between 0.60 and 0.80. The heritability of "fatness" (measured, for example, by skinfold thickness) is approximately 0.50 Research, aided substantially by mouse models, has shown that several genes likely play a role in human obesity. Important among these are the genes that encode leptin (Greek, "thin") and its receptor. The leptin hormone is secreted by adipocytes (fat storage cells) and binds to receptors in the hypothalamus, the site of the body's appetite control center. Increased fat stores lead to an elevated leptin level, which produces satiety and a loss of appetite. Lower leptin levels lead to increased appetite. Mice with loss-of-function mutations in the leptin gene have uncontrolled appetites and become obese. When injected with leptin, these mice lose weight. Mice with mutations in the leptin receptor gene cannot respond to increased leptin levels and also develop obesity. Identification of the leptin gene and its receptor in mice led to their identification in humans, which in turn prompted optimistic predictions that leptin could be a key to weight loss in humans (without the perceived unpleasantness of dieting and exercise). However, most obese humans have high levels of leptin, indicating that the leptin gene is functioning normally. Leptin receptor defects were then suspected, but these are also uncommon in humans. Although mutations in the human leptin gene and its receptor have now been identified in a few humans with severe obesity (BMI >40), they both appear to be extremely rare. Unfortunately, studying these genes will not solve the problem of human obesity. However, clinical trials using recombinant leptin have demonstrated moderate weight loss in a subset of obese individuals In addition, leptin participates in important interactions with other components of appetite control, such as neuropeptide Y, as well as α-melanocyte-stimulating hormone and its receptor, the melanocortin-4 receptor (MC4R). Mutations in the gene that encodes MC4R have been found in 3% to 5% of severely obese individuals. Several genomewide association studies have demonstrated an association between an intronic variant in the brain-expressed FTO gene and obesity in whites. Homozygosity for this variant, which is seen in about 16% of whites, confers increased risks of overweight and obesity of 40% and 70%, respectively. Recent evidence shows that the FTO variant is part of an enhancer that binds to the IRX gene, which is located 2 Mb away from FTO and is involved in regulation of fat mass. Identification of these and other obesity-predisposing genes is leading to a better understanding of appetite control in humans and could eventually lead to effective treatments for some cases of obesity. Multifactorial Inheritance and Common Diseases Adoption and twin studies indicate that at least half of the population variation in obesity may be caused by genes. Specific genes and gene products involved in appetite control and susceptibility to obesity, including leptin and its receptor, MC4R, and FTO, are now being studied
ABO system
There are four major ABO blood types: A, B, AB, and O. The first three groups respectively represent persons who carry the A, B, or A and B antigens on their erythrocyte surfaces. Those with type O have neither the A nor the B antigen. Persons who have one of these antigens on their erythrocyte surfaces possess antibodies against all other ABO antigens in their blood stream. (These antibodies are formed early in life as a result of exposure to antigens that are identical to the A and B antigens but are present in various microorganisms.) Thus, if a type B person received type A or AB blood, his or her anti-A antibodies would produce a severe and possibly fatal reaction. Type O persons, who have neither the A nor the B antigen and thus both anti-A and anti-B antibodies, would react strongly to blood of the other three types (A, B, and AB). It was once thought that type O persons, because they lack both types of antigens, could be "universal donors" (anyone could accept their blood). Similarly, type AB persons were termed "universal recipients" because they lacked both anti-A and anti-B antibodies. However, when patients are given transfusions of whole blood containing large volumes of serum, the donor's antibodies can react against the recipient's erythrocyte antigens. Hence, complete ABO matching is nearly always done for blood transfusions. The ABO locus encodes red cell antigens that can cause a transfusion reaction if donors and recipients are not properly matched.
How can Familial combined Hyperlipoproteinemia be diagnosed?
There are no definitive diagnostic tests, but family screening can help confirm the diagnosis The phenotype of FCHL is variable, with individuals displaying high LDL-cholesterol, high VLDL-triglyceride, or both based on the genetic defect and environmental factors Patients also typically have high apo B (>120 mg/dL) and a low ratio of LDL-cholesterol to apo B100 (<1.2). They accumulate small dense LDL particles, which are
Conclusion
There are two major classes o cholinergic receptors: nicotinic and muscarinic. Nicotinic receptors are ligand-gated channels that require the direct binding o two acetylcholine molecules to open. These receptors comprise all o the cholinergic receptors at the neuromuscular junction (N M ), and they predominate at autonomic ganglia (N N ). Thus, the primary cholinergic unctions mediated by nAChRs include skeletal muscle contraction and autonomic activity. The predominant applications o pharmacologic agents directed at nAChRs are (1) neuromuscular blockade, through competitive antagonists and depolarizing blockers, and (2) ganglionic blockade, which results in e ector organ responses that are opposite to those produced by physiologic autonomic tone. Muscarinic receptors are G protein-coupled receptors that bind acetylcholine and initiate signaling through several intracellular pathways. These receptors are expressed in the autonomic ganglia and e ector organs, where they mediate a parasympathetic response. The primary use o muscarinic receptor agonists and antagonists is to modulate autonomic responses o e ector organs. Both nicotinic and muscarinic receptors are ubiquitous in the CNS, where the e ects o acetylcholine include analgesia, arousal, and attention. The relative roles o mAChRs and nAChRs in the brain and spinal cord are not ully understood, and the most e ective currently available CNS drugs increase endogenous cholinergic transmission by inhibiting the action o acetylcholinesterase, the enzyme that hydrolyzes ACh. Although cholinergic pharmacology is a relatively mature f eld with several receptor-selective agents, the specif city o action o the various agents continues to be ref ned. The discovery o muscarinic receptor subtype diversity may lead to the development o agents selective or subtypes that are expressed in a tissue-specif c pattern. Similarly, elucidationo the role o nicotinic receptor subunit diversity in the CNS has spurred development o more selective agents that modulate the activity o these receptor subtypes. For example, a selective partial agonist at the 7 nicotinic ACh receptor is in late-stage clinical trial testing in AD dementia. Another avenue or uture investigation involves positive allosteric modulators o nicotinic receptors; these agents may augment endogenous cholinergic tone in a manner that is more spatially and temporally specif c, thus potentially providing di erential e f cacy and improved sa ety. Acetylcholinesterase inhibitors are widely used in clinical practice and are standard o care in the treatment o AD and other dementias. They may provide short-term (6-12 month) symptomatic benef ts in AD and, when used chronically, slow clinical decline. Several nicotinic and muscarinic agonists and receptor modulators are in clinical development or the treatment o cognitive impairment, AD dementia, neuropathic pain syndromes, and neuroprotection. Nicotinic receptors may also provide targets or uture treatment approaches in epilepsy. Finally, the physiologic and pathophysiologic roles o the non-neuronal cholinergic system remain to be ully delineated, and specif c therapies targeted at this system remain to be developed.
For multifactorial diseases that are either present or absent what has to be crossed before the disease is expressed?
Threshold of liability before the threshold, the person appears unaffected, above it, he or she is affected by the disease
What is performed in MG patients with MG and thymoma?
Thymoma resection
What does Nicotinic acid stimulate? which prevents?
Tissue plasminogen activator Prevents Thrombosis
What does the most commonly mutated gene in Dilated cardiomyopathy encode?
Titin a cytoskeletal protein other genes that can cause dilated cardiomyopathy encode cytoskeletal proteins including Troponin T, desmin and components of the dystroglycan-sacroglycan complex
principals of multifactorial inheritance- multifactorial model
Traits in which variation is thought to be caused by the combined effects of multiple genes are called polygenic ("many genes"). When environmental factors are also believed to cause variation in the trait, which is usually the case, the term multifactorial is used. Many quantitative traits (those, such as blood pressure, that are measured on a continuous numerical scale) are multifactorial. Because they are caused by the additive effects of many genetic and environmental factors, these traits tend to follow a normal, or bell-shaped, distribution in populations. Let us use an example to illustrate this concept. To begin with the simplest case, suppose (unrealistically) that height is determined by a single gene with two alleles, A and a. Allele A tends to make people tall, and allele a tends to make them short. If there is no dominance at this locus, then the three possible genotypes, AA, Aa, and aa, will produce three phenotypes: tall, intermediate, and short. Assume that the allele frequencies of A and a are each 0.50. If we assemble a population of individuals, we will observe the height distribution depicted in Figure 12-1A. Now suppose, a bit more realistically, that height is determined by two loci instead of one. The second locus also has two alleles, B (tall) and b (short), and they affect height in exactly the same way as alleles A and a do. There are now nine possible genotypes in our population: aabb, aaBb, aaBB, Aabb, AaBb, AaBB, AAbb, AABb, and AABB. Because an individual might have zero, one, two, three, or four "tall" alleles, there are now five distinct phenotypes (see Fig. 12-1B). Although the height distribution in our population is not yet normal, it approaches a normal distribution more closely than in the single-gene case We now extend our example so that many genes and environmental factors influence height, each having a small effect. Then there are many possible phenotypes, each differing slightly, and the height distribution approaches the bellshaped curve shown in Figure 12-1C. Genome-wide association studies (GWAS, see Chapter 8) have identified more than 200 loci associated with human height, affirming that this is indeed a polygenic, multifactorial trait. The loci underlying variation in a quantitative trait such as height are termed quantitative trait loci. It should be emphasized that the individual genes underlying a multifactorial trait such as height follow the mendelian principles of segregation and independent assortment, just like any other genes. The only difference is that many of them act together to influence the trait. Blood pressure is another example of a multifactorial trait. There is a correlation between parents' blood pressures (systolic and diastolic) and those of their children, and there is good evidence that this correlation is due in part to genes. But blood pressure is also influenced by environmental factors, such as diet and stress. One of the goals of genetic research is identification of the genes responsible for multifactorial traits such as blood pressure and of the interactions of those genes with environmental factors. Many traits are thought to be influenced by multiple genes as well as by environmental factors. These traits are said to be multifactorial. When they can be measured on a continuous scale, they often follow a normal distribution.
treatment
Treatment is initiated after two abnormal lipid findings. Treatment of elevated total cholesterol and LDL-cholesterol can slow the development and progression of CHD. Meta-analysis of primary and secondary prevention trials indicates that CHD mortality decreases by approximately 15% for 10% reduction in serum cholesterol. LDL-cholesterol treatment strategies are based on risk indicators (Table 69-5). There is strong evidence that dietary modifications can reduce LDL-cholesterol and triglyceride levels (Table 69-6). However, evidence that lifestyle-induced lipid modifications improve cardiovascular outcomes is limited (level C). If target goals are not achieved, then pharmacologic therapy is considered (Table 69-7). Ample evidence supports statin use in primary and secondary prevention of CHD (level A). Treatment effects of statin can be assessed after 1 to 2 months. Additional agents can be considered if target goals are not achieved with maximal drug dosing. A fasting lipid panel is required to diagnose hypertriglyceridemia. Triglyceride levels higher than 200 mg/dL are classified as abnormal. Borderline triglyceride levels range from 150 to 200 mg/dL, and normal values are lower than 150 mg/dL. A diet and exercise program is recommended for all individuals with abnormal triglyceride levels (level C). However, pharmacologic treatments to reduce triglyceride levels may be considered if fasting levels are higher than 200 mg/dL, especially if the individual is at risk for CHD or pancreatitis (see Table 69-7). Fibrates, fish oil, and nicotinic acid should be considered if the triglyceride level is higher than 500 mg/dL (level C). However, for levels lower than 500 mg/dL, statins are first-line therapy (level B). Low HDL concentrations (<40 mg/dL) can also increase the risk for CHD. In the Framingham Heart Study, every decrease in HDL of 5 mg/dL increased the risk for myocardial infarction. Both lifestyle modifications (e.g., diet low in saturated fat, exercise) and pharmacologic therapy (e.g., nicotinic acid, fibrate) can improve HDL levels. However, target goals and treatment recommendations have not been established due to a lack of evidence.
Treatment of what can result normalizing E2 homozygote levels in familial dysbetalipoproteinemia?
Treatment of coexisting conditions such as diabetes and hypothyroidism If target levels are not achieved, dietary therapy and lipid-lowering drugs such as fibric acid derivatives and HMG-CoA reductase inhibitors should also be considered.
What is the primary effect of Ach at the NMJ?
Triggering of muscle contraction
What does Apo C-II (cofactor for LPL) hydrolzye? what forms as a result?
Triglyceride core of VLDL particles VLDL remnant or intermediate density lipoprotein (IDL) The IDL, depleted of triglycerides (25%), can be cleared from the circulation by apo E-mediated LDL receptors, or it can be hydrolyzed further to form low-density lipoproteins (LDL) LDL particles are triglyceride poor (5% of LDL mass) and consist mostly of cholesterol esters (60%) and apolipoproteins
In the cell glycerol combines with 3 Fatty acid to form what? What is cholesterol esterified to form?
Triglycerides Cholesterol esters
What do hepatocytes synthesize in response to free fatty acid flux to the liver?
Triglycerides This typically occurs in response to fasting, thereby ensuring a continuous supply of fatty acids or delivery to muscle in the absence of triglycerides from the diet. dietary saturated fats as well as carbohydrates also stimulate the synthesis of triglycerides within the liver.
T or F Scavenger receptors are not down-regulated when the phagocytic cells begin to accumulate cholesterol?
True Unlike the LDL receptor, scavenger receptors are not down-regulated when the phagocytic cells begin to accumulate cholesterol. As a result, the continued accumulation of oxidized LDL in macrophages can lead to foam cell formation (cholesterol-rich macrophages) These foam cells may undergo apoptotic or necrotic death, releasing free radicals and proteolytic enzymes.
T or F Heterozygotes with familial hypercholesterolemia possess only 50% of the normal number of highaffinity LDL receptors
True because they have only one normal gene. As a result of this defect in transport, the catabolism of LDL by the receptor-dependent pathways is impaired, and the plasma level of LDL increases approximately two fold. Homozygotes have virtually no normal LDL receptors in their cells and have much higher levels of circulating LDL. addition to defective LDL clearance, both the homozygotes and heterozygotes have increased synthesis of LDL The mechanism of increased synthesis that contributes to hypercholesterolemia also results from a lack of LDL receptors
T or F ApoB48-containing chylomicron remnants are completely cleared from the plasma.
True By contrast, the presence of apoB100 alters the metabolism of VLDL remnants so that only approximately 50% are cleared by the pathways or remnant particles
T or F There is a strong correlation between obesity in parents and obesity in their children
True can be ascribed to environmental factors- children often share similar diet and exercise levels as parents Four adoption studies each showed that the body weights of adopted persons correlated significantly with their natural parents' body weights but not with those of their adoptive parents Twin studies also provide evidence for a genetic effect on body weight, with most studies yielding heritability estimates between 0.60 and 0.80
What is a classic example of CD4 T cell mediated delayed type hypersensitivity reaction? What is it produced by?
Tuberculin reaction (PPD Test) produced by the intracutaneous injection of purified protein derivative (PPD, also called tuberculin), a protein-containing antigen of the tubercle bacillus In a previously sensitized individual, reddening and induration of the site appear in 8 to 12 hours, reach a peak in 24 to 72 hours, and thereafter slowly subside
What are Dizygotic twins?
Twins that are the result of a double sperm ovulation followed by the fertilization of each egg by a different sperm thus, DZ twins are genetically no more similar than siblings it is possible for fraternal twins to have 2 different fathers
What are Monozygotic twins?
Twins that originate when the developing embryo divides to form two separate but genetically identical embryos AKA identical twins MZ twins are an example of natural clones and are strikingly similar in appearance
What is an example of disease that occurs as a result of T cell mediated (CD8) tissue destruction?
Type I diabetes
What type of diabetes accounts for more than 90% of all diabetes cases?
Type II diabetes It currently affects approximately 10% to 20% of the adult populations of many developed countries. MZ twin concordance rates are substantially higher than in type 1 diabetes, often exceeding 90% (because of age dependence, the concordance rate increases if older subjects are studied). The empirical recurrence risks for first-degree relatives of patients with type 2 diabetes are higher than those for type 1 patients, generally ranging from 15% to 40%.
What does the Vascular type of EDS result from abnormalities in?
Type III Collagen This form is genetically heterogeneous, because at least three distinct types of mutations affecting the COL3A1 gene encoding collagen type III can give rise to this variant Some affect the rate of synthesis of pro-αlpha1 (III) chains, others affect the secretion of type III procollagen and still others lead to the synthesis of structurally abnormal type III collagen
What is the prognosis of paraneoplastic LEMS determined by?
Underlying cancer The presence of LEMS in patients with small cell lung cancer (SCLC) is associated with longer survival from the malignancy. Nonparaneoplastic LEMS, when optimally treated, has an excellent prognosis and normal life expectancy, although patients may continue to experience various degrees of muscle weakness.
What blood type is considered universal donor? universal recipeints?
Universal donor: Type O Universal recipient: Type AB
multifactorial disorders in the adult population
Until recently, very little was known about specific genes responsible for common adult diseases. With more powerful laboratory and analytical techniques, this situation is changing. We next review recent progress in understanding the genetics of the major common adult diseases
mendelian disorders
Virtually all Mendelian disorders are the result of mutations in single genes that have large effects. It is not necessary to detail Mendel's laws here, since every student in biology, and possibly every garden pea, has learned about them at an early age. Only some comments of medical relevance are made. It is estimated that every individual is a carrier of five to eight deleterious genes, a number originally estimated from studies of populations that appears to be borne out by genomic sequencing of normal individuals. Most of these are recessive and therefore do not have serious phenotypic effects. About 80% to 85% of these mutations are familial. The remainder represents new mutations acquired de novo by an affected individual. Some autosomal mutations produce partial expression in the heterozygote and full expression in the homozygote. Sickle cell anemia is caused by substitution of normal hemoglobin (HbA) by hemoglobin S (HbS). When an individual is homozygous for the mutant gene, all the hemoglobin is of the abnormal, HbS, type, and even with normal saturation of oxygen the disorder is fully expressed (i.e., sickling deformity of all red cells and hemolytic anemia). In the heterozygote, only a proportion of the hemoglobin is HbS (the remainder being HbA), and therefore red cell sickling occurs only under unusual circumstances, such as exposure to lowered oxygen tension. This is referred to as the sickle cell trait to differentiate it from full-blown sickle cell anemia. Although Mendelian traits are usually described as dominant or recessive, in some cases both of the alleles of a gene pair contribute to the phenotype—a condition called codominance. Histocom patibility and blood group antigens are good examples of codominant inheritance A single mutant gene may lead to many end effects, termed pleiotropism; conversely, mutations at several genetic loci may produce the same trait (genetic heterogeneity). Sickle cell anemia is an example of pleiotropism. In this hereditary disorder not only does the point mutation in the gene give rise to HbS, which predisposes the red cells to hemolysis, but also the abnormal red cells tend to cause a logjam in small vessels, inducing, for example, splenic fibrosis, organ infarcts, and bone changes. The numerous differing end-organ derangements are all related to the primary defect in hemoglobin synthesis. On the other hand, profound childhood deafness, an apparently homogeneous clinical entity, results from many different types of autosomal recessive mutations. Recognition of genetic heterogeneity not only is important in genetic counseling but also is relevant in the understanding of the patho genesis of some common disorders, such as diabetes mellitus.
HDL metabolism and reverse cholesterol transport
Virtually all cells in the body are capable o synthesizing all the cholesterol they require. However, only the liver has the capacity to eliminate cholesterol, and it does so by secreting unesterif ed cholesterol into the bile or by converting cholesterol to bile acids. As noted above, HDL serves as a reservoir or exchangeable apolipoproteins or the metabolism o apoB-containing lipoproteins. HDL also plays a key role in cholesterol homeostasis by removing excess cholesterol rom cells and transporting it in plasma to the liver. This process is often re erred to as reverse cholesterol transport (Fig. 20-9A). The major apolipoproteins o HDL are apoAI and apoAII. ApoAI, the main structural determinant o HDL, participates in the ormation o the particle and its interaction with its receptor, scavenger receptor class B, type I (SR-BI). The unction o apoAII is not well understood—it appears to play a role in maintaining the structural integrity o HDL
PCSK9 inhibitors
When LDL-R degradation is reduced by genetic loss-o unction mutations in PCSK9, cardiovascular disease incidence is dramatically reduced by 88%. This observation has led to aggressive interest in PCSK9 as a therapeutic target. Monoclonal antibodies that target the LDL-R—interacting domain o PCSK9 are newly approved by the FDA or the treatment o patients with heterozygous FH or clinical atherosclerotic cardiovascular disease (ASCVD) who require additional therapy beyond diet and maximal statin therapy. These include evolocumab and alirocumab, which are administered subcutaneously and can lower LDL-C by 50-72%. These drugs have been well tolerated in short-term clinical trials and in post-marketing surveillance, with no major reported adverse e ects. The hal -li e o these antibodies is long, and the LDL-C lowering e ects persist or up to 2-4 weeks. Interestingly, PCSK9 degrades many receptor targets (LDL-R, apoE-R2, VLDL-R, and LRP1) that are also receptors or viruses, such as human rhinovirus and hepatitis C virus. This suggests that the incidence o viral in ection will need to be monitored closely in ongoing large-scale clinical trials o PCSK9 inhibitors.
When does Maternal-fetal incompatibility occur?
When a Rh- positive man and an Rh- negative women produce children If the man's genotype is DD, all of their offspring will be Rh-positive and will have Rh antigens on their erythrocytes. If the man is a heterozygote,with genotype Dd, half of their children will be Rh-positive, on average
Immune response: basic concepts innate immune system
When a pathogenic microorganism is encountered, the body's first line of defense includes phagocytes (a type of cell that engulfs and destroys the microorganism) and the complement system. The complement proteins can destroy microbes directly by perforating their cell membranes, and they can also attract phagocytes and other immune system agents to microbes by coating the microbial surface (it is because of this assisting role that the term complement originated). Natural killer cells, a specific type of lymphocyte, can respond to certain viral infections and some tumor cells. Phagocytes, complement, and natural killer cells are all part of the innate immune system, which is capable of responding very rapidly to pathogen The innate immune system is activated by general features that are detected in pathogens but are not found in the host. For example, gram-negative bacteria produce lipopolysaccharides, and gram-positive bacteria produce peptidoglycans. Some bacteria have a high percentage of unmethylated CG sequences, and some viruses produce double-stranded RNA. These distinctive features of pathogenic organisms can be detected by receptor molecules located on the surfaces of innate immune system cells. An important example is the Toll-like family of receptors, named after a cell-surface receptor, Toll, that was first described in fruit flies. The genes that encode the human and fruit-fly versions of Toll-like receptors have a remarkable similarity to one another, attesting to their importance in maintaining an innate immune response in a wide variety of organisms. Indeed, all multicellular organisms are thought to possess innate immune systems. The innate immune system, which includes some phagocytes, natural killer cells, and the complement system, is an early part of the immune response and recognizes general features of invading microorganisms
inflammation
When antibodies deposit in fixed tissues, such as basement membranes and extracellular matrix, the resultant injury is due to inflammation. The deposited antibodies activate complement, generating by-products, including chemotactic agents (mainly C5a), which direct the migration of polymorphonuclear leukocytes and monocytes, and anaphylatoxins (C3a and C5a), which increase vascular permeability (Fig. 6-16B). The leukocytes are activated by engagement of their C3b and Fc receptors. This results in the production of other substances that damage tissues, such as lysosomal enzymes, including proteases capable of digesting basement membrane, collagen, elastin, and cartilage, and reactive oxygen species. Antibody-mediated inflammation is the mechanism responsible for tissue injury in some forms of glomerulonephritis, vascular rejection in organ grafts, and other disorders (Table 6-3).
When does Secondary immunodeficiency occur?
When components of the immune system are altered or destroyed by other factors such as radiation, infection or drugs eg: HIV causes AIDS which attacks macrophages and helper T lymphocytes
When does an immunodeficiency disease occur?
When one or more components of the immune system (eg: T cells, B cells, MHC, complement proteins) are missing or fail to function normally
What is incomplete penetrance?
When some individuals inherit the mutant gene but are phenotypically normal 50% penetrance indicates that 50% of those who carry the gene express the trait
recurrence risks and transmission patterns
Whereas recurrence risks can be given with confidence for single-gene diseases (50% for a completely penetrant autosomal dominant disease, 25% for autosomal recessive diseases, and so on), risk estimation is more complex for multifactorial diseases. This is because the number of genes contributing to the disease is usually not known, the precise allelic constitution of the parents is not known, and the extent of environmental effects can vary substantially. For most multifactorial diseases, empirical risks (i.e., risks based on direct observation of data) have been derived. To estimate empirical risks, a large series of families is examined in which one child (the proband) has developed the disease. The relatives of each proband are surveyed in order to calculate the percentage who have also developed the disease. For example, in North America neural tube defects are seen in about 2% to 3% of the siblings of probands with this condition (Clinical Commentary 12-1). Thus, the recurrence risk for parents who have had one child with a neural tube defect is 2% to 3%. For conditions that are not lethal or severely debilitating, such as cleft lip and cleft palate, recurrence risks can also be estimated for the offspring of affected parents. Because risk factors vary among diseases, empirical recurrence risks are specific for each multifactorial disease. In contrast to most single-gene diseases, recurrence risks for multifactorial diseases can change substantially from one population to another (notice the differences between the London and Belfast populations in Table 12-1). This is because allele frequencies as well as environmental factors can differ among populations. Empirical recurrence risks for multifactorial diseases are based on studies of large collections of families. These risks are specific to a given population
What is Anaphylaxis caused by?
Widespread mast cell degranulation in response to the systemic distribution of the antigen it is life threatening because of the sudden fall in blood pressure and airway obstruction
A 19-year-old male college student presents to the clinic for his yearly physical. He is an undergrad biological science major and completing his pre-medical requirements. You complete his physical and he sparks up a conversation about how he learned all about the biochemistry of lipoprotein formation. Interested, you ask him what the difference is between the chylomicrons and VLDL particles, what would be the correct answer? a) Chylomicrons are composed of one apoB48 and VLDLs are composed of one apoB100 b) Chylomicrons are composed of one apoB100 and VLDLs are composed of one apoB48 c) Chylomicrons are the densest lipoproteins and VLDLs are the least dense lipoproteins d) Chylomicrons carry endogenous lipids and VLDLs carry exogenous lipids e) Both are the same but are in different locations.
a) Chylomicrons are composed of one apoB48 and VLDLs are composed of one apoB100
intravascular metabolism of apoB containing lipoproteins
Within the circulation, chylomicrons and VLDL particles must be activated in order to target triglyceride delivery to muscle and adipose tissue (Fig. 20-5). Activation requires the addition o an optimal complement o apoCII molecules, which occurs by aqueous trans er o apoCII rom HDL particles. Because there is an inherent delay in the trans er o apoCII to chylomicrons and VLDL particles, there is time or widespread circulation o triglyceride-rich particles throughout the body. Lipoprotein lipase (LPL) is a lipolytic enzyme expressed on the endothelial sur ace o capillaries in muscle and at tissue. LPL is a glycoprotein that is synthesized by myocytes and adipocytes and transported to the endothelial cell sur ace by a specif c glycosylphosphatidylinositol (GPI)-linked protein, GPIHBP1. On the endothelial cell membrane, GPIHBP1 also serves to anchor LPL in place. Once chylomicrons and VLDL particles acquire apoCII, they can bind to LPL, which hydrolyzes triglycerides rom the core o the lipoprotein (Fig. 20-5). LPL-mediated lipolysis liberates ree atty acids and glycerol. The ree atty acids are then taken up by the neighboring parenchymal cells. The expression level and intrinsic activity o LPL in muscle and adipose tissue are regulated according to the ed/ asting state, allowing the body to direct the delivery o atty acids pre erentially to muscle during asting and to adipose a ter a meal. The rate o lipolysis o chylomicron and VLDL triglycerides is also controlled by apoCIII, which is an inhibitor o LPL activity. LPL inhibition by apoCIII may be an additional mechanism promoting widespread distribution o triglyceride-rich particles in the circulation.
A 56-year-old male presents to his primary care physician for a yearly checkup. He has type 2 diabetes. His physician is concerned about his health because he admits that in the last year his alcohol consumption has increased dramatically. Both alcohol excess and type 2 diabetes can lead to hypertriglyceridemia. What is the main mechanism in which this is accomplished? a) Increased VLDL synthesis b) Decreased VLDL synthesis c) Decreased ApoCIII levels d) Decreased fatty acid synthesis e) Increased VLDL catabolism by LPL
a) Increased VLDL synthesis
A mother brings in her 14-year-old daughter to the emergency department due to sudden onset of abdominal pain that radiates to her back. The examining physician determines the child has pancreatitis as well as splenomegaly and hepatomegaly. Labs reveal the patient has extremely elevated triglycerides and the physician decides to perform an infusion of plasma that contains normal apoCII which does not reduce plasma triglyceride levels. The physician instructs the mother to place her daughter on a fat-free diet and to avoid the use of glucocorticoids and alcohol when she is older. The patient's condition is caused by the absence of which of the following? a) Lipoprotein lipase b) ApoCII c) Defect in bile acid metabolism d) ApoB100 e) PCSK9
a) Lipoprotein lipase
A 6'8" tall patient presents with pectus excavatum, bilateral ectopia lentis, and scoliosis. On physical exam, the doctor observes that the joint ligaments are lax, and easily hyperextendable. The doctor informs the patient that because of this underlying connective tissue disorder, he is at a much greater risk for an aortic dissection. An autosomal dominant inherited gene defect leads to a defect in which protein, causing the symptom manifestations? a. Fibrillin b. Lysyl Hydroxylase c. Procollagen N-peptidase d. Spectrin e. Dystrophin
a. Fibrillin
A very clever virus enters a lung cell and downregulates MHC class 1 receptors on the cell. This mechanism helps the virus evade what type of cell? a. Lymphocyte with CD8 b. Phagocyte with CD7 c. Lymphocyte with CD4 d. Phagocyte with CD26 e. Lymphocyte with CD12
a. Lymphocyte with CD8
What is the failure to inactivate tissue-damaging substrate best exemplified by?
alpha1 antitrypsin deficiency Individuals who have an inherited deficiency of serum α1-antitrypsin are unable to inactivate neutrophil elastase in their lungs. Unchecked activity of this protease leads to destruction of elastin in the walls of lung alveoli, leading eventually to pulmonary emphysema
What can be the causes that result in Polygenic Hypercholesterolemia?
apo E may play a role in the pathogenesis 1- Apo E4 on chylomicrons and VLDL remnants has a high affinity for the LDL receptor. Elevated binding of apo E4-containing lipoproteins to LDL receptors may downregulate LDL receptor synthesis and increase circulating LDL levels. 2- Environmental factors such as diet can influence production of chylomicrons and VLDL, resulting in downregulation of the LDL receptor in conditions with high apo E4
Once triglycerides and cholesteryl esters are packaged together with apoB48 what is added and what happens next?
apoA1 apoA1 is added as an additional structural apolipoprotein and the chylomicron particle is exocytosed into the lymphatics for transport to the circulation via the thoracic duct
What does Niacin increase the half life of?
apoAI the major apolipoprotein of HDL The increase in plasma apoAI increases plasma HDL concentrations by up to 30% and presumably augments reverse cholesterol transport.
What do VLDL particles produced by the liver contain?
apoB100
What do Chylomicrons (the apoB containing lipoprotein produced by the intestine) contain?
apoB48
What is the rate of lipolysis of chylomicron and VLDL triglycerides controlled by? which is an?
apoCIII Inhibitor of LPL activity LPL inhibition by apoCIII may be an additional mechanism promoting widespread distribution of triglyceride rich particles in the circulation
What is an important risk factor for the more common late-onset Alzheimer's disease?
apolipoprotein E (APOE) locus has three major alleles: ε2, ε3, and ε4 persons who have one copy of the ε4 allele are 2 to 5 times more likely to develop AD, and those with two copies of this allele are 5 to 10 times more likely to develop AD
What is an End plate potential (EPP) ? What does it result in?
arrival of an action potential at the motor axon terminal causes many vesicles (up to thousands) to use with the neuronal membrane and release their ACh. At the motor end plate, the result is a relatively large depolarization termed the end-plate potential (EPP) Triggering of an action potential
What areas can cholesterol plaques be depositied?
arteries (atheroma), skin or tendons (xanthoma), eyelids (xanthelasma), and iris (corneal arcus).
How can ApoCII deficiency be distinguished from LDL deficiency?
by demonstrating that the triglyceride levels of patients are reduced following in fusion of plasma that contains normal apoCII this does not occur in patients with familial LPL deficiency mutations in apoAV can present with chylomicronemia and severe hypertriglyceridemia, consistent with an apparent role or apoAV in facilitating the interaction between apoCII and LPL
How do antibodies that react with antigens present on cell surfaces or in extracellular matrix cause disease?
by: 1- Destroying these cells 2- Triggering inflammation 3- Interfering with normal functions
What do Twin studies show about Bipolar disorder?
concordance rates of 79% and 24% for MZ and DZ twins, respectively, yielding a heritability estimate of approximately 72%
A 48-year-old male presents to his general physician for a regular check-up, which included having blood work done. The results showed that the patient had a plasma cholesterol level of 350mg/dL, which is almost twice the normal amount. This level is very concerning as it could put the patient at risk for atherosclerosis and potentially fatal results. Knowing that this is a multifactorial disease, which of the following criteria is incorrect when defining multifactorial inheritance? a) Higher recurrence risk if more than 1 family member is affected b) Recurrence risk is higher if the proband is of the less commonly affected sex c) If expression of the disease in the proband is more severe, the recurrence risk is higher d) Recurrence risk for the disease usually increases rapidly in more remotely related relatives e) If the prevalence of the disease in a population is f (which varies btwn 0 and 1), the risk for offspring & siblings of probands is approximately sqrt(f)
d) Recurrence risk for the disease usually increases rapidly in more remotely related relatives
What does saturated fat downregulate and increase levels of?
downregulates hepatic LDL receptors and increases circulating LDL Because unsaturated fats (polyunsaturated and monounsaturated) generally do not have this effect, they are the preferred form of fat intake. However, polyunsaturated fats containing fatty acids with a trans rather than cis double bond configuration (trans-fatty acids) increase plasma cholesterol levels similarly to saturated fat.
A 67-year-old male presents to the clinic. Lab results show elevated cholesterol levels of 367 mg/dL, high LDL levels of 170 mg/dl, and newly appeared xanthomas on face. Patient also reports family history of Hypercholesterolemia through paternal and maternal lineage. What is characteristic of familial hypercholesterolemia? a. Mutation in the gene that encodes for the LDL receptor b. Mutation in the apolipoprotein B c. Mutation in the cardiac troponin T (TNNT2) d. Mutation in renal epithelial sodium channel subunits (SCNN1B, SCNN1G) e. Both A and B are correct
e. Both A and B are correct
What is the adverse effect associated with fibrate and statin use together?
ertain fibrates inhibit both the transport of statins into the liver and the glucuronidation of statins in the liver, thereby decreasing statin clearance These agents may there ore raise the plasma statin concentration and increase the risk of rhabdomyolysis. this effect ocurs with Gemfibrozil but not Fenofibrate
What does HDL serve as a reservoir for?
exchangeable apolipoproteins in the plasma, including apoAI, apoCII, and apoE.
What are the adverse effects associated with Muscarinic overstimulation? What is used to treat these symptoms?
nausea, vomiting, diarrhea, sweating, hypersalivation, cutaneous flushing, reflex tachycardia (sometimes bradycardia), and bronchoconstriction Atropine (competitive blockade)
What are the 2 mechanisms for the removal of IDL from the plasm?
one mediated by an LDL receptor and the other by a receptor for oxidized LDL (scavenger receptor)
The defect in the conversion of procollagen to collagen in the arthrochalasia type has been traced to what? What occurs as a result?
one of the two type I collagen genes, COL1A1 and COL1A2 structurally abnormal pro-α 1 (I) or pro-α2 (I) chains that resist cleavage of N-terminal peptides are formed
What is ApoCII deficiency? What is it caused by?
rare genetic disorder with presentation and treatment similar to familial lipoprotein lipase deficiency. caused by deficiency of apoCII, a cofactor protein of LPL